GATE Exam 2023 Chemistry (CY) Question Paper With Answer Key

GATE-2023

CY: Chemistry

GA-General Aptitude

Q.1 – Q.5 Carry ONE mark each.

1. “I cannot support this proposal. My ________ will not permit it.”

(A)  conscious

(B)  consensus

(C)  conscience

(D)  consent

Answer: (C)

2. Courts : _______ : : Parliament : Legislature

(By word meaning)

(A)  Judiciary

(B)  Executive

(C)  Governmental

(D)  Legal

Answer: (A)

3. What is the smallest number with distinct digits whose digits add up to 45?

(A)  123555789

(B)  123457869

(C)  123456789

(D)  99999

Answer: (C)

4. In a class of 100 students,

(i) there are 30 students who neither like romantic movies nor comedy movies,

(ii) the number of students who like romantic movies is twice the number of students who like comedy movies, and

(iii) the number of students who like both romantic movies and comedy movies is 20.

How many students in the class like romantic movies?

(A)  40

(B)  20

(C)  60

(D)  30

Answer: (C)

5. How many rectangles are present in the given figure?

(A)  8

(B)  9

(C)  10

(D)  12

Answer: (C)

Q.6 – Q.10 Carry TWO marks Each

6. Forestland is a planet inhabited by different kinds of creatures. Among other creatures, it is populated by animals all of whom are ferocious. There are also creatures that have claws, and some that do not. All creatures that have claws are ferocious.

Based only on the information provided above, which one of the following options can be logically inferred with certainty?

(A)  All creatures with claws are animals.

(B)  Some creatures with claws are non-ferocious.

(C)  Some non-ferocious creatures have claws.

(D)  Some ferocious creatures are creatures with claws.

Answer: (D)

7. Which one of the following options represents the given graph?

(A)  f(x) = x2 2|x|

(B)  f(x) = x 2|x|

(C)  f(x) = |x|2x

(D)  f(x) = x 2x

Answer: (A)

8. Which one of the following options can be inferred from the given passage alone?

When I was a kid, I was partial to stories about other worlds and interplanetary travel. I used to imagine that I could just gaze off into space and be whisked to another planet.

[Excerpt from The Truth about Stories by T. King]

(A)  It is a child’s description of what he or she likes.

(B)  It is an adult’s memory of what he or she liked as a child.

(C)  The child in the passage read stories about interplanetary travel only in parts.

(D)  It teaches us that stories are good for children.

Answer: (B)

9. Out of 1000 individuals in a town, 100 unidentified individuals are covid positive. Due to lack of adequate covid-testing kits, the health authorities of the town devised a strategy to identify these covid-positive individuals. The strategy is to:

(i) Collect saliva samples from all 1000 individuals and randomly group them into sets of 5.

(ii) Mix the samples within each set and test the mixed sample for covid.

(iii) If the test done in (ii) gives a negative result, then declare all the 5 individuals to be covid negative.

(iv) If the test done in (ii) gives a positive result, then all the 5 individuals are separately tested for covid.

Given this strategy, no more than _______ testing kits will be required to identify all the 100 covid positive individuals irrespective of how they are grouped.

(A)  700

(B)  600

(C)  800

(D)  1000

Answer: (A)

10. A 100 cm × 32 cm rectangular sheet is folded 5 times. Each time the sheet is folded, the long edge aligns with its opposite side. Eventually, the folded sheet is a rectangle of dimensions 100 cm×1 cm.

The total number of creases visible when the sheet is unfolded is _______.

(A)  32

(B)  5

(C)  31

(D)  63

Answer: (C)

CY: Chemistry

Q.11 – Q.35 Carry ONE mark Each

11. The major product formed in the given reaction is

DME: 1,2-Dimethoxyethane

Answer: (A)

12. The compound which gives a fragment at m/z = 124[M + H]+ is

Answer: (B)

13. The major product formed in the given reaction is

Answer: (A)

14. The major product formed in the given reaction is

Answer: (A)

15. On irradiation using UV light (>300 nm), compounds X and Y, predominantly, undergo

(A)  X: Norrish type I reaction and Y: Norrish type II reaction

(B)  X: Norrish type II reaction and Y: Norrish type I reaction

(C)  Both X and Y: Norrish type I reaction

(D)  Both X and Y: Norrish type II reaction

Answer: (B)

16. The topicity relationship of Ha and Hb in X, Y and Z are, respectively,

(A)  Diastereotopic, Homotopic and Enantiotopic

(B)  Homotopic, Enantiotopic and Enantiotopic

(C)  Homotopic, Homotopic and Enantiotopic

(D)  Diastereotopic, Enantiotopic and Homotopic

Answer: (B)

17. Compound P was prepared based on a four-component reaction at room temperature in methanol. The required starting materials for the synthesis are

Answer: (A)

18. The major product formed in the following reaction is

Answer: (B)

19. The reaction of Ph3PCl2 with PhNH2 primarily produces

(A)  Ph3P=NPh

(B)  PhP=NPh

(C)  PhCl2P=NPh

(D)  Ph2ClP=NPh

Answer: (A)

20. Formation of [M(en)3]2+ from [M(H2O)6]2+ and three equivalents of ethylenediamine (en) is LEAST favored when M is

(A)  Co

(B)  Ni

(C)  Cu

(D)  Fe

Answer: (D)

21. Wacker oxidation of alkenes is catalyzed by a combination of

(A)  Pd(II) and Cu(II)

(B)  Co(II) and Cu(II)

(C)  Pd(II) and Ni(II)

(D)  Pd(II) and Co(II)

Answer: (A)

22. For the conversion of [Pt(L)Cl3] to trans-[Pt(L)Cl2(H2O)], the trans-effect is LEAST when the ligand L is

(A)  H2O

(B)  NH3

(C)  DMSO

(D)  C2H4

Answer: (A)

23. The tetracoordinated copper center in the oxidized and reduced forms of plastocyanin exhibits longest bond with

(A)  cysteine-S and methionine-S, respectively

(B)  methionine-S and cysteine-S, respectively

(C)  cysteine-S and cysteine-S, respectively

(D)  methionine-S and methionine-S, respectively

Answer: (D)

24. The packing efficiency (in %) of spheres for a body-centered cubic (bcc) lattice is approximately

(A)  74

(B)  68

(C)  60

(D)  52

Answer: (B)

25. The magnitudes of CFSE in [M(H2O)6]n+ for Mn and Fe ions satisfy the relations

(A)  Mn2+ < Mn3+ and Fe2+ < Fe3+

(B)  Mn2+ > Mn3+ and Fe2+ > Fe3+

(C)  Mn2+ < Mn3+ and Fe2+ > Fe3+

(D)  Mn2+ > Mn3+ and Fe2+ < Fe3+

Answer: (C)

26. The organometallic catalyst for the following transformation is

Answer: (A)

27. Point group of naphthalene (C10H8) is

(A)  D2d

(B)  D2h

(C)  D3d

(D)  D­3h

Answer: (B)

28. The INCORRECT statement is

(A)  Zero-point energy of a quantum mechanical harmonic oscillator of frequency v is hv/2

(B)  Energy level of a quantum mechanical rigid rotor is inversely proportional to its moment of inertia

(C)  The time independent Schrödinger equation for Li2+ cannot be solved exactly

(D)  Total angular momentum of an atomic system is equal to the sum of orbital angular momentum and spin angular momentum

Answer: (C)

29. For an ideal gas, the molecular partition function in the canonical ensemble, that is proportional to the system volume (V), is the

(A)  vibrational partition function

(B)  rotational partition function

(C)  electronic partition function

(D)  translational partition function

Answer: (D)

30. Assertion (S): The total angular momentum for light atoms (low atomic number) is obtained by Russell-Saunders coupling, whereas jj-coupling is used for heavy atoms (high atomic number).

Reasoning (R): The spin-orbit interactions are weak in light atoms (low atomic number) and strong in heavy atoms (high atomic number).

The correct option is

(A)  S and R are true; and R is the correct reason for S

(B)  S and R are true; but R is NOT the correct reason for S

(C)  S is true but R is false

(D)  S is false but R is true

Answer: (A)

31. The acetolysis product(s) of the given reaction is(are)

Answer: (A, C)

32. Product(s) formed in the given reaction is(are)

Answer: (A, B, C)

33. The choice(s) that correctly identify radioisotopes (P, Q, R, S) shown in the following nuclear reaction is(are)

Answer: (A, B, D)

34. For the Lindemann-Hinshelwood mechanism of gas phase unimolecular reactions, the true statement(s) is(are)

(A)  Only molecules with three or more atoms can follow the Lindemann-Hinshelwood mechanism

(B)  Lindemann-Hinshelwood mechanism involves bimolecular elementary steps

(C)  The overall reaction is of second order at low pressure

(D)  The overall reaction is of second order at high pressure

Answer: (A, B, C)

35. The calculated magnetic moment of [Ce(NO3)5]2− is _______ BM. (rounded off to two decimal places)

(Given: atomic number of Ce is 58)

Answer: (2.51 to 2.55)

Q.36 – Q.65 Carry TWO marks Each

36. A compound, C15H16O2 , has the following spectral data;

1H NMR (ppm): 9.16 (s), 6.89 (d, J = 8 Hz), 6.64 (d, J = 8 Hz), 1.53 (s)

13C NMR (ppm): 154.7, 140.9, 127.1, 114.4, 40.7, 30.7

The structure of the compound is

Answer: (A)

37. The major product formed in the given reaction sequence is

Answer: (B)

38. E and F in the given reaction scheme are

Answer: (A)

39. M and N in the given reaction scheme are

NMO: N-Methylmorpholine-N-oxide

Answer: (C)

40. In the 1H NMR spectrum, multiplicity of the signal (bold and underlined H atom) in the following species is

(A)  I- pentet, II- quartet, III- doublet and IV- singlet

(B)  I- pentet, II- singlet, III- singlet and IV- doublet

(C)  I- triplet, II- triplet, III- doublet and IV- doublet

(D)  I- singlet, II- quartet, III- singlet and IV- singlet

Answer: (A)

41. The major product obtained by the treatment of (η5-C5H5)2Ni with Na/Hg in ethanol is

(A)  (η5-C5H5)( η3-C5H5)Ni

(B)  (η3-C5H5)2Ni

(C)  (η5-C5H5)( η3-C5H7)Ni

(D)  (η3-C5H7)2Ni

Answer: (C)

42. The number of shared corners of the constituent SiO4 units in orthosilicate, pyrosilicate, cyclic silicate and sheet silicate, respectively, are

(A)  0, 1, 2 and 3

(B)  2, 3, 0 and 1

(C)  0, 3, 1 and 2

(D)  1, 2, 3 and 0

Answer: (A)

43. Concentration of Q in a consecutive reaction  is given by  where [P]0 is the initial concentration of P.

If the value of k2 = 25 s−1, the value of k1 that leads to the longest waiting time for Q to reach its maximum is

(A)  k1 = 20 s1

(B)  k1 = 25 s1

(C)  k1 = 30 s1

(D)  k1 = 35 s1

Answer: (A)

44. The wave function for Be3+ in a certain state is given by  where N is the normalization constant, r is the distance of electron from the nucleus and 𝑎0 is the Bohr radius. The most probable distance of the electron from the nucleus in this state is

(A)  4a0

(B)  a0/4

(C)  8a0

(D)  a0/8

Answer: (B)

45. Match the following

(A)  P→III, Q→I, R→IV, S→II

(B)  P→III, Q→IV, R→II, S→I

(C)  P→IV, Q→I, R→III, S→II

(D)  P→II, Q→III, R→IV, S→I

Answer: (A)

46. In the scheme below,

Iα represents the intensity of the light absorbed. Assuming that the quantum yield of the first step is one, the steady state concentration of Q is given by

Answer: (A)

47. Product(s) formed in the given reaction sequence is(are)

Answer: (A, B)

48. Product(s) formed in the reaction below is(are)

Answer: (A, C)

49. The stereoisomer(s) of G giving the depicted product is(are)

Answer: (A, C)

50. Product(s) formed in the given reaction sequence is(are)

Answer: (A, C)

51. The reaction(s) in which inversion of configuration occur(s) is(are)

Answer: (A, B)

52. The correct statement(s) regarding myoglobin (Mb) and haemoglobin (Hb) is(are)

(A)  At low partial pressure of O2 (e.g., 5 kPa), the O2 affinity of Hb lowers upon lowering the pH

(B)  Binding of the first O2 molecule to Hb results in lower affinity for the binding of second O2 molecule

(C)  Metal center in deoxy-Mb is low-spin whereas it is high-spin in the case of oxy-Mb

(D)  One end of O2 binds to the metal center in oxy-Mb and the other end of the bound O2 is H-bonded with imidazole-NH of a distal histidine

Answer: (A, D)

53. The correct statement(s) regarding Co2(CO)8 is(are)

(A)  It reacts with Na to give Na[Co(CO)4]

(B)  It contains three bridging carbonyls

(C)  It can be prepared by reductive carbonylation of Co(OAc)2·4H2O

(D)  Two isomers exist in hexane solution

Answer: (A, C, D)

54. The compound(s) having [Xe]4f1 configuration is(are)

(Given the atomic numbers Ce:58, Lu:71, Pr:59 and Nd:60)

(A)  Na3[Ce(NO3)6]

(B)  Na3[LuCl6]

(C)  PrO2

(D)  Nd(NR2)3 (R = SiMe3)

Answer: (A, C)

55. The correct statement(s) for XeF2 is(are)

(A)  Its bonding is best explained by classical 2-centered-2-electron bonds

(B)  Its bonding is best explained by a non-classical 3-centered-4-electron bond

(C)  It contains nine lone pairs of electrons

(D)  Its point group is D∞h

Answer: (B, C, D)

56. For the non-dissociative adsorption of a gas on solid,

(i) the Freundlich isotherm is given by θ = kp1/n where θ is surface coverage, p is pressure, k and n are empirical constants; and

(ii) the BET isotherm is given by 

where p* and c empirical constants, and p < p*.

The correct statement(s) is(are)

(A)  At low surface coverage, the Langmuir isotherm reduces to the Freundlich isotherm with n = 1

(B)  At high surface coverage, the Langmuir isotherm reduces to the Freundlich isotherm with n = ∞

(C)  At very low pressure (p << p*), the BET isotherm reduces to the Langmuir isotherm

(D)  At very high pressure (p → p*), the BET isotherm reduces to the Langmuir isotherm

Answer: (A, B, C)

57. Two different enzyme catalysis reactions I and II have identical Y-intercepts for the Lineweaver-Burke (equation given below) plots. The slope for reaction I is twice than that of reaction II.

If the initial concentrations of enzymes in I and II are same, the correct statement(s) is(are)

where v and vmax are rate and maximum rate; KM is Michaelis-Menten constant, and [S] is substrate concentration.

(A)  Reactions I and II have same turn over number

(B)  Michaelis-Menten constants for reactions I and II are identical

(C)  Michaelis-Menten constant for reaction I is twice than that of reaction II

(D)  The rates of the elementary steps for reactions I and II are identical

Answer: (A, C)

58. The enthalpy change for the exothermic reaction between BeI2 and HgF2 is _______kJ mol−1 (rounded off to the nearest integer)

(Given: Bond dissociation energy (in kJ mol−1) for Be−F = 632, Be−I = 289, Hg−F = 268 and Hg−I = 145)

Answer: (-440 to -440)

59. Number of carbon atoms connected to the metal center in [W(C60)(CO)5] is _______ (rounded off to the nearest integer)

(Given: atomic number of W is 74)

Answer: (7 to 7)

60. Two-component solid-liquid system of naphthalene-benzene forms a simple eutectic mixture. Assuming that naphthalene-benzene forms an ideal solution, the mole fraction of naphthalene in benzene at 300 K and 1 bar is _______ (rounded off to two decimal places)

(Given: Freezing point (Tfp) and enthalpy of fusion (∆Hfus) of naphthalene are 353 K and 19.28 kJ mol−1, respectively and gas constant (R) =8.31 J K−1 mol−1)

Answer: (0.30 to 0.32)

61. The intrinsic viscosity of a sample of polystyrene in toluene is 84 cm3 g−1 at 30° It follows Mark-Houwink equation with empirical constant values of K = 1.05 × 10−2 cm3 g−1 and a = 0.75. The molecular weight of the polymer is ________ × 103 g mol−1 (rounded off to the nearest integer)

Answer: (158 to 160)

62. According to Debye-Hückel limiting law, the mean molal activity coefficient for 0.87 g K2SO4 (molar mass = 174 g mol−1) in 1 kg of water at 25°C is _________ (rounded off to two decimal places)

Answer: (0.74 to 0.76)

63. A solution is prepared by dissolving 128 g of naphthalene (C10H8) in 780 g of benzene (C6H6). The vapor pressure of pure benzene is 12.6 kPa at 25° Assuming that naphthalene in benzene is an ideal solution, the partial vapor pressure of benzene is _________ kPa (rounded off to two decimal places)

Answer: (11.34 to 11.47)

64. For the galvanic cell: H2 (g) | HCl (aq) | Cl2 (g)

the standard electromotive force (E0) value is given by

E0 = 1.73 – (1.25 × 103)T + (1.00 × 106)T2

where E0 is in Volts and T is in Kelvin.

For the cell reaction, the standard enthalpy change (∆rH0) at 300 K is _________ kJ mol1(rounded off to the nearest integer)

(Given: Faraday constant, F = 96500 C mol1)

Answer: (-164 to -154)

65. A solution of three non-interacting compounds P, Q, and R is taken in a cuvette of 1 cm path length. Their concentrations are [P] = 1 × 10−6 M, [Q] = 2 × 10−6 M, [R] = 3 × 10−6 M and the molar extinction coefficients at 300 nm are εP =1 × 105 M−1 cm−1, εQ = 2 × 105 M−1 cm−1 and εR = 3 × 105 M−1 cm−1. The % transmittance at 300 nm is __________ (rounded off to two decimal places)

Answer: (3.98 to 4.00)

GATE Exam 2023 Computer Science and Information Technology (CS) Question Paper With Answer Key

GATE-2023

CS: Computer Science and Information Technology

General Aptitude

Q.1 – Q.5 Carry ONE mark each.

1. We reached the station late, and _______ missed the train.

(A)  near

(B)  nearly

(C)  utterly

(D)  mostly

Answer: (B)

2. Kind : _______ : : Often : Frequently

(By word meaning)

(A)  Mean

(B)  Type

(C)  Cruel

(D)  Kindly

Answer: (MTA)

3. A series of natural numbers F­1, F2, F3, F4, F5, F6, F7, … obeys Fn+1 = Fn + Fn – 1 for all integers n ≥

If F6 = 37, and F7 = 60, then what is F1?

(A)  4

(B)  5

(C)  8

(D)  9

Answer: (A)

4. A survey for a certain year found that 90% of pregnant women received medical care at least once before giving birth. Of these women, 60% received medical care from doctors, while 40% received medical care from other healthcare providers.

Given this information, which one of the following statements can be inferred with certainty?

(A)  More than half of the pregnant women received medical care at least once from a doctor.

(B)  Less than half of the pregnant women received medical care at least once from a doctor.

(C)  More than half of the pregnant women received medical care at most once from a doctor.

(D)  Less than half of the pregnant women received medical care at most once from a doctor.

Answer: (A)

5. Looking at the surface of a smooth 3-dimensional object from the outside, which one of the following options is TRUE?

(A)  The surface of the object must be concave everywhere.

(B)  The surface of the object must be convex everywhere.

(C)  The surface of the object may be concave in some places and convex in other places.

(D)  The object can have edges, but no corners.

Answer: (C)

Q.6 – Q.10 Carry TWO marks Each

6. The country of Zombieland is in distress since more than 75% of its working population is suffering from serious health issues. Studies conducted by competent health experts concluded that a complete lack of physical exercise among its working population was one of the leading causes of their health issues. As one of the measures to address the problem, the Government of Zombieland has decided to provide monetary incentives to those who ride bicycles to work.

Based only on the information provided above, which one of the following statements can be logically inferred with certainty?

(A)  All the working population of Zombieland will henceforth ride bicycles to work.

(B)  Riding bicycles will ensure that all of the working population of Zombieland is free of health issues.

(C)  The health experts suggested to the Government of Zombieland to declare riding bicycles as mandatory.

(D)  The Government of Zombieland believes that riding bicycles is a form of physical exercise.

Answer: (D)

7. Consider two functions of time (t),

                             F(t) = 0.01 t2

                             g(t) = 4t

where 0 < t < ∞.

Now consider the following two statements:

(i) For some t > 0, g(t) > f(t)

(ii) There exists a T, such that f(t) > g(t) for all t > T.

Which one of the following options is TRUE?

(A)  only (i) is correct

(B)  only (ii) is correct

(C)  both (i) and (ii) are correct

(D)  neither (i) nor (ii) is correct

Answer: (C)

8. Which one of the following sentence sequences creates a coherent narrative?

(i) Once on the terrace, on her way to her small room in the corner, she notices the man right away.

(ii) She begins to pant by the time she has climbed all the stairs.

(iii) Mina has bought vegetables and rice at the market, so her bags are heavy.

(iv) He was leaning against the parapet, watching the traffic below.

(A)  (i), (ii), (iv), (iii)

(B)  (ii), (iii), (i), (iv)

(C)  (iv), (ii), (i), (iii)

(D)  (iii), (ii), (i), (iv)

Answer: (D)

9. f(x) and g(y) are functions of x and y, respectively, and f(x) = g(y) for all real values of x and y. Which one of the following options is necessarily TRUE for all x and y?

(A)  f(x) = 0 and g(y) = 0

(B)  f(x) = g(y) = constant

(C)  f(x) ≠ constant and g(y) ≠ constant

(D)  f(x) + g(y) = f(x) – g(y)

Answer: (B)

10. Which one of the options best describes the transformation of the 2-dimensional figure P to Q, and then to R, as shown?

(A)  Operation 1: A clockwise rotation by 90º about an axis perpendicular to the plane of the figure

Operation 2: A reflection along a horizontal line

(B)  Operation 1: A counter clockwise rotation by 90º about an axis perpendicular to the plane of the figure

Operation 2: A reflection along a horizontal line

(C)  Operation 1: A clockwise rotation by 90º about an axis perpendicular to the plane of the figure

Operation 2: A reflection along a vertical line

(D)  Operation 1: A counter clockwise rotation by 180º about an axis perpendicular to the plane of the figure

Operation 2: A reflection along a vertical line

Answer: (A)

CS: Computer Science and Information Technology

Q.11 – Q.35 Carry ONE mark each.

11. Consider the following statements regarding the front-end and back-end of a compiler.

S1: The front-end includes phases that are independent of the target hardware.

S2: The back-end includes phases that are specific to the target hardware.

S3: The back-end includes phases that are specific to the programming language used in the source code.

Identify the CORRECT option.

(A)  Only S1 is TRUE.

(B)  Only S1 and S2 are TRUE.

(C)  S1, S2, and S3 are all TRUE.

(D)  Only S1 and S3 are TRUE.

Answer: (B)

12. Which one of the following sequences when stored in an array at locations A[1], . . . , A[10] forms a max-heap?

(A)  23, 17, 10, 6, 13, 14, 1, 5, 7, 12

(B)  23, 17, 14, 7, 13, 10, 1, 5, 6, 12

(C)  23, 17, 14, 6, 13, 10, 1, 5, 7, 15

(D)  23, 14, 17, 1, 10, 13, 16, 12, 7, 5

Answer: (B)

13. Let SLLdel be a function that deletes a node in a singly-linked list given a pointer to the node and a pointer to the head of the list. Similarly, let DLLdel be another function that deletes a node in a doubly-linked list given a pointer to the node and a pointer to the head of the list.

Let n denote the number of nodes in each of the linked lists. Which one of the following choices is TRUE about the worst-case time complexity of SLLdel and DLLdel?

(A)  SLLdel is O(1) and DLLdel is O(n)

(B)  Both SLLdel and DLLdel are O(log(n))

(C)  Both SLLdel and DLLdel are O(1)

(D)  SLLdel is O(n) and DLLdel is O(1)

Answer: (D)

14. Consider the Deterministic Finite-state Automaton (DFA) 𝒜 shown below. The DFA runs on the alphabet {0, 1}, and has the set of states {s, p, q, r}, with s being the start state and p being the only final state.

Which one of the following regular expressions correctly describes the language accepted by A?

(A)  1(0∗11)∗

(B)  0(0 + 1)∗

(C)  1(0 + 11)∗

(D)  1(110∗)∗

Answer: (C)

15. The Lucas sequence Ln is defined by the recurrence relation:

Ln = Ln – 1 + Ln – 2, for n ≥ 3,

with L1 = 1 and L2 = 3.

Which one of the options given is TRUE?

Answer: (A)

16. Which one of the options given below refers to the degree (or arity) of a relation in relational database systems?

(A)  Number of attributes of its relation schema.

(B)  Number of tuples stored in the relation.

(C)  Number of entries in the relation.

(D)  Number of distinct domains of its relation schema.

Answer: (A)

17. Suppose two hosts are connected by a point-to-point link and they are configured to use Stop-and-Wait protocol for reliable data transfer. Identify in which one of the following scenarios, the utilization of the link is the lowest.

(A)  Longer link length and lower transmission rate

(B)  Longer link length and higher transmission rate

(C)  Shorter link length and lower transmission rate

(D)  Shorter link length and higher transmission rate

Answer: (B)

18. Let

Let det(A) and det(B) denote the determinants of the matrices A and B, respectively.

Which one of the options given below is TRUE?

(A)  det(A) = det(B)

(B)  det(B) = −det(A)

(C)  det(A) = 0

(D)  det(AB) = det(A)+det(B)

Answer: (B)

19. Consider the following definition of a lexical token id for an identifier in a programming language, using extended regular expressions:

Answer: (C)

20. An algorithm has to store several keys generated by an adversary in a hash table. The adversary is malicious who tries to maximize the number of collisions. Let k be the number of keys, m be the number of slots in the hash table, and k > m.

Which one of the following is the best hashing strategy to counteract the adversary?

(A)  Division method, i.e., use the hash function h(k) = k mod m.

(B)  Multiplication method, i.e., use the hash function  where A is a carefully chosen constant.

(C)  Universal hashing method.

(D)  If k is a prime number, use Division method. Otherwise, use Multiplication method.

Answer: (C)

21. The output of a 2-input multiplexer is connected back to one of its inputs as shown in the figure.

Match the functional equivalence of this circuit to one of the following options.

(A)  D Flip-flop

(B)  D Latch

(C)  Half-adder

(D)  Demultiplexer

Answer: (B)

22. Which one or more of the following need to be saved on a context switch from one thread (T1) of a process to another thread (T2) of the same process?

(A)  Page table base register

(B)  Stack pointer

(C)  Program counter

(D)  General purpose registers

Answer: (B, C, D)

23. Which one or more of the following options guarantee that a computer system will transition from user mode to kernel mode?

(A)  Function Call

(B)  malloc Call

(C)  Page Fault

(D)  System Call

Answer: (C, D)

24. Which of the following statements is/are CORRECT?

(A)  The intersection of two regular languages is regular.

(B)  The intersection of two context-free languages is context-free.

(C)  The intersection of two recursive languages is recursive.

(D)  The intersection of two recursively enumerable languages is recursively enumerable.

Answer: (A, C, D)

25. Which of the following statements is/are INCORRECT about the OSPF (Open Shortest Path First) routing protocol used in the Internet?

(A)  OSPF implements Bellman-Ford algorithm to find shortest paths.

(B)  OSPF uses Dijkstra’s shortest path algorithm to implement least-cost path routing.

(C)  OSPF is used as an inter-domain routing protocol.

(D)  OSPF implements hierarchical routing.

Answer: (A, C)

26. Geetha has a conjecture about integers, which is of the form

∀x(P(x) ⟹ ∃yQ(x, y),

where P is a statement about integers, and Q is a statement about pairs of integers. Which of the following (one or more) option(s) would imply Geetha’s conjecture?

(A)  ∃x(P(x) ˄ ∀yQ(x, y))

(B)  ∀x∀yQ(x, y)

(C)  ∃y∀x(P(x) ⟹ Q(x, y))

(D)  ∃x(P(x) ˄ ∃yQ(x, y))

Answer: (B, C)

27. Which one or more of the following CPU scheduling algorithms can potentially cause starvation?

(A)  First-in First-Out

(B)  Round Robin

(C)  Priority Scheduling

(D)  Shortest Job First

Answer: (A, C, D or C, D)

28. Let

f(x) = x3 + 15x2 – 3x – 36

be a real-valued function.

Which of the following statements is/are TRUE?

(A)  f(x) does not have a local maximum.

(B)  f(x) has a local maximum.

(C)  f(x) does not have a local minimum.

(D)  f(x) has a local minimum.

Answer: (B, D)

29. Let f and g be functions of natural numbers given by f(n) = n and g(n) = n2. Which of the following statements is/are TRUE?

(A)  f ∈ O(g)

(B)  f ∈ Ω(g)

(C)  f ∈ o(g)

(D)  f ∈ Θ(g)

Answer: (A, C)

30. Let A be the adjacency matrix of the graph with vertices {1, 2, 3, 4, 5}.

Let λ1, λ2, λ3, λ4, and λ5 be the five eigenvalues of A. Note that these eigenvalues need not be distinct.

The value of λ1 + λ2 + λ3 + λ4 + λ5 = _______.

Answer: (2 to 2 OR 4 to 4)

31. The value of the definite integral

is ___________. (Rounded off to the nearest integer)

Answer: (0 to 0)

32. A particular number is written as 132 in radix-4 representation. The same number in radix-5 representation is _________.

Answer: (110 to 110)

33. Consider a 3-stage pipelined processor having a delay of 10 ns (nanoseconds), 20 ns, and 14 ns, for the first, second, and the third stages, respectively. Assume that there is no other delay and the processor does not suffer from any pipeline hazards. Also assume that one instruction is fetched every cycle.

The total execution time for executing 100 instructions on this processor is ___________ ns.

Answer: (2040 to 2040)

34. A keyboard connected to a computer is used at a rate of 1 keystroke per second. The computer system polls the keyboard every 10 ms (milli seconds) to check for a keystroke and consumes 100 μs (micro seconds) for each poll. If it is determined after polling that a key has been pressed, the system consumes an additional 200 μs to process the keystroke. Let T1 denote the fraction of a second spent in polling and processing a keystroke.

In an alternative implementation, the system uses interrupts instead of polling. An interrupt is raised for every keystroke. It takes a total of 1 ms for servicing an interrupt and processing a keystroke. Let T2 denote the fraction of a second spent in servicing the interrupt and processing a keystroke.

The ratio T1/T2 is _________. (Rounded off to one decimal place)

Answer: (10.2 to 10.2)

35. The integer value printed by the ANSI-C program given below is ________.

Answer: (7 to 7)

Q.36 – Q.65 Carry TWO mark each.

36. Consider the following program:

Which one of the following options represents the activation tree corresponding to the main function?

Answer: (A)

37. Consider the control flow graph shown.

Which one of the following choices correctly lists the set of live variables at the exit point of each basic block?

(A)  B1: {}, B2: {a}, B3: {a}, B4: {a}

(B)  B1: {i, j}, B2: {a}, B3: {a}, B4: {i}

(C)  B1: {a, i, j}, B2: {a, i, j}, B3: {a, i}, B4: {a}

(D)  B1: {a, i, j}, B2: {a, j}, B3: {a, j}, B4: {a, i, j}

Answer: (D)

38. Consider the two functions incr and decr shown below.

There are 5 threads each invoking incr once, and 3 threads each invoking decr once, on the same shared variable X. The initial value of X is 10.

Suppose there are two implementations of the semaphore s, as follows:

I-1: s is a binary semaphore initialized to 1.

I-2: s is a counting semaphore initialized to 2.

Let V1, V2 be the values of X at the end of execution of all the threads with

implementations I-1, I-2, respectively.

Which one of the following choices corresponds to the minimum possible values of

V1, V2, respectively?

(A)  15, 7

(B)  7, 7

(C)  12, 7

(D)  12, 8

Answer: (C)

39. Consider the context-free grammar G below

S → aSb | X

X → aX | Xb |a| b,

where S and X are non-terminals, and a and b are terminal symbols. The starting non-terminal is S.

Which one of the following statements is CORRECT?

(A)  The language generated by G is (a + b)∗

(B)  The language generated by G is a∗(a + b)b∗

(C)  The language generated by G is a∗b∗(a + b)

(D)  The language generated by G is not a regular language

Answer: (B)

40. Consider the pushdown automaton (PDA) P below, which runs on the input alphabet {a, b}, has stack alphabet {⊥, A}, and has three states {s, p, q}, with s being the start state. A transition from state u to state v, labelled c/X/γ, where c is an input symbol, or ϵ, X is a stack symbol, and γ is a string of stack symbols, represents the fact that in state u, the PDA can read c from the input, with X on the top of its stack, pop X from the stack, push in the string γ on the stack, and go to state v. In the initial configuration, the stack has only the symbol ⊥ in it. The PDA accepts by empty stack.

Which one of the following options correctly describes the language accepted by P?

(A)  {ambn|1 ≤ m and n < m}

(B)  {ambn | 0 ≤ n ≤ m}

(C)  {ambn | 0 ≤ m and 0 ≤ n}

(D)  {am| 0 ≤ m}∪ {bn|0 ≤ n}

Answer: (A)

41. Consider the given C-code and its corresponding assembly code, with a few operands U1–U4 being unknown. Some useful information as well as the semantics of each unique assembly instruction is annotated as inline comments in the code. The memory is byte-addressable.

Which one of the following options is a CORRECT replacement for operands in the position (U1, U2, U3, U4) in the above assembly code?

(A)  (8, 4, 1, L02)

(B)  (3, 4, 4, L01)

(C)  (8, 1, 1, L02)

(D)  (3, 1, 1, L01)

Answer: (B)

42. A 4 kilobyte (KB) byte-addressable memory is realized using four 1 KB memory blocks. Two input address lines (IA4 and IA3) are connected to the chip select (CS) port of these memory blocks through a decoder as shown in the figure. The remaining ten input address lines from IA11–IA0 are connected to the address port of these blocks. The chip select (CS) is active high.

The input memory addresses (IA11–IA0), in decimal, for the starting locations (Addr = 0) of each block (indicated as X1, X2, X3, X4 in the figure) are among the options given below. Which one of the following options is CORRECT?

(A)  (0, 1, 2, 3)

(B)  (0, 1024, 2048, 3072)

(C)  (0, 8, 16, 24)

(D)  (0, 0, 0, 0)

Answer: (C)

43. Consider a sequential digital circuit consisting of T flip-flops and D flip-flops as shown in the figure. CLKIN is the clock input to the circuit. At the beginning, Q1, Q2 and Q3 have values 0, 1 and 1, respectively.

Which one of the given values of (Q1, Q2, Q3) can NEVER be obtained with this digital circuit?

(A)  (0, 0, 1)

(B)  (1, 0, 0)

(C)  (1, 0, 1)

(D)  (1, 1, 1)

Answer: (A)

44. A Boolean digital circuit is composed using two 4-input multiplexers (M1 and M2) and one 2-input multiplexer (M3) as shown in the figure. X0–X7 are the inputs of the multiplexers M1 and M2 and could be connected to either 0 or 1. The select lines of the multiplexers are connected to Boolean variables A, B and C as shown.

Which one of the following set of values of (X0, X1, X2, X3, X4, X5, X6, X7) will realise the Boolean function 

(A)  (1, 1, 0, 0, 1, 1, 1, 0)

(B)  (1, 1, 0, 0, 1, 1, 0, 1)

(C)  (1, 1, 0, 1, 1, 1, 0, 0)

(D)  (0, 0, 1, 1, 0, 1, 1, 1)

Answer: (C)

45. Consider the IEEE-754 single precision floating point numbers P=0xC1800000 and Q=0x3F5C2EF4.

Which one of the following corresponds to the product of these numbers (i.e., P × Q), represented in the IEEE-754 single precision format?

(A)  0x404C2EF4

(B)  0x405C2EF4

(C)  0xC15C2EF4

(D)  0xC14C2EF4

Answer: (C)

46. Let A be a priority queue for maintaining a set of elements. Suppose A is implemented using a max-heap data structure. The operation Extract-Max(A) extracts and deletes the maximum element from A. The operation Insert(A, key) inserts a new element key in A. The properties of a max-heap are preserved at the end of each of these operations.

When A contains n elements, which one of the following statements about the worst case running time of these two operations is TRUE?

(A)  Both Extract-Max(A) and Insert(A, key) run in O(1).

(B)  Both Extract-Max(A) and Insert(A, key) run in O(log(n)).

(C)  Extract-Max(A) runs in O(1) whereas Insert(A, key) runs in O(n).

(D)  Extract-Max(A) runs in O(1) whereas Insert(A, key) runs in O(log(n)).

Answer: (B)

47. Consider the C function foo and the binary tree shown.

When foo is called with a pointer to the root node of the given binary tree, what will it print?

(A)  3 8 5 13 11 10

(B)  3 5 8 10 11 13

(C)  3 8 16 13 24 50

(D)  3 16 8 50 24 13

Answer: (C)

48. Let U = {1, 2, . . . , n}, where n is a large positive integer greater than 1000. Let k be a positive integer less than n. Let A,B be subsets of U with |A| = |B| = k and A ∩ B = ∅. We say that a permutation of U separates A from B if one of the following is true.

– All members of A appear in the permutation before any of the members of B.

– All members of B appear in the permutation before any of the members of A.

How many permutations of U separate A from B?

Answer: (D)

49. Let f : A → B be an onto (or surjective) function, where A and B are nonempty sets. Define an equivalence relation ∼ on the set A as

a1 ∼ a2 if f(a1) = f(a2) ,

where a1, a2 ∈ A. Let E = {[x] : x ∈ A} be the set of all the equivalence classes under ∼. Define a new mapping F :E →B as

F([x]) = f(x), for all the equivalence classes [x] in E.

Which of the following statements is/are TRUE?

(A)  F is NOT well-defined.

(B)  F is an onto (or surjective) function.

(C)  F is a one-to-one (or injective) function.

(D)  F is a bijective function.

Answer: (B, C, D)

50. Suppose you are asked to design a new reliable byte-stream transport protocol like TCP. This protocol, named myTCP, runs over a 100 Mbps network with Round Trip Time of 150 milliseconds and the maximum segment lifetime of 2 minutes.

Which of the following is/are valid lengths of the Sequence Number field in the myTCP header?

(A)  30 bits

(B)  32 bits

(C)  34 bits

(D)  36 bits

Answer: (B, C, D)

51. Let X be a set and 2X denote the powerset of X.

Define a binary operation Δ on 2X as follows:

A∆B = (A – B) ∪ (B – A).

Let H = (2X, Δ). Which of the following statements about H is/are correct?

(A)  H is a group.

(B)  Every element in H has an inverse, but H is NOT a group.

(C)  For every A ∈ 2X, the inverse of A is the complement of A.

(D)  For every A ∈ 2X, the inverse of A is A.

Answer: (A, D)

52. Suppose in a web browser, you click on the gate-2023.in URL. The browser cache is empty. The IP address for this URL is not cached in your local host, so a DNS lookup is triggered (by the local DNS server deployed on your local host) over the 3-tier DNS hierarchy in an iterative mode. No resource records are cached anywhere across all DNS servers.

Let RTT denote the round trip time between your local host and DNS servers

in the DNS hierarchy. The round trip time between the local host and the web server hosting www.gate-2023.in is also equal to RTT. The HTML file associated with the URL is small enough to have negligible transmission time and negligible rendering time by your web browser, which references 10 equally small objects on the same web server.

Which of the following statements is/are CORRECT about the minimum elapsed time between clicking on the URL and your browser fully rendering it?

(A)  7 RTTs, in case of non-persistent HTTP with 5 parallel TCP connections.

(B)  5 RTTs, in case of persistent HTTP with pipelining.

(C)  9 RTTs, in case of non-persistent HTTP with 5 parallel TCP connections.

(D)  6 RTTs, in case of persistent HTTP with pipelining.

Answer: (C, D)

53. Consider a random experiment where two fair coins are tossed. Let A be the event that denotes HEAD on both the throws, B be the event that denotes HEAD on the first throw, and C be the event that denotes HEAD on the second throw. Which of the following statements is/are TRUE?

(A)  A and B are independent.

(B)  A and C are independent.

(C)  B and C are independent.

(D)  Prob(B|C) = Prob(B)

Answer: (C, D)

54. Consider functions Function 1 and Function 2 expressed in pseudocode as follows:

Let f1(n) and f2(n) denote the number of times the statement “x = x + 1” is executed in Function 1 and Function 2, respectively.

Which of the following statements is/are TRUE?

(A)  f1(n) ∈ Θ(f2(n))

(B)  f1(n) ∈ o(f2(n))

(C)  f1(n) ∈ ω(f2(n))

(D)  f1(n) ∈ O(n)

Answer: (A, D)

55. Let G be a simple, finite, undirected graph with vertex set {v1, . . . , vn}. Let Δ(G) denote the maximum degree of G and let ℕ = {1, 2, . . .} denote the set of all possible colors. Color the vertices of G using the following greedy strategy:

for i = 1, . . . , n

color(vi) ← min{j ∈ ℕ : no neighbour of vi is colored j}

Which of the following statements is/are TRUE?

(A)  This procedure results in a proper vertex coloring of G.

(B)  The number of colors used is at most Δ(G) + 1.

(C)  The number of colors used is at most Δ(G).

(D)  The number of colors used is equal to the chromatic number of G.

Answer: (A, B)

56. Let U = {1, 2, 3}. Let 2U denote the powerset of U. Consider an undirected graph G whose vertex set is 2U. For any A,B ∈ 2U, (A,B) is an edge in G if and only if (i) A = B, and (ii) either A B or B A. For any vertex A in G, the set of all possible orderings in which the vertices of G can be visited in a Breadth First Search (BFS) starting from A is denoted by B(A).

If ∅ denotes the empty set, then the cardinality of B(∅) is ________.

Answer: (5040 to 5040)

57. Consider the following two-dimensional array D in the C programming language, which is stored in row-major order:

int D[128][128];

Demand paging is used for allocating memory and each physical page frame holds 512 elements of the array D. The Least Recently Used (LRU) page-replacement policy is used by the operating system. A total of 30 physical page frames are allocated to a process which executes the following code snippet:

The number of page faults generated during the execution of this code snippet is ________.

Answer: (4096 to 4096)

58. Consider a computer system with 57-bit virtual addressing using multi-level tree-structured page tables with L levels for virtual to physical address translation. The page size is 4 KB (1 KB = 1024 B) and a page table entry at any of the levels occupies 8 bytes.

The value of L is _______.

Answer: (5 to 5)

59. Consider a sequence a of elements a0 = 1, a1 = 5, a2 = 7, a3 = 8, a4 = 9, and a5 = 2. The following operations are performed on a stack S and a queue Q, both of which are initially empty.

I: push the elements of a from a0 to a5 in that order into S.

II: enqueue the elements of a from a0 to a5 in that order into Q.

III: pop an element from S.

IV: dequeue an element from Q.

V: pop an element from S.

VI: dequeue an element from Q.

VII: dequeue an element from Q and push the same element into S.

VIII: Repeat operation VII three times.

IX: pop an element from S.

X: pop an element from S.

The top element of S after executing the above operations is ________.

Answer: (8 to 8)

60. Consider the syntax directed translation given by the following grammar and semantic rules. Here N, I, F and B are non-terminals. N is the starting non-terminal, and #, 0 and 1 are lexical tokens corresponding to input letters “#”, “0” and “1”, respectively. X.val denotes the synthesized attribute (a numeric value) associated with a non-terminal X. I1 and F1 denote occurrences of I and F on the right hand side of a production, respectively. For the tokens 0 and 1, 0.val = 0 and 1.val = 1.

The value computed by the translation scheme for the input string

10 # 011

is __________. (Rounded off to three decimal places)

Answer: (2.374 to 2.376)

61. Consider the following table named Student in a relational database. The primary key of this table is rollNum.

The SQL query below is executed on this database.

The number of rows returned by the query is ______.

Answer: (2 to 2)

62. Consider a database of fixed-length records, stored as an ordered file. The database has 25,000 records, with each record being 100 bytes, of which the primary key occupies 15 bytes. The data file is block-aligned in that each data record is fully contained within a block. The database is indexed by a primary index file, which is also stored as a block-aligned ordered file. The figure below depicts this indexing scheme.

Suppose the block size of the file system is 1024 bytes, and a pointer to a block occupies 5 bytes. The system uses binary search on the index file to search for a record with a given key. You may assume that a binary search on an index file of b blocks takes  block accesses in the worst case.

Given a key, the number of block accesses required to identify the block in the data file that may contain a record with the key, in the worst case, is _______.

Answer: (6 to 6)

63. Consider the language L over the alphabet {0, 1}, given below:

L = {w ∈ {0, 1}∗ | w does not contain three or more consecutive 1’s}.

The minimum number of states in a Deterministic Finite-State Automaton (DFA) for L is _______.

Answer: (4 to 4)

64. An 8-way set associative cache of size 64 KB (1 KB = 1024 bytes) is used in a system with 32-bit address. The address is sub-divided into TAG, INDEX, and BLOCK OFFSET.

The number of bits in the TAG is ______.

Answer: (19 to 19)

65. The forwarding table of a router is shown below.

A packet addressed to a destination address 200.150.68.118 arrives at the router. It will be forwarded to the interface with ID _______.

Answer: (3 to 3)

GATE Exam 2023 Chemical Engineering (CH) Question Paper With Answer Key

GATE-2023

CH: Chemical Engineering

GA-General Aptitude

Q.1 – Q.5 Carry ONE mark each.

1. “You are delaying the completion of the task. Send _______ contributions at the earliest.”

(A)  you are

(B)  your

(C)  you’re

(D)  yore

Answer: (B)

2. References : ______ : : Guidelines : Implement

(By word meaning)

(A)  Sight

(B)  Site

(C)  Cite

(D)  Plagiarise      

Answer: (C)

3. In the given figure, PQRS is a parallelogram with PS = 7 cm, PT = 4 cm and PV = 5 cm. What is the length of RS in cm? (The diagram is representative.)

(A)  20/7

(B)  28/5

(C)  9/2

(D)  35/4

Answer: (B)

4. In 2022, June Huh was awarded the Fields medal, which is the highest prize in Mathematics.

When he was younger, he was also a poet. He did not win any medals in the International Mathematics Olympiads. He dropped out of college.

Based only on the above information, which one of the following statements can be logically inferred with certainty?

(A)  Every Fields medalist has won a medal in an International Mathematics Olympiad.

(B)  Everyone who has dropped out of college has won the Fields medal.

(C)  All Fields medalists are part-time poets.

(D)  Some Fields medalists have dropped out of college.

Answer: (D)

5. A line of symmetry is defined as a line that divides a figure into two parts in a way such that each part is a mirror image of the other part about that line.

The given figure consists of 16 unit squares arranged as shown. In addition to the three black squares, what is the minimum number of squares that must be coloured black, such that both PQ and MN form lines of symmetry? (The figure is representative)

(A)  3

(B)  4

(C)  5

(D)  6

Answer: (C)

Q.6 – Q.10 Carry TWO marks Each

6. Human beings are one among many creatures that inhabit an imagined world. In this imagined world, some creatures are cruel. If in this imagined world, it is given that the statement “Some human beings are not cruel creatures” is FALSE, then which of the following set of statement(s) can be logically inferred with certainty?

(i) All human beings are cruel creatures.

(ii) Some human beings are cruel creatures.

(iii) Some creatures that are cruel are human beings.

(iv) No human beings are cruel creatures.

(A)  only (i)

(B)  only (iii) and (iv)

(C)  only (i) and (ii)

(D)  (i), (ii) and (iii)

Answer: (D)

7. To construct a wall, sand and cement are mixed in the ratio of 3:1. The cost of sand and that of cement are in the ratio of 1:2.

If the total cost of sand and cement to construct the wall is 1000 rupees, then what is the cost (in rupees) of cement used?

(A)  400

(B)  600

(C)  800

(D)  200

Answer: (A)

8. The World Bank has declared that it does not plan to offer new financing to Sri Lanka, which is battling its worst economic crisis in decades, until the country has an adequate macroeconomic policy framework in place. In a statement, the World Bank said Sri Lanka needed to adopt structural reforms that focus on economic stabilisation and tackle the root causes of its crisis. The latter has starved it of foreign exchange and led to shortages of food, fuel, and medicines. The bank is repurposing resources under existing loans to help alleviate shortages of essential items such as medicine, cooking gas, fertiliser, meals for children, and cash for vulnerable households.

Based only on the above passage, which one of the following statements can be inferred with certainty?

(A)  According to the World Bank, the root cause of Sri Lanka’s economic crisis is that it does not have enough foreign exchange.

(B)  The World Bank has stated that it will advise the Sri Lankan government about how to tackle the root causes of its economic crisis.

(C)  According to the World Bank, Sri Lanka does not yet have an adequate macroeconomic policy framework.

(D)  The World Bank has stated that it will provide Sri Lanka with additional funds for essentials such as food, fuel, and medicines.

Answer: (C)

9. The coefficient of x4 in the polynomial (x − 1)3 (x − 2)3 is equal to _______.

(A)  33

(B)  −3

(C)  30

(D)  21

Answer: (A)

10. Which one of the following shapes can be used to tile (completely cover by repeating) a flat plane, extending to infinity in all directions, without leaving any empty spaces in between them? The copies of the shape used to tile are identical and are not allowed to overlap.

(A)  circle

(B)  regular octagon

(C)  regular pentagon

(D)  rhombus

Answer: (D)

CH: Chemical Engineering

Q.11 – Q.35 Carry ONE mark Each

11. Which one of the following is the CORRECT value of y, as defined by the expression given below?

(A)  1

(B)  2

(C)  0

(D)  ∞

Answer: (B)

12. The vector  is defined as

Which one of the following is the CORRECT value of divergence of  evaluated at the point (x, y, z) = (3, 2, 1)?

(A)  0

(B)  3

(C)  14

(D)  13

Answer: (D)

13. Given that

where z1 = 2 + 3i and z2 = −2 + 3i with i = √−1, which one of the following options is CORRECT?

(A)  F < 0

(B)  F < 1

(C)  F > 1

(D)  F = 1

Answer: (B)

14. For a two-dimensional plane, the unit vectors,  of the polar coordinate system and  of the Cartesian coordinate system, are related by the following two equations.

Which one of the following is the CORRECT value of 

(A)  1

(B) 

(C) 

(D) 

Answer: (D)

15. Which one of the following statements related to octane number is NOT correct?

(A)  Linear alkanes with higher carbon number have higher octane number.

(B)  Branching in linear alkanes increases their octane number.

(C)  Catalytic reforming of hydrocarbons increases their octane number.

(D)  Gasoline quality is measured in terms of octane number.

Answer: (A)

16. Which one of the following options represents the major components of oleum?

(A)  Sulfuric acid and nitric acid

(B)  Concentrated sulfuric acid and petroleum jelly

(C)  Sulfuric acid and hydrochloric acid

(D)  Sulfuric acid and sulfur trioxide

Answer: (D)

17. For a reversible endothermic chemical reaction with constant heat of reaction over the operating temperature range, K is the thermodynamic equilibrium constant. Which one of the following figures shows the CORRECT dependence of K on temperature T?

Answer: (A)

18. Nitrile rubber is manufactured via polymerization process. Which one of the following options is the CORRECT pair of monomers used in this process?

(A)  Acrylonitrile and styrene

(B)  Acrylonitrile and butadiene

(C)  Butadiene and styrene

(D)  Butadiene and isoprene

Answer: (B)

19. John and Jane independently performed a thermodynamic experiment, in which X and Y represent the initial and final thermodynamic states of the system, respectively. John performed the experiment under reversible conditions, for which the change in entropy of the system was ∆Srev. Jane performed the experiment under irreversible conditions, for which the change in entropy of the system was ∆Sirr. Which one of the following relationships is CORRECT?

(A)  ∆Srev = ∆Sirr

(B)  ∆Srev > ∆Sirr

(C)  ∆Srev < ∆Sirr

(D)  ∆Srev = 2∆Sirr

Answer: (A)

20. For a packed-bed comprising of uniform-sized spherical particles of diameter Dp, the pressure drop across the bed is given by the Kozeny-Carman equation when the particle Reynolds number (Rep) < 1. Under this condition, minimum fluidization velocity is proportional to Dpn. Which one of the following is the CORRECT value of exponent n?

(A)  2

(B)  −1

(C)  −2

(D)  1

Answer: (A)

21. Match the quantities in Group 1 with their units in Group 2 listed in the table below.

(A)  P-II, Q-I, R-IV, S-III

(B)  P-I, Q-II, R-III, S-IV

(C)  P-III, Q-IV, R-II, S-I

(D)  P-IV, Q-I, R-III, S-II

Answer: (A)

22. A slab of thickness L, as shown in the figure below, has cross-sectional area A and constant thermal conductivity k. T1 and T2 are the temperatures at x = 0 and x = L, respectively. Which one of the following options is the CORRECT expression of the thermal resistance for steady-state one-dimensional heat conduction?

(A)  L/kA

(B)  k/LA

(C) 

(D)  A/Lk

Answer: (A)

23. Spray dryers have many advantages. Which one of the following is NOT an advantage of a typical spray dryer?

(A)  Has short drying time

(B)  Produces hollow spherical particles

(C)  Has high heat efficiency

(D)  Is suitable for heat sensitive materials

Answer: (C)

24. Which one of the following quantities of a flowing fluid is measured using a rotameter?

(A)  Static pressure

(B)  Dynamic pressure

(C)  Volumetric flow rate

(D)  Viscosity

Answer: (C)

25. A liquid surge tank has Fin and Fout as the inlet and outlet flow rates respectively, as shown in the figure below. Fout is proportional to the square root of the liquid level ℎ. The cross-sectional area of the tank is 20 cm2. Density of the liquid is constant everywhere in the system. At steady state, Fin = Fout =10 cm3s−1 and h = 16 cm. The variation of h with Fin is approximated as a first order transfer function. Which one of the following is the CORRECT value of the time constant (in seconds) of this system?

(A)  20

(B)  32

(C)  64

(D)  128

Answer: (C)

26. A packed distillation column, with vapor having an average molecular weight of 45 kg.kmol−1, density of 2 kg.m−3 and a molar flow rate of 0.1 kmol.s−1, has a flooding velocity of 0.15 m.s−1. The column is designed to operate at 60 % of the flooding velocity. Which one of the following is the CORRECT value for the column diameter (in m)?

(A)  5/√π

(B)  5√π

(C)  4π

(D)  10/√π

Answer: (D)

27. An isothermal jacketed continous stirred tank reactor (CSTR) operating at 150°C is shown in the figure below. The cold feed entering the system at 30 oC is preheated to a temperature T (T < 150°C) using a heat exchanger HX1. This preheated feed is further heated to 150°C using the utility heater HX2. The mass flow rate and heat capacity are same for all the process streams, and the overall heat transfer coefficient is independent of temperature. Which one of the following statements is the CORRECT action to take if it is desired to increase the value of T?

(A)  Increase both heat transfer area of HX1 and heat duty of HX2.

(B)  Decrease both heat transfer area of HX1 and heat duty of HX2.

(C)  Increase the heat transfer area of HX1 and decrease the heat duty of HX2.

(D)  Decrease the heat transfer area of HX1 and increase the heat duty of HX2.

Answer: (C)

28. Consider a system where a Carnot engine is operating between a source and a sink. Which of the following statements about this system is/are NOT correct?

(A)  This engine is reversible.

(B)  The engine efficiency is independent of the source and sink temperatures.

(C)  This engine has the highest efficiency among all engines that operate between the same source and sink.

(D)  The total entropy of this system increases at the completion of each cycle of the engine.

Answer: (B, D)

29. For a fully developed turbulent flow of an incompressible Newtonian fluid through a pipe of constant diameter, which of the following statements is/are CORRECT?

(A)  Reynolds stress, averaged over a sufficiently long time, is zero everywhere inside the pipe.

(B)  Reynolds stress at the pipe wall is zero.

(C)  Average velocity of the fluid is half of its center-line velocity.

(D)  Average pressure gradient in the flow direction is constant.

Answer: (B, D)

30. Given that E (in W.m−2) is the total hemispherical emissive power of a surface maintained at a certain temperature, which of the following statements is/are CORRECT?

(A)  E does not depend on the direction of the emission.

(B)  E depends on the viewfactor.

(C)  E depends on the wavelength of the emission.

(D)  E does not depend on the frequency of the emission.

Answer: (-1 to -1)

31. The position x(t) of a particle, at constant ω, is described by the equation

The initial conditions are x(t = 0) = 1 and  The position of the particle at t = (3π/ω) is __________ (in integer).

Answer: (199.5 to 200.5)

32. Burning of methane in a combustor yields carbon monoxide, carbon dioxide, and water vapor. Methane is fed to the combustor at 100 mol.hr−1, of which 50 % reacts. The theoretical oxygen requirement (in mol.hr−1) is ________ (rounded off to one decimal place).

Answer: (1.01 to 1.03)

33. The viscosity of an incompressible Newtonian fluid is measured using a capillary tube of diameter 0.5 mm and length 1.5 m. The fluid flow is laminar, steady and fully developed. For a flow rate of 1 cm3s−1, the pressure drop across the length of the tube is 1 MPa. If the viscosity of the fluid is k × 103s, the value of k is ___________ (rounded off to two decimal places).

Answer: (3 to 3)

34. A liquid L containing a dissolved gas S is stripped in a countercurrent operation using a pure carrier gas V. The liquid phase inlet and outlet mole fractions of S are 0.1 and 0.01, respectively. The equilibrium distribution of S between V and L is governed by ye = xe, where ye and xe are the mole fractions of S in V and L, respectively. The molar feed rate of the carrier gas stream is twice as that of the liquid stream. Under dilute solution conditions, the minimum number of ideal stages required is __________ (in integer).

Answer: (1.17 to 1.19)

35. In a binary gas-liquid system, NA,EMD is the molar flux of a gas A for equimolar counter diffusion with a liquid B. NA,UMD is the molar flux of A for steady one-component diffusion through stagnant B. Using the mole fraction of A in the bulk of the gas phase as 0.2 and that at the gas-liquid interface as 0.1 for both the modes of diffusion, the ratio of NA,UMD to NA,EMD is equal to ______ (rounded off to two decimal places).

Answer: (A)

Q.36 – Q.65 Carry TWO marks Each

36. An exhibition was held in a hall on 15 August 2022 between 3 PM and 4 PM during which any person was allowed to enter only once. Visitors who entered before 3:40 PM exited the hall exactly after 20 minutes from their time of entry. Visitors who entered at or after 3:40 PM, exited exactly at 4 PM. The probability distribution of the arrival time of any visitor is uniform between 3 PM and 4 PM. Two persons X and Y entered the exhibition hall independent of each other. Which one of the following values is the probability that their visits to the exhibition overlapped with each other?

(A)  5/9

(B)  4/9

(C)  2/9

(D)  7/9

Answer: (B)

37. Simpson’s one-third rule is used to estimate the definite integral

with an interval length of 0.5. Which one of the following is the CORRECT estimate of I obtained using this rule?

Answer: (B)

38. Match the products in Group 1 with the manufacturing processes in Group 2 listed in the table below.

(A)  P-III, Q-IV, R-I, S-II

(B)  P-III, Q-I, R-IV, S-II

(C)  P-IV, Q-I, R-II, S-III

(D)  P-I, Q-IV, R-II, S-III

Answer: (A)

39. Match the reactions in Group 1 with the catalysts in Group 2 listed in the table below.

(A)  P-III, Q-IV, R-II, S-I

(B)  P-III, Q-IV, R-I, S-II

(C)  P-IV, Q-II, R-I, S-III

(D)  P-IV, Q-III, R-I, S-II

Answer: (B)

40. Water in a container at 290 K is exposed to air containing 3 % CO2 by volume. Air behaves like an ideal gas and is maintained at 100 kPa pressure. The liquid phase comprising of dissolved CO2 in water behaves like an ideal solution. Use Henry’s constant of CO2 dissolved in water at 290 K as 12 MPa. Under equilibrium conditions, which one of the following is the CORRECT value of the mole fraction of CO2 dissolved in water?

(A)  2.9 × 104

(B)  0.9 × 104

(C)  2.5 × 104

(D)  0.5 × 104

Answer: (C)

41. The enthalpy (H, in J.mol−1) of a binary liquid system at constant temperature and pressure is given as

H = 40x1 + 60x2 + x1x2(4x1 + 2x2),

where x1 and x2 represent the mole fractions of species 1 and 2 in the liquid, respectively. Which one of the following is the CORRECT value of the partial molar enthalpy of species 1 at infinite dilution, 

(A)  100

(B)  42

(C)  64

(D)  40

Answer: (B)

42. Which one of the following represents the CORRECT effects of concentration polarization in a reverse osmosis process?

(A)  Reduced water flux and reduced solute rejection

(B)  Increased water flux and increased solute rejection

(C)  Reduced water flux and increased solute rejection

(D)  Increased water flux and reduced solute rejection

Answer: (A)

43. CO and H2 participate in a catalytic reaction. The partial pressures (in atm) of the reacting species CO and H2 in the feed stream are pCO and pH2, respectively. While CO undergoes molecular adsorption, H2 adsorbs via dissociative adsorption, that is, as hydrogen atoms. The equilibrium constants (in atm−1) corresponding to adsorption of CO and H2 to the catalyst sites are KCO and KH2, respectively. Total molar concentration of active sites per unit mass of the catalyst is Ct (in mol.(g cat)−1). Both the adsorption steps are at equilibrium. Which one of the following expressions is the CORRECT ratio of the concentration of catalyst sites occupied by CO to that by hydrogen atoms?

Answer: (A)

44. A cascade control strategy is shown in the figure below. The transfer function between the output (y) and the secondary disturbance (d2) is defined as 

Which one of the following is the CORRECT expression for the transfer function Gd2(s)?

Answer: (A)

45. Level (h) in a steam boiler is controlled by manipulating the flow rate (F) of the make-up (fresh) water using a proportional (P) controller. The transfer function between the output and the manipulated input is

The measurement and valve transfer functions are both equal to 1. A process engineer wants to tune the controller so that the closed-loop response gives decaying oscillations under servo mode. Which one of the following is the CORRECT value of the controller gain to be used by the engineer?

(A)  0.25

(B)  2

(C)  4

(D)  6

Answer: (B)

46. Which of the following statements is/are CORRECT?

(A)  Bond number includes surface tension.

(B)  Jakob number includes latent heat.

(C)  Prandtl number includes liquid-vapor density difference.

(D)  Biot number includes gravity.

Answer: (A, B)

47. If a matrix M is defined as  the sum of all the eigenvalues of M3 is equal to ________ (in integer).

Answer: (4160 to 4160)

48. The first derivative of the function  evaluated at r = 1 is ________ (in integer).

Answer: (-24 to -24)

49. Wet air containing 10 mole percent water vapor is dried by continuously passing it through a column of CaCl2 The pellets remove 50 percent of water from wet air entering the column. The mole percent of water vapor in the product stream exiting the column is ___________ (rounded off to two decimal places).

Answer: (5.20 to 5.30)

50. Orsat analysis showing the composition (in mol %, on a dry basis) of a stack gas is given in the table below. The humidity measurement reveals that the mole fraction of H2O in the stack gas is 0.07. The mole fraction of N2 calculated on a wet basis is ________ (rounded off to two decimal places).

Answer: (0.59 to 0.62)

51. A pump draws water (density =1000 kg.m−3) at a steady rate of 10 kg.s−1. The pressures at the suction and discharge sides of the pump are −20 kPa (gauge) and 350 kPa (gauge), respectively. The pipe diameters at the suction and discharge side are 70 mm and 50 mm, respectively. The suction and discharge lines are at the same elevation, and the pump operates at an efficiency of 80 %. Neglecting frictional losses in the system, the power (in kW) required to drive the pump is ______ (rounded off to two decimal places).

Answer: (4.70 to 4.78)

52. A cylindrical tank with a diameter of 500 mm contains water (density =1 g.cm−3) upto a height ℎ. A 5 mm diameter round nozzle, whose center is 1 cm above the base of the tank, has its exit open to the atmosphere as shown in the schematic below. The pressure above the water level in the tank is maintained at 2 bar (absolute). Neglect all frictional and entry/exit losses. Use acceleration due to gravity as 10 m.s−2 and atmospheric pressure as 1 bar. The absolute value of initial  when h = 51 cm is equal to _______ (round off to two decimal places).

Answer: (1.40 to 1.50)

53. A large tank is filled with water (density =1 g.cm−3) upto a height of 5 m. A 100 μm diameter solid spherical particle (density =0.8 g.cm−3) is released at the bottom of the tank. The particle attains its terminal velocity (vt) after traveling to a certain height in the tank. Use acceleration due to gravity as 10 m.s−2 and water viscosity as 10−3s . Neglect wall effects on the particle. If Stokes law is applicable, the absolute value of vt (in mm.s−1) is ____________ (rounded off to two decimal places).

Answer: (1.00 to 1.20)

54. A fluid is flowing steadily under laminar conditions over a thin rectangular plate at temperature Ts as shown in the figure below. The velocity and temperature of the free stream are u and T, respectively. When the fluid flow is only in the x- direction, hx is the local heat transfer coefficient. Similarly, when the fluid flow is only in the y-direction, hy is the corresponding local heat transfer coefficient. Use the correlation Nu = 0.332 (Re)1/2 (Pr)1/3 for the local heat transfer coefficient, where, Nu, Re, and Pr, respectively are the appropriate Nusselt, Reynolds and Prandtl numbers. The average heat transfer coefficients are defined as  and  If w = 1 m and l = 4 m, the value of the ratio of   is ________ (in integer).

Answer: (2 to 2)

55. A perfectly insulated, concentric tube countercurrent heat exchanger is used to cool lubricating oil using water as a coolant (see figure below). Oil enters the outer annulus at a mass flow rate of 2 kg.s−1 with a temperature of 100°C and leaves at 40° Water enters the inner tube at a mass flow rate of 1 kg.s−1 with a temperature of 20°C and leaves at 80°C. Use specific heats of oil and water as 2089 J.kg−1K−1 and 4178 J.kg−1K−1, respectively. There is no phase change in both the streams. Under steady-state conditions, the number of transfer units (NTU) is _______ (in integer).

Answer: (3 to 3)

56. Partially saturated air at 1 bar and 50°C is contacted with water in an adiabatic saturator. The air is cooled and humidified to saturation, and exits at 25°C with an absolute humidity of 0.02 kg water per kg dry air. Use latent heat of vaporization of water as 2450 kJ.kg−1, and average specific heat capacity for dry air and water, respectively as 1.01 kJ.kg−1K−1 and 4.18 kJ.kg−1K−1. If the absolute humidity of air entering the adiabatic saturator is H × 103 kg water per kg dry air, the value of H is __________ (rounded off to two decimal places).

Answer: (9.10 to 9.50)

57. Distillation of a non-reactive binary mixture with components A and B is carried out in a batch still as shown in the figure below. The initial charge of the mixture in the still is 1 kmol. The initial and final amounts of 𝐴 in the still are 0.1 kmol and 0.01 kmol, respectively. Use a constant relative volatility of 4.5. The mole fraction of 𝐵 remaining in the vessel is _______ (rounded off to three decimal places).

Answer: (0.002 to 0.004 OR 0.981 to 0.983)

58. Fresh catalyst is loaded into a reactor before the start of the following catalytic reaction.

A products

The catalyst gets deactivated over time. The instantaneous activity a(t), at time t, is defined as the ratio of the rate of reaction −rꞌA(t) (mol.(g cat)1hr1) to the rate of reaction with fresh catalyst. Controlled experimental measurements led to an empirical correlation

−rꞌA(t) = −0.5t + 10

where t is in hours. The activity of the catalyst at t = 10 hr is _______ (rounded off to one decimal place).

Answer: (0.5 to 0.5)

59. A unimolecular, irreversible liquid-phase reaction

A P

was carried out in an ideal batch reactor at temperature T. The rate of the reaction (−rA) measured at different conversions XA is given in the table below. This reaction is also carried out in an ideal continuous stirred tank reactor (CSTR) at the same temperature T with a feed concentration of 1 mol.m−3, under steady-state conditions. For a conversion of 0.8, the space time (in s) of the CSTR is __________ (in integer).

Answer: (16 to 16)

60. An irreversible liquid-phase second-order reaction  with rate constant k = 0.2 liter.mol−1min−1, is carried out in an isothermal non-ideal reactor. A tracer experiment conducted on this reactor resulted in a residence time distribution (𝐸-curve) as shown in the figure below. The areas of the rectangles (i), (ii), and (iii) are equal. Pure A at a concentration of 1.5 mol.liter−1 is fed to the reactor. The segregated model mimics the nonideality of this reactor. The percentage conversion of A at the exit of the reactor is ______ (rounded off to the nearest integer).

Answer: (71 to 73)

61. The outlet concentration CA of a plug flow reactor (PFR) is controlled by manipulating the inlet concentration CA0. The following transfer function describes the dynamics of this PFR.

In the above equation, V = 1 m3, F = 0.1 m3min1 and k = 0.5 min1. The measurement and valve transfer functions are both equal to 1. The ultimate gain, defined as the proportional controller gain that produces sustained oscillations, for this system is _______ (rounded off to one decimal place).

Answer: (148.0 to 148.8)

62. The transfer function of a measuring instrument is 

At time t = 0, a step change of +1 unit is introduced in the input of this instrument. The time taken by the instrument to show an increase of 1 unit in its output is _____ (rounded off to two decimal places).

Answer: (6.99 to 7.19)

63. A design engineer needs to purchase a membrane module (M) for a plant. Details about the two available options, M1 and M2, are given in the table below. The overall plant has an expected life of 7 years. If the interest rate is 8 % per annum, compounded annually, the difference in the net present value (NPV) of these two options, in lakhs of rupees, is ______ (rounded off to one decimal place).

Answer: (-4.8 to -4.6 OR 4.6 to 4.8)

64. The purchase cost of a new distillation column is Rs.10 lakhs with an installation factor of 5.8. The cost of the capital is to be annualized over a period of 6 years at a fixed rate of interest of 5 % per annum, compounded annually. The annual cost (in lakhs of rupees) of the installed capital is _______ (rounded off to one decimal place).

Answer: (11.0 to 11.8)

65. Pumps A and B are being considered for purchase in a chemical plant. Cost details for these two pumps are given in the table below. The interest rate is 10 % per annum, compounded annually. For both the pumps to have the same capitalized cost, the salvage value (in Rs.) of pump B should be ______ (rounded off to the nearest integer).

Answer: (2080 to 2300)

GATE Exam 2023 Civil Engineering (CE-2) Question Paper With Answer Key

GATE-2023

CE2: Civil Engineering

GA-General Aptitude

Q.1 – Q.5 Carry ONE mark each.

1. The line ran _______ the page, right through the centre, and divided the page into two.

(A)  across

(B)  of

(C)  between

(D)  about

Answer: (A)

2. Kind : ________ : : Often : Seldom

(By word meaning)

(A)  Cruel

(B)  Variety

(C)  Type           

(D)  Kindred

Answer: (A)

3. In how many ways can cells in a 3 × 3 grid be shaded, such that each row and each column have exactly one shaded cell? An example of one valid shading is shown.

(A)  2

(B)  9

(C)  3

(D)  6

Answer: (D)

4. There are 4 red, 5 green, and 6 blue balls inside a box. If N number of balls are picked simultaneously, what is the smallest value of N that guarantees there will be at least two balls of the same colour?

One cannot see the colour of the balls until they are picked.

(A)  4

(B)  15

(C)  5

(D)  2

Answer: (A)

5. Consider a circle with its centre at the origin (O), as shown. Two operations are allowed on the circle.

Operation 1: Scale independently along the x and y axes.

Operation 2: Rotation in any direction about the origin.

Which figure among the options can be achieved through a combination of these two operations on the given circle?

Answer: (A)

Q.6 – Q.10 Carry TWO marks Each

6. Elvesland is a country that has peculiar beliefs and practices. They express almost all their emotions by gifting flowers. For instance, if anyone gifts a white flower to someone, then it is always taken to be a declaration of one’s love for that person. In a similar manner, the gifting of a yellow flower to someone often means that one is angry with that person.

Based only on the information provided above, which one of the following sets of statement(s) can be logically inferred with certainty?

(i) In Elvesland, one always declares one’s love by gifting a white flower.

(ii) In Elvesland, all emotions are declared by gifting flowers.

(iii) In Elvesland, sometimes one expresses one’s anger by gifting a flower that is not yellow.

(iv) In Elvesland, sometimes one expresses one’s love by gifting a white flower.

(A)  only (ii)

(B)  (i), (ii) and (iii)

(C)  (i), (iii) and (iv)

(D)  only (iv)

Answer: (D)

7. Three husband-wife pairs are to be seated at a circular table that has six identical chairs. Seating arrangements are defined only by the relative position of the people. How many seating arrangements are possible such that every husband sits next to his wife?

(A)  16

(B)  4

(C)  120

(D)  720

Answer: (A)

8. Based only on the following passage, which one of the options can be inferred with certainty?

When the congregation sang together, Apenyo would also join, though her little screams were not quite audible because of the group singing. But whenever there was a special number, trouble would begin; Apenyo would try singing along, much to the embarrassment of her mother. After two or three such mortifying Sunday evenings, the mother stopped going to church altogether until Apenyo became older and learnt to behave.

At home too, Apenyo never kept quiet; she hummed or made up silly songs to sing by herself, which annoyed her mother at times but most often made her become pensive. She was by now convinced that her daughter had inherited her love of singing from her father who had died unexpectedly away from home.

[Excerpt from These Hills Called Home by Temsula Ao]

(A)  The mother was embarrassed about her daughter’s singing at home.

(B)  The mother’s feelings about her daughter’s singing at home were only of annoyance.

(C)  The mother was not sure if Apenyo had inherited her love of singing from her father.

(D)  When Apenyo hummed at home, her mother tended to become thoughtful.

Answer: (D)

9. If x satisfies the equation  then x is equal to _____.

(A)  1/2

(B)  log16 8

(C)  2/3

(D)  log4 8

Answer: (C)

10. Consider a spherical globe rotating about an axis passing through its poles. There are three points P, Q, and R situated respectively on the equator, the north pole, and midway between the equator and the north pole in the northern hemisphere. Let P, Q, and R move with speeds vP , vQ, and vR, respectively.

Which one of the following options is CORRECT?

(A)  vP < vR < vQ

(B)  vP < vQ < vR

(C)  vP > vR > vQ

(D)  vP = vR ≠ vQ

Answer: (C)

CE2: Civil Engineering

Q.11 – Q.35 Carry ONE mark Each

11. Let ϕ be a scalar field, and u be a vector field. Which of the following identities is true for div (ϕ u)?

(A)  div(ϕu) = ϕdiv(u) + u ∙ grad(ϕ)

(B)  div(ϕu) = ϕdiv(u) + u × grad(ϕ)

(C)  div(ϕu) = ϕgrad(u) + u ∙ grad(ϕ)

(D)  div(ϕu) = ϕgrad(u) + u × grad(ϕ)

Answer: (A)

12. Which of the following probability distribution functions (PDFs) has the mean greater than the median?

(A)  Function 1

(B)  Function 2

(C)  Function 3

(D)  Function 4

Answer: (B)

13. A remote village has exactly 1000 vehicles with sequential registration numbers starting from 1000. Out of the total vehicles, 30% are without pollution clearance certificate. Further, even- and odd-numbered vehicles are operated on even- and odd-numbered dates, respectively.

If 100 vehicles are chosen at random on an even-numbered date, the number of vehicles expected without pollution clearance certificate is ______.

(A)  15

(B)  30

(C)  50

(D)  70

Answer: (B)

14. A circular solid shaft of span L = 5 m is fixed at one end and free at the other end. A torque T = 100 kN.m is applied at the free end. The shear modulus and polar moment of inertia of the section are denoted as G and J, respectively. The torsional rigidity GJ is 50,000 kN.m2/rad. The following are reported for this shaft:

Statement i) The rotation at the free end is 0.01 rad

Statement ii) The torsional strain energy is 1.0 kN.m

With reference to the above statements, which of the following is true?

(A)  Both the statements are correct

(B)  Statement i) is correct, but Statement ii) is wrong

(C)  Statement i) is wrong, but Statement ii) is correct

(D)  Both the statements are wrong

Answer: (B)

15. M20 concrete as per IS 456: 2000 refers to concrete with a design mix having ________

(A)  an average cube strength of 20 MPa

(B)  an average cylinder strength of 20 MPa

(C)  a 5-percentile cube strength of 20 MPa

(D)  a 5-percentile cylinder strength of 20 MPa

Answer: (C)

16. When a simply-supported elastic beam of span L and flexural rigidity EI (E is the modulus of elasticity and I is the moment of inertia of the section) is loaded with a uniformly distributed load w per unit length, the deflection at the mid-span is 

If the load on one half of the span is now removed, the mid-span deflection _______.

(A)  reduces to Δ0/2

(B)  reduces to a value less than Δ0/2

(C)  reduces to a value greater than Δ0/2

(D)  remains unchanged at Δ0

Answer: (A)

17. Muller-Breslau principle is used in analysis of structures for _____________.

(A)  drawing an influence line diagram for any force response in the structure

(B)  writing the virtual work expression to get the equilibrium equation

(C)  superposing the load effects to get the total force response in the structure

(D)  relating the deflection between two points in a member with the curvature diagram in-between

Answer: (A)

18. A standard penetration test (SPT) was carried out at a location by using a manually operated hammer dropping system with 50% efficiency. The recorded SPT value at a particular depth is 28. If an automatic hammer dropping system with 70% efficiency is used at the same location, the recorded SPT value will be _________.

(A)  28

(B)  20

(C)  40

(D)  25

Answer: (B)

19. A vertical sheet pile wall is installed in an anisotropic soil having coefficient of horizontal permeability, kH and coefficient of vertical permeability, kV. In order to draw the flow net for the isotropic condition, the embedment depth of the wall should be scaled by a factor of ______, without changing the horizontal scale.

Answer: (A)

20. Identify the cross-drainage work in the figure.

(A)  Super passage

(B)  Aqueduct

(C)  Siphon aqueduct

(D)  Level crossing

Answer: (A)

21. Which one of the following options provides the correct match of the terms listed in Column-1 and Column-2?

(A)  P-IV, Q-V, R-III

(B)  P-III, Q-IV, R-I

(C)  P-IV, Q-III, R-II

(D)  P-III, Q-I, R-IV

Answer: (A)

22. In the context of Municipal Solid Waste Management, ‘Haul’ in ‘Hauled Container System operated in conventional mode’ includes the _________.

(A)  time spent by the transport truck at the disposal site

(B)  time spent by the transport truck in traveling between a pickup point and the disposal site with a loaded container

(C)  time spent by the transport truck in picking up a loaded container at a pickup point

(D)  time spent by the transport truck in driving from the depot to the first pickup point

Answer: (B)

23. Which of the following is equal to the stopping sight distance?

(A)  (braking distance required to come to stop) + (distance travelled during the perception-reaction time)

(B)  (braking distance required to come to stop) – (distance travelled during the perception-reaction time)

(C)  (braking distance required to come to stop)

(D)  (distance travelled during the perception-reaction time)

Answer: (A)

24. The magnetic bearing of the sun for a location at noon is 183˚ 30ˊ. If the sun is exactly on the geographic meridian at noon, the magnetic declination of the location is _______.

(A)  3˚ 30ʹ W

(B)  3˚ 30ʹ E

(C)  93˚ 30ʹ W

(D)  93˚ 30ʹ E

Answer: (A)

25. For the matrix

which of the following statements is/are TRUE?

(A)  [A]{x} = {b} has a unique solution

(B)  [A]{x} = {b} does not have a unique solution

(C)  [A] has three linearly independent eigenvectors

(D)  [A] is a positive definite matrix

Answer: (B, C)

26. In the frame shown in the figure (not to scale), all four members (AB, BC, CD, and AD) have the same length and same constant flexural rigidity. All the joints A, B, C, and D are rigid joints. The midpoints of AB, BC, CD, and AD, are denoted by E, F, G, and H, respectively. The frame is in unstable equilibrium under the shown forces of magnitude 𝑃 acting at E and G. Which of the following statements is/are TRUE?

(A)  Shear forces at H and F are zero

(B)  Horizontal displacements at H and F are zero

(C)  Vertical displacements at H and F are zero

(D)  Slopes at E, F, G, and H are zero

Answer: (A, B, D)

27. With regard to the shear design of RCC beams, which of the following statements is/are TRUE?

(A)  Excessive shear reinforcement can lead to compression failure in concrete

(B)  Beams without shear reinforcement, even if adequately designed for flexure, can have brittle failure

(C)  The main (longitudinal) reinforcement plays no role in the shear resistance of beam

(D)  As per IS456:2000, the nominal shear stress in the beams of varying depth depends on both the design shear force as well as the design bending moment

Answer: (A, B, D)

28. The reason(s) of the nonuniform elastic settlement profile below a flexible footing, resting on a cohesionless soil while subjected to uniform loading, is/are:

(A)  Variation of friction angle along the width of the footing

(B)  Variation of soil stiffness along the width of the footing

(C)  Variation of friction angle along the depth of the footing

(D)  Variation of soil stiffness along the depth of the footing

Answer: (B)

29. Which of the following is/are NOT active disinfectant(s) in water treatment?

(A)  •OH (hydroxyl radical)

(B)  O3 (ozone)

(C)  OCl (hypochlorite ion)

(D)  Cl (chloride ion)

Answer: (D)

30. As per the Indian Roads Congress guidelines (IRC 86: 2018), extra widening depends on which of the following parameters?

(A)  Horizontal curve radius

(B)  Superelevation

(C)  Number of lanes

(D)  Longitudinal gradient

Answer: (A, C)

31. The steady-state temperature distribution in a square plate ABCD is governed by the 2-dimensional Laplace equation. The side AB is kept at a temperature of 100°C and the other three sides are kept at a temperature of 0 °C. Ignoring the effect of discontinuities in the boundary conditions at the corners, the steady-state temperature at the center of the plate is obtained as T0°C. Due to symmetry, the steady-state temperature at the center will be same (T0°C), when any one side of the square is kept at a temperature of 100°C and the remaining three sides are kept at a temperature of 0°C. Using the principle of superposition, the value of T0 is ____________ (rounded off to two decimal places).

Answer: (24.90 to 25.10)

32. An unconfined compression strength test was conducted on a cohesive soil. The test specimen failed at an axial stress of 76 kPa. The undrained cohesion (in kPa, in integer) of the soil is _________.

Answer: (38 to 38)

33. The pressure in a pipe at X is to be measured by an open manometer as shown in figure. Fluid A is oil with a specific gravity of 0.8 and Fluid B is mercury with a specific gravity of 13.6. The absolute pressure at X is __________ kN/m2 (round off to one decimal place).

[Assume density of water as 1000 kg/m3 and acceleration due to gravity as 9.81 m/s2 and atmospheric pressure as 101.3 kN/m2]

Answer: (140.0 to 141.0)

34. For the elevation and temperature data given in the table, the existing lapse rate in the environment is _______ ℃/100 m (round off to two decimal places).

Answer: (0.84 to 0.85)

35. If the size of the ground area is 6 km × 3 km and the corresponding photo size in the aerial photograph is 30 cm × 15 cm, then the scale of the photograph is 1 : ________ (in integer).

Answer: (20000 to 20000)

Q.36 – Q.65 Carry TWO marks Each

36. The solution of the differential equation is expressed as y = C1e2.5x + C2eα x + C3eβx, where C1, C2, C3, α, and β are constants, with α and β being distinct and not equal to 2.5. Which of the following options is correct for the values of α and β ?

(A)  1 and 2

(B)  −1 and −2

(C)  2 and 3

(D)  −2 and −3

Answer: (A)

37. Two vectors  and  belong to the null space of a 4×4 matrix of rank 2. Which one of the following vectors also belongs to the null space?

Answer: (A)

38. Cholesky decomposition is carried out on the following square matrix [A].

Let lij and aij be the (i, j)th elements of matrices [L] and [A], respectively. If the element l22 of the decomposed lower triangular matrix [L] is 1.968, what is the value (rounded off to the nearest integer) of the element a22?

(A)  5

(B)  7

(C)  9

(D)  11

Answer: (B)

39. In a two-dimensional stress analysis, the state of stress at a point is shown in the figure. The values of length of PQ, QR, and RP are 4, 3, and 5 units, respectively. The principal stresses are _________. (round off to one decimal place)

(A)  σx = 26.7 MPa, σy = 172.5 MPa

(B)  σx = 54.0 MPa, σy = 128.5 MPa

(C)  σx = 67.5 MPa, σy = 213.3 MPa

(D)  σx = 16.0 MPa, σy = 138.5 MPa

Answer: (C)

40. Two plates are connected by fillet welds of size 10 mm and subjected to tension, as shown in the figure. The thickness of each plate is 12 mm. The yield stress and the ultimate stress of steel under tension are 250 MPa and 410 MPa, respectively. The welding is done in the workshop (partial safety factor, γmw = 1.25). As per the Limit State Method of IS 800: 2007, what is the minimum length (in mm, rounded off to the nearest higher multiple of 5 mm) required of each weld to transmit a factored force P equal to 275 kN?

(A)  100

(B)  105

(C)  110

(D)  115 

Answer: (B)

41. In the given figure, Point O indicates the stress point of a soil element at initial non-hydrostatic stress condition. For the stress path (OP), which of the following loading conditions is correct?

(A)  σv is increasing and σh is constant

(B)  σv is constant and σh is increasing 

(C)  σv is increasing and σh is decreasing

(D)  σv is decreasing and σh is increasing

Answer: (A)

42. The figure shows a vertical retaining wall with backfill consisting of cohesive-frictional soil and a failure plane developed due to passive earth pressure. The forces acting on the failure wedge are: P as the reaction force between the wall and the soil, R as the reaction force on the failure plane, C as the cohesive force along the failure plane and W as the weight of the failure wedge. Assuming that there is no adhesion between the wall and the wedge, identify the most appropriate force polygon for the wedge.

Answer: (C)

43. A compound symmetrical open channel section as shown in the figure has a maximum of ________ critical depth(s).

(A)  3

(B)  2

(C)  1

(D)  0

Answer: (A)

44. The critical flow condition in a channel is given by ________.

Note: α – kinetic energy correction factor; Q – discharge; Ac – cross-sectional area of flow at critical flow condition; Tc – top width of flow at critical flow condition; g – acceleration due to gravity]

Answer: (A)

45. Match the following air pollutants with the most appropriate adverse health effects:

(A)  (P) – (II), (Q) – (I), (R) – (IV), (S) – (III)

(B)  (P) – (IV), (Q) – (I), (R) – (III), (S) – (II)

(C)  (P) – (III), (Q) – (I), (R) – (II), (S) – (IV)

(D)  (P) – (IV), (Q) – (I), (R) – (II), (S) – (III)

Answer: (D)

46. A delivery agent is at a location R. To deliver the order, she is instructed to travel to location P along straight-line paths of RC, CA, AB and BP of 5 km each. The direction of each path is given in the table below as whole circle bearings. Assume that the latitude (L) and departure (D) of R is (0, 0) km. What is the latitude and departure of P (in km, rounded off to one decimal place)?

(A)  L = 2.5; D = 5.0

(B)  L = 0.0; D = 5.0

(C)  L = 5.0; D = 2.5

(D)  L = 0.0; D = 0.0

Answer: (B)

47. Which of the following statements is/are TRUE?

(A)  The thickness of a turbulent boundary layer on a flat plate kept parallel to the flow direction is proportional to the square root of the distance from the leading edge

(B)  If the streamlines and equipotential lines of a source are interchanged with each other, the resulting flow will be a sink

(C)  For a curved surface immersed in a stationary liquid, the vertical component of the force on the curved surface is equal to the weight of the liquid above it

(D)  For flow through circular pipes, the momentum correction factor for laminar flow is larger than that for turbulent flow

Answer: (C, D)

48. In the context of water and wastewater treatments, the correct statements are:

(A)  particulate matter may shield microorganisms during disinfection

(B)  ammonia decreases chlorine demand

(C)  phosphorous stimulates algal and aquatic growth

(D)  calcium and magnesium increase hardness and total dissolved solids

Answer: (A, C, D)

49. Which of the following statements is/are TRUE for the aerobic composting of sewage sludge?

(A)  Bulking agent is added during the composting process to reduce the porosity of the solid mixture

(B)  Leachate can be generated during composting

(C)  Actinomycetes are involved in the process

(D)  In-vessel composting systems cannot be operated in the plug-flow mode

Answer: (B, C)

50. The figure presents the time-space diagram for when the traffic on a highway is suddenly stopped for a certain time and then released. Which of the following statements are true?

(A)  Speed is higher in Region R than in Region P

(B)  Volume is lower in Region Q than in Region P

(C)  Volume is higher in Region R than in Region P

(D)  Density is higher in Region Q than in Region R

Answer: (B, C, D)

51. Consider the Marshall method of mix design for bituminous mix. With the increase in bitumen content, which of the following statements is/are TRUE?

(A)  the Stability decreases initially and then increases

(B)  the Flow increases monotonically

(C)  the air voids (VA) increases initially and then decreases

(D)  the voids filled with bitumen (VFB) increases monotonically

Answer: (B, D)

52. A 5 cm long metal rod AB was initially at a uniform temperature of T0 °C. Thereafter, temperature at both the ends are maintained at 0 °C. Neglecting the heat transfer from the lateral surface of the rod, the heat transfer in the rod is governed by the one-dimensional diffusion equation  where D is the thermal diffusivity of the metal, given as 1.0 cm2/s.

The temperature distribution in the rod is obtained as  where x is in cm measured from A to B with x = 0 at A, t is in s, Cn are constants in °C, T is in °C, and β is in s−1.

The value of β (in s−1, rounded off to three decimal places) is ___________.

Answer: (0.394 to 0.396)

53. A beam is subjected to a system of coplanar forces as shown in the figure. The magnitude of vertical reaction at Support P is ________N (round off to one decimal place).

Answer: (195.0 to 200.0)

54. For the frame shown in the figure (not to scale), all members (AB, BC, CD, GB, and CH) have the same length, L and flexural rigidity, EI. The joints at B and C are rigid joints, and the supports A and D are fixed supports. Beams GB and CH carry uniformly distributed loads of w per unit length. The magnitude of the moment reaction at A is wL2/k. What is the value of k (in integer)? _________

Answer: (6 to 6)

55. Consider the singly reinforced section of a cantilever concrete beam under bending, as shown in the figure (M25 grade concrete, Fe415 grade steel). The stress block parameters for the section at ultimate limit state, as per IS 456: 2000 notations, are given. The ultimate moment of resistance for the section by the Limit State Method is __________ kN.m (round off to one decimal place).

Note: Here, As is the total area of tension steel bars, b is the width of the section, d is the effective depth of the bars, fck is the characteristic compressive cube strength of concrete, fy is the yield stress of steel, and xu is the depth of neutral axis.]

Answer: (295.0 to 305.0)

56. A 2D thin plate with modulus of elasticity, E = 1.0 N/m2, and Poisson’s ratio, μ = 0.5, is in plane stress condition. The displacement field in the plate is given by u = Cx2y and 𝑣=0, where u and v are displacements (in m) along the X and Y directions, respectively, and C is a constant (in m−2). The distances x and y along X and Y, respectively, are in m. The stress in the X direction is σXX = 40 xy N/m2, and the shear stress is τXY = αx2 N/m2. What is the value of 𝛼 (in N/m4, in integer)? ___________

Answer: (5 to 5)

57. An idealised frame supports a load as shown in the figure. The horizontal component of the force transferred from the horizontal member PQ to the vertical member RS at P is __________ N (round off to one decimal place).

Answer: (17.5 to 18.5)

58. A square footing is to be designed to carry a column load of 500 kN which is resting on a soil stratum having the following average properties: bulk unit weight = 19 kN/m3; angle of internal friction = 0° and cohesion = 25 kPa. Considering the depth of the footing as 1 m and adopting Meyerhof’s bearing capacity theory with a factor of safety of 3, the width of the footing (in m) is _____________ (round off to one decimal place)

[Assume the applicable shape and depth factor values as unity; ground water level at greater depth.]

Answer: (3.0 to 3.5)

59. A circular pile of diameter 0.6 m and length 8 m was constructed in a cohesive soil stratum having the following properties: bulk unit weight = 19 kN/m3; angle of internal friction = 0° and cohesion = 25 kPa.

The allowable load the pile can carry with a factor of safety of 3 is __________ kN (round off to one decimal place).

[Adopt: Adhesion factor, α = 1.0 and Bearing capacity factor, Nc = 9.0]

Answer: (145.0 to 149.0)

60. For the flow setup shown in the figure (not to scale), the hydraulic conductivities of the two soil samples, Soil 1 and Soil 2, are 10 mm/s and 1 mm/s, respectively. Assume the unit weight of water as 10 kN/m3 and ignore the velocity head. At steady state, what is the total head (in m, rounded off to two decimal places) at any point located at the junction of the two samples? ____________

Answer: (4.50 to 4.60)

61. A consolidated drained (CD) triaxial test was carried out on a sand sample with the known effective shear strength parameters, cʹ = 0 and ϕʹ = 30°. In the test, prior to the failure, when the sample was undergoing axial compression under constant cell pressure, the drainage valve was accidentally closed. At the failure, 360 kPa deviatoric stress was recorded along with 70 kPa pore water pressure. If the test is repeated without such error, and no back pressure is applied in either of the tests, what is the deviatoric stress (in kPa, in integer) at the failure? ________

Answer: (500 to 500)

62. The annual rainfall recorded at these gauges in a particular year are given below.

Using the Thiessen polygon method, what is the average rainfall (in mm, rounded off to two decimal places) over the catchment in that year? ___________

Answer: (912.28 to 912.82)

63. The cross-section of a small river is sub-divided into seven segments of width 1.5 m each. The average depth, and velocity at different depths were measured during a field campaign at the middle of each segment width. The discharge computed by the velocity area method for the given data is ___________ m3/s (round off to one decimal place).

Answer: (8.4 to 8.6)

64. The theoretical aerobic oxidation of biomass (C5H7O2N) is given below:

C5H7O2N + 5O2 → 5CO2 + NH3 + 2H2O

The biochemical oxidation of biomass is assumed as a first-order reaction with a rate constant of 0.23/d at 20ºC (logarithm to base e). Neglecting the second-stage oxygen demand from its biochemical oxidation, the ratio of BOD5 at 20ºC to total organic carbon (TOC) of biomass is ________ (round off to two decimal places).

[Consider the atomic weights of C, H, O and N as 12 g/mol, 1 g/mol, 16 g/mol and 14 g/mol, respectively]

Answer: (1.80 to 2.00)

65. A system of seven river segments is shown in the schematic diagram. The Ri’s, Qi’s, and Ci’s (i = 1 to 7) are the river segments, their corresponding flow rates, and concentrations of a conservative pollutant, respectively. Assume complete mixing at the intersections, no additional water loss or gain in the system, and steady state condition. Given: Q1 = 5 m3/s ; Q2 = 15 m3/s ; Q4 = 3 m3/s ; Q6 = 8 m3/s ; C1 = 8 kg/m3 ; C2 = 12 kg/m3 ; C6 = 10 kg/m3. What is the steady state concentration (in kg/m3, rounded off to two decimal place) of the pollutant in the river segment 7 ? ___________

Answer: (10.58 to 10.78)

GATE Exam 2023 Civil Engineering (CE-1) Question Paper With Answer Key

GATE-2023

CE1: Civil Engineering

GA-General Aptitude

Q.1 – Q.5 Carry ONE mark each.

1. “I have not yet decided what I will do this evening; I ______ visit a friend.”

(A)  mite

(B)  would

(C)  might

(D)  didn’t

Answer: (C)

2. Eject : Insert : : Advance : _______

(By word meaning)

(A)  Advent

(B)  Progress

(C)  Retreat

(D)  Loan

Answer: (C)

3. In the given figure, PQRSTV is a regular hexagon with each side of length 5 cm. A circle is drawn with its centre at V such that it passes through P. What is the area (in cm2) of the shaded region? (The diagram is representative)

(A)  25π/3

(B)  20π/3

(C)  6π

(D)  7π

Answer: (A)

4. A duck named Donald Duck says “All ducks always lie.”

Based only on the information above, which one of the following statements can be logically inferred with certainty?

(A)  Donald Duck always lies.

(B)  Donald Duck always tells the truth.

(C)  Donald Duck’s statement is true.

(D)  Donald Duck’s statement is false.

Answer: (D)

5. A line of symmetry is defined as a line that divides a figure into two parts in a way such that each part is a mirror image of the other part about that line.

The figure below consists of 20 unit squares arranged as shown. In addition to the given black squares, upto 5 more may be coloured black. Which one among the following options depicts the minimum number of boxes that must be coloured black to achieve two lines of symmetry? (The figure is representative)

(A)  d

(B)  c, d, i

(C)  c, i

(D)  c, d, i, f, g

Answer: (B)

Q.6 – Q.10 Carry TWO marks Each

6. Based only on the truth of the statement ‘Some humans are intelligent’, which one of the following options can be logically inferred with certainty?

(A)  No human is intelligent.

(B)  All humans are intelligent.

(C)  Some non-humans are intelligent.

(D)  Some intelligent beings are humans.

Answer: (D)

7. Which one of the options can be inferred about the mean, median, and mode for the given probability distribution (i.e. probability mass function), P(x), of a variable x?

(A)  mean = median ≠ mode

(B)  mean = median = mode

(C)  mean ≠ median = mode

(D)  mean ≠ mode = median

Answer: (A)

8. The James Webb telescope, recently launched in space, is giving humankind unprecedented access to the depths of time by imaging very old stars formed almost 13 billion years ago. Astrophysicists and cosmologists believe that this odyssey in space may even shed light on the existence of dark matter. Dark matter is supposed to interact only via the gravitational interaction and not through the electromagnetic-, the weak- or the strong-interaction. This may justify the epithet “dark” in dark matter.

Based on the above paragraph, which one of the following statements is FALSE?

(A)  No other telescope has captured images of stars older than those captured by the James Webb telescope.

(B)  People other than astrophysicists and cosmologists may also believe in the existence of dark matter.

(C)  The James Webb telescope could be of use in the research on dark matter.

(D)  If dark matter was known to interact via the strong-interaction, then the epithet “dark” would be justified.

Answer: (D)

9. Let a = 30! , b = 50! , and c = 100! . Consider the following numbers:

loga c, logc a, logb a, loga b

Which one of the following inequalities is CORRECT?

(A)  logc a < logb a < loga b < loga c

(B)  logc a < loga b < logb a < logb c

(C)  logc a < logb a < loga c < loga b         

(D)  logb a < logc a < loga b < loga c

Answer: (A)

10. A square of side length 4 cm is given. The boundary of the shaded region is defined by one semi-circle on the top and two circular arcs at the bottom, each of radius 2 cm, as shown.

The area of the shaded region is _______ cm2.

(A)  8

(B)  4

(C)  12

(D)  10

Answer: (A)

CE1: Civil Engineering

Q.11 – Q.35 Carry ONE mark Each

11. For the integral

which of the following statements is TRUE?

(A)  I = 0

(B)  I = 2

(C)  I = −2

(D)  The integral does not converge

Answer: (D)

12. A hanger is made of two bars of different sizes. Each bar has a square cross-section. The hanger is loaded by three-point loads in the mid vertical plane as shown in the figure. Ignore the self-weight of the hanger. What is the maximum tensile stress in N/mm2 anywhere in the hanger without considering stress concentration effects?

(A)  15.0

(B)  25.0

(C)  35.0

(D)  45.0

Answer: (B)

  1. Creep of concrete under compression is defined as the _________.

(A)  increase in the magnitude of strain under constant stress

(B)  increase in the magnitude of stress under constant strain

(C)  decrease in the magnitude of strain under constant stress

(D)  decrease in the magnitude of stress under constant strain

Answer: (A)

14. A singly reinforced concrete beam of balanced section is made of M20 grade concrete and Fe415 grade steel bars. The magnitudes of the maximum compressive strain in concrete and the tensile strain in the bars at ultimate state under flexure, as per IS 456: 2000 are _______, respectively. (round off to four decimal places)

(A)  0.0035 and 0.0038

(B)  0.0020 and 0.0018

(C)  0.0035 and 0.0041

(D)  0.0020 and 0.0031

Answer: (A)

15. In cement concrete mix design, with the increase in water-cement ratio, which one of the following statements is TRUE?

(A)  Compressive strength decreases but workability increases

(B)  Compressive strength increases but workability decreases

(C)  Both compressive strength and workability decrease

(D)  Both compressive strength and workability increase

Answer: (A)

16. The specific gravity of a soil is 2.60. The soil is at 50% degree of saturation with a water content of 15%. The void ratio of the soil is _______.

(A)  0.35

(B)  0.78

(C)  0.87

(D)  1.28

Answer: (B)

17. A group of 9 friction piles are arranged in a square grid maintaining equal spacing in all directions. Each pile is of diameter 300 mm and length 7 m. Assume that the soil is cohesionless with effective friction angle ϕ′=32°. What is the center-to-center spacing of the piles (in m) for the pile group efficiency of 60%?

(A)  0.582

(B)  0.486

(C)  0.391

(D)  0.677

Answer: (MTA)

18. A possible slope failure is shown in the figure. Three soil samples are taken from different locations (I, II and III) of the potential failure plane. Which is the most appropriate shear strength test for each of the sample to identify the failure mechanism? Identify the correct combination from the following options:

P: Triaxial compression test

Q: Triaxial extension test

R: Direct shear or shear box test

S: Vane shear test

(A)  I-Q, II-R, III-P

(B)  I-R, II-P, III-Q

(C)  I-S, II-Q, III-R

(D)  I-P, II-R, III-Q

Answer: (A)

19. When a supercritical stream enters a mild-sloped (M) channel section, the type of flow profile would become _______.

(A)  M1

(B)  M2

(C)  M3

(D)  M1 and M2

Answer: (C)

20. Which one of the following statements is TRUE for Greenhouse Gas (GHG) in the atmosphere?

(A)  GHG absorbs the incoming short wavelength solar radiation to the earth surface, and allows the long wavelength radiation coming from the earth surface to pass through

(B)  GHG allows the incoming long wavelength solar radiation to pass through to the earth surface, and absorbs the short wavelength radiation coming from the earth surface

(C)  GHG allows the incoming long wavelength solar radiation to pass through to the earth surface, and allows the short wavelength radiation coming from the earth surface to pass through

(D)  GHG allows the incoming short wavelength solar radiation to pass through to the earth surface, and absorbs the long wavelength radiation coming from the earth surface

Answer: (D)

21. G1 and G2 are the slopes of the approach and departure grades of a vertical curve, respectively.

Given |G1| < |G2| and |G1| ≠ |G2| ≠ 0

Statement 1: +G1 followed by +G2 results in a sag vertical curve.

Statement 2: −G1 followed by −G2 results in a sag vertical curve.

Statement 3: +G1 followed by −G2 results in a crest vertical curve.

Which option amongst the following is true?

(A)  Statement 1 and Statement 3 are correct; Statement 2 is wrong

(B)  Statement 1 and Statement 2 are correct; Statement 3 is wrong

(C)  Statement 1 is correct; Statement 2 and Statement 3 are wrong

(D)  Statement 2 is correct; Statement 1 and Statement 3 are wrong

Answer: (A)

22. The direct and reversed zenith angles observed by a theodolite are 56o 00′ 00″ and 303o 00′ 00″, respectively. What is the vertical collimation correction?

(A)  +1° 00′ 00″

(B)  −1° 00′ 00″

(C)  −0° 30′ 00″

(D)  +0° 30′ 00″

Answer: (D)

23. A student is scanning his 10 inch × 10 inch certificate at 600 dots per inch (dpi) to convert it to raster. What is the percentage reduction in number of pixels if the same certificate is scanned at 300 dpi?

(A)  62

(B)  88

(C)  75

(D)  50

Answer: (C)

24. If M is an arbitrary real n × n matrix, then which of the following matrices will have non-negative eigenvalues?

(A)  M2

(B)  MMT

(C)  MTM

(D)  (MT)2

Answer: (B, C)

25. The following function is defined over the interval [−L, L]:

f(x) = px4 + qx5.

If it is expressed as a Fourier series,

which options amongst the following are true?

(A)  an, n = 1, 2 …, ∞ depend on p

(B)  an, n = 1, 2 …, ∞ depend on q

(C)  bn, n = 1, 2 …, ∞ depend on p

(D)  bn, n = 1, 2 …, ∞ depend on q

Answer: (B, C)

26. Consider the following three structures:

Which of the following statements is/are TRUE?

(A)  Structure I is unstable

(B)  Structure II is unstable

(C)  Structure III is unstable

(D)  All three structures are stable

Answer: (A, B, C)

27. Identify the waterborne diseases caused by viral pathogens:

(A)  Acute anterior poliomyelitis

(B)  Cholera

(C)  Infectious hepatitis

(D)  Typhoid fever

Answer: (A, C)

28. Which of the following statements is/are TRUE for the Refuse-Derived Fuel (RDF) in the context of Municipal Solid Waste (MSW) management?

(A)  Higher Heating Value (HHV) of the unprocessed MSW is higher than the HHV of RDF processed from the same MSW

(B)  RDF can be made in the powdered form

(C)  Inorganic fraction of MSW is mostly converted to RDF

(D)  RDF cannot be used in conjunction with oil

Answer: (B)

29. The probabilities of occurrences of two independent events A and B are 0.5 and 0.8, respectively. What is the probability of occurrence of at least A or B (rounded off to one decimal place)? ______

Answer: (0.9 to 0.9)

30. In the differential equation  α is a positive constant. If y = 1.0 at x = 0.0, and y = 0.8 at x = 1.0, the value of α is _______ (rounded off to three decimal places).

Answer: (0.445 to 0.447)

31. Consider the fillet-welded lap joint shown in the figure (not to scale). The length of the weld shown is the effective length. The welded surfaces meet at right angle. The weld size is 8 mm, and the permissible stress in the weld is 120 MPa. What is the safe load P (in kN, rounded off to one decimal place) that can be transmitted by this welded joint? ____________

Answer: (134.0 to 136.0)

32. A drained direct shear test was carried out on a sandy soil. Under a normal stress of 50 kPa, the test specimen failed at a shear stress of 35 kPa. The angle of internal friction of the sample is ________ degree (round off to the nearest integer).

Answer: (35 to 35)

33. A canal supplies water to an area growing wheat over 100 hectares. The duration between the first and last watering is 120 days, and the total depth of water required by the crop is 35 cm. The most intense watering is required over a period of 30 days and requires a total depth of water equal to 12 cm. Assuming precipitation to be negligible and neglecting all losses, the minimum discharge (in m3/s, rounded off to three decimal places) in the canal to satisfy the crop requirement is ___________.

Answer: (0.045 to 0.047)

34. The ordinates of a one-hour unit hydrograph for a catchment are given below:

Using the principle of superposition, a D-hour unit hydrograph for the catchment was derived from this one-hour unit hydrograph. The ordinates of the D-hour unit hydrograph were obtained as 3 m3/s at t = 1 hour and 10 m3/s at t = 2 hour. The value of D (in integer) is _____________.

Answer: (3 to 3)

35. For a horizontal curve, the radius of a circular curve is obtained as 300 m with the design speed as 15 m/s. If the allowable jerk is 0.75 m/s3, what is the minimum length (in m, in integer) of the transition curve? _________

Answer: (15 to 15 OR 26 to 27)

Q.36 – Q.65 Carry TWO marks Each

36. A function f(x), that is smooth and convex-shaped between interval (x­l, xu) is shown in the figure. This function is observed at odd number of regularly spaced If the area under the function is computed numerically, then ________

(A)  the numerical value of the area obtained using the trapezoidal rule will be less than the actual

(B)  the numerical value of the area obtained using the trapezoidal rule will be more than the actual

(C)  the numerical value of the area obtained using the trapezoidal rule will be exactly equal to the actual

(D)  with the given details, the numerical value of area cannot be obtained using trapezoidal rule

Answer: (A)

37. Consider a doubly reinforced RCC beam with the option of using either Fe250 plain bars or Fe500 deformed bars in the compression zone. The modulus of elasticity of steel is 2 × 105 N/mm2. As per IS456:2000, in which type(s) of the bars, the stress in the compression steel (fsc) can reach the design strength (0.87 fy) at the limit state of collapse?

(A)  Fe250 plain bars only

(B)  Fe500 deformed bars only

(C)  Both Fe250 plain bars and Fe500 deformed bars

(D)  Neither Fe250 plain bars nor Fe500 deformed bars

Answer: (A)

38. Consider the horizontal axis passing through the centroid of the steel beam cross-section shown in the figure. What is the shape factor (rounded off to one decimal place) for the cross-section?

(A)  1.5

(B)  1.7

(C)  1.3

(D)  2.0

Answer: (B)

39. Consider the pin-jointed truss shown in the figure (not to scale). All members have the same axial rigidity, AE. Members QR, RS, and ST have the same length L. Angles QBT, RCT, SDT are all 90°. Angles BQT, CRT, DST are all 30°. The joint T carries a vertical load P. The vertical deflection of joint T is  What is the value of k?

(A)  1.5

(B)  4.5

(C)  3.0

(D)  9.0

Answer: (B)

40. With reference to the compaction test conducted on soils, which of the following is INCORRECT?

(A)  Peak point of the compaction curve gives the maximum dry unit weight and optimum moisture content

(B)  With increase in the compaction effort, the maximum dry unit weight increases

(C)  With increase in the compaction effort, the optimum moisture content decreases

(D)  Compaction curve crosses the zero-air-voids curve

Answer: (D)

41. Consider that a force P is acting on the surface of a half-space (Boussinesq’s problem). The expression for the vertical stress (σz) at any point (r, z), within the half-space is given as,

where, r is the radial distance, and z is the depth with downward direction taken as positive. At any given r, there is a variation of σz along z, and at a specific z, the value of σz will be maximum. What is the locus of the maximum σz?

Answer: (A)

42. A square footing of size 2.5 m × 2.5 m is placed 1.0 m below the ground surface on a cohesionless homogeneous soil stratum. Considering that the groundwater table is located at the base of the footing, the unit weights of soil above and below the groundwater table are 18 kN/m3 and 20 kN/m3, respectively, and the bearing capacity factor Nq is 58, the net ultimate bearing capacity of the soil is estimated as 1706 kPa (unit weight of water = 10 kN/m3).

Earlier, a plate load test was carried out with a circular plate of 30 cm diameter in the same foundation pit during a dry season, when the water table was located beyond the plate influence zone. Using Terzaghi’s bearing capacity formulation, what is the ultimate bearing capacity (in kPa) of the plate?

(A)  110.16

(B)  61.20

(C)  204.00

(D)  163.20

Answer: (A)

43. A very wide rectangular channel carries a discharge (Q) of 70 m3/s per meter width. Its bed slope changes from 0.0001 to 0.0009 at a point P, as shown in the figure (not to scale). The Manning’s roughness coefficient of the channel is 0.01. What water surface profile(s) exist(s) near the point P?

(A)  M2 and S2

(B)  M2 only

(C)  S2 only

(D)  S2 and hydraulic jump

Answer: (A)

44. A jet of water having a velocity of 20 m/s strikes a series of plates fixed radially on a wheel revolving in the same direction as the jet at 15 m/s. What is the percentage efficiency of the plates? (round off to one decimal place)

(A)  37.5

(B)  66.7

(C)  50.0

(D)  88.9

Answer: (A)

45. In the following table, identify the correct set of associations between the entries in Column-1 and Column-2.

(A)  P-II, Q-I, S-III

(B)  Q-III, R-II, S-IV

(C)  P-IV, R-I, S-II

(D)  P-III, Q-I, R-IV

Answer: (D)

46. A plot of speed-density relationship (linear) of two roads (Road A and Road B) is shown in the figure.

If the capacity of Road A is CA and the capacity of Road B is CB, what is CA/CB?

(A)  kA/kB

(B)  uA/uB

(C)  kAuA/kBuB

(D)  kAuB/kBuA

Answer: (A, B, D)

47. For the matrix

which of the following statements is/are TRUE?

(A)  The eigenvalues of [A]T are same as the eigenvalues of [A]

(B)  The eigenvalues of [A]1 are the reciprocals of the eigenvalues of [A]

(C)  The eigenvectors of [A]T are same as the eigenvectors of [A]

(D)  The eigenvectors of [A]1 are same as the eigenvectors of [A]

Answer: (A, B, D)

48. For the function f(x) = ex|sin x|; x ∈ ℝ, which of the following statements is/are TRUE?

(A)  The function is continuous at all x

(B)  The function is differentiable at all x

(C)  The function is periodic

(D)  The function is bounded

Answer: (A)

49. Consider the beam shown in the figure (not to scale), on a hinge support at end A and a roller support at end B. The beam has a constant flexural rigidity, and is subjected to the external moments of magnitude 𝑀 at one-third spans, as shown in the figure. Which of the following statements is/are TRUE?

(A)  Support reactions are zero

(B)  Shear force is zero everywhere

(C)  Bending moment is zero everywhere

(D)  Deflection is zero everywhere

Answer: (A, B)

50. Which of the following statements is/are TRUE in relation to the Maximum Mixing Depth (or Height) ‘Dmax’ in the atmosphere?

(A)  Dmax is always equal to the height of the layer of unstable air

(B)  Ventilation coefficient depends on Dmax

(C)  A smaller Dmax will have a smaller air pollution potential if other meteorological conditions remain same

(D)  Vertical dispersion of pollutants occurs up to Dmax

Answer: (B, D)

51. Which of the following options match the test reporting conventions with the given material tests in the table?

(A)  (P) – (VI); (Q) – (I); (R) – (II); (S) – (VII)

(B)  (P) – (VI); (Q) – (III); (R) – (IV); (S) – (V)

(C)  (P) – (VI); (Q) – (I); (R) – (II); (S) – (V)

(D)  (P) – (VI); (Q) – (III); (R) – (IV); (S) – (VII)

Answer: (B, C)

52. The differential equation,

is solved by employing a backward difference scheme within the finite difference framework. The value of u at the (n – 1)th time-step, for some n, is 1.75. The corresponding time (t) is 3.14 s. Each time step is 0.01 s long. Then, the value of (un – un – 1) is __________ (round off to three decimal places).

Answer: (-0.152 to -0.149)

53. The infinitesimal element shown in the figure (not to scale) represents the state of stress at a point in a body. What is the magnitude of the maximum principal stress (in N/mm2, in integer) at the point? _________

Answer: (7 to 7)

54. An idealised bridge truss is shown in the figure. The force in Member U2L3 is _________ kN (round off to one decimal place).

Answer: (13.5 to 14.5)

55. The cross-section of a girder is shown in the figure (not to scale). The section is symmetric about a vertical axis (Y-Y). The moment of inertia of the section about the horizontal axis (X-X) passing through the centroid is ________ cm4 (round off to nearest integer).

Answer: (464000 to 472000)

56. A soil having the average properties, bulk unit weight = 19 kN/m3; angle of internal friction = 25° and cohesion = 15 kPa, is being formed on a rock slope existing at an inclination of 35° with the horizontal. The critical height (in m) of the soil formation up to which it would be stable without any failure is _________ (round off to one decimal place).

[Assume the soil is being formed parallel to the rock bedding plane and there is no ground water effect.]

Answer: (4.8 to 5.2)

57. A smooth vertical retaining wall supporting layered soils is shown in figure. According to Rankine’s earth pressure theory, the lateral active earth pressure acting at the base of the wall is ________ kPa (round off to one decimal place).

Answer: (34.0 to 37.0)

58. A vertical trench is excavated in a clayey soil deposit having a surcharge load of 30 kPa. A fluid of unit weight 12 kN/m3 is poured in the trench to prevent collapse as the excavation proceeds. Assume that the fluid is not seeping through the soil deposit. If the undrained cohesion of the clay deposit is 20 kPa and saturated unit weight is 18 kN/m3, what is the maximum depth of unsupported excavation (in m, rounded off to two decimal places)? ________

Answer: (3.30 to 3.35)

59. A 12-hour storm occurs over a catchment and results in a direct runoff depth of 100 mm. The time-distribution of the rainfall intensity is shown in the figure (not to scale). The ϕ-index of the storm is (in mm, rounded off to two decimal places) __________.

Answer: (MTA)

60. A hydraulic jump occurs in a 1.0 m wide horizontal, frictionless, rectangular channel, with a pre-jump depth of 0.2 m and a post-jump depth of 1.0 m. The value of g may be taken as 10 m/s2. The values of the specific force at the pre-jump and post-jump sections are same and are equal to (in m3, rounded off to two decimal places) _____________.

Answer: (0.60 to 0.64)

61. In Horton’s equation fitted to the infiltration data for a soil, the initial infiltration capacity is 10 mm/h; final infiltration capacity is 5 mm/h; and the exponential decay constant is 0.5 /h. Assuming that the infiltration takes place at capacity rates, the total infiltration depth (in mm) from a uniform storm of duration 12 h __________. (round off to one decimal place)

Answer: (69.7 to 70.1)

62. The composition and energy content of a representative solid waste sample are given in the table. If the moisture content of the waste is 26%, the energy content of the solid waste on dry-weight basis is __________ MJ/kg (round off to one decimal place).

Answer: (18.0 to 19.0)

63. A flocculator tank has a volume of 2800 m3. The temperature of water in the tank is 15°C, and the average velocity gradient maintained in the tank is 100/s. The temperature of water is reduced to 5°C, but all other operating conditions including the power input are maintained as the same. The decrease in the average velocity gradient (in %) due to the reduction in water temperature is _______ (round off to nearest integer).

[Consider dynamic viscosity of water at 15°C and 5°C as 1.139 × 103 N-s/m2 and 1.518 × 103 N-s/m2, respectively]

Answer: (12 to 15)

64. The wastewater inflow to an activated sludge plant is 0.5 m3/s, and the plant is to be operated with a food to microorganism ratio of 0.2 mg/mg-d. The concentration of influent biodegradable organic matter of the wastewater to the plant (after primary settling) is 150 mg/L, and the mixed liquor volatile suspended solids concentration to be maintained in the plant is 2000 mg/L. Assuming that complete removal of biodegradable organic matter in the tank, the volume of aeration tank (in m3, in integer) required for the plant is _________.

Answer: (16200 to 16200)

65. Trigonometric levelling was carried out from two stations P and Q to find the reduced level (R. L.) of the top of hillock, as shown in the table. The distance between Stations P and Q is 55 m. Assume Stations P and Q, and the hillock are in the same vertical plane. The R. L. of the top of the hillock (in m) is ________ (round off to three decimal places).

Answer: (137.500 to 137.730)

GATE Exam 2023 Biotechnology (BT) Question Paper With Answer Key

GATE-2023

BT: Biotechnology

General Aptitude

Q.1 – Q.5 Carry ONE mark each.

1. “You are delaying the completion of the task. Send _______ contributions at the earliest.”

(A)  you are

(B)  your

(C)  you’re

(D)  yore

Answer: (B)

2. References : ______ : : Guidelines : Implement

(By word meaning)

(A)  Sight

(B)  Site

(C)  Cite

(D)  Plagiarise      

Answer: (C)

3. In the given figure, PQRS is a parallelogram with PS = 7 cm, PT = 4 cm and PV = 5 cm. What is the length of RS in cm? (The diagram is representative.)

(A)  20/7

(B)  28/5

(C)  9/2

(D)  35/4

Answer: (B)

4. In 2022, June Huh was awarded the Fields medal, which is the highest prize in Mathematics.

When he was younger, he was also a poet. He did not win any medals in the International Mathematics Olympiads. He dropped out of college.

Based only on the above information, which one of the following statements can be logically inferred with certainty?

(A)  Every Fields medalist has won a medal in an International Mathematics Olympiad.

(B)  Everyone who has dropped out of college has won the Fields medal.

(C)  All Fields medalists are part-time poets.

(D)  Some Fields medalists have dropped out of college.

Answer: (D)

5. A line of symmetry is defined as a line that divides a figure into two parts in a way such that each part is a mirror image of the other part about that line.

The given figure consists of 16 unit squares arranged as shown. In addition to the three black squares, what is the minimum number of squares that must be coloured black, such that both PQ and MN form lines of symmetry? (The figure is representative)

(A)  3

(B)  4

(C)  5

(D)  6

Answer: (C)

Q.6 – Q.10 Carry TWO marks Each

6. Human beings are one among many creatures that inhabit an imagined world. In this imagined world, some creatures are cruel. If in this imagined world, it is given that the statement “Some human beings are not cruel creatures” is FALSE, then which of the following set of statement(s) can be logically inferred with certainty?

(i) All human beings are cruel creatures.

(ii) Some human beings are cruel creatures.

(iii) Some creatures that are cruel are human beings.

(iv) No human beings are cruel creatures.

(A)  only (i)

(B)  only (iii) and (iv)

(C)  only (i) and (ii)

(D)  (i), (ii) and (iii)

Answer: (D)

7. To construct a wall, sand and cement are mixed in the ratio of 3:1. The cost of sand and that of cement are in the ratio of 1:2.

If the total cost of sand and cement to construct the wall is 1000 rupees, then what is the cost (in rupees) of cement used?

(A)  400

(B)  600

(C)  800

(D)  200

Answer: (A)

8. The World Bank has declared that it does not plan to offer new financing to Sri Lanka, which is battling its worst economic crisis in decades, until the country has an adequate macroeconomic policy framework in place. In a statement, the World Bank said Sri Lanka needed to adopt structural reforms that focus on economic stabilisation and tackle the root causes of its crisis. The latter has starved it of foreign exchange and led to shortages of food, fuel, and medicines. The bank is repurposing resources under existing loans to help alleviate shortages of essential items such as medicine, cooking gas, fertiliser, meals for children, and cash for vulnerable households.

Based only on the above passage, which one of the following statements can be inferred with certainty?

(A)  According to the World Bank, the root cause of Sri Lanka’s economic crisis is that it does not have enough foreign exchange.

(B)  The World Bank has stated that it will advise the Sri Lankan government about how to tackle the root causes of its economic crisis.

(C)  According to the World Bank, Sri Lanka does not yet have an adequate macroeconomic policy framework.

(D)  The World Bank has stated that it will provide Sri Lanka with additional funds for essentials such as food, fuel, and medicines.

Answer: (C)

9. The coefficient of x4 in the polynomial (x − 1)3 (x − 2)3 is equal to _______.

(A)  33

(B)  −3

(C)  30

(D)  21

Answer: (A)

10. Which one of the following shapes can be used to tile (completely cover by repeating) a flat plane, extending to infinity in all directions, without leaving any empty spaces in between them? The copies of the shape used to tile are identical and are not allowed to overlap.

(A)  circle

(B)  regular octagon

(C)  regular pentagon

(D)  rhombus

Answer: (D)

BT: Biotechnology

Q.11 – Q.35 Carry ONE mark Each

11. Eukaryotic transcription is carried out by

(A)  DNA-dependent RNA polymerase

(B)  DNA-dependent DNA polymerase

(C)  RNA-dependent DNA polymerase

(D)  RNA-dependent RNA polymerase

Answer: (A)

12. Acetylcholine released by the parasympathetic nerves has which one of the following functions in the heart pacemaker cells?

(A)  It binds to GPCR and activates G protein to slow the heart rate

(B)  It stimulates GABA-activated ion-channel coupled receptor to increase the heart rate

(C)  It binds to GPCR and inhibits G protein to slow the heart rate

(D)  It inhibits GABA-activated ion-channel coupled receptor to increase the heart rate

Answer: (A)

13. Determine the correctness or otherwise of the following Assertion [a] and the Reason [r].

Assertion [a]: In multicellular organisms, cells of different lineages have different gene expression profiles.

Reason [r]: Alternative splicing is the only mechanism to generate protein diversity.

(A)  Both [a] and [r] are false

(B)  Both [a] and [r] are true and [r] is the correct reason for [a]

(C)  Both [a] and [r] are true but [r] is not the correct reason for [a]

(D)  [a] is true but [r] is false

Answer: (D)

14. Determine the correctness or otherwise of the following Assertion [a] and the Reason [r].

Assertion [a]: Chromosome mutations can change the structure of chromosomes.

Reason [r]: All chromosome mutations arise due to nondisjunction of chromosomes during mitosis or meiosis.

(A)  Both [a] and [r] are false

(B)  [a] is true but [r] is false

(C)  Both [a] and [r] are true and [r] is the correct reason for [a]

(D)  Both [a] and [r] are true but [r] is not the correct reason for [a]

Answer: (B)

15. C-value paradox refers to

(A)  the lack of correlation between genome size and genetic complexity of an organism

(B)  the presence of genetic sequences that propagate themselves within a genome

(C)  the coexistence of multiple alleles at a genetic locus

(D)  the concept that two or more genes may have the same function

Answer: (A)

16. Which one of the following drugs is NOT an immune checkpoint inhibitor?

(A)  Ipilimumab

(B)  Pembrolizumab

(C)  Nivolumab

(D)  Trastuzumab

Answer: (D)

17. Dendritic cells are involved in cross-presentation of antigens. Which of the following protein(s) is(are) required for cross-presentation?

(P) Basic leucine zipper ATF-like transcription factor 3 (BATF3)

(Q) Membrane associated ring-CH-type finger 1 (MARCH-1)

(R) Solute carrier family 10 member 1 (SLC10A1)

(S) Class II-associated invariant chain peptide (CLIP)

(A)  P only

(B)  P and R only

(C)  P, Q and R only

(D)  S only

Answer: (A)

18. Which one of the following is required for the development of B-cells in the bone marrow?

(A)  Stromal cells

(B)  Dendritic cells

(C)  Kupffer cells

(D)  NK cells

Answer: (A)

19. Which one of the following statements is TRUE about leghemoglobin?

(A)  It binds oxygen to protect nitrogenase

(B)  It binds hemoglobin to protect oxygenase

(C)  It binds oxygen to protect hydrogenase

(D)  It binds oxygen to protect oxygenase

Answer: (A)

20. The correct sequence of events during bacteriophage infection of a bacterial cell is

(A)  landing → attachment → tail contraction → penetration and unplugging → DNA ejection

(B)  attachment → landing → penetration and unplugging → tail contraction → DNA ejection

(C)  landing → tail contraction → attachment → DNA ejection → penetration and unplugging

(D)  attachment → tail contraction → landing → penetration and unplugging → DNA ejection

Answer: (A)

21. Intracellular proteins are targeted for proteolytic degradation in proteasomes upon conjugation with

(A)  ubiquitin

(B)  integrin

(C)  peptidase

(D)  calreticulin

Answer: (A)

22. In ELISA, which of the following enzymes are conjugated to antibodies for detection of the analyte?

(P) Alkaline phosphatase

(Q) Trypsinase

(R) Horseradish peroxidase

(S) Amylase

(A)  P and R

(B)  P and Q

(C)  Q and S

(D)  R and S

Answer: (A)

23. In hybridoma technology, which one of the following enzymes is absent in the myeloma cells that are used for monoclonal antibody production?

(A)  Hypoxanthine-guanine phosphoribosyltransferase

(B)  Alanine aminotransferase

(C)  Triose phosphate isomerase

(D)  Glycosyltransferase

Answer: (A)

24. Which of the following methods are used for detection of DNA and RNA, respectively?

(A)  Southern and Northern blotting

(B)  Southern and Western blotting

(C)  Northern and Southern blotting

(D)  Northern and Western blotting

Answer: (A)

25. Match the types of RNA in Group I with their corresponding function in Group II.

(A)  P-3, Q-4, R-2, S-1

(B)  P-3, Q-4, R-1, S-2

(C)  P-4, Q-3, R-2, S-1

(D)  P-2, Q-1, R-4, S-3

Answer: (A)

26. Which one of the following programs is used for finding distantly related (or remote) protein homologs?

(A)  BLASTN

(B)  BLASTX

(C)  PSI-BLAST

(D)  TBLASTX

Answer: (C)

27. Which one of the following is used for global alignment of two protein sequences?

(A)  Chou-Fasman method

(B)  Garnier-Osguthorpe-Robson (GOR) method

(C)  Needleman-Wunsch algorithm

(D)  Smith-Waterman algorithm

Answer: (C)

28. Which one of the following methods CANNOT be used to determine the secondary structure content of a protein?

(A)  Circular dichroism spectroscopy

(B)  Fourier transform infrared spectroscopy

(C)  Mass spectrometry

(D)  X-ray crystallography

Answer: (C)

29. Which one of the following plant growth regulators facilitate adventitious root formation?

(A)  Auxin

(B)  Zeatin

(C)  Dihydrozeatin

(D)  Kinetin

Answer: (A)

30. Fabry disease in humans is a X-linked disease. The probability (in percentage) for a phenotypically normal father and a carrier mother to have a son with Fabry disease is __________.

Answer: (25 to 25)

31. The value of  is _________.

Answer: (6 to 6)

32. A series (S) is given as

S = 1 + 3 + 5 + 7 + 9 + ……

The sum of the first 50 terms of S is __________.

Answer: (2500 to 2500)

33. Two fair six-sided dice are thrown. The probability of getting 12 as the product of

the numbers on the dice (rounded off to two decimal places) is __________.

Answer: (0.10 to 0.12)

34. If 73x = 216, the value of 7−x (rounded off to three decimal places) is ____________.

Answer: (0.166 to 0.168)

35. The distance between the two points of intersection of x2 + y = 7 and x + y = 7 (rounded off to two decimal places) is ____________.

Answer: (1.40 to 1.42)

Q.36 – Q.65 Carry TWO marks Each

36. Match the immune tolerance mechanisms in Group I with their respective outcomes in Group II.

(A)  P-3, Q-1, R-4, S-2

(B)  P-4, Q-3, R-1, S-2

(C)  P-3, Q-4, R-2, S-1

(D)  P-3, Q-2, R-4, S-1

Answer: (A)

37. Match the type of bacteria in Group I with their respective growth properties in Group II.

(A)  P-3, Q-4, R-1, S-2

(B)  P-2, Q-3, R-4, S-1

(C)  P-3, Q-1, R-2, S-4

(D)  P-4, Q-3, R-1, S-2

Answer: (A)

38. Match the virus in Group I with the type of genome it contains in Group II.

(A)  P-3, Q-4, R-1, S-2

(B)  P-2, Q-1, R-4, S-3

(C)  P-4, Q-3, R-2, S-1

(D)  P-1, Q-4, R-2, S-3

Answer: (A)

39. The event(s) that lead(s) to inactivation of tumor suppressor genes in cancer cells is(are)

(A)  gene amplification

(B)  promoter methylation

(C)  loss of heterozygosity

(D)  histone acetylation

Answer: (B, C)

40. Methylation of CpG islands near the promoter of a gene can inhibit transcription by

(A)  preventing RNA polymerase binding

(B)  facilitating repressor binding

(C)  facilitating heterochromatin formation

(D)  inducing euchromatin formation

Answer: (A, B, C)

41. Which of the following statement(s) is(are) TRUE about induced pluripotent stem cells?

(A)  They can self-renew

(B)  They require specific signals to maintain their stemness

(C)  They cannot be genetically manipulated

(D)  They can form organoids in vitro

Answer: (A, B, D)

42. Which of the following statement(s) is(are) TRUE about fluoroquinolone drugs?

(A)  They contain quinolone ring(s)

(B)  They inhibit RNA polymerase

(C)  They bind to bacterial topoisomerase

(D)  They bind to 23S rRNA within the 50S ribosome subunit

Answer: (A, C)

43. Which of the following is(are) plant protoplast fusogenic agent(s)?

(A)  Sodium nitrate

(B)  Polyvinyl alcohol

(C)  Polyethylene glycol

(D)  Bromoxynil

Answer: (A, B, C)

44. Direct DNA transfer method(s) used for plant genetic engineering is(are)

(A)  microparticle bombardment

(B)  electroporation

(C)  polyethylene glycol treatment

(D)  Agrobacterium-mediated transformation

Answer: (A, B, C)

45. Which of the following vector(s) is(are) used to clone a DNA fragment of size 220 kb?

(A)  Bacterial artificial chromosome

(B)  Yeast artificial chromosome

(C)  Cosmids

(D)  pUC19 plasmid

Answer: (A, B)

46. The following reaction represents biomass synthesis from hexadecane

C16H34 + 12.5O2 + 2.13NH3 → 10.6CH1.66O0.27N0.27 + 5.37CO2 + 11.4H2O

where CH1.66O0.27N0.27 represents the biomass. The value of respiratory quotient (rounded off to two decimal places) is __________.

Answer: (0.41 to 0.44)

47. Temperature of a reaction with an activation energy value of 15kcal .mol−1 is increased from 300 K to 310 K. If the value of the ideal gas constant (R) is 1.9872 cal.mol1.K1, the ratio of the reaction rate constants (k310/k300) (rounded off to two decimal places) is _______.

Answer: (2.24 to 2.26)

48. E. coli is cultivated in a chemostat operated at a dilution rate of 0.2 h1 values of biomass yield due to oxygen consumption and the steady state biomass concentration are 0.2g.g1 and 10 g.L1, respectively. The oxygen transfer rate (in g.L1.h1) is ________.

Answer: (10 to 10)

49. Aqueous two-phase extraction is used to recover α-amylase from a solution. A polypropylene glycol-dextran mixture is added and the solution separates into upper and lower phases. The partition coefficient is 4.0 and the ratio of upper to lower phase volume is 5.0. The enzyme recovery or yield (in percentage, rounded off to the nearest integer) is __________.

Answer: (94 to 96)

50. E. coli cultivated at 298 K uptakes an uncharged compound (A) by passive diffusion. The intracellular and extracellular concentrations of A are 0.001 M and 0.1 M, respectively. If the value of the ideal gas constant R is 1.9872 cal.mol1.K1, the free-energy change (in kcal.mol1) for this passive diffusion of A (rounded off to two decimal places) is _______.

Answer: (-2.74 to -2.71)

51. If there are three unrooted trees for four protein sequences, the number of rooted

trees for the same number of sequences is ___________.

Answer: (15 to 15)

52. The number of different possible ways of forming five intramolecular disulfide

bonds with ten cysteine residues of a protein is __________.

Answer: (945 to 945)

53. The following schematic diagram shows a chemostat with cell recycle

where F0 and Fr are the volumetric flow rates (in L.h1) of feed and recycle streams, respectively. X1, X0 and X are the cell concentrations (in g.L1) in the reactor, recycle-stream and product-stream, respectively. If  and X1 is 7.3 g.L1, the value of X (in g.L1, rounded off to one decimal place) is _________.

Answer: (4.6 to 4.9)

54. An enzyme (E) catalyzes the biochemical reaction A → B with kcat equal to 500 s1. If the initial reaction velocity (V0) is 10 μs1 at the total enzyme concentration [Et] of 30 nM and substrate concentration [A] of 40 μM, the value of Km(in μM) is _________.

Answer: (20 to 20)

55. DNA sample collected from an unidentified bacterial species (Y) contains 13% of adenine. The G+C content (in percentage) of Y is _________.

Answer: (74 to 74)

56. If 1000 bp of a double-helical DNA weighs 1 × 1018 gm and distance between two bp is 0.34 nm , the total amount of DNA (in mg, rounded off to one decimal place) required to stretch from Earth to Moon (assuming the distance between Earth and Moon to be 3,74,000 km) is __________.

Answer: (1.0 to 1.2)

57. A protein has three identical sites arranged at the vertices of an equilateral triangle. If one site is filled with a dye (donor), the measured quantum yield (ϕD) is 0.5. Filling one site with a donor dye and a second site with an acceptor dye results in ϕD of 0.25. The measured ϕD of one site filled with donor and the other two sites filled with acceptor dye (rounded off to three decimal places) is __________.

Answer: (0.165 to 0.168)

58. If  the value of |A4 + 3A2 – 5A + 6I| is _______.

Answer: (10551 to 10551)

59. If  the value of fʹ(x) at x = 0 is _________.

Answer: (-2 to -2)

60. If f(2) = 5 and (f(x))(f(x + 1)) = 3 for all real values of x, the value of f(10) is __________.

Answer: (5 to 5)

61. Ten playing cards numbered 1, 2, 3, …., 10 are placed face down on a table. One card is drawn at random, its number recorded, and then replaced face down. A card is drawn again at random. The probability that the number on the second draw is greater than the number on the first draw (rounded off to two decimal places) is __________.

Answer: (0.45 to 0.45)

62. The values of the consistency index ‘ K ’ and the flow behavior index ‘ n ’ of a dilatant fluid are 0.415 (in CGS units) and 1.23, respectively. The value of the apparent viscosity (in g.cm1.s1) of this fluid at a shear rate of 60 s1 (rounded off to the nearest integer) is ________.

Answer: (1 to 1)

63. An evaporator is insulated using glass wool material of 0.15 m thickness. The inner most surface and the outer surface of the insulation are at 700°C and 80°C, respectively. The mean thermal conductivity of the glass wool under these conditions is 0.29 W.m1.K1. The rate of heat loss (in W) through 1.2 m2 of the evaporator wall surface (rounded off to the nearest integer) is _________.

Answer: (1437 to 1439)

64. A proportional controller is used to control the temperature of an autoclave from 60°C to 130° If the proportional band setting of the controller is 25%, the proportional gain value is _________.

Answer: (4 to 4)

65. A dNTP master-mix is prepared by combining 40 μL of each 20 mM dNTP stock (dATP, dCTP, dGTP and dTTP). 4 μL of this dNTP master-mix is added to a PCR mix and the final volume is adjusted to 50 μ The concentration (in μM) of total dNTPs in the PCR mix is _________.

Answer: (1600 to 1600)

GATE Exam 2023 Biomedical Engineering (BM) Question Paper With Answer Key

GATE-2023

BM: Biomedical Engineering

General Aptitude

Q.1 – Q.5 Carry ONE mark each.

1. “I cannot support this proposal. My ________ will not permit it.”

(A)  conscious

(B)  consensus

(C)  conscience

(D)  consent

Answer: (C)

2. Courts : _______ : : Parliament : Legislature

(By word meaning)

(A)  Judiciary

(B)  Executive

(C)  Governmental

(D)  Legal

Answer: (A)

3. What is the smallest number with distinct digits whose digits add up to 45?

(A)  123555789

(B)  123457869

(C)  123456789

(D)  99999

Answer: (C)

4. In a class of 100 students,

(i) there are 30 students who neither like romantic movies nor comedy movies,

(ii) the number of students who like romantic movies is twice the number of students who like comedy movies, and

(iii) the number of students who like both romantic movies and comedy movies is 20.

How many students in the class like romantic movies?

(A)  40

(B)  20

(C)  60

(D)  30

Answer: (C)

5. How many rectangles are present in the given figure?

(A)  8

(B)  9

(C)  10

(D)  12

Answer: (C)

Q.6 – Q.10 Carry TWO marks Each

6. Forestland is a planet inhabited by different kinds of creatures. Among other creatures, it is populated by animals all of whom are ferocious. There are also creatures that have claws, and some that do not. All creatures that have claws are ferocious.

Based only on the information provided above, which one of the following options can be logically inferred with certainty?

(A)  All creatures with claws are animals.

(B)  Some creatures with claws are non-ferocious.

(C)  Some non-ferocious creatures have claws.

(D)  Some ferocious creatures are creatures with claws.

Answer: (D)

7. Which one of the following options represents the given graph?

(A)  f(x) = x22|x|

(B)  f(x) = x 2|x|

(C)  f(x) = |x|2x

(D)  f(x) = x 2x

Answer: (A)

8. Which one of the following options can be inferred from the given passage alone?

When I was a kid, I was partial to stories about other worlds and interplanetary travel. I used to imagine that I could just gaze off into space and be whisked to another planet.

[Excerpt from The Truth about Stories by T. King]

(A)  It is a child’s description of what he or she likes.

(B)  It is an adult’s memory of what he or she liked as a child.

(C)  The child in the passage read stories about interplanetary travel only in parts.

(D)  It teaches us that stories are good for children.

Answer: (B)

9. Out of 1000 individuals in a town, 100 unidentified individuals are covid positive. Due to lack of adequate covid-testing kits, the health authorities of the town devised a strategy to identify these covid-positive individuals. The strategy is to:

(i) Collect saliva samples from all 1000 individuals and randomly group them into sets of 5.

(ii) Mix the samples within each set and test the mixed sample for covid.

(iii) If the test done in (ii) gives a negative result, then declare all the 5 individuals to be covid negative.

(iv) If the test done in (ii) gives a positive result, then all the 5 individuals are separately tested for covid.

Given this strategy, no more than _______ testing kits will be required to identify all the 100 covid positive individuals irrespective of how they are grouped.

(A)  700

(B)  600

(C)  800

(D)  1000

Answer: (A)

10. A 100 cm × 32 cm rectangular sheet is folded 5 times. Each time the sheet is folded, the long edge aligns with its opposite side. Eventually, the folded sheet is a rectangle of dimensions 100 cm×1 cm.

The total number of creases visible when the sheet is unfolded is _______.

(A)  32

(B)  5

(C)  31

(D)  63

Answer: (C)

BM: Biomedical Engineering

Q.11 – Q.35 Carry ONE mark Each

11. What is the magnitude of the difference between the mean and the median of the dataset {1, 2, 3, 4, 6, 8}?

(A)  0

(B)  1

(C)  0.5

(D)  0.25

Answer: (C)

12. For a Binomial random variable X, E(X) and Var(X) are the expectation and variance, respectively. Which one of the following statements CANNOT be true?

(A)  E(X) = 20 and Var(X) = 16

(B)  E(X) = 6 and Var(X) = 5.4

(C)  E(X) = 10 and Var(X) = 15

(D)  E(X) = 64 and Var(X) = 12.8

Answer: (C)

13. is a 2 × 2 matrix. Which one of the following statements is TRUE?

(A)  Q is equal to its transpose.

(B)  Q is equal to its inverse.

(C)  Q is of full rank.

(D)  Q has linearly dependent columns.

Answer: (C)

14. Which one of the following vectors is an eigenvector corresponding to the eigenvalue = 1 for the matrix A?

Answer: (D)

15. For the function f(x, y) = ex cos (y), what is the value of  (x = 0, y = π/2)?

(A)  0

(B)  1

(C)  −1

(D)  eπ/2

Answer: (C)

16. For the circuit given below, choose the angular frequency ω0 (in rad/s) at which the voltage across the capacitor has maximum amplitude?

(A)  1000

(B)  100

(C)  1

(D)  0

Answer: (D)

17. A finite impulse response (FIR) filter has only two non-zero samples in its impulse response h[n], namely h[0] = h[1] = 1. The Discrete Time Fourier Transform (DTFT) of h[n] equals H(ejω), as a function of the normalized angular frequency ω. For the range |ω| ≤ π, |H(ejω)| is equal to ________.

(A)  2|cos(ω)|

(B)  2|sin(ω)|

(C)  2|cos(ω/2)|

(D)  2|sin(ω/2)|

Answer: (C)

18. An 8 bit successive approximation Analog to Digital Converter (ADC) has a clock frequency of 1 MHz. Assume that the start conversion and end conversion signals occupy one clock cycle each. Among the following options, what is the maximum frequency that this ADC can sample without aliasing?

(A)  0.9 kHz

(B)  9.9 kHz

(C)  49.9 kHz

(D)  99.9 kHz

Answer: (C)

19. In the following circuit with an ideal operational amplifier, the capacitance of the parallel plate capacitor C is given by the expression C = (ϵA/x), where ϵ is the dielectric constant of the medium between the capacitor plates, and A is the cross-sectional area. In the above relation, x is the separation between the two parallel plates, given by x = x0 + kt, where t is time; x0 and k are positive non-zero constants. If the input voltage vi is constant, then the output voltage v0 is given by _______.

(A)  RviCk/x

(B)  RviC/kx

(C)  vik/RCx

(D)  0

Answer: (A)

20. Which one of the following techniques makes use of Korotkoff sounds?

(A)  Sphygmomanometry

(B)  Audiometry

(C)  Spirometry

(D)  Tonometry

Answer: (A)

21. The pulmonary artery and pulmonary vein ________.

(A)  carry deoxygenated blood and oxygenated blood, respectively

(B)  carry oxygenated blood and deoxygenated blood, respectively

(C)  both carry oxygenated blood

(D)  both carry deoxygenated blood

Answer: (A)

22. Which one of the following bridges CANNOT be used for measuring inductance?

(A)  Schering Bridge

(B)  Maxwell Wien Bridge

(C)  Hay Bridge

(D)  Series Owen Bridge

Answer: (A)

23. A polychromatic beam of X-Rays has an energy spectrum as shown in Figure P below. Which of the following graphs (in the options A to D) depicts the energy spectrum after passing through a human body? In each figure, the horizontal axis represents Energy in keV and the vertical axis represents Relative X-ray Intensity.

Answer: (B)

24. M, L and T correspond to dimensions representing mass, length and time, respectively. What is the dimension of viscosity?

(A)  M1L2T1

(B)  M1L1T1

(C)  M1L1T1

(D)  M1L2T2

Answer: (B)

25. Choose the option that has the biomaterials arranged in order of decreasing tensile strength.

(PMMA : poly-methyl-methacrylate)

(A)  Human compact bone > PMMA bone cement > Polymer foams > Graphite-epoxy

(B)  Human compact bone > Graphite-epoxy > PMMA bone cement > Polymer foams

(C)  Graphite-epoxy > Human compact bone > PMMA bone cement > Polymer foams

(D)  PMMA bone cement > Human compact bone > Polymer foams > Graphite-epoxy

Answer: (C)

26. A causal, discrete time system is described by the difference equation

y[n] = 0.5y[n – 1] + x[n], for all n,

where 𝑦[𝑛] denotes the output sequence and x[n] denotes the input sequence. Which of the following statements is/are TRUE?

(A)  The system has an impulse response described by 0.5n u[−n] where u[n] is the unit step sequence.

(B)  The system is stable in the bounded input, bounded output sense.

(C)  The system has an infinite number of non-zero samples in its impulse response.

(D)  The system has a finite number of non-zero samples in its impulse response.

Answer: (B, C)

27. Which of the following constituents is/are NOT normally found in serum obtained from human blood?

(A)  Platelets

(B)  Albumin

(C)  Glucose

(D)  Fibrinogen

Answer: (A, D)

28. Q, R, S are Boolean variables and ⊕ is the XOR operator. Select the CORRECT option(s).

(A)  (Q ⊕ R) ⊕ S = Q ⊕ (R ⊕ S)

(B)  (Q ⊕ R) ⊕ S = 0 when any two of the Boolean variables (Q, R, S) are 0 and the third variable is 1

(C)  (Q ⊕ R) ⊕ S = 1 when Q = R = S = 1

(D)  ((Q ⊕ R) ⊕(R ⊕ S) )⊕(Q ⊕ S) = 1

Answer: (A, C)

29. In the human pancreas, which cell types secrete insulin and glucagon?

(A)  Alpha cells and delta cells, respectively

(B)  Beta cells and delta cells, respectively

(C)  Alpha cells and beta cells, respectively

(D)  Beta cells and alpha cells, respectively

Answer: (D)

30. In the following circuit, the switch S is open for t < 0 and closed for t ≥ 0. What is the steady state voltage (in Volts) across the capacitor when the switch is closed? (Round off the answer to one decimal place.)

Answer: (6.9 to 7.1)

31. For a tissue with Young’s modulus of 3.6 kPa and Poisson’s ratio of 0.2, what is the value of its shear modulus (in kPa)? (Round off the answer to one decimal place.)

Answer: (1.5 to 1.5)

32. In the circuit shown below, the amplitudes of the voltage across the resistor and the capacitor are equal. What is the value of the angular frequency ω0 (in rad/s)? (Round off the answer to one decimal place.)

Answer: (9.9 to 10.1)

33. A continuous time, band limited signal x(t) has its Fourier transform described by:

The signal is uniformly sampled at a sampling rate of 600 Hz. The Fourier transform of the sampled signal is XS(f). What is the value of   (Round off the answer to one decimal place.)

Answer: (1.9 to 2.1)

34. At time t, the cardiac dipole is oriented at – 45° (minus forty five degrees) to the horizontal axis. The magnitude of the dipole is 3 mV. Assuming Einthoven frontal plane configuration, what is the magnitude (in mV) of the electrical signal in lead II? (Round off the answer to two decimal places.)

Answer: (2.8 to 3.0)

35. A 5 MHz ultrasound transducer is being used to measure the velocity of blood. When the transducer is placed at an angle of 45° to the direction of blood flow, a frequency shift of 200 Hz is observed in the echo. Assume that the velocity of sound is 1500 m/s. What is the velocity (in cm/s) of the blood flow? (Round off the answer to one decimal place.)

Answer: (4.1 to 4.3)

Q.36 – Q.65 Carry TWO marks Each

36. The time-dependent growth of a bacterial population is governed by the equation

where x is the population size at time t. The initial population size is x0 = 100 at t = 0. As t → ∞, the population size of bacteria asymptotically approaches __________.

(A)  150

(B)  200

(C)  300

(D)  500

Answer: (B)

37. A 20 mV DC signal has been superimposed with a 10 mV RMS band-limited Gaussian noise with a flat spectrum upto 5 kHz. If an integrating voltmeter is used to measure this DC signal, what is the minimum averaging time (in seconds) required to yield a 99 % accurate result with 95 % certainty?

(A)  0.1

(B)  1.0

(C)  5.0

(D)  10.0

Answer: (B)

38. In the circuit below, the two DC voltage sources have voltages of value V1 and V2. The expression for the power dissipated in the 60 kΩ resistor is proportional to _________.

(A)  (V1 + V2)2

(B)  (3V1 + V2)2

(C)  (2V1 + V2)2

(D)  (V1 + 2V2)2

Answer: (C)

39. The Laplace transform of x­1(t) = et u(t) is X1(s), where u(t) is the unit step function. The Laplace transform of x2(t) = etu(−t) is X2(s). Which one of the following statements is TRUE?

(A)  The region of convergence of X­1(s) is Re(s) > 0.

(B)  The region of convergence of X2(s) is confined to the left  half-plane of s.

(C)  The region of convergence of X1(s) is confined to the right half-plane of 𝑠.

(D)  The imaginary axis in the s-plane is included in both the region of convergence of X­1(s) and the region of convergence of X2(s).

Answer: (D)

40. A circular disc of radius R (in cm) has a uniform absorption coefficient of 1 cm−1. Consider a single ray passing through the disc in the plane of the disc. The shortest distance from the center of the disc to the ray is 𝑡 (in cm). If Ii is the intensity of the incident ray and I0 is the intensity of the transmitted ray, then log(Ii/I0) is given by _________.

(A) 

(B)  2R

(C)  1

(D) 

Answer: (A)

41. The free induction decay (FID) in the MRI of an object can be approximated as

Here Gx and Gy are pulses of identical period and are in-phase. By changing the amplitude of the pulses, one can obtain the two dimensional Fourier transform of the object __________.

(A)  over radial lines in (Kx, Ky)

(B)  over a parabolic contour in (Kx, Ky) space

(C)  along Ky only

(D)  along Kx only

Answer: (A)

42. In the circuit shown below, it is observed that the amplitude of the voltage across the resistor is the same as the amplitude of the source voltage. What is the angular frequency ω0 (in rad/s)?

(A)  104

(B)  103

(C)  103π

(D)  104π

Answer: (A)

43. In a biomaterial, formation of hydrogen bonds on alcoholic groups will lead to a _________.

(A)  shift in the infra-red peak around 1700 cm–1

(B)  shift in the infra-red peak around 2800 cm–1

(C)  broadening of the infra-red peak around 3500 cm–1

(D)  disappearance of the infra-red peak around 1700 cm–1

Answer: (C)

44. In the circuit shown below, the input voltage is sinusoidal and 2.5 V peak to peak. The capacitors are 20 μF each. Assume that the knee voltage of the diodes is 0 V and RL is very large (almost infinite). Which one of the following options is closest to the peak to peak voltage across RL, after a large number of cycles?

(A)  1.25 V

(B)  2.50 V

(C)  5.00 V

(D)  10.0 V

Answer: (C)

45. An ultrasound plane wave of amplitude P0 hits the semi-infinite boundary of two media having acoustic impedances Z1 and Z2. The sum of the amplitudes of the reflected and the incident waves at the boundary is equal to ________.

Answer: (A)

46. In the circuit given below, what should be the value of the resistance R for maximum dissipation of power in R?

(A)  1.2 kΩ

(B)  2.2 kΩ

(C)  3.2 kΩ

(D)  4.2 kΩ

Answer: (B)

47. Two sequences x1[n] and x2[n] are described as follows:

x1[0] = x2[0] = 1

1[1] = x2[2] = 2

x1[2] = x2[1] = 1

x1[n] = x2[n] = 0 for all n < 0 and n > 2

If x[n] is obtained by convolving x1[n] with x2[n], which of the following equations is/are TRUE?

(A)  x[2] = x[3]

(B)  x[1] = 2

(C)  x[4] = 3

(D)  x[2] = 5

Answer: (A, D)

48. The function  of a complex variable Z is integrated on a closed contour in an anti-clockwise direction. For which of the following contours, does this integral have a non-zero value?

(A)  |Z – 2| = 0.01

(B)  |Z – 1| = 0.1

(C)  |Z – 3| = 5

(D)  |Z| = 2

Answer: (B, C, D)

49. The continuous time signal x(t) is described by

If y(t) represents x(t) convolved with itself, which of the following statements is/are TRUE?

(A)  y(t) = 0 for all t < 0

(B)  y(t) = 0 for all t > 1

(C)  y(t) = 0 for all t > 3

(D) 

Answer: (A, C, D)

50. Which of the following relations is/are CORRECT in terms of various lung volume measurements?

(A)  Vital capacity minus expiratory reserve volume equals inspiratory capacity.

(B)  Vital capacity plus expiratory reserve volume equals inspiratory capacity.

(C)  Total lung capacity equals the sum of inspiratory capacity and functional residual capacity.

(D)  Functional residual capacity is the difference between expiratory reserve volume and residual volume.

Answer: (A, C)

51. Assuming the operational amplifier in the circuit shown below to be ideal, which of the following properties hold(s) TRUE for the circuit?

(A)  It acts as a voltage follower.

(B)  It is bistable.

(C)  It is astable.

(D)  The output voltage is at saturation.

Answer: (B, D)

52. A water insoluble polymeric biomaterial can become water soluble in vivo by which of the following mechanisms?

(A)  Cleavage of crosslinks between water soluble polymer chains

(B)  Cleavage of side chains leading to formation of non-polar groups

(C)  Cleavage of backbone linkages between polymer repeat units leading to the formation of polar groups

(D)  Enzymatic degradation of crosslinks between water soluble polymer chains

Answer: (A, C, D)

53. For the function f(x) = x4 – x2, which of the following statements is/are TRUE?

(A)  The function is symmetric about x = 0.

(B)  The minimum value of the function is –0.5.

(C)  The function has two minima.

(D)  The function is an odd function.

Answer: (A, C)

54. A system is described by the following differential equation

where time (t) is in seconds. If x(t) is the unit step input applied at t = 0 s to this system, the magnitude of the output at t = 1 s is _______. (Round off the answer to two decimal places.)

Answer: (5.80 to 6.00)

55. The resistance of a thermistor is 1 kΩ at 25 °C and 500 Ω at 50 °C. Find the temperature coefficient of resistance (in units of °C –1) at 35 °C. (Round off the answer to three decimal places.)

Answer: (-0.03 to -0.025)

56. A normally incident X-ray of energy 140 keV passes through a tissue phantom and is detected by the detector as shown in the figure below. The phantom consists of tissue P with an absorption coefficient of 1 cm–1 and a thickness of 1 cm, and tissue Q with an absorption coefficient of 10 cm–1 and a thickness of 2 cm. Calculate the intensity (in μeV) detected by the detector. (Round off the answer to one decimal place.)

Answer: (106.0 to 106.4)

57. A two-dimensional square plate (20 mm sides) contains a homogeneous circular inclusion of 5 mm diameter in it. A parallel beam of X-rays (beam width 30 mm) is used in a tomography system to determine the location of the inclusion. What is the minimum number of views required to approximately determine the location of the inclusion?

Answer: (2 to 2)

58. Calculate the reciprocal of the coefficient of z3 in the Taylor series expansion of the function f(z) = sin(z) around z = 0. (Provide the answer as an integer.)

Answer: (-6 to -6)

59. In a cell viability experiment, 10,000 cells were cultured in the absence and presence of a compound Q for 24 h. The absorbance of a dye associated with cellular metabolic activity was measured at a wavelength of 570 nm at 24 h. The measured absorbances were 0.8 a.u. in the absence of the compound Q, and 0.5 a.u. in its presence.

If the dye gives an absorbance (at 570 nm) of 0.1 a.u. in the absence of cells, what is the percentage cell growth inhibition caused by the compound Q? (Round off the answer to one decimal place.)

Answer: (42.0 to 44.0)

60. The volume percentage of oxygen in inspired air is 20 % and that of expired air is 16 %. A person is breathing at a rate of 12 breaths per minute. Each breath is 500 ml in volume. The cardiac output is 5 liters per minute.

Assuming ideal, healthy lung and cardiac conditions, what is the change in percentage of oxygen in blood over 1 minute? (Round off the answer to one decimal place.)

Answer: (4.8 to 4.8)

61. The intracellular and extracellular concentrations (in mM) of three important ions are given in the table below. The relative permeability of the cell membrane to each ion is provided. Universal gas constant is 8.31 J/(mol.K) and Faraday’s constant is 96500 C/mol.

What is the absolute value of the resting membrane potential (in mV) across the cell membrane at 27 °C? (Round off the answer to one decimal place.)

Answer: (82.6 to 86.0)

62. A metallic strain gauge with negligible piezoresistive effect is subjected to a strain of 50 × 10−6. For the metal, Young’s Modulus = 80 GPa and Poisson’s Ratio = 0.42. What is the change in resistance (in mΩ), if the unstrained resistance of the strain gauge is 200 Ω ? (Round off the answer to one decimal place.)

Answer: (18.4 to 18.4)

63. Consider the total hip joint prosthesis as shown in the figure. The geometric parameters of the prosthesis are such that L1 = 40 mm, L2 = 60 mm, θ1 = 45°, θ2 = 90°. Assume that, when standing symmetrically on both feet, a joint reaction force of 400 N is acting vertically at the femoral head (point A) due to the body weight of the subject. Calculate the magnitude of the moment (in Nm) about point C. (Round off the answer to one decimal place.)

Answer: (11.0 to 12.0)

64. A Wheatstone bridge strain gauge transducer is constructed on a diaphragm in such a way that when a force is applied on the diaphragm, the resistors R1 and R4 will be in compression, and the resistors R2 and R3 will be in tension.

The bridge excitation voltage (Ein) is 10 Volts. If all the resistors have a resistance of 200 Ω in the absence of any force, and each resistance changes by 20 Ω upon application of a force, what is the output voltage Vout (in Volts) from the Wheatstone bridge? (Round off your answer to the nearest integer.)

Answer: (-1 to -1 OR 1 to 1)

65. A stone is thrown from an elevation of 2 m above ground level, at an angle of 30° to the horizontal axis. If the stone hits the ground at a horizontal distance of 6 m from the point of release, at what speed (in m/s) was the stone thrown? Use g = 10 m/s2 and assume that there is no air resistance. (Round off your answer to one decimal place.)

Answer: (6.3 to 6.9)

GATE Exam 2023 Architecture and Planning (AR) Question Paper With Answer Key

GATE-2023

AR: Architecture and Planning

General Aptitude.

Q.1 – Q.5 Carry ONE mark each.

1. He did not manage to fix the car himself, so he _______ in the garage.

(A)  got it fixed

(B)  getting it fixed

(C)  gets fixed

(D)  got fixed

Answer: (A)

2. Planting : Seed : : Raising : _____

(By word meaning)

(A)  Child

(B)  Temperature

(C)  Height

(D)  Lift

Answer: (A)

3. A certain country has 504 universities and 25951 colleges. These are categorised into Grades I, II, and III as shown in the given pie charts.

What is the percentage, correct to one decimal place, of higher education institutions (colleges and universities) that fall into Grade III?

(A)  22.7

(B)  23.7

(C)  15.0

(D)  66.8

Answer: (A)

4. The minute-hand and second-hand of a clock cross each other _______ times between 09:15:00 AM and 09:45:00 AM on a day.

(A)  30

(B)  15

(C)  29

(D)  31

Answer: (A)

5. The symbols  are to be filled, one in each box, as shown below.

The rules for filling in the four symbols are as follows.

(1) Every row and every column must contain each of the four symbols.

(2) Every 2 × 2 square delineated by bold lines must contain each of the four symbols.

Which symbol will occupy the box marked with ‘?’ in the partially filled figure?

Answer: (B)

Q.6 – Q.10 Carry TWO marks Each

6. In a recently held parent-teacher meeting, the teachers had very few complaints about Ravi. After all, Ravi was a hardworking and kind student. Incidentally, almost all of Ravi’s friends at school were hardworking and kind too. But the teachers drew attention to Ravi’s complete lack of interest in sports. The teachers believed that, along with some of his friends who showed similar disinterest in sports, Ravi needed to engage in some sports for his overall development.

Based only on the information provided above, which one of the following statements can be logically inferred with certainty?

(A)  All of Ravi’s friends are hardworking and kind.

(B)  No one who is not a friend of Ravi is hardworking and kind.

(C)  None of Ravi’s friends are interested in sports.

(D)  Some of Ravi’s friends are hardworking and kind.

Answer: (D)

7. Consider the following inequalities

p2 – 4q < 4

3p + 2q < 6

where p and q are positive integers.

The value of (p + q) is ______.

(A)  2

(B)  1

(C)  3

(D)  4

Answer: (A)

8. Which one of the sentence sequences in the given options creates a coherent narrative?

(i) I could not bring myself to knock.

(ii) There was a murmur of unfamiliar voices coming from the big drawing room and the door was firmly shut.

(iii) The passage was dark for a bit, but then it suddenly opened into a bright kitchen.

(iv) I decided I would rather wander down the passage.

(A)  (iv), (i), (iii), (ii)

(B)  (iii), (i), (ii), (iv)

(C)  (ii), (i), (iv), (iii)

(D)  (i), (iii), (ii), (iv)

Answer: (C)

9. How many pairs of sets (S,T) are possible among the subsets of {1, 2, 3, 4, 5, 6} that satisfy the condition that S is a subset of T?

(A)  729

(B)  728

(C)  665

(D)  664

Answer: (A)

10. An opaque pyramid (shown below), with a square base and isosceles faces, is suspended in the path of a parallel beam of light, such that its shadow is cast on a screen oriented perpendicular to the direction of the light beam. The pyramid can be reoriented in any direction within the light beam. Under these conditions, which one of the shadows P, Q, R, and S is NOT possible?

(A)  P

(B)  Q

(C)  R

(D)  S

Answer: (B)

AR: Architecture and Planning

PART A: Common FOR ALL CANDIDATES

Q.11 – Q.28 Carry ONE mark Each

11. The triad of secondary colours in the additive colour system is _________.

(A)  Cyan, Magenta, Yellow

(B)  Red, Green, Blue

(C)  Purple, Green, Orange

(D)  Magenta, Blue, Yellow

Answer: (A)

12. The criterion that is specifically mentioned in Special Conditions of Contract (SCC) is__________.

(A)  Scope and performance of the work

(B)  Site mobilization advance

(C)  Labour regulation

(D)  Arbitration and law

Answer: (MTA)

13. The command employed in AutoCAD® to create a mesh from a line or curve that is swept along a straight path (as shown in the figure below) is _________.

(A)  TABSURF

(B)  REVSURF

(C)  RULESURF

(D)  EDGESURF

Answer: (A)

14. As per the Burra Charter (2013) ‘Cultural Significance’ means __________ for past, present or future generations.

(A)  historic, aesthetic, scientific, social or spiritual value

(B)  archaeological, architectural, environmental, cultural value

(C)  natural, cultural, mixed, intangible heritage

(D)  heritage value, authenticity, integrity

Answer: (A)

15. As per URDPFI (2015), the density range (in persons per Hectare) suggested for overall planning approach for small towns in hill areas is ________.

(A)  20-30

(B)  45-75

(C)  100-125

(D)  125-150

Answer: (B)

16. In ecology, the term ‘niche’ refers to ______.

(A)  the ways in which species interact with biotic and abiotic factors of the environment

(B)  only the abiotic factors such as temperature and rainfall

(C)  the gradient change of physiochemical characteristics between two ecosystems

(D)  the zone of junction or a transition area between two biomes

Answer: (A)

17. Lowry’s model of Metropolis (1964) includes two _____________ spatial interaction models.

(A)  Singly constrained

(B)  Doubly constrained

(C)  Unconstraine

(D)  Triply constrained

Answer: (A)

18. Select the method(s) that involve(s) a pairwise comparison matrix for quantifying the weights of decision criteria.

(A)  Analytical hierarchy process

(B)  Exploratory factor analysis

(C)  Latent class analysis

(D)  Multiple linear regression

Answer: (A)

19. Select the micro-organism which is NOT an enteric pathogen.

(A)  Staphylococcus aureus

(B)  Vibrio cholerae

(C)  Escherichia coli

(D)  Salmonella typhi

Answer: (A)

20. Select the publication by Ministry of Statistics and Programme Implementation (MoSPI) related to Environmental Accounts as per UN-SEEA framework.

(A)  EnviStats India 2022

(B)  Energy Conservation Building Code 2017

(C)  Eco Niwas Samhita 2018

(D)  Climate Change 2022: Impacts, Adaptation and Vulnerability

Answer: (A)

21. Ebenezer Howard suggested the maximum population of ‘Garden City’ as________ persons.

(A)  10,000

(B)  22,000

(C)  32,000

(D)  58,000

Answer: (C)

22. In eighteenth century English gardens, __________was used to eliminate visual boundaries between the garden and the landscape.

(A)  Stroll garden

(B)  Sunken fence

(C)  Topiary

(D)  Qanat

Answer: (B)

23. The figure below shows the spatial arrangement of rooms in a building with access from the exterior, marked as ‘entry’. Identify the appropriate diagram showing the access to rooms starting from the entry.

Answer: (B)

24. In high-rise buildings, the method adopted to prevent ingress of smoke in an enclosed fire staircase is _________.

(A)  Polarization

(B)  Pressurization

(C)  Perpetuation

(D)  Fumigation

Answer: (B)

25. Select the Act which stipulates prohibited area of 100 m around centrally protected monuments in India.

(A)  The Antiquities and Art Treasures Act, 1972

(B)  The AMASR (Amendment and Validation) Act, 2010

(C)  Urban Land (Ceiling and Regulation) Act, 1976

(D)  Environment Protection Act, 1986

Answer: (B)

26. Select the option(s) which include(s) a pair of ‘Gestalt’ principles.

(A)  Proximity and Similarity

(B)  Continuity and Closure

(C)  Grain and Texture

(D)  Scale and Proportion

Answer: (A, B)

27. Select the option(s) which is/are NOT considered as primary air pollutant(s).

(A)  Suspended particulate matter

(B)  Oxides of nitrogen

(C)  Volatile organic compounds

(D)  Peroxyacetyl Nitrate

Answer: (D)

28. Select the Biosphere Reserve(s) in India which is/are listed in the ‘Man and the Biosphere’ program of UNESCO.

(A)  Sunderban

(B)  Sena Oura

(C)  Majang Forest

(D)  Gulf of Mannar

Answer: (A, D)

Q.29 – Q.49 Carry TWO marks Each

29. Match the buildings in Group I with their dominant spatial pattern in Group II.

(A)  P-1, Q-3, R-2, S-5

(B)  P-5, Q-3, R-1, S-4

(C)  P-3, Q-4, R-1, S-5

(D)  P-1, Q-4, R-2, S-3

Answer: (A)

30. Match the Parts of Residential Buildings in Group-I with their respective minimum width (in m) in Group-II as per the National Building Code 2016

(A)  P-2, Q-1, R-5, S-3

(B)  P-5, Q-3, R-4, S-1

(C)  P-2, Q-3, R-5, S-4

(D)  P-5, Q-1, R-4, S-3

Answer: (D)

31. Match the following City Planning concepts in Group-I with their proponents in Group-II

(A)  P-2, Q-4, R-5, S-3

(B)  P-3, Q-4, R-5, S-2

(C)  P-3, Q-2, R-1, S-4

(D)  P-1, Q-5, R-3, S-2

Answer: (B)

32. With reference to planning and design of housing, identify the correct statements.

(P) Gross population density is higher than net population density

(Q) Gross population density is lower than net population density

(R) Net population density is directly proportional to area of the plot

(S) Net population density is inversely proportional to area of the plot

(A)  Both Q and S are correct

(B)  Both Q and R are correct

(C)  Both P and R are correct

(D)  Both P and S are correct

Answer: (A)

33. Match the Mission in Group I with their objectives in Group II.

(A)  P-3, Q-5, R-4, S-1

(B)  P-2, Q-5, R-4, S-3

(C)  P-3, Q-4, R-5, S-1

(D)  P-2, Q-5, R-3, S-4

Answer: (A)

34. Select the option(s) that is/are listed among the Sustainable Development Goals as articulated by the United Nations.

(A)  Globalization and Free Trade

(B)  Sustainable Cities and Communities

(C)  Protection of Indigenous Culture and Architecture

(D)  Good Health and Well-being

Answer: (B, D)

35. Select the statement(s) that are TRUE regarding ‘Building Security Services’.

(A)  ‘Radio Frequency Identification Device’ is used for electronic access control system.

(B)  ‘Magnetic Loop Detector’ is used for fire detection system.

(C)  ‘Infrared Sensor’ is used in public broadcasting system.

(D)  ‘Iris Scan’ is a type of biometric access control system.

Answer: (A, D)

36. Select the statement(s) that are TRUE regarding ‘Quality and Cost-Based Selection (QCBS)’ system for tendering.

(A)  Financial bid is opened before technical bid.

(B)  Earnest Money Deposit (EMD) is submitted before the opening of technical bid.

(C)  Technically qualified bidder with lowest financial bid is always awarded the job.

(D)  A composite scoring system considering both the financial and technical bids is adopted for awarding the job.

Answer: (B, D)

37. Design of septic tank requires consideration of space for the following item(s).

(A)  Settling of incoming sewage

(B)  Storage of digested sludge

(C)  Installation of liner to allow seepage of effluent

(D)  Digestion of the settled sludge

Answer: (A, B, D)

38. Select the place(s) which have adopted the “star pattern” of the French Garden in the design of its /their urban form(s).

(A)  Versailles

(B)  Washington D.C.

(C)  Islamabad

(D)  Jaipur

Answer: (A, B)

39. Select the parameter(s) required for determining peak rates of runoff using the Rational formula.

(A)  Intensity of rainfall

(B)  Coefficient of runoff

(C)  Velocity of flow

(D)  Hydraulic mean depth of flow

Answer: (A, B)

40. As per Solid Waste Management Rules 2016 (Ministry of Environment, Forest and Climate, Govt. of India) select the correct statement(s).

(A)  “dry waste” means waste other than bio-degradable waste and inert street sweepings.

(B)  “combustible waste” means biodegradable, recyclable, reusable, hazardous solid waste having maximum calorific value of 800 kcal/kg.

(C)  “domestic hazardous waste” includes expired medicine, CFL bulbs, discarded paint drums.

(D)  “biodegradable waste” means any inorganic material that cannot be degraded by micro-organisms into simpler stable compounds.

Answer: (A, C)

41. Select the correct statement(s) from the following.

(A)  Introduction of automobiles led to urban sprawl.

(B)  Compact cities show relatively higher carbon emissions.

(C)  Land use and transportation planning is inter-dependent on each other.

(D)  Addition of a transport mode in an urban area does not change accessibility.

Answer: (A, C)

42. Choose the correct statement(s) with regard to composting.

(A)  It produces natural soil amendment and enhances the effectiveness of fertilizer.

(B)  Warm temperature of tropical regions is least suitable for composting.

(C)  Composting is an aerobic thermophilic process.

(D)  Windrow composting and in-vessel composting are two of the common composting methods.

Answer: (A, C, D)

43. Select the item(s) that are NOT stipulated as obligatory function(s) of the urban local bodies as per the 12th Schedule of the Indian Constitution.

(A)  Urban poverty alleviation

(B)  Promotion of cultural, educational and aesthetic aspects

(C)  Special measures for disaster mitigation

(D)  Prevention of cruelty to animals

Answer: (C)

44. The annual precipitation recorded in a town is 400 mm. Rainwater is being collected from the flat roof of a building, and then treated to potable standards, and stored. Water loss due to evaporation, transmission and treatment is 40 percent of the total harvested volume. The roof area is 500 sq.m. There are 3 occupants, with average daily water demand as 200 lpcd. The stored rainwater will be adequate for the household’s daily use for ________ days [in integer].

Answer: (200 to 200)

45. A primary school is having 8 class rooms, each having internal dimensions of 15m × 10m × 4m (height). Only the internal walls of all the class rooms are proposed to be painted. Assume a deduction of 10% internal wall area due to doors, windows etc. The specification suggests two coats of paint application. The spreading rates of the selected paint during base coat and finish coat are 4.5 l/sq.m and 2.5 l/sq.m respectively. The amount of paint (in liters) required for the job will be _______ [in integer].

Answer: (10080 to 10080)

46. A construction project consists of four activities. The quantity of work, manpower requirement, and the productivity of the activities are listed in the table below. The interrelationship between the activities are also mentioned in the table. The construction project will start on January 29. Assume no holidays for the entire duration of the project. The project will finish on February _____ [mention date in number].

Answer: (17 to 17)

47. For a privately developed group housing project, the ratio of total number of dwelling units of HIG: MIG: LIG is 3:2:1. The proposed average size of HIG, MIG and LIG units in sq.m are 100, 60 and 30 respectively. The ratio of the total built up area between (MIG + LIG) to HIG will be 1: ______ [in integer].

Answer: (2 to 2)

48. The surface development of a three dimensional object is shown in the figure below. The dotted lines indicate the folds. The dimensions given in the figure are in centimeter. The volume of the three-dimensional object (in cu.cm) is_________ [ rounded off to one decimal place].

Answer: (25.5 to 25.5)

49. A residential housing project is designed in a plot measuring 1 hectare. The car parking area is equally distributed between the ground floor and the basement. Considering the data given below, the number of cars accommodated in the basement will be________ [in integer].

Data:

FAR consumed = 2.0 Car parking area is exempted from built up area for FAR calculations. One car parking to be given for each 100 sq.m of built up area. Area required for accommodating each car in ground floor = 15 sq.m Area required for accommodating each car in basement = 25 sq.m

Answer: (75 to 75)

PART B1: FOR Architecture CANDIDATES ONLY

Q.50 – Q.56 Carry ONE mark Each

50. As per the CPWD Specifications (2019), the material used for cleaning marble flooring after polishing is__________.

(A)  Oxalic Acid

(B)  Caustic Soda

(C)  Bleaching Power

(D)  White Cement

Answer: (A)

51. The proportion of the sides of a traditional Japanese tatami mat is _______.

(A)  1 : 1.414

(B)  1 : 1.5

(C)  1 : 2

(D)  1 : 1.618

Answer: (C)

52. As per IS:4954 – 1964, the acceptable noise level (in dB) for urban residential areas is____________.

(A)  35-45

(B)  65-75

(C)  20-30

(D)  15-25

Answer: (A)

53. Identify the Indian tribe that is associated with the vernacular dwelling illustrated in the image below.

(A)  Bhotia, Uttarakhand

(B)  Toda, Tamil Nadu

(C)  Naga, Nagaland

(D)  Kutia Kondh, Odisha

Answer: (B)

54. Thermal diffusivity of a wall is influenced by the choice of building material. Identify the statement(s) that is/are correct.

(A)  Thermal diffusivity is inversely proportional to thermal conductivity.

(B)  Increase in specific heat capacity increases the thermal diffusivity.

(C)  Materials with low thermal diffusivity have a high amplitude dampening effect.

(D)  Thermal diffusivity is inversely proportional to the density of material.

Answer: (C, D)

55. Select the statement(s) which are NOT correct with respect to burnt clay bricks.

(A)  Lime (<10% of clay) in carbonated form lowers the fusion point of bricks.

(B)  Magnesia (>1% of clay) imparts red colour to the bricks.

(C)  Iron Pyrites tend to oxidize and decompose the brick during burning.

(D)  Alkalis (alkaline salts) when present in excess (>10% of clay) decrease the probability of efflorescence.

Answer: (B, D)

56. Select the example(s) of Art Nouveau architecture.

(A)  Basilica of the Sagrada Familia, Barcelona

(B)  Chrysler Building, New York

(C)  Eiffel Tower, Paris

(D)  Mackintosh Building of the Glasgow School of Art, Glasgow

Answer: (A, D)

Q.57 – Q.65 Carry TWO marks Each

57. Match the buildings in Group I with their architectural feature in Group II.

(A)  P-2, Q-1, R-3, S-5

(B)  P-1, Q-2, R-4, S-3

(C)  P-3, Q-1, R-5, S-2

(D)  P-2, Q-3, R-4, S-5

Answer: (A)

58. Match the architects in Group I with their key architectural ideas in Group II.

(A)  P-2, Q-5, R-1, S-3

(B)  P-4, Q-1, R-3, S-5

(C)  P-2, Q-1, R-5, S-3

(D)  P-4, Q-5, R-1, S-2

Answer: (A)

59. Match the pump types in Group-I with their key components in Group-II.

(A)  P-2, Q-1, R-3, S-5

(B)  P-1, Q-2, R-5, S-3

(C)  P-2, Q-5, R-4, S-1

(D)  P-1, Q-2, R-3, S-4

Answer: (A)

60. Match the geometric forms in Group I with the buildings in Group II.

(A)  P-5, Q-3, R-4, S-2

(B)  P-3, Q-5, R-2, S-4

(C)  P-5, Q-3, R-1, S-4

(D)  P-3, Q-1, R-4, S-2

Answer: (A)

61. Match the instruments in Group I with the physical quantities they measure in Group II.

(A)  P-5, Q-2, R-1, S-3

(B)  P-2, Q-3, R-1, S-4

(C)  P-2, Q-3, R-1, S-5

(D)  P-5, Q-1, R-2, S-4

Answer: (B)

62. Match the terms in Group I with their associated items in Group II.

(A)  P-1, Q-4, R-5, S-2

(B)  P-4, Q-3, R-1, S-2

(C)  P-1, Q-5, R-4, S-2

(D)  P-4, Q-2, R-1, S-5

Answer: (C)

63. Choose the correct statement(s) from the following:

(A)  Waste water from sinks, baths, etc. enters through the top inlet of a gully trap, while foul water from sweeping of rooms or courtyards enters from side inlet.

(B)  Anti-siphon traps have a reduced water-way at the inlet side, while the outlet being larger prevents the pipe from filling full and causing siphonic action.

(C)  Intercepting traps prevent foul gases from street sewer to enter into the house.

(D)  P, Q and S traps are classified according to their shape.

Answer: (C, D)

64. A steel wire of 5.65 mm diameter and 50 m length is used for a hoisting crane. The wire is used to vertically lift a weight of 200 kg attached to its lowest end. Assume the Young’s Modulus of Elasticity of Steel as 2 × 105 N/mm2 and gravitational acceleration as 10 m/sec2. The elongation of the steel wire (in mm ) will be ____ [rounded off to two decimal places].

Answer: (19.75 to 20.15)

65. A simply supported RCC beam of span 4 m is supporting a brick wall over its entire span. The brick wall is 250 mm thick and 2 m high. The RCC beam has a depth of 600 mm and width of 250 mm. The density of brick masonry and RCC can be assumed as 18 KN/m3 and 25 KN/m3 Considering the load of the wall and self-weight of the RCC beam, the maximum bending moment in the beam (in KN-m) will be _______ [rounded off to two decimal places].

Answer: (25.4 to 25.6)

PART B2: FOR Planning CANDIDATES ONLY

Q.66 – Q.72 Carry ONE mark Each

66. Select the most appropriate scale to measure Attitude, Opinion and Perception.

(A)  Likert scale

(B)  Ratio scale

(C)  Richter scale

(D)  Armstrong scale

Answer: (A)

67. Jal Shakti Abhiyan initiated by the Ministry of Jal Shakti does NOT include ____.

(A)  Water conservation and rain water harvesting

(B)  Renovation of traditional water bodies

(C)  Hydroelectric power generation

(D)  Intensive afforestation

Answer: (C)

68. Select the correct sequence of activities for transit-operation planning process.

(A)  Network Route Design → Timetable Development → Vehicle Scheduling → Crew Scheduling

(B)  Timetable Development → Crew Scheduling → Vehicle Scheduling → Network route design

(C)  Vehicle Scheduling → Crew Scheduling → Network Route Design → Timetable Development

(D)  Crew Scheduling → Vehicle Scheduling → Timetable Development → Network Route Design

Answer: (A)

69. Select the correct sequence of steps for designing the operation of a signalized intersection.

(A)  Signal Phasing → Green Allocation → Cycle Length Selection

(B)  Green Allocation → Cycle Length Selection → Signal Phasing

(C)  Cycle Length Selection → Signal Phasing → Green Allocation

(D)  Signal Phasing → Cycle Length Selection → Green Allocation

Answer: (D)

70. Considering the following statements (P, Q, and R), select the correct option.

(P) Prediction of travel demand depends on target year modal alternatives.

(Q) Prediction of travel demand depends on target year population.

(R) Prediction of travel demand depends on target year land use.

(A)  Only P is correct

(B)  Only P & R are correct

(C)  Only Q & R are correct

(D)  P, Q, and R are all correct

Answer: (D)

71. During Covid-19 pandemic, the ARHC scheme was launched in 2021 by the Government of India to address the problems of poor urban migrants. The term ARHC refers to__________.

(A)  Accessible Rural Health Centre

(B)  Affordable Rental Housing Complexes

(C)  Affordable Rentals for Homeless Citizens

(D)  Accessible Rural Housing Complexes

Answer: (B)

72. Choose the non-probability sampling method where the sample is taken from a group of people easy to contact or reach.

(A)  Simple random sampling

(B)  Snowball sampling

(C)  Convenience sampling

(D)  Stratified random sampling

Answer: (C)

Q.73 – Q.81 Carry TWO marks Each

73. Match the items in Group-I with the most appropriate stages of travel demand modelling in Group-II.

(A)  P-4, Q-3, R-2, S-1

(B)  P-3, Q-4, R-5, S-1

(C)  P-4, Q-3, R-1, S-5

(D)  P-3, Q-4, R-2, S-5

Answer: (A)

74. Match the Acts in Group-I with the corresponding organizations empowered by the Act in Group-II.

(A)  P-4, Q-1, R-2, S-3

(B)  P-2, Q-3, R-4, S-5

(C)  P-3, Q-1, R-4, S-5

(D)  P-3, Q-1, R-5, S-2

Answer: (C)

75. As per IRC 11:1962, separate bicycle tracks may be provided when the peak hour ________.

Which of the following statement(s) can be used to correctly fill in the blank?

(P) Bicycle traffic is 400 bicycles/hour or more and the volume of motorized vehicles is 100-200 vehicles/hour

(Q) Bicycle traffic is 100 bicycles/hour or more and the volume of motorized vehicles exceed 200 vehicles/hour

(R) Bicycle traffic is 100-200 bicycles/hour and the volume of motorized vehicle is 100-200 vehicles/hour

(A)  Only P & Q

(B)  Only P & R

(C)  Only R

(D)  P, Q & R

Answer: (A)

76. As per URDPFI Guidelines (2015), match the following settlement types in Group-I to their population range in Group-II.

(A)  P-5, Q-2, R-3, S-1

(B)  P-2, Q-4, R-1, S-5

(C)  P-5, Q-4, R-1, S-2

(D)  P-4, Q-2, R-3, S-5

Answer: (A)

77. Match the application areas in Group I with the Satellites/Satellite sensors in Group II.

(A)  P-5, Q-3, R-4, S-1

(B)  P-3, Q-5, R-1, S-4

(C)  P-5, Q-2, R-4, S-3

(D)  P-2, Q-3, R-5, S-1

Answer: (A)

78. Select the institution(s) that are mandated as per the 73rd Constitutional Amendment Act, 1992 of India.

(A)  Panchayat

(B)  Municipal council

(C)  Ward committee

(D)  Gram Sabha

Answer: (A, D)

79. Select the method(s) that can be used for landuse classification based on satellite images.

(A)  Maximum Likelihood

(B)  Northwest Corner Method

(C)  K Means

(D)  ANN

Answer: (A, C, D)

80. The figure below shows a contour diagram and two points (A & B) on the continuously ascending surface. The horizontal projection of AB is 200 m long, and the gradient of AB is 1 in 25. The constant contour interval (in m) is ______ [in integer].

Answer: (2 to 2)

81. A given zone is characterized in the following tables in terms of household size, and vehicle ownership. Table I shows the trip rates of households, and Table II shows the household composition. For households of size two and above, having one or more vehicles, the total daily home-based trips made are __________ [in integer].

Answer: (2350 to 2350)

GATE Exam 2023 Agricultural Engineering (AG) Question Paper With Answer Key

GATE-2023

AG: Agricultural Engineering

GA-General Aptitude

Q.1 – Q.5 Carry ONE mark each.

1. “You are delaying the completion of the task. Send _______ contributions at the earliest.”

(A)  you are

(B)  your

(C)  you’re

(D)  yore

Answer: (B)

2. References : ______ : : Guidelines : Implement

(By word meaning)

(A)  Sight

(B)  Site

(C)  Cite

(D)  Plagiarise

Answer: (C)

3. In the given figure, PQRS is a parallelogram with PS = 7 cm, PT = 4 cm and PV = 5 cm. What is the length of RS in cm? (The diagram is representative.)

(A)  20/7

(B)  28/5

(C)  9/2

(D)  35/4

Answer: (B)

4. In 2022, June Huh was awarded the Fields medal, which is the highest prize in Mathematics.

When he was younger, he was also a poet. He did not win any medals in the International Mathematics Olympiads. He dropped out of college.

Based only on the above information, which one of the following statements can be logically inferred with certainty?

(A)  Every Fields medalist has won a medal in an International Mathematics Olympiad.

(B)  Everyone who has dropped out of college has won the Fields medal.

(C)  All Fields medalists are part-time poets.

(D)  Some Fields medalists have dropped out of college.

Answer: (D)

5. A line of symmetry is defined as a line that divides a figure into two parts in a way such that each part is a mirror image of the other part about that line.

The given figure consists of 16 unit squares arranged as shown. In addition to the three black squares, what is the minimum number of squares that must be coloured black, such that both PQ and MN form lines of symmetry? (The figure is representative)

(A)  3

(B)  4

(C)  5

(D)  6

Answer: (C)

Q.6 – Q.10 Carry TWO marks Each

6. Human beings are one among many creatures that inhabit an imagined world. In this imagined world, some creatures are cruel. If in this imagined world, it is given that the statement “Some human beings are not cruel creatures” is FALSE, then which of the following set of statement(s) can be logically inferred with certainty?

(i) All human beings are cruel creatures.

(ii) Some human beings are cruel creatures.

(iii) Some creatures that are cruel are human beings.

(iv) No human beings are cruel creatures.

(A)  only (i)

(B)  only (iii) and (iv)      

(C)  only (i) and (ii)

(D)  (i), (ii) and (iii)

Answer: (D)

7. To construct a wall, sand and cement are mixed in the ratio of 3:1. The cost of sand and that of cement are in the ratio of 1:2.

If the total cost of sand and cement to construct the wall is 1000 rupees, then what is the cost (in rupees) of cement used?

(A)  400

(B)  600

(C)  800

(D)  200

Answer: (A)

8. The World Bank has declared that it does not plan to offer new financing to Sri Lanka, which is battling its worst economic crisis in decades, until the country has an adequate macroeconomic policy framework in place. In a statement, the World Bank said Sri Lanka needed to adopt structural reforms that focus on economic stabilisation and tackle the root causes of its crisis. The latter has starved it of foreign exchange and led to shortages of food, fuel, and medicines. The bank is repurposing resources under existing loans to help alleviate shortages of essential items such as medicine, cooking gas, fertiliser, meals for children, and cash for vulnerable households.

Based only on the above passage, which one of the following statements can be inferred with certainty?

(A)  According to the World Bank, the root cause of Sri Lanka’s economic crisis is that it does not have enough foreign exchange.

(B)  The World Bank has stated that it will advise the Sri Lankan government about how to tackle the root causes of its economic crisis.

(C)  According to the World Bank, Sri Lanka does not yet have an adequate macroeconomic policy framework.

(D)  The World Bank has stated that it will provide Sri Lanka with additional funds for essentials such as food, fuel, and medicines.

Answer: (C)

9. The coefficient of x4 in the polynomial (x – 1)3 (x – 2)3 is equal to _______.

(A)  33

(B)  −3

(C)  30

(D)  21

Answer: (A)

10. Which one of the following shapes can be used to tile (completely cover by repeating) a flat plane, extending to infinity in all directions, without leaving any empty spaces in between them? The copies of the shape used to tile are identical and are not allowed to overlap.

(A)  circle

(B)  regular octagon

(C)  regular pentagon

(D)  rhombus

Answer: (D)

AG: Agricultural Engineering

Q.11 – Q.35 Carry ONE mark Each

11. If A and B are square matrices of order 3 such that |B|=−1, |B|=3, then |3AB| equals

(A)  -81

(B)  -27

(C)  -9

(D)  81

Answer: (A)

12. is equal to

(A)  0

(B)  1/2

(C)  1

(D)  2

Answer: (B)

13. The value of  is

(A)  0

(B)  1

(C)  2

(D)  3

Answer: (C)

14. y = aemx + bemx is the solution of the differential equation

Answer: (D)

15. In rotary tiller, the total energy requirement for carrying out tillage will decrease if

(A)  the bite length is increased

(B)  the bite length is decreased

(C)  the cone index of soil is higher

(D)  forward speed of the machine is reduced

Answer: (A)

16. The effectiveness of the turbocharger of a diesel engine increases when

(A)  the ambient temperature increases

(B)  the pressure ratio across the compressor decreases

(C)  the load on the engine increases

(D)  the displacement volume of the engine decreases

Answer: (C)

17. In a thresher, the cylinder separation efficiency can be improved by increasing

(A)  cylinder diameter

(B)  cylinder speed

(C)  cylinder-concave clearance

(D)  feed rate

Answer: (B)

18. In a 4-stroke single cylinder diesel engine, the inlet valve opens at 10⁰ before TDC and closes at 40⁰ after BDC. The exhaust valve opens at 25⁰ before BDC and closes at 15⁰ after TDC. The percentage of time for which both the valves remain closed in one cycle of the engine is

(A)  32.29

(B)  40.97

(C)  46.53

(D)  75.01

Answer: (B)

19. The torque available at maximum power developed by the tractor is 150 N m. If the reserve torque is 20%, the peak torque that can be developed by the tractor in N m is

(A)  100

(B)  120

(C)  180

(D)  210

Answer: (C)

20. The statement which is not correct for the porous medium is

(A)  Seepage velocity is always greater than the Darcy’s velocity

(B)  Darcy’s velocity is not exclusively controlled by soil porosity

(C)  Seepage velocity increases with increasing surface ponding of water

(D)  Darcy’s velocity in unsaturated soil is always greater than that in saturated soil

Answer: (D)

21. A sprinkler irrigation system has been designed for a crop with the water application rate of 1.17 cm h1 and sprinkler discharge of 1.3 L s1. The coefficient of discharge and uniformity coefficient are 0.9 and 0.8, respectively. If the sprinkler spacing along the lateral is 20 m, the lateral spacing in m is

(A)  14.4

(B)  16.0

(C)  18.0

(D)  20.0

Answer: (D)

22. The average discharge, operating pressure and emitter constant of a drip emitter are 4 L h1, 110 kPa and 0.3, respectively. The type of emitter is

(A)  orifice

(B)  long path

(C)  pressure compensating

(D)  disc

Answer: (A)

23. If the departure and latitude of a line are 70 m and −130 m, respectively, then the whole circle bearing of the line in degrees is

(A)  28

(B)  62

(C)  152

(D)  208

Answer: (C)

24. Match the Columns:

(A)  1 – c, 2 – b, 3 – a, 4 – e, 5 – d

(B)  1 – c, 2 – d, 3 – a, 4 – e, 5 – b

(C)  1 – d, 2 – c, 3 – e, 4 – a, 5 – b

(D)  1 – c, 2 – d, 3 – a, 4 – b, 5 – e

Answer: (B)

25. The information needed for estimating the design flood using Rational formula is

(A)  cumulative infiltration

(B)  antecedent moisture condition of soil

(C)  shape factor of the catchment

(D)  time of concentration of the catchment

Answer: (D)

26. The microbial death kinetics for a food suspension follows the equation:

where No= initial microbial load, N = microbial load after time t, tl = lag time and D = decimal reduction time.

The correct statement for this equation is

(A)  the time required to reduce 10% of the initial population is lag time.

(B)  the time required to reduce the initial 90% of population is lag time

(C)  time required to kill the first 90% population is lower than D value at the same temperature

(D)  lag time approaches D value as N0 becomes smaller and temperature decreases.

Answer: (B)

27. If the diameter of fat globule in a cream separator is reduced to half and the rotational speed of the centrifuge increased to three times, the terminal settling velocity of fat globule is

(A)  decreased to 0.44 times

(B)  increased to 0.44 times

(C)  decreased to 2.25 times

(D)  increased to 2.25 times

Answer: (D)

28. The log mean temperature difference (LMTD) correction factor is not required during heat transfer rate calculation in

(A)  plate heat exchanger

(B)  1 shell pass and 1 tube pass heat exchanger

(C)  1 shell pass and 2 tube pass heat exchanger

(D)  2 shell pass and 4 tube pass heat exchanger

Answer: (B)

29. Identify the dimensionless parameter(s) from the following:

(A)  Cone index

(B)  Puddling index

(C)  Performance index

(D)  Reel index

Answer: (B, D)

30. The probability that a storm event with a return period of 20 years will occur once in 5-year period is __________(rounded off to 2 decimal places).

Answer: (0.19 to 0.21)

31. Considering declining balance method, the constant rate of depreciation at which the value of the tractor will come down to 50% of its purchase price at the end of 4th year in per cent is _________ (rounded off to 2 decimal places).

Answer: (15.80 to 16.00)

32. A trapezoidal grassed waterway with side slope (H:V) of 1:1 carries a design discharge of 1 m3 s1. The bed slope and Manning’s roughness coefficient of this channel are 1% and 0.04, respectively. The design depth of the best hydraulic trapezoidal grassed waterway section in m is ________ (rounded off to 2 decimal places).

Answer: (0.66 to 0.69)

33. The minimum fluidization height of 1.20 m is maintained during fluidized bed drying of carrots. The bed diameter of the fluidized bed dryer is 0.6 m. If mass and solid density of carrots are 250 kg and 1040 kg m3, respectively, then the porosity of the bed at the minimum fluidization condition is ____________ (rounded off to 3 decimal places, Consider π = 3.14).

Answer: (0.290 to 0.296)

34. The lighter liquid layer and the interphase layer in a basket centrifuge, rotating at a speed of 1000 rpm, are 0.1025 m and 0.105 m away from the center, respectively. Considering the densities of lighter and heavier liquids as 920 kg m3 and 1015 kg m3, the differential pressure in horizontal direction required to maintain the interphase layer in kPa is ___________ (rounded off to 3 decimal places, Consider π = 3.14).

Answer: (2.610 to 2.620)

35. The upstream and downstream pressures in a homogenizer during homogenization of milk are maintained at 250 bar and 10 bar, respectively. If density of milk is 1030 kg m3, then the velocity at which milk comes out of the homogenizing valve in m s1 is ___________ (rounded off to 3 decimal places).

Answer: (215.850 to 215.900)

Q.36 – Q.65 Carry TWO marks Each

36. If  and (A + B)2, then the values of a and b are:

(A)  a = 4, b = 1

(B)  a = 1, b = 4

(C)  a = 0, b = 4

(D)  a = 2, b = 4

Answer: (B)

37. A vector  is passing through the origin of a 3-D frame. Considering the tendency of rotation in the counter clockwise direction as positive, the moment about a point A : (3, 4, 8) is

Answer: (D)

38. A vertical disc plough with 5 discs is operated at a depth of 0.15 m. The disc angle and disc diameter are 40⁰ and 0.6 m, respectively. If overlap between two consecutive discs is 0.12 m at 0.15 m depth of cut, the total width of cut at the specified depth in m is

(A)  1.19

(B)  1.55

(C)  2.11

(D)  2.36

Answer: (A)

39. In a 9 × 20 cm fluted roller type seed drill, each fluted roller is discharging 4.25 g of seed per revolution of fluted roller shaft. The fluted roller shaft rotates once for two complete rotation of the ground drive wheel of the seed drill. The rolling diameter of the ground drive wheel is 0.35 m. Considering no skid of the ground drive wheel, the seed rate in kg ha1 is

Consider π = 3.14

(A)  96.62

(B)  141.55

(C)  187.35

(D)  386.42

Answer: (A)

40. A field sprayer with 12 nozzles fitted to the boom at a spacing of 0.5 m is used for spraying at a height of 0.75 m from the ground. The angle of spraying is 75⁰. If the height of spraying is reduced to 0.6 m, the change in swath in m is

(A)  0.23

(B)  0.48

(C)  0.65

(D)  0.91

Answer: (A)

41. The ordinates of a 6-hour S-hydrograph of a catchment are given in Table below. The catchment has phi-index of 0.25 cm h1 and base flow of 10.5 m3 s1. The peak of the flood hydrograph generated from this catchment due to a storm of 45 mm received during the first 6 h in m3 s1 is

(A)  259.5

(B)  270.0

(C)  280.5

(D)  349.5

Answer: (C)

42. It is planned to provide irrigation in a crop field having field capacity and permanent wilting point of the soil as 0.21 cm3 cm3 and 0.09 cm3 cm3, respectively. The crop root zone depth is 0.90 m. The growing period of this crop is 1st January to 31st March, during which the observed reference evapotranspiration (ETr), effective rainfall (Pe) and crop coefficients (Kc) are listed below. Considering management allowable deficit (MAD) for this crop as 50%, the average irrigation interval during the growing period in days is

(A)  4

(B)  6

(C)  8

(D)  11

Answer: (B)

43. The infiltration capacity of a basin is described by the Horton’s equation, I = 2 + e3t, where I is in cm h1 and the duration, t is in hours. If the duration of the storm event is 2 hours, the depth of the infiltration in the last 1 hour of the storm event in mm is

(A)  5

(B)  10

(C)  20

(D)  25

Answer: (C)

44. In a juice filtration process, solid concentration per m3 of filtrate is 0.2 kg. During filtration of 12.49 m3 of juice, 0.02 m thick cake (porosity of 0.32) is deposited. If 2.5 kg of solid is collected in 180 s, the pressure drop across the cake in kPa is

[Absolute viscosity of juice is 2.12 ×103 kg m-1 s1, and specific cake resistance is 1.2 × 108 m kg1]

(A)  0.18

(B)  1.81  

(C)  18.06

(D)  180.60

Answer: (C)

45. Cheese is packed in a bilayer plastic package made up of low density polyethylene (LDPE) and polyethylene terephthalate (PET). The thickness of LDPE and PET in the package are 1.5 mm and 1.3 mm, respectively. The surface area of the plastic package is 6.25 cm2. The partial pressure difference of oxygen across the package wall is 0.30 atm. The permeability coefficient of oxygen in LDPE and PET are 4.18 × 108 cm3 cm cm2 s1 atm1 and 1.67 × 1010 cm3 cm cm2 s1 atm-1, respectively. If the food gets spoiled when it absorbs 0.025 ml of oxygen, then the shelf life of food in days is

(A)  121

(B)  103

(C)  73

(D)  61

Answer: (A)

46. The rotor shaft of an ice cream freezer consists of 3 scraper blades. The temperature difference between the ice cream mix and the refrigerant during freezing of ice cream is 30 ⁰C. Density and latent heat of fusion of ice are 917 kg m3 and 335 kJ kg1, respectively. The overall heat transfer coefficient is 2000 kJ m2 h1 °C1. If the maximum thickness of ice formed before being scraped off is 10 μm, the minimum speed of the scraper shaft in rpm is

(A)  88

(B)  109

(C)  121

(D)  149

Answer: (B)

47. The percentage absolute humidity of air becomes equal to the percentage relative humidity, when

(A)  absolute humidity of air is equal to relative humidity

(B)  saturated humidity of air is equal to relative humidity

(C)  air is almost or completely dry

(D)  air is almost or completely saturated

Answer: (C, D)

48. Dimensionless numbers play an important role in correlating transfer coefficients during forced convection. In relation to the dimensionless numbers, the correct statement(s) is/are

(A)  Prandtl number in heat transfer is analogous to Schmidt number in mass transfer

(B)  Small value of Prandtl number signifies lower thermal diffusion as compared to momentum diffusion

(C)  Prandtl number is the ratio of momentum diffusivity to the thermal diffusivity of the fluid

(D)  Lewis number is the product of Schmidt number and Prandtl number

Answer: (A, C)

49. In a locality ‘A’, the probability of a convective storm event is 0.7 with a density function,  The probability of tropical cyclone-induced storm in the same location is given by the density function  The probability of occurring more than 1 unit of storm event is _________ (rounded off to 2 decimal places).

Answer: (0.28 to 0.32)

50. Given that  and y = 1, when x = 0. Using Runge-Kutta fourth order method, the value of y at x = 0.2 is ________ (rounded off to 3 decimal places).

Answer: (1.264 to 1.265)

51. A power operated chaff cutter with a mean cutting radius of 0.25 m is fitted with two cutting knives and is rotating at 300 rpm. Thirty maize stalks with a mean diameter of 12 mm are fed through the throat at a time. The dynamic shear strength of the stalk is 0.05 N mm2. The mass and radius of gyration of the flywheel (including knives) are 40 kg and 0.27 m, respectively. The total shaft power requirement in kW is _________ (rounded off to 2 decimal places).

Answer: (2.70 to 2.85)

52. A two-wheel drive tractor with a total weight of 24 kN has a static weight distribution of 30% and 70% at the front and rear axles, respectively. When the tractor is operated on a level ground of pure sand, the maximum tractive force developed is 13 kN. If external weight of 1.5 kN is added to the rear axle, neglecting weight transfer, the change in maximum tractive force in kN is _________ (rounded off to 2 decimal places).

Answer: (1.15 to 1.20)

53. A 4-stroke diesel engine can be operated with either diesel (heating value 45 MJ kg1) or biodiesel blend, B20 (heating value 42.1 MJ kg1). The brake specific fuel consumption of the engine when operated with diesel and B20 is 260 g kW1 h1 and 310 g kW1 h1, respectively. For developing a brake power of 20 kW, the change in brake thermal efficiency of the engine when B20 is used in place of diesel is ________ (rounded off to 2 decimal places).

Answer: (3.15 to 3.25 OR 0.3 to 0.3)

54. A solar photovoltaic system is used to generate power from total solar radiations varying from 400 to 750 W m2. The maximum conversion efficiency of solar photovoltaic system is 14%. The open circuit voltage, short circuit current and fill factor of solar cells are 21.6 V, 3.22 A and 0.72, respectively. To generate maximum power, the minimum cell area required in m2 is _________ (rounded off to 3 decimal places).

Answer: (0.885 to 0.895)

55. A single disc clutch is used to transmit 10 kW power at 1400 rpm. The axial pressure exerted on the contact surface is 0.07 N mm2 and the coefficient of friction is 0.25. Considering the ratio of diameter to face width of the clutch lining as 8 and assuming uniform wear theory, the required face width of friction lining in mm is ___________ (rounded off to 2 decimal places).

Answer: (26.75 to 26.90)

56. In a tractor seat system, the chassis frequency and seat suspension damping rate are 20 rad s1 and 400 N m1 s, respectively. The critical damping rate of tractor seat system is 1600 N m1 If the combined mass of the seat and operator is 80 kg, the transmissibility of vibration is _________ (rounded off to 2 decimal places).

Answer: (0.43 to 0.46)

57. Two cylindrical reservoirs ‘A’ and ‘B’ are connected by a 30 m long pipe of 250 mm internal diameter as shown in Figure below. The Darcy-Weisbach friction factor for the pipe is 0.025. Initially the reservoir ‘A’ was full at the indicated level and reservoir ‘B’ was empty. If the entrance and exit losses in this pipe are neglected, the time required to empty the reservoir ‘A’ in hour is _________ (rounded off to 3 decimal places).

Consider π = 3.14 and acceleration due to gravity, g = 9.81 m s2.

Answer: (3.30 to 3.40)

58. A homogenous anisotropic earthen dam of height 52 m with a free board of 2 m is constructed on an impermeable foundation. The horizontal and vertical hydraulic conductivities of soil used for the construction of the dam are 4.5 × 108 m s1 and 2.0 × 108 m s1, respectively. There are 6 flow channels and 25 equipotential drops in a square flow net drawn in the transformed dam section. If the downstream dam side is dry, the quantity of seepage per unit length through the dam in m3 day1 m1 is __________ (rounded off to 3 decimal places).

Answer: (0.030 to 0.032)

59. A salt affected crop field is to be leached with irrigation water having salt concentration of 3.5 meq L1. Salt concentration in the saturation extract of soil is 15.2 meq L1. Leaching efficiency of the field is 55%. In the month of March, the observed reference evapotranspiration and effective rainfall in this area are 150 mm and 75 mm, respectively. If the average crop coefficient in this month is 1.05, the leaching requirement for the entire month in mm is _________ (rounded off to 2 decimal places).

Answer: (19.50 to 19.55)

60. A 10 m long concrete pipe is required to carry a peak discharge of 1.0 m3 s1 in a drop inlet spillway with a head of 4 m. The entrance loss coefficient is 0.5 and the friction loss coefficient is 0.02. Consider acceleration due to gravity = 9.81 m s2. The neutral slope of the water level in per cent is _________ (rounded off to 2 decimal places).

Answer: (4.69 to 4.80)

61. Discharge from a centrifugal pump operating at 1000 rpm with a total head of 30 m is 300 L min1. The pump efficiency is 65%. If speed of the pump is increased to 1200 rpm, the power required to operate the pump in kW is ____________ (rounded off to 2 decimal places).

Consider acceleration due to gravity = 9.81 m s2.

Answer: (3.90 to 4.00)

62. A 0.30 m diameter well penetrates an unconfined aquifer with a saturated depth of 40 m. After 8 hours of pumping at a steady rate of 0.03 m3 s1, the drawdown in two observation wells located at 20 m and 50 m away from the pumping well are found to be 3 m and 2 m, respectively. The drawdown in the pumping well in m is _________ (rounded off to 1 decimal place, Consider π = 3.14).

Answer: (8.6 to 8.8)

63. The apparent wall shear stress in a 0.6 m long pipe line carrying refined oil is 12.5 Pa. If the pressure drop along the length is 300 Pa and flow rate is 0.25 m3 s1, the absolute viscosity of oil in 103 Pa s is ______________ (rounded off to 3 decimal places).

Answer: (4.902 to 4.908)

64. The carrot slices (water activity = 0.89) are to be preserved using osmo-dehydration. Addition of salt (NaCl) to 20% sucrose solution (water activity = 0.987) reduces the water activity to 0.85. Percentage of NaCl added to the solution is ______________ (rounded off to 2 decimal places).

Consider molecular mass of Sucrose = 342 and molecular mass of NaCl = 58.44

Answer: (16.40 to 16.60)

65. A copper ball and a steel ball having diameters d1 and d2, respectively, are initially at a uniform temperature of 200⁰C. Both the balls are exposed to the atmosphere at 30⁰C. If both the balls attain a temperature of 120⁰C after equal exposure duration, then the ratio of d1 to d2 is _____________ (rounded off to 3 decimal places).

Assume Biot Number to be less than 0.1. The thermo-physical properties of copper and steel are given below:

Answer: (MTA)

GATE Exam 2023 Aerospace Engineering (AE) Question Paper With Answer Key

GATE-2023

AE: Aerospace Engineering

GA-General Aptitude

Q.1 – Q.5 Carry ONE mark each.

1. “You are delaying the completion of the task. Send _______ contributions at the earliest.”

(A)  you are

(B)  your

(C)  you’re

(D)  yore

Answer: (B)

2. References : ______ : : Guidelines : Implement

(By word meaning)

(A)  Sight

(B)  Site

(C)  Cite

(D)  Plagiarise      

Answer: (C)

3. In the given figure, PQRS is a parallelogram with PS = 7 cm, PT = 4 cm and PV = 5 cm. What is the length of RS in cm? (The diagram is representative.)

(A)  20/7

(B)  28/5

(C)  9/2

(D)  35/4

Answer: (B)

4. In 2022, June Huh was awarded the Fields medal, which is the highest prize in Mathematics.

When he was younger, he was also a poet. He did not win any medals in the International Mathematics Olympiads. He dropped out of college.

Based only on the above information, which one of the following statements can be logically inferred with certainty?

(A)  Every Fields medalist has won a medal in an International Mathematics Olympiad.

(B)  Everyone who has dropped out of college has won the Fields medal.

(C)  All Fields medalists are part-time poets.

(D)  Some Fields medalists have dropped out of college.

Answer: (D)

5. A line of symmetry is defined as a line that divides a figure into two parts in a way such that each part is a mirror image of the other part about that line.

The given figure consists of 16 unit squares arranged as shown. In addition to the three black squares, what is the minimum number of squares that must be coloured black, such that both PQ and MN form lines of symmetry? (The figure is representative)

(A)  3

(B)  4

(C)  5

(D)  6

Answer: (C)

Q.6 – Q.10 Carry TWO marks Each

6. Human beings are one among many creatures that inhabit an imagined world. In this imagined world, some creatures are cruel. If in this imagined world, it is given that the statement “Some human beings are not cruel creatures” is FALSE, then which of the following set of statement(s) can be logically inferred with certainty?

(i) All human beings are cruel creatures.

(ii) Some human beings are cruel creatures.

(iii) Some creatures that are cruel are human beings.

(iv) No human beings are cruel creatures.

(A)  only (i)

(B)  only (iii) and (iv)

(C)  only (i) and (ii)

(D)  (i), (ii) and (iii)

Answer: (D)

7. To construct a wall, sand and cement are mixed in the ratio of 3:1. The cost of sand and that of cement are in the ratio of 1:2.

If the total cost of sand and cement to construct the wall is 1000 rupees, then what is the cost (in rupees) of cement used?

(A)  400

(B)  600

(C)  800

(D)  200

Answer: (A)

8. The World Bank has declared that it does not plan to offer new financing to Sri Lanka, which is battling its worst economic crisis in decades, until the country has an adequate macroeconomic policy framework in place. In a statement, the World Bank said Sri Lanka needed to adopt structural reforms that focus on economic stabilisation and tackle the root causes of its crisis. The latter has starved it of foreign exchange and led to shortages of food, fuel, and medicines. The bank is repurposing resources under existing loans to help alleviate shortages of essential items such as medicine, cooking gas, fertiliser, meals for children, and cash for vulnerable households.

Based only on the above passage, which one of the following statements can be inferred with certainty?

(A)  According to the World Bank, the root cause of Sri Lanka’s economic crisis is that it does not have enough foreign exchange.

(B)  The World Bank has stated that it will advise the Sri Lankan government about how to tackle the root causes of its economic crisis.

(C)  According to the World Bank, Sri Lanka does not yet have an adequate macroeconomic policy framework.

(D)  The World Bank has stated that it will provide Sri Lanka with additional funds for essentials such as food, fuel, and medicines.

Answer: (C)

9. The coefficient of x4 in the polynomial (x − 1)3 (x − 2)3 is equal to _______.

(A)  33

(B)  −3

(C)  30

(D)  21

Answer: (A)

10. Which one of the following shapes can be used to tile (completely cover by repeating) a flat plane, extending to infinity in all directions, without leaving any empty spaces in between them? The copies of the shape used to tile are identical and are not allowed to overlap.

(A)  circle

(B)  regular octagon

(C)  regular pentagon

(D)  rhombus

Answer: (D)

AE: Aerospace Engineering

Q.11 – Q.35 Carry ONE mark Each

11. The direction in which a scalar field ϕ(x, y, z) has the largest rate of change at any point with position vector

Answer: (A)

12. If a monotonic and continuous function y = f(x) has only one root in the interval x1 < x < x2, then

(A)  f(x1)f(x2) > 0

(B)  f(x1)f(x2) = 0

(C)  f(x1)f(x2) < 0

(D)  f(x1) – f(x2) = 0

Answer: (C)

13. Consider the one-dimensional wave equation  for −∞ < x < ∞, t ≥ For an initial condition  the solution at t = 1 is

Answer: (A)

14. A two-dimensional potential flow solution for flow past an airfoil has a streamline pattern as shown in the figure. Which of the following conditions is additionally required to satisfy the Kutta condition?

(A)  Addition of a source of strength Q > 0

(B)  Addition of a source of strength Q < 0

(C)  Addition of a circulation of strength Γ > 0 (counter-clockwise)

(D)  Addition of a circulation of strength Γ < 0 (clockwise)

Answer: (D)

15. Consider the Blasius solution for the incompressible laminar flat plate boundary layer. Among the following options, select the correct relation for the development of the momentum thickness θ with distance x from the leading edge along the length of the plate.

(A)  θ ∝ x2/3

(B)  θ ∝ x1/2

(C)  θ ∝ x1/7

(D)  θ ∝ x2/3

Answer: (B)

16. In a two-dimensional potential flow, the doublet is a limit of the superposition of

(A)  a uniform stream and a source

(B)  a source and a sink of equal strength

(C)  a uniform stream and a sink

(D)  a source and a vortex

Answer: (B)

17. An ideal glider has drag characteristics given by  where  is the induced drag coefficient, CL is the lift coefficient, and K is a constant. For maximum range of the glider, the ratio  is

(A)  1

(B)  1/3

(C)  3

(D)  3/2

Answer: (A)

18. The figures shown in the options are schematics of airfoil shapes (not to scale). For a civilian transport aircraft designed for a cruise Mach number of 0.8, which among them is aerodynamically best suited as a wing section?

Answer: (D)

19. For a longitudinally statically stable aircraft, which one of the following represents the relationship between the coefficient of pitching moment about the center of gravity Cmcg and absolute angle of attack αa ?

(Note : nose-up moment is positive.)

Answer: (D)

20. In a single-spool aviation turbojet engine, which of the following is the correct relationship between the total work output WT of a 2-stage axial turbine and the total work required WC by a 6-stage axial compressor, neglecting losses?

(A)  WT = 2WC

(B)  WT = 6WC

(C)  WT = WC

(D)  WT = 3WC

Answer: (C)

21. For a stage of a 50% reaction ideal axial flow compressor (symmetrical blading), select the correct statement from the options given.

(A)  The stagnation enthalpy rise across the rotor is 50% of the rise across the stage.

(B)  The static enthalpy rise across the rotor is 50% of the rise across the stage.

(C)  Axial velocity component of the flow at the rotor exit is 50% of that at the rotor entry.

(D)  The static pressure rise across the rotor is 50% of the rise across the stator.

Answer: (B)

22. An aircraft is cruising with a forward speed Va and the jet exhaust speed relative to the engine at the exit is Vj. If Vj/Va = 2, what is the propulsive efficiency?

(A)  0.50

(B)  1.00

(C)  0.33

(D)  0.67

Answer: (D)

23. Consider the four basic symmetrical flight loading conditions corresponding to the corners of a typical V-n diagram. For one of these flight loading conditions, it is observed that (i) the compressive bending stresses have a maximum value in the bottom aft region (see figure) of the wing cross-section; and (ii) the tensile bending stresses are maximum in the upper forward region (see figure) of the wing cross-section. For the preceding observations, select the corresponding flight loading condition from the options given.

(A)  Positive high angle of attack

(B)  Positive low angle of attack

(C)  Negative high angle of attack

(D)  Negative low angle of attack

Answer: (D)

24. Which one of the following figures represents the qualitative variation of absolute deceleration |dV/dt| with altitude h(measured from the mean sea level) for a space vehicle undergoing a ballistic entry into the Earth’s atmosphere?

Answer: (D)

25. Which of the following statement(s) is/are true about harmonically excited forced vibration of a single degree-of-freedom linear spring-mass-damper system?

(A)  The total response of the mass is a combination of free vibration transient and steady-state response.

(B)  The free vibration transient dies out with time for each of the three possible conditions of damping (under-damped, critically damped, and over-damped).

(C)  The steady-state periodic response is dependent on the initial conditions at the time of application of external forcing.

(D)  The rate of decay of free vibration transient response depends on the mass, spring stiffness and damping constant.

Answer: (A, B, D)

26. Which of the following statement(s) is/are true about the state of stress in a plane?

(A)  Maximum or major principal stress is algebraically the largest direct stress at a point.

(B)  The magnitude of minor principal stress cannot be greater than the magnitude of major principal stress.

(C)  The planes of maximum shear stress are inclined at 90 degrees to the principal axes.

(D)  The normal stresses along the planes of maximum shear stress are equal.

Answer: (A, D)

27. The normal stresses along the planes of maximum shear stress are equal.

(A)  For a rectangular planform wing, the dimensions of the ribs DO NOT depend on their spanwise position in the wing.

(B)  Ribs increase the column buckling stress of longitudinal stiffeners connected to them.

(C)  Ribs increase plate buckling stress of the skin panels.

(D)  Ribs help in maintaining aerodynamic shape of the wing.

Answer: (B, C, D)

28. From the options given, select all that are true for turbofan engines with afterburners.

(A)  Turning afterburner ON increases specific fuel consumption.

(B)  Turbofan engines with afterburners have variable area nozzles.

(C)  Turning afterburner ON decreases specific fuel consumption.

(D)  Turning afterburner ON increases stagnation pressure across the engine.

Answer: (A, B)

29. Which of the following statement(s) is/are true with respect to eigenvalues and eigenvectors of a matrix?

(A)  The sum of the eigenvalues of a matrix equals the sum of the elements of the principal diagonal.

(B)  If λ is an eigenvalue of a matrix A, then 1/λ is always an eigenvalue of its transpose (AT).

(C)  If λ is an eigenvalue of an orthogonal matrix A, then 1/λ is also an eigenvalue of A.

(D)  If a matrix has n distinct eigenvalues, it also has n independent eigenvectors.

Answer: (A, C, D)

30. For studying wing vibrations, a wing of mass M and finite dimensions has been idealized by assuming it to be supported using a linear spring of equivalent stiffness k and a torsional spring of equivalent stiffness kθ as shown in the figure.

The centre of gravity (CG) of the wing idealized as an airfoil is marked in the figure. The number of degree(s) of freedom for this idealized wing vibration model is _________. (Answer in integer)

Answer: (2 to 2)

31. The system of equations

x – 2y + αz = 0

2x + y – 4z = 0

x – y + z = 0

has a non-trivial solution for α = _______. (Answer integer)

Answer: (3 to 3)

32. An airplane weighting 40 kN is landing on a horizontal runway during which it is retarded by an arresting cable mechanism. The tension in the arresting cable at a given instant, as shown in the figure, is 100 kN. Assuming that the thrust from the engine continues to balance airplane drag, the magnitude of horizontal load factor is ________. (round off to one decimal place)

Answer: (2.4 to 2.6)

33. The ratio of the speed of sound in H2 (molecular weight 2 kg/kmol) to that in N2 (molecular weight 28 kg/kmol) at temperature 300 K and pressure 2 bar is ____. (round off to two decimal places)

Answer: (3.5 to 4.0)

34. Airplane A and Airplane B are cruising at altitudes of 2 km and 4 km, respectively. The free stream density and static pressure at altitude 2 km are 1.01 kg/m3 and 79.50 kPa, respectively, and at altitude 4 km, they are 0.82 kg/m3 and 61.70 kPa, respectively. The differential pressure reading from the pitot-static tubes is 3 kPa for both the airplanes. Assuming incompressible flow, the ratio of

cruise speeds of Airplane A to Airplane B is ______. (round off to two decimal places)

Answer: (0.89 to 0.91)

35. A supersonic vehicle powered by a ramjet engine is cruising at a speed of 1000 m/s. The ramjet engine burns hydrogen in a subsonic combustor to produce thrust. The heat of combustion for hydrogen is 120 MJ/kg. The overall efficiency of the engine η0, defined as the ratio of propulsive power to the total heat release in the combustor, is 40%. Taking acceleration due to gravity g0 = 10 m/s2, the specific impulse of the engine is __________ seconds.

(round off to the nearest integer).

Answer: (4800 to 4900)

Q.36 – Q.65 Carry TWO marks Each

36. Given the function y(x) = (x + 3) (x – 2), for −4 < x < 4. What is the value of x at which the function has a minimum?

(A)  −3/2

(B)  −1/2

(C)  1/2

(D)  3/2

Answer: (B)

37. A supersonic aircraft has an air intake ramp that can be rotated about the leading edge O such that the shock from the leading edge meets the cowl lip as shown in the figure. Select all the correct statement(s) as per oblique shock theory when the flight Mach number M is increased.

(A)  It is always possible to find a ramp setting θRAMP such that the shock still meets the cowl lip (βSHOCK remains the same).

(B)  If θRAMP is held fixed, the shock angle βSHOCK will increase.

(C)  If M exceeds a critical value, it would NOT be possible to find a ramp setting θRAMP such that the shock still meets the cowl lip (βSHOCK­ remains the same).

(D)  Shock angle βSHOCK < sin1(1/M)

Answer: (A)

38. Two missiles A and B powered by solid rocket motors have identical specific impulse, liftoff mass of 5600 kg each, and burn durations of tA = 30 s and tB = 70 s, respectively. The propellant mass flow rates,  for missiles A and B, respectively, are given by

Neglecting gravity and aerodynamic forces, the relationship between the final velocities VA and VB of missiles A and B, respectively, is given by

(A)  VA = 4.1 VB

(B)  VA = VB

(C)  VA = 0.5 VB

(D)  VA = 0.7 VB

Answer: (C)

39. A perfect gas stored in a large reservoir exhausts into the atmosphere through a convergent duct. The reservoir pressure is P0 and temperature is T0. The jet emerges from the nozzle at choked conditions with average velocity u, Mach number M, pressure p, temperature T, and density ρ. If the reservoir pressure is increased, then

(A)  u, M, p, T and ρ increase

(B)  u, p, T, and ρ increase, but M remains the same

(C)  u, M and T remain the same, but p and ρ increase

(D)  u, M, T and ρ remain the same but p increases

Answer: (C)

40. Consider a general aviation airplane with weight 10 kN and a wing planform area of 15 m2. The drag coefficient of the airplane is given as CD = CD0 + KCL2 with CD0 = 0.025 and K = 0.05. For level flight at an altitude where the density is 0.60 kg/m3 and thrust 1 kN, the maximum cruise speed is

(rounded off to the nearest integer)

(A)  87 m/s

(B)  30 m/s

(C)  36 m/s

(D)  101 m/s

Answer: (A)

41. A scramjet engine features an intake, isolator, combustor, and a nozzle, as shown in the schematic. Station 3 indicates the combustor entry point. Assume stagnation enthalpy to be constant between Stations 1 and 3, and air to be a calorically perfect gas with specific heat ratio γ. Select the correct expression for Mach number M3 at the inlet to the combustor from the options given.

Answer: (B)

42. Consider the equation  where a and ω are constant. Given y = 1 at x = 0, select all correct statement(s) from the following is x → ∞.

(A)  y → 0 if a ≠ 0

(B)  y → 1 if a = 0

(C)  y → A exp(|a|x) if a < 0; A is a constant

(D)  y → B sin (ωx + C) if a > 0; B and C are constants

Answer: (C, D)

43. Given the vectors

which of the following statement(s) is/are TRUE?

(A)  Vectors  are coplanar

(B)  The scalar triple product of the vectors  is zero

(C)   are perpendicular

(D)  is parallel to 

Answer: (A, B)

44. Consider a one-dimensional inviscid supersonic flow in a diverging duct with heat addition (Qin) as shown. Which of the following statement(s) is/are always TRUE?

(A)  Mach number, M2 > M1

(B)  Stagnation pressure, P10 > P20

(C)  Static pressure, P2 > P1

(D)  Stagnation temperature, T10 < T20

Answer: (B, D)

45. Consider the International Standard Atmosphere (ISA) with h being the geopotential altitude (in km) and dT/dh being the temperature gradient (in K/m). Which of the following combination(s) of (h, dT/dh) is/are as per ISA?

(A)  (7, −6.5 × 103)

(B)  (9, 4 × 103)

(C)  (15, 0)

(D)  (35, 3 × 103)

Answer: (A, C, D)

46. For an airfoil, which of the relations given about the critical Mach number Mcr and drag divergence Mach number Mdd is/are correct?

(A)  Mcr < Mdd

(B)  Mcr < 1.0

(C)  Mdd < 1.0

(D)  Mcr > 1.0

Answer: (A, B, C)

47. Which of the following statement(s) about the elastic flexural buckling load of columns is/are correct?

(A)  The buckling load increases with increase in flexural rigidity of the column.

(B)  The buckling load increases with increase in the length of the column.

(C)  The boundary conditions of the column affect the buckling load.

(D)  The buckling load is NOT directly dependent on the density of the material used

for the column.

Answer: (A, C, D)

48. The thickness of a uniform hollow circular shaft is equal to the difference between the outer radius and the inner radius. The ratio of the inner diameter to outer diameter of the shaft is 0.5. For the shaft reacting to an applied torque, the ratio of the maximum shear stress τ to the maximum shear stress τthin wall obtained using the thin-wall approximation is _________. (round off to one decimal place)

Answer: (1.1 to 1.3)

49. A rigid bar AB is subjected to a uniformly distributed load of 100 N/m as shown in the figure. The bar is supported by rod CD, with A, C, and D as pin joints. The rod CD has axial stiffness of 40 N/mm. The vertical deflection at point D is __________ mm. (round off to nearest integer)

Answer: (10 to 10)

50. A cantilever beam of length 2a is loaded at the tip with force F as shown in the figure. The beam is supported in the middle by a roller with a pin. The magnitude of moment reaction at the built-in end of the beam is α Fa, where α is ______ . (round off to one decimal place)

Answer: (0.5 to 0.5)

51. A single degree-of-freedom spring-mass-damper system has viscous damping ratio of 0.1. The mass is given an initial displacement of 10 cm without imparting any velocity. After exactly two complete cycles of oscillation (i.e., after time 2Td, where Td is the period of the damped vibration), the amplitude of the displacement is ______ cm. (round off to two decimal place)

Answer: (2.80 to 2.86)

52. The shear flow distribution in a single cell, thin-walled beam under the action of an arbitrary shear load Sy applied at the shear centre S is shown in the figure. The cell has horizontal symmetry with booms marked by 1 to 4 that carry direct stresses. The shear modulus G is the same for all the walls, and the area of the cell is 135000 mm2. With respect to the point O marked in the figure, the distance to the shear centre S is ______ mm. (round off to the nearest integer)

Answer: (196 to 198)

53. Consider a thin-walled cylindrical pressure vessel made of an alloy with yield strength of 300 MPa. The vessel has end caps to contain the pressure. The ratio of radius of the vessel to its wall thickness is 100. As per the von Mises yield criterion, the internal pressure that would cause the failure of the vessel is _____ MPa. (round off to two decimal places)

Answer: (3.40 to 3.50)

54. Consider the differential equation

with initial conditions  at x = 1. The value of y at x = 2 is ______. (round off to two decimal places)

Answer: (0.24 to 0.26)

55. The operating characteristic of a pump were measured to be CP = qΦ2, where power coefficient  is the flow coefficient, a is a constant, D is a length scale, ω is the rotation rate, ρ is fluid density, and P is the power required. The flow coefficient is a dimensionless volume flow rate scaled with ω and D. Assuming that the flow rate remains the same, if the rotation rate is increased to 1.25 ω, the power changes to α The value of α is ______. (round off to two decimal places)

Answer: (1.24 to 1.26)

56. A thin cambered airfoil has lift coefficient c1 = 0 at an angle of attack α = −1°. Assuming that stall occurs at much larger α, the c1 at α = 4° is _______. (round off to two decimal places)

Answer: (0.55 to 0.57)

57. In a potential flow, a uniform stream of strength U directed along the x-axis and four line sources (2-dimensional) of strengths π/2, −π/3, π/4, −π/5 are placed along the x-axis at x = 0, 1, 2, and 3, respectively. The strength of an additional line source to be placed at x = 4 such that a closed streamline encircles all five sources is ________. (round off to two decimal places).

Answer: (-0.69 to -0.67)

58. Enstrophy is defined as the square of the magnitude of vorticity.

For the three-dimensional velocity field

the enstrophy at location (1, 1, 1) is _______.

(round off to two decimal places)

Answer: (12.61 to 12.81)

59. An airplane with wing planform area of 20 m2 and weight 8 kN is flying straight and level with a speed of 100 m/s. The total drag coefficient is 0.026 and the air density is 0.7 kg/m3. The total thrust required to introduce a steady climb angle of 0.1 radians is ______ N. (round off to the nearest integer)

Answer: (2615 to 2625)

60. The maximum permissible load factor and the maximum lift force coefficient for an airplane is 7 and 2, respectively. For a wing loading of 6500 N/m2 and air density 1.23 kg/m3, the speed yielding the highest possible turn rate in the vertical plane is ___________ m/s. (round off to the nearest integer)

Answer: (190 to 195)

61. A gas turbine combustor is burning methane and air at an equivalence ratio ϕ = 0.5, where  and [F/A]stoich is the ratio of mass flow rate of fuel to the mass flow rate of air at stoichiometry. If the air flow rate is  then the mass flow rate of methane is ______ kg/s. (round off to two decimal places)

Answer: (0.57 to 0.60)

62. The universal gravitational constant is 11 6.67 × 10−11Nm2/kg2. For a planet of mass 6.4169 × 1023 kg and radius 3390 km, the escape velocity is _____ km/s.

(round off to one decimal place).

Answer: (4.9 to 5.1)

63. A satellite is in a circular orbit around Earth with a time period of 90 minutes. The radius of Earth is 6370 km, mass of Earth is 5.98 ×1024 kg and the universal gravitational constant is 6.67 × 10−11 Nm2/kg2. The altitude of the satellite above mean sea level is ______ km. (round off to the nearest integer)

Answer: (260 to 300)

64. A centrifugal air compressor has inlet root diameter of 0.25 m and the outlet diameter of the impeller is 0.6 m. The pressure ratio is 5.0. The air at the inlet of the rotor is at 1 atm and 25° The polytropic efficiency is 0.8 and slip factor is 0.92. Use Cp = 1.004 kJ/kg-K and γ = 1.4. The impeller speed in revolutions per minute (RPM) is ________. (round off to the nearest integer)

Answer: (15948 to 16048)

65. Consider a cryogenic liquid rocket engine using an expander cycle with liquid hydrogen and liquid oxygen as the two propellants. The mass flow rate of hydrogen  into the combustion chamber is 32 kg/s, and the mass flow rate of oxygen  into the chamber is such that  The combustion of hydrogen and oxygen is at stoichiometry. Assuming that the rate of the forward reaction is much larger than that of the reverse reaction, the rate of formation of H2O is _______ kmol/s. (round off to the nearest integer)

Answer: (16 to 16)

VITEEE Examination Previous Year Question Paper 2022 With Answer Key

VITEEE SOLVED PAPER-2022

PART –I (PHYSICS)

1. The root mean square speed of smoke particles of mass 5 × 1017 kg in their Brownian motion in air at NTP is approximately

[Given k = 1.38 × 1023 JK1]

(a)   60 mms−1

(b)   12 mms−1

(c)   15 mms−1

(d)   36 mms−1

Answer: (c)

2. The equation of a particle executing simple harmonic motion is given by  At t = 1s, the speed of particle will be (Given : π = 3.14)

(a)   0 cms1

(b)   157 cms1

(c)   272 cms1

(d)   314 cms1

Answer: (b)

3. Following are expressions for four plane simple harmonic waves

The paris of waves which will produce destructive interference and stationary waves respectively in a medium, are

(a)   (iii, iv), (i, ii)

(b)   (i, iii), (ii, iv)

(c)   (i, iv), (ii, iii)

(d)   (i, ii), (iii, iv)

Answer: (d)

4. If a charge q is placed at the centre of a closed hemispherical non-conducting surface, the total flux passing through the flat surface would be :

(a)   zero

(b)   q/2ε0

(c)   q/4ε0

(d)   q/2πε0

Answer: (a)

5. The electric potential V(x) in a region around the origin is given by V(x) = 4x2 The electric charge enclosed in a cube of 1 m side with its centre at the origin is (in coulomb)

(a)   8ε0

(b)   −4ε0

(c)   0

(d)   −8ε0

Answer: (d)

6. A heater coil is cut into two equal parts and only one part is now used in the The heat generated will now be

(a)   four times

(b)   doubled

(c)   halved

(d)   one fourth

Answer: (b)

7. In a region, steady and uniform electric and magnetic fields are present. These two fields are parallel to each other. A charged particle is released from rest in this region. The path of the particle will be a

(a)   helix

(b)   straight line

(c)   ellipse

(d)   circle

Answer: (b)

8. An object is thrown vertically upwards. At its maximum height, which of the following quantity becomes zero?

(a)   Momentum

(b)   Potential energy

(c)   Acceleration

(d)   Force

Answer: (a)

9. The self induced emf of a coil is 25 volts. When the current in it is changed at uniform rate from 10 A to 25 A in 1s, the change in the energy of the inductance is:

(a)   740 J

(b)   437.5 J

(c)   540 J

(d)   637.5 J

Answer: (b)

10. Alternating current can not be measured by D.C. ammeter because

(a)   Average value of current for complete cycle is zero

(b)   A.C. Changes direction

(c)   A.C. can not pass through D.C. Ammeter

(d)   D.C. Ammeter will get damaged.

Answer: (a)

11. The magnetic field of a plane electromagnetic wave is given by:

The amplitude of the electric field would be:

(a)   6 Vm1 along x-axis

(b)   3 Vm1 along z-axis

(c)   6 Vm1 along z-axis

(d)   2 × 108 Vm1 along z-axis

Answer: (a)

12. An ideal gas is expanding such that PT3 = constant. The coefficient of volume expansion of the gas is:

(a)   1/T

(b)   2/T

(c)   4/T

(d)   3/T

Answer: (c)

13. Two light beams of intensities in the ratio of 9 : 4 are allowed to interfere. The ratio of the intensity of maxima and minima will be:

(a)   2 : 3

(b)   16 : 81

(c)   25 : 169

(d)   25 : 1

Answer: (d)

14. The deBroglie wavelength of a proton and α-particle are equal. The ratio of their velocities is :

(a)   4 : 3

(b)   4 : 1

(c)   4 : 2

(d)   1 : 4

Answer: (b)

15. The recoil speed of a hydrogen atom after it emits a photon in going from n = 5 state to n = 1 state will be

(a)   4.34 m/s

(b)   2.19 m/s

(c)   4.17 m/s

(d)   3.25 m/s

Answer: (c)

16. Which of the following figure represents the variation of ln(R/R0) with ln A(If R = radius of a nucleus and A = its mass number)?

Answer: (b)

17. Zener breakdown occurs in a p-n junction having p and n both :

(a)   lightly doped and have wide depletion layer

(b)   heavily doped and have narrow depletion layer

(c)   lightly doped and have narrow depletion layer

(d)   heavily doped and have side depletion layer

Answer: (b)

18. If E and H represents the intensity of electric field and magnetizing field respectively, then the unit of E/H will be:

(a)   ohm

(b)   mho

(c)   joule

(d)   newton

Answer: (a)

19. A stone of mass m, tied to a string is being whirled in a vertical circle with a uniform speed. The tension in the string is :

(a)   the same throughout the motion

(b)   minimum at the highest position of the circular path

(c)   minimum at the lowest position of the circular path

(d)   minimum when the rope is in the horizontal position

Answer: (b)

20. A particle is moving with a velocity  where K is a constant. The general equation for its path is :

(a)   y = x2 + constant

(b)   y2 = x + constant

(c)   y2 = x2 + constant

(d)   xy = constant

Answer: (c)

21. A particle of mass M originally at rest is subjected to a force whose direction is constant but magnitude varies with time according to the relation

Where F0 and T are constants. The force acts only for the time interval 2T. The velocity v of the particle after time 2T is:

(a)   2F0T/M

(b)   F0T/2M

(c)   4F0T/3M

(d)   F0T/3M

Answer: (c)

22. The magnetic moment of an electron (e) revolving in an orbit around nucleus with an orbital angular momentum is given by:

Answer: (b)

23. Angular momentum of the particle rotating with a central force is constant due to

(a)   constant torque

(b)   constant force

(c)   constant linear momentum

(d)   zero torque

Answer: (d)

24. The escape velocity of a body depends upon mass as

(a)   m0

(b)   m1

(c)   m2

(d)   m3

Answer: (a)

25. A Potential energy as a function of r is given by  where r is the interatomic distance, A and B are positive constants. The equilibrium distance between the two atoms will be:

(a)   (A/B)1/5

(b)   (B/A)1/5

(c)   (2A/B)1/5

(d)   (B/2A)1/5

Answer: (c)

26. If two soap bubbles of different radii are connected by a tube

(a)   air flows from the smaller bubble to the bigger

(b)   air flows from bigger bubble to the smaller bubble till the sizes are interchanged

(c)   air flows from the bigger bubble to the smaller bubble till the sizes become equal

(d)   there is no flow of air.

Answer: (a)

27. The focal length f is related to the radius of curvature r of the spherical convex mirror by :

(a) 

(b)   f = −r

(c) 

(d)   f = r

Answer: (a)

28. A thermodynamic system is taken from an original state D to an intermediate state E by the linear process shown in the figure. Its volume is then reduced to the original volume from E to F by an isobaric process. The total work done by the gas from D to E to F will be

(a)   −450 J

(b)   450 J

(c)   900 J

(d)   1350 J

Answer: (b)

29. A vertical electric field of magnitude 4.9 × 105 N/C just prevents a water droplet of a mass 0.1 g from falling. The value of charge on the droplet will be:

(Given g = 9.8 m/s2)

(a)   1.6 × 109 C

(b)   2.0 × 109 C

(c)   3.2 × 109 C

(d)   0.5 × 109 C

Answer: (b)

30. In the circuit shown in the figure, the total charge is 750 μC and the voltage across capacitor C2 is 20 V. Then the charge on capacitor C2 is :

(a)   450 μC

(b)   590 μC

(c)   160 μC

(d)   650 μC

Answer: (b)

31. For a transistor α and β are given as  Then the correct relation between α and β will be:

Answer: (b)

32. A current I flows along the length of an infinitely long, straight, thin walled pipe. Then

(a)   the magnetic field at all points inside the pipe is the same, but not zero

(b)   the magnetic field is zero only on the axis of the pipe

(c)   the magnetic field is different at different points inside the pipe

(d)   the magnetic field at any point inside the pipe is zero

Answer: (d)

33. A Carnot engine has efficiency of 50%. If the temperature of sink is reduced by 40°C, its efficiency increases by 30%. The temperature of the source will be :

(a)   166.7 K

(b)   255.1 K

(c)   266.7 K

(d)   367.7 K

Answer: (c)

34. When you walk through a metal detector carrying a metal object in your pocket, it raises an alarm. This phenomenon works on

(a)   Electromagnetic induction

(b)   Resonance in ac circuits

(c)   Mutual induction in ac circuits

(d)   interference of electromagnetic waves

Answer: (b)

35. An electron moving with speed v and a photon moving with speed c, have same D-Broglie wavelength. The ratio of kinetic energy of electron to that of photon is :

(a)   3c/v

(b)   v/3c

(c)   v/2c

(d)   2c/v

Answer: (c)

PART-II (CHEMISTRY)

36. Assuming fully decomposed, the volume of CO2 released at STP on heating 9.85 g of BaCO3 (Atomic mass, Ba = 137) will be

(a)   1.12 L

(b)   2.24 L

(c)   4.06 L

(d)   0.84 L

Answer: (a)

37. Among the following, the species having the smallest bond is

(a)   NO

(b)   NO+

(c)   O2

(d)   NO

Answer: (b)

38. The oxidation number of phosphorus in Ba(H2PO2)2 is

(a)   +3

(b)   +2

(c)   +1

(d)   −1

Answer: (c)

39. The correct order of thermal stability of hydroxides is:

(a)   Ba(OH)2 < Ca(OH)2 < Sr(OH)2 < Mg(OH)2

(b)   Mg(OH)2 < Sr(OH)2 < Ca(OH)2 < Ba(OH)2

(c)   Mg(OH)2 < Ca(OH)2 < Sr(OH)2 < Ba(OH)2

(d)   Ba(OH)2 < Sr(OH)2 < Ca(OH)2 < Mg(OH)2

Answer: (d)

40. Which of the following has correct increasing basic strength?

(a)   MgO < BeO < CaO < BaO

(b)   BeO < MgO < CaO < BaO

(c)   BaO < CaO < MgO < BeO

(d)   CaO < BaO < BeO < MgO

Answer: (b)

41. Water sample is reported to be highly polluted if BOD (Biological Oxygen Demand) value of sample becomes

(a)   more than 17 ppm.

(b)   equal to 10 ppm.

(c)   equal to 5 ppm.

(d)   less than 5 ppm.

Answer: (a)

42. 200 mL of an aqueous solution of a protein contains its 1.26 g. The osmotic pressure of this solution at 300 K is found to be 2.57 × 103 The molar mass of protein will be

(R = 0.083 L bar mol1 K1)

(a)   51022 g mol1

(b)   122044 g mol1

(c)   31011 g mol1

(d)   61038 g mol1

Answer: (d)

43. Lyophilic sols are more stable than lyophobic sols because:

(a)   the colloidal particles have positive charge

(b)   the colloidal particles have negative charge

(c)   the colloidal particles are solvated

(d)   there is strong electrostatic repulsion between the colloidal particles

Answer: (c)

44. Which of the following is not permissible arrangement of electrons in an atom?

(a)   n = 5, l = 3, m = 0, s = +1/2

(b)   n = 3, l = 2, m = −3, s = −1/2

(c)   n = 3, l = 2, m = −2, s = −1/2

(d)   n = 4, l = 0, m = 0, s = −1/2

Answer: (b)

45. The value of van der Waals constant ‘a’ for gases O2, N2, NH3 and CH4 are 1.360, 1.390, 4.170 and 2.253 litre2 atm mol2 The gas which can most easily be liquefied is :

(a)   O2

(b)   N2

(c)   NH3

(d)   CH4

Answer: (c)

46. Which one of the following does not have a pyramidal shape?

(a)   (CH3)3N

(b)   (SiH3)3N

(c)   P(CH3)3

(d)   P(SiH3)3

Answer: (b)

47. Boric acid is polymeric due to

(a)   its acidic nature

(b)   the presence of hydrogen bonds

(c)   its monobasic nature

(d)   its geometry

Answer: (b)

48. Which of the following order is not correct?

(a)   MeBr > Me2CHBr > Me3CBr > Et3CBr(SN2)

(b)   PhCH2Br > PhCHBrMe2 > PhCBrMePh(SN1)

(c)   MeI > MeBr > MeCl > MeF (SN2)

(d)   All are correct

Answer: (b)

49. A catalyst is a substance which :

(a)   is always in the same phase as in the reaction

(b)   alters the equilibrium in a reaction

(c)   does not participate in the reaction but alters the rate of reaction

(d)   participates in the reaction and provides an easier pathway for the same

Answer: (a)

50. Which of the following is a non-reducing sugar?

(a)   Lactose

(b)   Fructose

(c)   Sucrose

(d)   Maltose

Answer: (c)

51. An ideal gas expands against a constant external pressure of 2.0 atmosphere from 20 litre to 40 litre and absorbs 10 kJ of heat from surrounding. What is the change in internal energy of the system?

(given : 1 atm-litre = 101.3 J)

(a)   4052 J

(b)   5948 J

(c)   14052 J

(d)   9940 J

Answer: (b)

52. The polymer used for optical lenses is:

(a)   polypropylene

(b)   polyvinyl chloride

(c)   polythene

(d)   polymethyl methacrylate

Answer: (d)

53. Which of the following arrangements represents the increasing order (smallest to largest) of ionic radii of the given species O2, S2, N3, P3?

(a)   O2− < N3− < S2− < P3−

(b)   O2− < P3− < N3− < S2−

(c)   N3 < O2− < P3− < S2−

(d)   N3− < S2− < O2− < P3−

Answer: (a)

54. The IUPAC name of the following compound is

(a)   (E)-2-hepten-4-yne

(b)   (Z)-5-hepten-3-yne

(c)   (E)-5-hepten-3-yne

(d)   (Z)-2-hepten-4-yne

Answer: (a)

55. In CsCl type structure, the co-ordination number of Cs+ and Cl respectively are

(a)   6, 6

(b)   6, 8

(c)   8, 8

(d)   8, 6

Answer: (c)

56. Which one of the following reactions will not result in the formation of carbon-carbon bond?

(a)   Reimer-Tiemann reaction

(b)   Friedel Craft’s acylation

(c)   Wurtz reaction

(d)   Cannizzaro reaction

Answer: (d)

57. Water is :

(a)   more polar than H2S

(b)   more or less identical in polarity with H2S

(c)   less polar than H2S

(d)   None of these

Answer: (a)

58. Carboxylic acids are more acidic than phenol and alcohol because of

(a)   intermolecular hydrogen bonding

(b)   formation of dimers

(c)   highly acidic hydrogen

(d)   resonance stabilization of their conjugate

Answer: (d)

59. The order of increasing sizes of atomic radii among the elements O, S, Se and As is:

(a)   As < S < O < Se

(b)   Se < S < As < O

(c)   O < S < As < Se

(d)   O < S < Se < As

Answer: (d)

60. Bauxite ore is generally contaminated with impurity of oxides of two elements X and Y. Which of the following statement is correct?

(a)   X is a non-metal and belongs to the third period while Y is a metal and belongs to the fourth period.

(b)   One of two oxides has three-dimensional polymeric structure.

(c)   Both (a) and (b) are correct.

(d)   None of the above

Answer: (c)

61. The partial pressure of CH­3OH(g), CO(g) and H2(g) in equilibrium mixture for the reaction, CO(g) + 2H2(g) ⇌ CH3OH(g) are 2.0, 1.0 and 0.1 atm respectively at 427° The value of Kp for the decomposition of CH3OH to CO and H2 is

(a)   102 atm

(b)   2 × 102 atm−1

(c)   50 atm2

(d)   5 × 10−3 atm2

Answer: (d)

62. The conjugate base of (CH3)2NH2+ is

(a)   (CH3)2NH

(b)   (CH3)2N+

(c)   (CH3)3N+

(d)   (CH3)2N

Answer: (a)

63. Which of the following is not present in a nucleotide?

(a)   Guanine

(b)   Cytosine

(c)   Adenine

(d)   Tyrosine

Answer: (d)

64. The shape of [Cu(NH3)4]2+ is

(a)   tetrahedral

(b)   square planar

(c)   pyramidal

(d)   octahedral

Answer: (b)

65. Heroin is a derivative of

(a)   cocaine

(b)   morphine

(c)   caffeine

(d)   nicotine

Answer: (b)

66. The limiting equivalent conductivity of NaCl, KCl and KBr are 126.5, 150.0 and 151.5 S cm2 eq1, respectively. The limiting equivalent ionic conductivity for Br is 78 Scm2eq1. The limiting equivalent ionic conductivity for Na+ ions would be :

(a)   128

(b)   125

(c)   49

(d)   50

Answer: (d)

67. Rate of dehydration of alcohols follows the order:

(a)   2° > 1° > CH3OH > 3°

(b)   3° > 2° > 1° > CH3OH

(c)   2° > 3° > 1° > CH3OH

(d)   CH3OH > 1°> 2° > 3°

Answer: (b)

68. An alkene having molecular formula C7H14 was subjected to ozonolysis in the presence of zinc dust. An equimolar amount of the following two compounds was obtained

The IUPAC name of the alkene is

(a)   3, 4-dimethyl-3-pentene

(b)   3, 4-dimethyl-2-pentene

(c)   2, 3-dimethyl-3-pentene

(d)   2, 3-dimethyl-2-pentene

Answer: (d)

69. Lanthanoid contraction can be observed in

(a)   At

(b)   Gd

(c)   Ac

(d)   Lw

Answer: (b)

70.  The form of iron obtained from blast furnace is:

(a)   Steel

(b)   Cast Iron

(c)   Pig Iron

(d)   Wrought Iron

Answer: (b)

PART – III (MATHEMATICS)

71. A class has 175 students. The following data shows the number of students opting one or more subjects. Maths-100, Physics-70, Chemistry-40, Maths and Physics-30, Maths and Chemistry-28, Physics and Chemistry-23, Maths, Physics and Chemistry-18. How many have offered Maths alone?

(a)   35

(b)   48

(c)   60

(d)   22

Answer: (c)

72. Let R be a relation on the set N be defined by |(x, y)|x, y ∈ N, 2x + y = 41}. Then, R is

(a)   Reflexive

(b)   Symmetric

(c)   Transitive

(d)   None of these

Answer: (d)

73. The function f : R → R defined by f(x) = x2 + x is.

(a)   one-one

(b)   onto

(c)   many-one

(d)   None of these

Answer: (c)

74. If 12 cot2 θ – 31 cosec 0 + 32 = 0, then the value of sin θ is

(a)   3/5 or 1

(b)   2/3 or −2/3

(c)   4/5 or 3/4

(d)   ±1/2

Answer: (c)

75. The modulus of  is

(a)   2

(b)   4

(c)   3√2

(d)   2√2

Answer: (d)

76. If α, β are the roots of the equation ax2 + bx + c = 0, then 

(a)   2/a

(b)   2/b

(c)   2/c

(d)   −2/a

Answer: (d)

77. The solution set of the inequality 37 – (3x + 5) ≥ 9x – 8 (x – 3) is

(a)   (−∞, 2)

(b)   (−∞, −2)

(c)   (−∞, 2]

(d)   (−∞, −2]

Answer: (c)

78. If n+2C8 : n – 2 P4 = 57 : 16, then the value of n is:

(a)   20

(b)   19

(c)   18

(d)   17

Answer: (b)

79. The middle term in the expansion of  is

(a)   10C5

(b)   10C6

(c) 

(d)   10C5x10

Answer: (a)

80. The fourth, seventh and tenth terms of a GP. Are p, q, r respectively. then :

(a)   p2 = q2 + r2

(b)   q2 = pr

(c)   p2 = qr

(d)   pqr + pq + 1 = 0

Answer: (b)

81. The point (t2 + 2t + 5, 2t2 + t – 2) lies on the line x + y = 2 for

(a)   All real values of t

(b)   Some real values of t

(c) 

(d)   None of these

Answer: (d)

82. The equations of the lines which cuts off an intercept −1 from y-axis and equally inclined to the axes are

(a)   x – y + 1 = 0, x + y + 1 = 0

(b)   x – y – 1 = 0, x + y – 1 = 0

(c)   x – y – 1 = 0, x + y + 1 = 0

(d)   None of these

Answer: (c)

83. The distance between the parallel lines 3x – 4y + 7 = 0 and 3x – 4y + 5 = 0 is a/b. Value of a + b is

(a)   2

(b)   5

(c)   7

(d)   3

Answer: (c)

84. For what value of k, does the equation

9x2 + y2 = k(x2 – y2 – 2x)

represent equation of a circle?

(a)   1

(b)   2

(c)   −1

(d)   4

Answer: (d)

85. A parabola has the origin as its focus and the line x = 2 as the directrix. Then the vertex of the parabola is at

(a)   (0, 2)

(b)   (1, 0)

(c)   (0, 1)

(d)   (2, 0)

Answer: (b)

86. Equation of the ellipse whose axes are the axes of coordinates and which passes through the point (−3, 1) and has eccentricity  is

(a)   5x2 + 3y2 – 48 = 0

(b)   3x2 + 5y2 – 15 = 0

(c)   5x2 + 3y2 – 32 = 0

(d)   3x2 + 5y2 – 32 = 0

Answer: (d)

87. The co-ordinates of the point which divides the join of the points (2, −1, 3) and (4, 3, 1) in the ratio 3 : 4 internally are given by :

(a)   (2/7, 20/7, 10/7)

(b)   (10/7, 15/7, 2/7)

(c)   (20/7, 5/7, 15/7)

(d)   (15/7, 20/7, 3/7)

Answer: (c)

88. The relationship between a and b, so that the function f defined by

is continuous at x = 3, is

(a)   a = b + 2/3

(b)   a – b = 3/2

(c)   a + b = 2/3

(d)   a + b = 2

Answer: (a)

89. at x = 0 is

(a)   continuous as well as differentiable

(b)   differentiable but not continuous

(c)   continuous but not differentiable

(d)   neither continuous nor differentiable

Answer: (c)

90. The variance of the data 2, 4, 6, 8, 10 is

(a)   8

(b)   7

(c)   6

(d)   None of these

Answer: (a)

91. Find the probability of getting the sum as a perfect square number when two dice are thrown together.

(a)   5/12

(b)   7/18

(c)   7/36

(d)   None of these

Answer: (c)

92. The principal value of  is

(a)   −5π/3

(b)   5π/3

(c)   −π/3

(d)   4π/3

Answer: (c)

93. If the system of linear equations

x + ky + 3z = 0

3x + ky – 2z = 0

2x + 4y – 3z = 0

Has a non-zero solution (x, y, z), then xz/y2 is equal to :

(a)   10

(b)   −30

(c)   30

(d)   −10

Answer: (a)

94. The value of definite integral 

(a)   0

(b)   π/4

(c)   π/2

(d)   π

Answer: (a)

95. The area enclosed between the graph of y = x3 and the lines x = 0, y = 1,  y = 8 is

(a)   45/4

(b)   14

(c)   7

(d)   None of these

Answer: (a)

96. The total number of 3-digit numbers, the sum of whose digits is even, is equal to

(a)   450

(b)   350

(c)   250

(d)   325

Answer: (a)

97. To fill 12 vacancies, there are 25 candidates of which five are from scheduled caste. If 3 of the vacancies are reserved for scheduled caste candidates while the rest are open to all, then the number of ways in which the selection can be made

(a)   5C3 × 22C9

(b)   22C95C3

(c)   22C3 + 5C3

(d)   None of these

Answer: (a)

98. are in A.P. then,

(a)   p, q, r are in A.P.

(b)   p2, q2, r2 are in A.P

(c)   1/p, 1/q, 1/r are in A.P

(d)   p + q + r are in A.P

Answer: (b)

99. The sum of the first n terms of the series 12 + 2.22 + 32 + 2.42 + 52 + 2.62 + … is  when n is even. When n is odd the sum is

Answer: (b)

100. The locus of a point that is equidistant from the lines x + y – 2√2 = 0 and x + y – √2 = 0 is

(a)   x + y – 5√2 = 0

(b)   x + y – 3√2 = 0

(c)   2x + 2y – 3√2 = 0

(d)   2x + 2y – 5√2 = 0

Answer: (c)

101. The point diametrically opposite to the point P(1, 0) on the circle x2 + y2 + 2x + 4y – 3 = 0 is

(a)   (3, −4)

(b)   (−3, 4)

(c)   (−3, −4)

(d)   (3, 4)

Answer: (c)

102. For the parabola y2 = −12x, equation of directrix is x = a. The value of ‘a’ is

(a)   3

(b)   4

(c)   2

(d)   6

Answer: (a)

103. The eccentricity of the curve 2x2 + y2 – 8x – 2y = 0 is :

(a)   1/2

(b)   1/√2

(c)   2/3

(d)   3/4

Answer: (b)

104. The equation of the hyperbola with vertices at (0, ±6) and e = 5/3 is

Answer: (b)

105. is equal to:

(a)   0

(b)   12 cos2x − 10 sin2x

(c)   12 cos2x – 10 sin2x – 2

(d)   10 sin 2x

Answer: (a)

106. The function  is

(a)   differentiable at x = 2

(b)   not differentiable at x = 2

(c)   continuous at x = 2

(d)   None of these

Answer: (b)

107. The local minimum value of the function f given by f(x) = 3 + |x|, x ∈ R is

(a)   1

(b)   2

(c)   3

(d)   0

Answer: (c)

108. Value of  is

(a)   π/2

(b)   −π/2

(c)   π/4

(d)   None of these

Answer: (c)

109. The equation of the plane which bisects the angle between the planes 3x – 6y + 2z + 5 = 0 and 4x – 12y + 3z – 3 = 0 which contains the origin is

(a)   33x – 13y + 32x + 45 = 0

(b)   x – 3y + z – 5 =0

(c)   33x + 13y + 32x + 45 = 0

(d)   None of these

Answer: (d)

110. An urn contains five balls. Two balls are drawn and found to be white. The probability that all the balls are white is

(a)   1/10

(b)   3/10

(c)   3/5

(d)   1/2

Answer: (d)

PART-IV (APTITUDE TEST)

Directions (Qs. 111-113): Study the following table to answer the given questions:

111. What is the total marks obtained by Meera in all the subject?

(a)   448

(b)   580

(c)   470

(d)   74.67

Answer: (c)

112. What is the average marks obtained by these seven students in History? (rounded off to two digits)

(a)   72.86

(b)   27.32

(c)   24.86

(d)   29.14

Answer: (a)

113. How many students have got 60% or more marks in all the subjects?

(a)   One

(b)   Two

(c)   Three

(d)   Four

Answer: (b)

114. A series is given, with one term missing. Choose the correct alternative from the given ones that will complete the series.

5, 11, 24, 51, 106, _____?

(a)   122

(b)   217

(c)   120

(d)   153

Answer: (b)

115. In a certain code BANKER is written as LFSCBO. How will CONFER be written in that code?

(a)   GFSDPO

(b)   FGSDOP

(c)   GFSEPO

(d)   FHSDPO

Answer: (a)

116. Kailash faces towards north. Turnings to his right, he walks 25 metres. He then turns to his left and walks 30 metres. Next, he moves 25 metres to his right. He then turns to the right again and walks 55 metres. Finally, he turns to the right and moves 40 metres. In which direction is he now from his starting point?

(a)   South-West

(b)   South

(c)   North-West

(d)   South-East

Answer: (d)

117. An accurate clock shows 8 O’clock in the morning. Through how many degrees will the hour hand rotate when the clock shows 2 O’clock in the afternoon?

(a)   144°

(b)   150°

(c)   168°

(d)   180°

Answer: (d)

118. Two statements are given followed by three conclusions numbered I, II and III. Assuming the statements to be true, even if they seem to be at variance with commonly known facts, decide which of the conclusions logically follow(s) from the statements.

Statements:

All utensils are spoons.

All bowls are spoons.

Conclusions:

(I) No utensil is a bowls.

(II) Some utensils are bowls

(III) No spoon is a utensil.

(a)   Only conclusions I follows

(b)   Conclusions I and III follow

(c)   Either conclusion I or II follows

(d)   Only conclusion III follows

Answer: (c)

Directions (Qs. 119-120) : Each of the questions below consists of a question and two statements numbered I and II given below it. You have to decide whether the data provided in the statements are sufficient to answer the question.

Read both the statements and Give answer

(a) if the data in statement I alone are sufficient to answer the question, while the data in statement II alone are not sufficient to answer the question.

(b) If the data in statement II alone are sufficient to answer the question, while the data is statement I alone are not sufficient to answer the question.

(c) if the data in both the statements I and II together are not sufficient to answer the question.

(d) if the data in both the statements I and II together are necessary to answer the question.

119. What was the ratio between the ages of P and Q four years ago?

(I) The ratio between the present ages of P and Q is 3:4.

(II) The ratio between the present ages of Q and R is 4:5.

Answer: (c)

120. What was the cost price of the suitcase purchased by Samir?

(I) Samir got 25 percent concession on the labeled price.

(II) Samir sold the suitcase for Rs. 2000 with 25 percent profit on the labeled price:

Answer: (d)

PART-V (ENGLISH)

121. Read the following passage and answer the question that follows. Choose the correct answer.

His instrument struck against something hard, dangerously near the kidney…. “ It is not quite at the kidney my friend,” Sadao murmured…. “My friend,” he always called his parents and so he did now, forgetting that this was his enemy.

To whom does Sadao attend to in the lines above?

(a)   A relative

(b)   His friend

(c)   His enemy

(d)   A patient

Answer: (c)

122. Choose the correct pronunciation for the word ‘sorbet’ from the following options:

(a)   sore-bet

(b)   sore-bay

(c)   sore-bye

(d)   shore-bay

Answer: (b)

123. What is the correct syllable division of the word ‘indomitable’?

(a)   in – do – mit – able

(b)   in – dom-i – t  – ble

(c)   in – do – mo – ta – be

(d)   in – dom – i – table

Answer: (b)

124. Read the following passage and the question below. Choose the correct answer.

Gandhi never contented himself with large political or economic solutions. He saw the cultural and social backwardness in the Champaran villages and wanted to do something about it immediately. He appealed to teachers.

Which of the following statements is true about the passage?

(a)   Gandhi was dissatisfied with political or economic solutions

(b)   Gandhi was interested in the welfare of teachers of Champaran villages

(c)   Gandhi was happy about the cultural and social backwardness of Champaran villages

(d)   Gandhi was hopeful that teachers could save villages from cultural and social backwardness

Answer: (d)

125. Choose the correct meaning of the idiom ‘a bolt out of the blue’ from the given options:

(a)   Something totally unexpected

(b)   Lightning and thunderstorm

(c)   To do something kind

(d)   To mourn after someone

Answer: (a)

VITEEE Examination Previous Year Question Paper 2021 With Answer Key

VITEEE SOLVED PAPER-2021

PART –I (PHYSICS)

1. The distance of the centres of moon and earth is D. The mass of earth is 81 times the mass of the moon. At what distance from the centre of the earth, the gravitational force will be zero?

(a)   D/2

(b)   2D/3

(c)   4D/3

(d)   9D/10

Answer: (d)

2. Two wires A and B are of the same material. Their lengths are in the ratio of 1 : 2 and the diameter are in the ratio 2 : 1. If they are pulled by the same force, then increase in length will be in the ratio of

(a)   2 : 1

(b)   1 : 4

(c)   1 : 8

(d)   8 : 1

Answer: (c)

3. If x = at + bt2, where x is the distance travelled by the body in kilometers while t is the time in seconds, then the unit of b is

(a)   km/s

(b)   kms

(c)   km/s2

(d)   kms2           

Answer: (c)

4. A soap bubble of radius r1 is placed on another soap bubble of radius r2(r1 < r2). The radius R of the soapy film separating the two bubbles is

Answer: (c)

5. A charge q is moving with a velocity v parallel to a magnetic field B. Force on the charge due to magnetic field is

(a)   q v B

(b)   q B/v

(c)   zero

(d)   B v/q

Answer: (c)

6. Two spheres A and B of masses m and 2m and radii 2R and R respectively are placed in contact as shown. The COM of the system lies

(a)   inside A

(b)   inside B

(c)   at  the point of contact

(d)   None of these

Answer: (c)

7. Identify the correct statement.

(a)   Static friction depends on the area of contact

(b)   Kinetic friction depends on the area of contact

(c)   Coefficient of kinetic friction is more than the coefficient of static friction

(d)   Coefficient of kinetic friction is less than the coefficient of static friction

Answer: (d)

8. The distance travelled by a particle starting from rest and moving with an acceleration 4/3 ms2, in the third second is:

(a)   6m

(b)   4m

(c)   10/3 m

(d)   19/3 m

Answer: (c)

9. Photoelectric work function of a metal is 1 eV. Light of wavelength λ = 3000 Å falls on it. The photo electrons come out with a maximum velocity of :

(a)   10 metres/sec

(b)   102 metres/sec

(c)   104 metres/sec

(d)   106 metres/sec

Answer: (d)

10. The coefficient of apparent expansion of mercury in a glass vessel is 153 × 106/°C and in a steel vessel is 144 × 106/° If α for steel is 12 106/°C, then that of glass is

(a)   9 × 106/°C

(b)   6 × 106/°C

(c)   36 × 106/°C

(d)   27 × 106/°C

Answer: (a)

11. A step-up transformer operates on a 230 V line and supplies a load of 2 ampere. The ratio of the primary and secondary windings is 1 : 25. The current in the primary is

(a)   15 A

(b)   50 A

(c)   25 A

(d)   12.5 A

Answer: (b)

12. Two bodies of same mass are projected with the same velocity at an angle 30° and 60° The ratio of their horizontal ranges will be

(a)   1 : 1

(b)   1 : 2

(c)   1 : 3

(d)   2 : √2

Answer: (a)

13. Two point charges +3 μC and +8 μC repel each other with a force of 40 N. If a charge of −5 μC is added to each of them, then the force between them will become

(a)   −10N

(b)   +10N

(c)   +20N

(d)   −20N

Answer: (a)

14. A sphere rolls down on an inclined plane of inclination θ. What is the acceleration as the sphere reaches the bottom?

Answer: (a)

15. A given ray of light suffers minimum deviation in an equilateral prism P. Additional prisms Q and R of identical shape and of same material as that of P are now combined as shown in figure. The ray will now suffer

(a)   greater deviation

(b)   no deviation

(c)   same deviation as before

(d)   total internal reflection

Answer: (c)

16. The current in the 1Ω resistor shown in the circuit is

(a)   2/3 A

(b)   3 A

(c)   6 A

(d)   2 A

Answer: (d)

17. The root mean square velocity of hydrogen molecules at 300 K is 1930 metre/sec. Then the r.m.s velocity of oxygen molecules at 1200 K will be

(a)   482.5 metre/sec

(b)   965 metre/sec

(c)   1930 metre/sec

(d)   3860 metre/sec

Answer: (b)

18. Lenz’s law gives

(a)   the magnitude of the induced e.m.f.

(b)   the direction of the induced current

(c)   both the magnitude and direction of the induced current

(d)   the magnitude of the induced current

Answer: (b)

19. A parallel plate capacitor with air between the plates has a capacitance of 8 pF. Calculate the capacitance if the distance between the plates is reduced by half and the space between them is filled with a substance of dielectric constant. (k = 6)

(a)   72 pF

(b)   81 pF

(c)   84 pF

(d)   96 pF

Answer: (d)

20. For a particle executing S.H.M. the displacement x is given by x = A cos ω Identify the graph which represents the variation of potential energy (P.E.) as a function of time t and displacement x.

(a)   I, III

(b)   II, IV

(c)   II, III

(d)   I, IV

Answer: (a)

21. A radioactive sample contains 103 kg each of two nuclear species A and B with half-life 4 days and 8 days respectively. The ratio of the amounts of A and B after a period of 16 days is

(a)   1 : 2

(b)   4 : 1

(c)   1 : 4

(d)   2 : 1

Answer: (c)

22. A string of 7 m length has a mass of 0.035 kg. If tension in the string is 60.5 N, then speed of a wave on the string is

(a)   77 m/s

(b)   102 m/s

(c)   110 m/s

(d)   165 m/s

Answer: (c)

23. The following circuit represents

(a)   OR gate

(b)   AND gate

(c)   NAND gate

(d)   None of these

Answer: (d)

24. A straight section PQ of a circuit lies along the X-axis from x = −a/2 to x = a/2 and carries a steady current i. The magnetic field due to the section PQ at a point = +a will be

(a)   proportional to a

(b)   proportional to a2

(c)   proportional to 1/a

(d)   zero

Answer: (d)

25. A source producing sound of frequency 170 Hz is approaching a stationary observer with a velocity 17 ms1. The apparent change in the wavelength of sound heard by the observer is (speed of sound in air = 340 ms1)

(a)   0.1 m

(b)   0.2 m

(c)   0.4 m

(d)   0.5 m

Answer: (a)

PART-II (CHMISTRY)

26. Consider the following reactions:

NaCl + K2Cr2O7 (Conc.) → (A) + Side products

(A) + NaOH → (B) + Side products

(B) + H2SO4 (dilute) + H2O2 → (C) + Side products

The sum of the total number of atoms in one molecule each of (A), (B) and (C) is ______.

(a)   18

(b)   15

(c)   21

(d)   20

Answer: (a)

27. Xenon hexafluoride on partial hydrolysis produces compounds ‘X’ and ‘Y’. Compounds ‘X’, ‘Y’ and the oxidation state of Xe are respectively :

(a)   XeOF4(+6) and XeO3(+6)

(b)   XeO2(+4) and XeO3 (+6)

(c)   XeOF4(+6) and XeO2F2(+6)

(d)   XeO2F2(+6) and XeO2(+4)

Answer: (c)

28. The edge length of unit cell of a metal having molecular weight 75 g/mol is 5 Å which crystallizes in cubic lattice. If the density is 2g/cc then find the radius of metal atom. (NA = 6 × 1023). Give the answer in pm.

(a)   217 pm

(b)   210 pm

(c)   220 pm

(d)   205 pm

Answer: (a)

29. Consider the following statements :

(I) Increase in concentration of reactant increases the rate of a zero order reaction.

(II) Rate constant k is equal to collision frequency A if Ea = 0.

(III) Rate constant k is equal to collision frequency A if Ea = ∞.

(IV) In k vs T is a straight line.

(V) In k vs 1/T is a straight line.

Correct statements are

(a)   I and IV

(b)   II and V

(c)   III and IV

(d)   II and III

Answer: (b)

30. To deposit 0.634 g of copper by electrolysis of aqueous cupric sulphate solution, the amount of electricity required (in coulombs) is

(a)   1930

(b)   3960

(c)   4825

(d)   9650

Answer: (a)

31. In the following skew conformation of ethane, Hʹ−C−C−Hʹʹ dihedral angle is :

(a)   58°

(b)   149°

(c)   151°

(d)   120°

Answer: (b)

32. What is the product of following reaction?

Answer: (d)

33. In the following sequence of reactions,

the compound D is

(a)   propanal     

(b)   butanal

(c)   n-butyl alcohol

(d)   n-propyl alcohol.

Answer: (d)

34. Which of the following reactions can produce aniline as main product?

(a)   C6H5NO2 + Zn/KOH

(b)   C6H5NO2 + Zn/NH4Cl

(c)   C6H5NO2 + LiAlH4

(d)   C6H5NO2 + Zn/HCl

Answer: (d)

35. Secondary structure of protein refers to

(a)   mainly denatured proteins and structure of prosthetic groups

(b)   three-dimensional structure, especially the bond between amino acid residues that are distinct from each other in the polypeptide chain

(c)   linear sequence of amino acid residues in the polypeptide chain

(d)   regular folding patterns of continuous portions of the polypeptide chain

Answer: (d)

36. The increasing order for the values of e/m (charge/mass) is

(a)   e, p, n, α

(b)   n, p, e, α

(c)   n, p, α, e

(d)   n, α, p, e

Answer: (d)

37. In which of the following pairs both the ions are coloured in aqueous solutions?

(a)   Sc3+, Ti3+

(b)   Sc3+, Co2+

(c)   Ni2+, Cu+

(d)   Ni2+, Ti3+

Answer: (d)

38. The total number of possible isomers for square-planar [Pt(Cl)(NO2)(NO3)(SCN)]2 is:

(a)   16

(b)   12

(c)   8

(d)   24

Answer: (b)

39. For the reaction,

2SO2(g) + O2(g) ⇌ 2SO3(g),

∆H = −57.2 kJ mol1 and Kc = 1.7 × 1016

Which of the following statement is INCORRECT?

(a)   The equilibrium constant is large suggestive of reaction going to completion and so no catalyst is required.

(b)   The equilibrium will shift in forward direction as the pressure increases.

(c)   The equilibrium constant decreases as the temperature increases.

(d)   The addition of inert gas at constant volume will not affect the equilibrium constant.

Answer: (a)

40. The half-life of a reaction is inversely proportional to the square of the initial concentration of the reactant. Then the order of the reaction is

(a)   0

(b)   1

(c)   2

(d)   3

Answer: (d)

41. A galvanic cell is set up from electrodes A and B

Electrode A : Cr2O72/Cr3+, E°red = +1.33 V

Electrode B : Fe3+/Fe2+, E°red = 0.77 V

Which of the following statements is false?

(a)   Standard e.m.f of the cell is 0.56 V

(b)   Current will flow from electrode A to B in the external circuit

(c)   A will act as cathode and have positive polarity

(d)   None of these

Answer: (d)

42. Keto-enol tautomerism is observed in :

Answer: (d)

43. In a set of reactions, ethylbenzene yield a product D.

Answer: (d)

44. What will be the final product in the following reaction sequence –

(a)   CH3CH2CONH2

(b)   CH3CH2COBr

(c)   CH3CH2NH2

(d)   CH3CH2CH2NH2

Answer: (c)

45. In a set of reactions acetic acid yielded a product D.

The structure of (D) would be –

Answer: (d)

46. In fructose, the possible optical isomers are

(a)   12

(b)   8

(c)   16

(d)   4

Answer: (b)

47. The position of both, an electron and a helium atom is known within 1.0 nm. Further the momentum of the electron is known within 5.0 × 1026 kg ms1. The minimum uncertainty in the measurement of the momentum of the helium atom is

(a)   50 kg ms1

(b)   80 kg ms1

(c)   8.0 × 1026 kg ms1

(d)   5.0 × 1026 kg ms1

Answer: (d)

48. The value of log10K for a reaction A ⇌ B is

(Given : ∆r298 K = −54.07 kJ mol1,

r298K = 10 JK1 mol1 and R = 8.314 JK1 mol1; 2.303 × 8.314 × 298 = 5705)

(a)   5

(b)   10

(c)   95

(d)   100

Answer: (b)

49. If C(s) + O2(g) → CO2(g); ∆H = R and

then heat of formation of CO is:

(a)   R + S

(b)   R – S

(c)   R × S

(d)   S – R

Answer: (b)

50. Which of the following compounds does not follow Markownikoff’s law?

(a)   CH3CH = CH2

(b)   CH2CHCl

(c)   CH3CH = CHCH3

(d)   None

Answer: (c)

PART – III (MATHEMATICS)

51. The value of c in Rolle’s Theorem for the function f(x) = ex sin x, x ∈ [0, π] is

(a)   π/6

(b)   π/4

(c)   π/2

(d)   3π/4

Answer: (d)

52. The equations 2x + 3y + 4 = 0; 3x + 4y + 6 = 0 and 4x + 5y + 8 = 0 are

(a)   consistent with unique solution

(b)   inconsistent

(c)   consistent with infinitely many solutions

(d)   None of the above

Answer: (a)

53. The shortest distance between the lines x = y + 2 = 6z – 6 and x + 1 = 2y = −12z is

(a)   1/2

(b)   2

(c)   1

(d)   3/2

Answer: (b)

54. If the tangent at P(1, 1) on y2 = x(2 – x)2 meets the curve again at Q, then Q is

(a)   (2, 2)

(b)   (−1, −2)

(c)   (9/4, 3/8)

(d)   None of these

Answer: (c)

55. If  then at x = 0, f(x)

(a)   has no limit

(b)   is discontinuous

(c)   is continuous but not differentiable

(d)   is differentiable

Answer: (b)

56. Radius of the circle (x + 5)2 + (y – 3)2 = 36 is

(a)   2

(b)   3

(c)   6

(d)   5

Answer: (c)

57. If  and  is equal to :

Answer: (b)

58. If (−4, 5) is one vertex and 7x – y + 8 = 0 is one diagonal of a square, then the equation of second diagonal is

(a)   x + 3y = 21

(b)   2x – 3y = 7

(c)   x + 7y = 31

(d)   2x + 3y = 21

Answer: (c)

59. p ⇒ q can also be written as

(a)   p ⇒ ~ q

(b)   ~p ⋁ q

(c)   ~ q ⇒ ~ p

(d)   None of these

Answer: (b)

60. Let  then

(a)   f(x) = √x

(b)   f(x) = x3/2 and g(x) = sin1x

(c)   f(x) = x2/3

(d)   None of these

Answer: (b)

61. Which one of the following is an infinite set?

(a)   The set of human beings on the earth

(b)   The set of water drops in a glass of water

(c)   The set of trees in a forest

(d)   The set of all primes

Answer: (d)

62. The domain of the function

is

(a)   [2, 3]

(b)   [−2, 4]

(c)   [−2, 2] ⋃ [3, 4]

(d)   [−2, 1] ⋃ [2, 4]

Answer: (c)

63. Area bounded by the curve y = log x and the coordinate axes is

(a)   2

(b)   1

(c)   5

(d)   2√2

Answer: (b)

64. The angle of intersection to the curve y = x2, 6y = 7 – x3 at(1, 1) is :

(a)   π/2

(b)   π/4

(c)   π/3

(d)   π

Answer: (a)

65. Angle formed by the positive Y-axis and the tangent to y = x2 + 4x – 17 at (5/2, −3/4) is

(a)   tan1 9

(b)  

(c)  

(d)   π/2

Answer: (b)

66. The value of  is

(a)   12

(b)   2

(c)   8

(d)   16

Answer: (d)

67. The relation R defined on the set A = {1, 2, 3, 4, 5} by R = {(x, y) : |x2 – y2| < 16} is given by

(a)   {(1, 1), (2, 1), (3, 1), (4, 1), (2, 3)}

(b)   {(2, 2), (3, 2), (4, 2), (2, 4)}

(c)   {(3, 3), (4, 3), (5, 4), (3, 4)}

(d)   None of these

Answer: (d)

68. 

Answer: (a)

69. The value of tan1 (1) + tan1(0) + tan1(2) + tan1(3) is equal to

(a)   π

(b)   5π/4

(c)   π/2

(d)   None of these

Answer: (a)

70. In a culture the bacteria count is 1,00,000. The number is increased by 10% in 2 hours. In how many hours will the count reach 2,00,000 if the rate of growth of bacteria is proportional to the number present.

Answer: (b)

71. What is the angle between the two straight lines y = (2 – √3)x + 5 and y = (2 + √3) x – 7?

(a)   60°

(b)   45°

(c)   30°

(d)   15°

Answer: (a)

72. If the angle θ between the line  and the plane 2x – y + √λz + 4 = 0 is such that sin θ = 1/3 then the value of λ is

(a)   5/3

(b)   −3/5

(c)   3/4

(d)   −4/3

Answer: (a)

73. The distance of the point (−5, −5, −10) from the point of intersection of the line  and the plane  is

(a)   13

(b)   12

(c)   4√15

(d)   10√2

Answer: (a)

74. is equal to :

(a)   √3

(b)   1/√3

(c)   3√3/2

(d)   1/2√3

Answer: (a)

75. If  then x ∈

(a)   (−5, −2)

(b)   (−1, ∞)

(c)   (−5, −2) ∪ (−1, ∞)

(d)   None of these

Answer: (c)

PART-IV (ENGLISH & LOGICAL REASONING)

Directions (76-78): Study the paragraph and answer the questions that follow.

A training calendar and schedule for Fire Agency Specialties Team (F.A.S.T) membership is available in this office to all applicants for F.A.S.T. membership. Training will take place the third week of each month. Classes will be taught on Monday afternoons, Wednesday evenings, and Saturday afternoons.

So that the F.A.S.T. can maintain a high level of efficiency and preparedness for emergency response situations, its members must meet certain requirements.

First, in order for you to be considered for membership on F.A.S.T., your department must be a member of the F.A.S.T. organization, and you must have written permission from your fire chief or your department’s highest ranking administrator.

Once active, you must meet further requirements to maintain active status. These include completion of technician-level training and certification in hazardous material (hazmat) operations. In addition, after becoming a member, you must also attend a minimum of 50% of all drills conducted by F.A.S.T. and go to at least one F.A.S.T. conference. You may qualify for alternative credit for drills by proving previous experience in actual hazmat emergency response.

If you fail to meet minimum requirements, you will be considered inactive, and the director of your team will be notified. You will be placed back on active status only after you complete the training necessary to meet the minimum requirements.

76. Potential F.A.S.T. members can attend less than half of F.A.S.T. drills if they

(a)   complete technician-level training requirements.

(b)   indicate prior real emergency experience.

(c)   receive permission from their fire chief.

(d)   enroll in three weekly training sessions.

Answer: (b)

77. Which of the following is the main subject of the passage?

(a)   preparing for hazmat certification

(b)   the main goal of F.A.S.T.

(c)   completing F.A.S.T. membership requirements

(d)   learning about your department’s F.A.S.T. membership

Answer: (c)

78. Applicants must be available for training

(a)   three days each months.

(b)   three days each week.

(c)   every third month.

(d)   for 50% of classes.

Answer: (a)

79. Jatin starting from a fixed point, goes 15 m towards North and then after turning to his right, he goes 15 m. Then, he goes 10 m, 15 m and 15 m after turning to his left each time. How far is he from this starting point?

(a)   15 m

(b)   5 m

(c)   10 m

(d)   20 m

Answer: (c)

80. Examine the following statements:

(1) All members of Mohan’s family are honest.

(2) Some members of Mohan’s family are not employed.

(3) Some employed persons are not honest.

(4) Some hones persons are not employed.

Which one of the following inferences can be drawn from the above statements?

(a)   All members of Mohan’s family are employed

(b)   The employed members of Mohan’s family are honest

(c)   The honest members of Mohan’s family are not employed

(d)   The employed member of Mohan’s family are not honest

Answer: (b)

BITSAT Examination Previous Year Question Paper 2021 With Answer Key

BITSAT SOLVED PAPER-2021

PART-I

PHYSICS

1. An ideal monoatomic gas is taken round the cycle of ABCDA as shown in the p-V diagram.

The work done during the cycle is

(a)   pV

(b)   2pV

(c)   pV/2

(d)   zero

Answer: (a)

2. The initial speed of a body of mass 2.0 kg is 5 m/s. A force acts for 4 s in the direction of motion of the body, as shown in force-time graph. The impulse of force is

(a)   8.50 Ns

(b)   8 Ns

(c)   5.5 Ns

(d)   6 Ns

Answer: (a)

3. For a constant hydraulic stress on an object, the fractional change in the object’s volume (∆V/V) and its bulk modulus B are related as

Answer: (c)

4. A shunt is connected in parallel with a galvanometer. Why?

(a)   To prevent galvanometer from strong current

(b)   To convert galvanometer into ammeter

(c)   To increase the range of ammeter

(d)   All of the above

Answer: (d)

5. The phase difference between the Vout­ and V­in of CE-amplifier circuit is

(a)   90°

(b)   180°

(c)   0°

(d)   270°

Answer: (b)

6. If nth division of main scale coincides with (n + 1)th divisions of vernier scale. The least count of the vernier is (Given, one main scale division is equal to a units)

Answer: (c)

7. A uniform rod AB of length L = 1 m is sliding along two mutually perpendicular surfaces OP and OQ as shown in figure.

When the rod subtends an angle θ = 30° with OQ, then the end B has a velocity √3 m/s. The velocity of end A at that time is

(a)   1 m /s

(b)   0.5 m/s

(c)   √3 m/s

(d)   1/√3 m/s

Answer: (a)

8. A tray of mass (M) 12 kg is supported by a spring as shown in figure.

When the tray is pressed down and released, it executes SHM with a period of 1.5 s. When a block of mass m placed on the tray, then the period of SHM changes to 3.0 s. The mass of block is

(a)   36 kg

(b)   48 kg

(c)   12 kg

(d)   24 kg

Answer: (a)

9. The phasor diagram of a load represents which circuit?

(a)   Purely capacitive

(b)   Purely inductive

(c)   R-L-C circuit with XL more than XC

(d)   R-L-C circuit with XL less than XC

Answer: (c)

10. The equation of progressive wave is given by  Which one of the following is correct?

(a)   υ = 5 cm/s

(b)   λ = 18 cm

(c)   A = 0.04 cm

(d)   f = 50 Hz

Answer: (b)

11. The property of light used in optical fibre cable is

(a)   total internal reflection

(b)   refraction

(c)   interference

(d)   polarization

Answer: (a)

12. A man walks in a straight line for 5 mine with a velocity of 45 m/s. What is the speed with which he has to move in order to comeback to its original position in 1.5 min?

(a)   90 m/s

(b)   150 m/s

(c)   135 m/s

(d)   115 m/s

Answer: (b)

13. From a solid sphere of mass M and radius R, spherical portion of radius R/2 is removed as shown in the figure.

Taking gravitational potential V = 0 at r = ∞, the potential at the centre of the cavity thus formed is

(a)   −GM/2R

(b)   −GM/R

(c)   −2GM/3R

(d)   −2GM/R

Answer: (b)

14. Four charges equal to +Q are placed at the four corners of a square and a charge (−q) is at its centre. If the system is in equilibrium, then the value of –q is

Answer: (c)

15. A force F = a + bx acts on a particle in the x-direction, where a and b are constants. The work done by this force during a displacement from x =0 to x = d is

Answer: (a)

16. A proton has kinetic energy E = 100 eV which is equal to that of a photon. The wavelength of photon is λ2 and that of proton is λ1. The ratio λ21 is proportional to

(a)   E2

(b)   E1/2

(c)   E−1

(d)   E1/2

Answer: (d)

17. A block of mass 10 kg rests on a rough inclined plane making an angle of 30° with the horizontal. The coefficient of static friction between the block and the plane is 0.1. The friction force on the block is

(a)   4.9 N

(b)   49√3 N

(c)   49 N

(d)   0.1 × 49√3 N

Answer: (c)

18. Which of the following graphs show the correct relation between conductivity and temperature for a metallic conductor?

Answer: (b)

19. The radius of a muonic hydrogen atom is 2.5 × 1013 The total atomic volume (in m3) of a mole of such hydrogen atoms is

(Take, π = 3.14)

(a)   3.94 × 1014

(b)   3.09 × 1014

(c)   4 × 1014

(d)   3.9 × 1014

Answer: (a)

20. The angular momentum of a body placed at origin of mass 1 kg and having position vector  is

(a)   time dependent

(b)  

(c)  

(d)   0

Answer: (c)

21. If the earth stops rotating about its axis, then what will be the change in the value of g at a place in the equatorial plane?

(Radius of earth = 6400 km)

(a)   3.7 cm/s2

(b)   9.8 m/s2

(c)   0

(d)   3.4 cm/s2

Answer: (d)

22. Two cars approach a stationary observer from opposite sides as shown in the figure.

The observer hears to beats. If the frequency of the horn of the car B is 504 Hz, then the frequency of the horn of the car A will be

(a)   529.2 Hz

(b)   440.5 Hz

(c)   295.2 Hz

(d)   None of these

Answer: (a)

23. 

In the following circuit, assuming point A to be at zero potential, then what is the potential at point B?

(a)   1 V

(b)   2 V

(c)   4 V

(d)   3 V

Answer: (b)

24. Two plane mirrors A and B are aligned parallel to each other as shown in the figure.

A ray of light is incident at an angle 30° at a point just inside one end of A. The plane of incidence coincides with the plane of the figure. The maximum number of times, the ray undergoes reflection (excluding the first one) before it emerges out is

(a)   28

(b)   34

(c)   30

(d)   29

Answer: (d)

25. A projectile is given an initial velocity of  where  is along the ground and  is along the vertical. If g = 10 m/s2, then the equation of its trajectory is

(a)   y = x  + 5x2

(b)   y = x – 5x2

(c)   y = x2 + 5x

(d)   y = x2 – 5x

Answer: (b)

26. Two drops of equal radius R coalesce to form a bigger drop. What is the ratio of surface energy of bigger drop to smaller one?

(a)   21/3; 1

(b)   22/3 : 1

(c)   1 : 1

(d)   21/2 : 1

Answer: (a)

27. Two particles A and B of masses mA and mB respectively, are having same charge and moving on same plane. A uniform magnetic field exists perpendicular to this plane. The speeds of the particles are vA and v respectively and the trajectories are as shown in figure. Then,

(a)   mA = mB and vA = vB

(b)   mAvA > mB­vB

(c)   mA < mB and vA < υB

(d)   mAvA < mBvB

Answer: (b)

28. The potential difference applied to an X-ray tube is decreased. As a result, in the emitted radiation,

(a)   the intensity increases

(b)   the intensity decreases

(c)   the minimum wavelength increases

(d)   the minimum wavelength decreases

Answer: (c)

29. The ratio of the specific heats  in terms of degrees of freedom n is given by

Answer: (b)

30. A cyclist speeding at 6 m/s in a circle of 18 m radius makes an angle θ with the vertical. The minimum possible value of coefficient of friction between the tyres and the ground is

(a)   12.041

(b)   0.2041

(c)   11.32

(d)   10.020

Answer: (b)

31. Three particles each of mass m are kept at the vertices of an equilateral triangle of side b. Moment of inertia of the system about an axis passing through the centroid and perpendicular to its plane is

(a)   3 mb2

(b)   mb2

(c)   mb2/3

(d)  

Answer: (b)

32. An electric dipole is placed at an angle of 30° in a non-uniform electric field. The dipole will experience

(a)   torque only

(b)   translational force only in the direction of the field

(c)   translational force only in a direction normal to the direction of the field

(d)   torque as well as a translational force

Answer: (d)

33. Two coils A and B have a mutual inductance 0.001 H. The current changes in the first coil according to the equation i = i0 sin ωt, where i0 = 10A and ω = 10π rads1. The maximum value of emf in the second coil is

(a)   0.01 π V

(b)   1 πV

(c)   0.1 π V

(d)   0.05 π V

Answer: (c)

34. The current in the circuit will be

(a)   0.125 A

(b)   0.1 A

(c)   0.5 A

(d)   0.25 A

Answer: (b)

35. The bob of a pendulum of length 2m lies at P, when it reaches Q, it loses 10% of its total energy due to air resistance.

The velocity of bob at Q is

(a)   6 m/s

(b)   1 m/s

(c)   2 m/s

(d)   8 m/s

Answer: (a)

36. A particle executes SHM with a frequency f. The frequency with which its kinetic energy oscillates is

(a)   f/2

(b)   f

(c)   2f

(d)   4f

Answer: (c)

37. A radioactive sample at any instant has its disintegration rate 5000 disintegrations per min. After 5 min, the rate is 1250 disintegrations per min. Then, the disintegration constant (per min) is

(a)   0.4 loge 2

(b)   0.2 loge 2

(c)   0.1 loge 2

(d)   0.8 loge 2

Answer: (a)

38. The separation between two parallel plates of capacitor is 1 mm. What is the electric potential generates between the plates of capacitor, when electric field of 2000 N/C is applied on it?

(a)   2 V

(b)   2000 V

(c)   0.2 V

(d)   200 V

Answer: (a)

39. Choose the correct statement.

(a)   The speed of light in the meta-material is υ = c|n|.

(b)   The speed of light in the meta-material is υ = c/|n|.

(c)   The speed of light in the meta-material is υ = c.

(d)   The wavelength of the light in the meta-material (λm) is given by λm = λair|n|.

Answer: (b)

40. A T-shaped object with dimensions shown in figure, is lying on a smooth floor. A force F is applied at the point P parallel to AB, such that the object has only the translational motion without rotation. Find the location of P with respect to C.

Answer: (d)

PART II

Chemistry

41. The most volatile compound among the given option is

(a)   o-nitrophenol

(b)   p-nitrophenol

(c)   m-nitrophenol

(d)   can’t say

Answer: (a)

42. What is the magnetic moment of Ti2+?

(Given : Atomic number = 22)

(a)   √8

(b)   √10

(c)   √6

(d)   √9

Answer: (a)

43. Why only Xe can form compounds with fluorine among noble gases?

(a)   Large size

(b)   Low electronegativity

(c)   High ionization energy

(d)   Low electron gain enthalpy

Answer: (a)

44. Which of the following inert gas is used as cryogenic agent?

(a)   He

(b)   Ne

(c)   Ar

(d)   Kr

Answer: (a)

45. For the following reaction,

2A + 3B → 3C + 4D

Expression for rate of reaction is

Answer: (c, d)

46. Cyclohexanol on reacting with H2SO4 and then heating gives

(a)   cyclohexene

(b)   cyclohexanone

(c)   cyclohexanol

(d)   None of these

Answer: (a)

47. The major product obtained on reaction of 3-methylbutene with HCl is

(a)   2-chloro-2-methylbutane

(b)   3-chloro-2-methylbutane

(c)   1-chloro-2-methylbutane

(d)   3-chloro-3-methylbutane

Answer: (a)

48. If Rydberg constant is same for all elements, the angular momentum and energy of Li2+ of which orbital is equal to angular momentum and energy of 1s-orbital of hydrogen atom?

(a)   3s

(b)   4s

(c)   2p

(d)   3d

Answer: (a)

49. Sodium iodide reacts with ammonia to give

(a)   [Na(NH3)4]I

(b)   [Na(NH3)4]I3

(c)   [Na(NH3)I3]I

(d)   [Na(NH3)3I]

Answer: (a)

50. In photography, which compound is used as a fixing agent

(a)   sodium thiosulphate

(b)   ammonium thiosulphate

(c)   sodium chloride

(d)   Both (a) and (b)

Answer: (d)

51. The compound formed on reaction of epoxy ethane with NH3 and H2O is

(a)   mono-ethanol amines

(b)   di-ethanol amines

(c)   tri-ethanol amines

(d)   All of these

Answer: (d)

52. The process of removal of excess electrolyte from colloidal solution is

(a)   coagulation

(b)   dialysis

(c)   ultra-filtration

(d)   peptisation

Answer: (b)

53. Maltase converts maltose into

(a)   glucose

(b)   sucrose

(c)   fructose

(d)   starch

Answer: (a)

54. Starch is a polymer of

(a)   glucose

(b)   fructose

(c)   Both (a) and (b)

(d)   None of these

Answer: (a)

55. Methyl alcohol can be distinguished from ethyl alcohol using

(a)   Fehling solution

(b)   Schiff’s reagent

(c)   Sodium hydroxide and iodine

(d)   Phthalein fusion test

Answer: (c)

56. Sodium stearate (C17H35COONa+) is

(a)   anionic soap

(b)   cationic detergent

(c)   anionic detergent

(d)   Non-ionic soap

Answer: (a)

57. Which is not an anti-fluorite structure

(a)   Rb2S

(b)   BaF2

(c)   K2O

(d)   Li2O

Answer: (b)

58. The structure of ClF3 is

(a)   T-shape

(b)   bent shape

(c)   linear

(d)   trigonal planar

Answer: (a)

59. Which of the following is diamagnetic in nature?

(a)   O2

(b)   O22

(c)   N22+

(d)   B2

Answer: (b)

60. The correct order of bond order in SO2, SO3, SO42, SO32 is

(a)   SO32 > SO42 > SO3 = SO2

(b)   SO42 > SO3 > SO2 > SO32−

(c)   SO2 = SO3 > SO42 > SO32

(d)   Can’t say

Answer: (c)

61. Ionisation energy for H+ ion is proportional to rn, then value of n is

(a)   1

(b)   −1

(c)   2

(d)   −2

Answer: (b)

62. Role of BHA in food industries

(a)   prevent oxidation

(b)   prevent reduction

(c)   add sweeteners

(d)   None of these

Answer: (a)

63. Permanganate ion (MnO4) is dark purple coloured though Mn is in +7 oxidation state with d0 This is due to

(a)   d – d transition

(b)   charge transfer from metal to ligand

(c)   charge transfer from ligand to metal

(d)   All of the above

Answer: (c)

64. Rutherford model could not explain

(a)   electronic structure of an atom

(b)   stability of an atom

(c)   Both (a) and (b)

(d)   None of the above

Answer: (c)

65. Which is true in case of [Ni(CO)4]?

(a)   Hybridization of Ni is sp3

(b)   Hybridization of Ni is dsp2

(c)   Paramagnetic

(d)   Square planar

Answer: (a)

66. SI unit of Boltzmann’s constant is

(a)   JK1

(b)   eVK1

(c)   ergK−1

(d)   JK

Answer: (a)

67. Acetone does not undergo which type of reaction?

(a)   Substitution reaction

(b)   Polymerization reaction

(c)   Condensation reaction

(d)   Addition reaction

Answer: (a)

68. Magnetic moment of Co in [CoF6]2 of unpaired electron is

(a)   √35

(b)   √45

(c)   √40

(d)   √30

Answer: (a)

69. Some statement about heavy water are given below

(1) Heavy water is used as a moderator in nuclear

(2) Heavy water is more associated than ordinary water.

(3) Heavy water in more effective solvent than ordinary water.

Which of the above statements are correct?

(a)   1 and 2

(b)   1, 2 and 3

(c)   2 and 3

(d)   1 and 3

Answer: (a)

70. CH3−CHCl−CH2−C3 has a chiral centre, which of the following represents its R configuration?

Answer: (c)

71. In Victor Meyer’s method 0.2 g of an organic substance displaced 56 mL of air at STP the molecular weight of the compound is

(a)   56

(b)   112

(c)   80

(d)   28

Answer: (c)

72. For the complete combustion of ethanol, C2H5OH(l) + 3O2(g) → 2CO2(g) + 3H2O(l)the amount of heat produced as measured in bomb calorimeter is 1364.47 kJ mol1 at 25° Assuming ideality the enthalpy of combustion, ∆HC, for the reaction will be

(R = 8.314 JK1 mol1)

(a)   −1366.95 kJ mol1

(b)   −1361.95 kJ mol1

(c)   −1460.50 kJ mol1

(d)   −1350.50 kJ mol1

Answer: (a)

73. The incorrect expression among the following is

(a) 

(b)   In isothermal process, 

(c)  

(d)

Answer: (c)

74. The solubility of Pb(OH)2 in water is 6.7 × 106 Its solubility in a buffer solution of pH = 8 would be

(a)   1.2 × 102

(b)   1.6 × 103

(c)   1.6 × 102

(d)   1.2 × 103

Answer: (d)

75. The density (in g mL1) of a 60 M sulphuric acid solution that is 29% H2SO4 (molar mass = 98 g mol1) by mass will be

(a)   1.64

(b)   1.88

(c)   1.22

(d)   1.45

Answer: (c)

76. The relative lowering of vapour pressure of a dilute aqueous solution containing non-volatile solute is 0.0125. The molality of the solution is about

(a)   0.70

(b)   0.50

(c)   0.90

(d)   0.80

Answer: (a)

77. Equal masses of methane and oxygen are mixed in an empty container at 25° The fraction of the total pressure exerted by oxygen is

(a)   2/3

(b)

(c)   1/3

(d)   1/2

Answer: (c)

78. The molar conductivities of KCl NaCl and KNO3 are 152, 128 and 111 S cm2 mol1 What is the molar conductivity of NaNO3?

(a)   101 S cm2 mol1

(b)   87 S cm2 mol1

(c)   −101 S cm2 mol1

(d)   −391 S cm2 mol1

Answer: (b)

79. The approximate time duration in hours to electroplate 30 g of calcium form molten calcium chloride using a current of 5 A is (Atomic mass of Ca = 40)

(a)   80

(b)   10

(c)   16

(d)   8

Answer: (d)

80. Given, the reduction potential of Na+, Mg2+, Al3+ and Ag+ as 

The least stable oxide is

(a)   Ag2O

(b)   Al2O3

(c)   MgO

(d)   Na2O

Answer: (a)

PART III

(a) English Proficiency

Directions (Q. Nos. 81-83) Choose the word which best expresses the meaning of the underlined word in the sentence.

81. Decay is an immutable factor of human life.

(a)   important

(b)   unique

(c)   unchangeable

(d)   awful

Answer: (c)

82. It was an ignominious defect for the team.

(a)   shameful

(b)   admirable

(c)   unaccountable

(d)   worthy

Answer: (a)

83. His conjecture was the better than mine.

(a)   guess

(b)   fact

(c)   surprise

(d)   doubt

Answer: (a)

Directions (Q. Nos. 84-86) Fill in the blanks.

84. Freedom and equality are the ………. Rights of every human.

(a)   inalienable

(b)   inscrutable

(c)   incalculable

(d)   institutional

Answer: (d)

85. Pradeep’s face spoke …………. of the happiness he was feeling.

(a)   elegantly

(b)   tons

(c)   volumes

(d)   much

Answer: (c)

86. His speech was disappointing : it …………. All the major issues.

(a)   projected

(b)   revealed

(c)   skirted

(d)   analysed

Answer: (c)

Directions (Q. Nos. 87-89) Choose the word which is closest to the opposite in meaning of the given italicized word.

87. Hydra is biologically believed to be immortal.

(a)   undying

(b)   perishable

(c)   ancient

(d)   eternal

Answer: (b)

88. The Gupta rulers patronized all cultural activities and thus Gupta period was called the golden era in Indian History.

(a)   criticized

(b)   rejected

(c)   opposed

(d)   spurned

Answer: (c)

89. This is a barbarous

(a)   bad

(b)   good

(c)   civilized

(d)   exemplary

Answer: (c)

Directions (Q. Nos. 90-92) In each of the following questions, out of the four alternatives, choose the one which can be substituted for the given words/sentence.

90. A person who does not believe in any religion

(a)   Philatelist

(b)   Rationalist

(c)   Atheist

(d)   Pagan

Answer: (c)

91. A person who believes that pleasure is the chief good

(a)   Stoic

(b)   Hedonist

(c)   Epicure

(d)   Sensual

Answer: (c)

92. One who loves mankind

(a)   Anthropologist

(b)   Philanthropist

(c)   Seismologist

(d)   Optometrist

Answer: (b)

Directions (Q. Nos. 93-95) Choose the order of the sentence marked A, B, C, D and E to form a logical paragraph.

93. (A) Tasty and healthy food can help you bring out their best.

(B) One minute they are toddlers and next you see them in their next adventure.

(C) Your young ones seem to be growing so fast.

(D) Being their loving custodians, you always want to see them doing well.

(E) Their eyes sparkle with curiosity and endless questions on their tongues.

Codes

(a)   DBCEA

(b)   CADEB

(c)   CBEDA

(d)   ECABD

Answer: (c)

94. (A) It is hoping that overseas friends will bring in big money and lift the morale of the people.

(B) But a lot needs to be done to kick start industrial revival.

(C) People had big hopes from the new government.

(D) So far government has only given an incremental push to existing policies and programmes.

(E) Government is to go for big time reforms, which it promised.

Codes

(a)   BCDAE

(b)   EADCB

(c)   DABEC

(d)   CDEAB

Answer: (d)

95. (A) However, women hiring is catching up at a slow and steady rate in the recent times.

(B) Gender ratio has been inclined more towards male employees.

(C) As a result, recent reports have highlighted the rise in demand for women employees.

(D) Women constitute a little over half of world’s total population.

(E) But, their contribution to measured economic activity is far below the potential.

Codes

(a)   DEBAC

(b)   CDAEB

(c)   BCDEA

(d)   AEDBC

Answer: (a)

(b) Logical Reasoning

96. Choose the correct answer figure which will make a complete square on joining with the problem figure.

Answer: (c)

97. In the following question, five figures are given. Out of them, find the three figures that can be joined to form a square.

Answer: (c)

98. Choose the answer figure which completes the problem figure matrix.

Answer: (b)

99. From the given four positions of a single dice, find the colour at the face opposite to the face having red colour.

Answer: (d)

100. In the following questions, one or more dots are placed in the figure marked as (A). The figure is followed by four alternatives marked as (a), (b), (c) and (d). One out of these four options contains region(s) common to the circle, square, triangle, similar to that marked by the dot in figure (A).

Answer: (c)

101. Complete the series by replacing ‘?’ mark.

G4T, J9R, M20P, P43N, S90L, ?

(a)   S90L

(b)   V185J

(c)   M20P

(d)   P43N

Answer: (b)

102. Neeraj starts walking towards South. After walking 15 m, he turns towards North. After walking 20 m, he turns towards East and walks 10 m. He then turns towards South and walks 5 m. How far is he from his original position and in which direction?

(a)   10 m, East

(b)   10 m, South-East

(c)   10 m, West

(d)   10 m, North-East

Answer: (a)

103. The average age of 8 men is increased by 2 yrs. when one of them whose age is 20 yr is replaced by a new man. What is the age of the new man?

(a)   28 y r

(b)   36 yr

(c)   34 yr

(d)   35 yr

Answer: (b)

104. Shikha is mother-in-law of Ekta who is sister-in-law of Ankit. Pankaj is father of Sanjay, the only brother of Ankit. How is Shikha related to Ankit?

(a)   Mother-in-law

(b)   Aunt

(c)   Wife

(d)   Mother

Answer: (d)

105. In a row of forty children, P is thirteenth from the left end and Q is ninth from the right end. How many children are there between P and R, if R is fourth to the left of Q?

(a)   12

(b)   13

(c)   14

(d)   15

Answer: (c)

PART IV

Mathematics

106. If Re(z + 2) = |z – 2|, then the locus of z is

(a)   parabola

(b)   circle

(c)   ellipse

(d)   hyperbola

Answer: (a)

107. If a ∈ R, b ∈ R, then the equation

x2 – abx – a2 = 0 has

(a)   one positive root and one negative root

(b)   Both positive roots

(c)   Both negative roots

(d)   Non-real roots

Answer: (a)

108. If a + 2b + 3c = 12 (a, b, c ∈ R+), then the maximum value of ab2c3 is

(a)   23

(b)   24

(c)   26

(d)   25

Answer: (c)

109. Sum of n terms of the infinite series

1.32 + 2.52 + 3.72 + … ∞ is

Answer: (a)

110. If log7 5 = a, log5 3 = b and log3 2 = c, then the logarithm of the number 70 to the base 225 is

Answer: (d)

111. The maximum number of points of intersection of 10 circles is

(a)   80

(b)   90

(c)   85

(d)   95

Answer: (b)

112. 

(a)   120

(b)   260

(c)   210

(d)   180

Answer: (c)

113. If p ≠ q ≠ r and  then the value of x which satisfy the equation is

(a)   x = p

(b)   x = q

(c)   x = r

(d)   x = 0

Answer: (c)

114. Matrix  if xyz = 60 and 8x + 4y + 3z = 20, then A(adj A) is equal to

Answer: (c)

115. If f(x) = 4x – x2, x ∈ R, and f(a + 1) – f(a – 1) = 0, then a is equal to

(a)   0

(b)   2

(c)   1

(d)   3

Answer: (b)

116. Which of the following is not an equivalence relation in z?

(a)   aRb ⇔ a + b is an even integer

(b)   aRb ⇔ a − b is an even integer

(c)   aRb ⇔ a < b

(d)   aRb ⇔ a = b

Answer: (c)

117. Which of the following is always true?

(a)   (~ p ˅ ~ q) ≡ (p ˄ q)

(b)   (p → q) ≡ (~ p → ~ p)

(c)   ~(p → ~ q) ≡ (p ˄ ~ q)

(d)   ~(p ⟷ q) ≡ (p → q) → (q → p)

Answer: (b)

118. The solution of the inequation 4x + 0.5 – 7.2x < 4, x ∈ R is

(a)   (−2, ∞)

(b)   (2, ∞)

(c)   (2, 7/2)

(d)   None of these

Answer: (a)

119. If  is identity in x, then

(a)   a3 = 3/8, a2 = 0

(b)   n =6, a1 = 1/2

(c)   n = 5, a1 = 3/4

(d)   ∑am = 1/4

Answer: (a)

120. Total number of solutions of  x ∈ [0, 3π] is equal to

(a)   1

(b)   2

(c)   3

(d)   0

Answer: (b)

121. The minimum value of (sin1 x)3 + (cos1 x)3 is equal to

(a)   π3/32

(b)   5π3/32

(c)   9π3/32

(d)   11π3/32

Answer: (a)

122. The origin is shifted to (1, 2). The equation y2 – 8x – 4y + 12 = 0 changes to y2 = 4ax, then a is equal to

(a)   1

(b)   2

(c)   −2

(d)   −1

Answer: (b)

123. The equations of the bisector of the angles between the straight lines 3x + 4y + 7 = 0 and 12x + 5y – 8 = 0 are

(a)   7x + 9y + 17 = 0, 99x + 77y + 51 = 0

(b)   7x – 9y – 17 = 0, 99x + 77y – 51 = 0

(c)   7x – 9y + 17 = 0, 99x + 77y + 51 = 0

(d)   None of the above

Answer: (d)

124. Equation of circle which passes through the points (1, −2) and (3, −4) and touch the X-axis is

(a)   x2 + y2 + 6x + 2y + 9 = 0

(b)   x2 + y2 + 10x + 20y + 25 = 0

(c)   x2 + y2 + 6x + 4y + 9 = 0

(d)   None of the above

Answer: (b)

125. If x = 9 is the chord of contact of the hyperbola x2 – y2 = 9, then the equation of the corresponding pair of tangent is

(a)   9x2 – 8y2 + 18x – 9 = 0

(b)   9x2 – 8y2 – 18x + 9 = 0

(c)   9x2 – 8y2 – 18x – 9 = 0

(d)   9x2 – 8y2 + 18x + 9 = 0

Answer: (b)

126. The points with position vectors  and  are collinear, if a is

(a)   −8

(b)   4

(c)   2

(d)   82/9

Answer: (d)

127. Let a, b, c be vectors of lengths 3, 4, 5 respectively and a be perpendicular to (b + c), to (c + a) and c to (a + b), then the value of (a + b + c) is

(a)   2√5

(b)   2√2

(c)   10√5

(d)   5√2

Answer: (d)

128. For non-zero vectors, a, b, c; |(a × b) ∙ c| = |a| |b| |c| holds if and only if

(a)   a ∙ b = 0, b ∙ c = 0

(b)   b ∙ c = 0, c ∙ a = 0

(c)   c ∙ a = 0, a ∙ b = 0

(d)   a ∙ b = b ∙ c = c ∙ a = 0

Answer: (d)

129. Angle between the diagonals of a cube is

(a)   π/3

(b)   π/2

(c)   cos1(1/3)

(d)   cos1(1/√3)

Answer: (c)

130. Consider the two lines

The unit vector perpendicular to both the lines L1 and L2 is

Answer: (b)

131. The distance between the line  and the plane  is

(a)   10/9

(b)   10/3√3

(c)   10/3

(d)   None of these

Answer: (b)

132. Two cards are drawn from a pack of 52 cards. What is the probability that either both are red or both are kings?

(a)   7/13

(b)   63/221

(c)   55/221

(d)   3/26

Answer: (c)

133. If A and B are two independent events such that P(A) = 1/2 and P(B) = 1/5, then which of the following is correct?

Answer: (d)

134. Box 1 contains 5 red and 2 blue balls, while box II contains 2 red and 6 blue balls. A fair coin is tossed. If it turns up head, a ball is drawn from box I, else a ball is drawn from box II. The probability ball drawn is from box I, if it is blue, is

(a)   27/56

(b)   8/29

(c)   21/29

(d)   29/56

Answer: (b)

135. For a random variable X, E(X) = 3 and E(X2) = 11. The variable of X is

(a)   8

(b)   5

(c)   2

(d)   1

Answer: (c)

136. The sum of 10 items is 12 and the sum of their squares is 18, then the standard deviation will be

(a)   −3/5

(b)   6/5

(c)   4/5

(d)   3/5

Answer: (d)

137. The height of the chimney when it is found that on walking towards it 50 m in the horizontal line through its base, the angle of elevation of its top changes from 30° to 60° is

(a)   25 m

(b)   25√2 m

(c)   25√3 m

(d)   None of these

Answer: (c)

138. The value of  is

(a)   1/2

(b)   2

(c)   √2

(d)   None of these

Answer: (c)

139. If  is differentiable at x = 1, then

(a)   a = 1, b = 1

(b)   a = 1, b = 0

(c)   a = 2, b = 0

(d)   a = 2, b = 1

Answer: (c)

140. The slope the tangent to the curve x = t2 + 3t – 8 y = 2t2 – 2t – 5 at the point t = 2 is

(a)   7/6

(b)   5/6

(c)   6/7

(d)   1

Answer: (c)

141. is equal to

Answer: (c)

142. is equal to

Answer: (a)

143. The area of one curvilinear triangle formed by curves y = sin x, y = cos x and X-axis, is

(a)   2 sq units

(b)   (2 + √2) sq units

(c)   (2 – √2) sq units

(d)   None of the above

Answer: (c)

144. Solution of  given that y = 1 when x = 1 is

Answer: (b)

145. If  is

(a)   1

(b)   −1

(c)  

(d) 

Answer: (d)

146. The maximum value of the function y = x(x – 1)2, is

(a)   0

(b)   4/27

(c)   −4

(d)   None of these

Answer: (b)

147. The solution of  satisfying y(1) = 0, is

(a)   tan y = (x – 2)ex log x

(b)   sin y = ex(x – 1) x1

(c)   tan y = (x – 1)ex x3

(d)   sin y = ex(x – 1)x3

Answer: (b)

148. The runs of two players for 10 innings each are as follows

The more consistent player is

(a)   player A

(b)   player B

(c)   both player A and B

(d)   None of the above

Answer: (a)

149. The linear programming problem minimize z = 3x + 2y subject to constraints x + y ≥ 8, 3x + 5y ≤ 15, x ≥ 0 and y ≥ 0, has

(a)   one solution

(b)   no feasible solution

(c)   two solutions

(d)   infinitely many solution

Answer: (b)

150. Find the area enclosed by the loop in the curve 4y2 = 4x2 – x3.

(a)   128/5

(b)   15/128

(c)   130/17

(d)   17/130

Answer: (a)

BITSAT Examination Previous Year Question Paper 2020 With Answer Key

BITSAT SOLVED PAPER-2020

PART-I

PHYSICS

1. Three solenoid coils of same dimensions, same number of turns and same number of layers of winding are taken. Coil 1 has inductance L1 wounded by Mn wire of resistance 6 Ωm1, coil 2 with inductance L2 wounded by similar wire but in reverse direction in each layer. Coil 3 with inductance L3 wounded by a superconducting wire. The relation between their self inductances will be

(a)   L1 = L2, L3 = 0

(b)   L1 = L2 = L3

(c)   L1 = L3, L2 = 0

(d)   L1 > L2 > L3

Answer: (c)

2. Three capacitors C1, C2 and C3 are connected as shown in the figure below. If capacitor C3 breaks down electrically, then the change in total charge on the combination of capacitors, is

Answer: (a)

3. A black body radiates energy at the rate EWm2 at high temperature T K. When the temperature is reduced to (T/4) K, then the new radiant energy is

(a)   E/256

(b)   4E

(c)   E/4

(d)   E/16

Answer: (a)

4. The length of the rectangle is l = 15.2 cm and breadth is b = 2.9 cm and the minimum possible measurement by scale = 0.1 cm. Then, the area of the rectangle is

(Taking, significant figures into consideration)

(a)   44.08 cm2

(b)   24.8 cm2

(c)   44 cm2

(d)   94.008 cm2

Answer: (c)

5. In an adiabatic process, where pressure is decreased by 3/4%, if  then the volume increases by

(a)   3/4%

(b)   9/16%

(c)   16/9%

(d)   4/3%

Answer: (b)

6. The vibrations of a string of length 60 cm fixed at both ends are represented by the equation

where, x and y are cm and t is in second. Calculate the velocity of the particle at x = 7.5 cm and t = 0.25 s.

(a)   4  m/s

(b)   Zero

(c)   16 m/s

(d)   9.8 m/s

Answer: (b)

7. A charge +q is placed at the origin O of XY-axes as shown in the figure. The work done in taking a charge Q from A to B along the straight line AB is

Answer: (a)

8. The hydrogen-like element that has a spectrum whose lines have wavelength four times shorter than those of atomic hydrogen is

(a)   lithium

(b)   helium

(c)   berilliyum

(d)   potassium

Answer: (b)

9. Two small conducting spheres of equal radius have charges +20 μC and −40 μC respectively and placed at a distance R from each other experience force F1. If they are brought in contact and separated to the same distance, they experience force F2. The ratio of F1 to F2 is

(a)   1 : 4

(b)   8 : 1

(c)   −8 : 1

(d)   1 : 8

Answer: (c)

10. A Carnot engine has the same efficiency between 600 K to 300 K and 1600 K to x K, then the value of x is

(a)   1600 K

(b)   800 K

(c)   819 K

(d)   900 K

Answer: (b)

11. A ball of mass 0.5 kg is thrown up with initial speed 16 ms1 and reaches maximum height of 9 m. How much energy is dissipated by air drag acting on the ball during the ascent?

(a)   199 J

(b)   19.9 J

(c)   20.9 J

(d)   9.9 J

Answer: (b)

12. In the circuit shown, if the 10 Ω resistor is replaced by a resistor of 15 Ω, then what is the amount of current drawn from the battery?

(a)   100 A

(b)   10 A

(c)   1 A

(d)   2.4 A

Answer: (b)

13. A time dependent force F(= 8t) acts on a particle of mass 2 kg. If the particle starts form rest, the work done by the force during the first 1 s will be

(a)   0.4 J

(b)   4 J

(c)   19 J

(d)   4.5 J

Answer: (b)

14. If L, R, C and V represent inductance, resistance, capacitance and potential difference respectively, then dimensions L/RCV are the same as those of

(a)   current

(b)   1/current

(c)   charge

(d)   1/charge

Answer: (b)

15. The magnetic field of a beam emerging from a fitter facing a flood light is given by

B = 10 × 108 sin(1 × 107 z – 3.6 × 1015 t)T. The average intensity of the beam is

(a)   1.82 W/m2

(b)   1.19 W/m2

(c)   1.18 W/m2

(d)   1.17 W/m2

Answer: (b)

16. From the velocity-time graph of a body moving in a straight line, the distance travelled and the average velocity in the time interval t = 0 to t = 20 s are, respectively,

(a)   0, 0

(b)   120 m, 60 m

(c)   60 m, 0

(d)   0, 60 m

Answer: (c)

17. A thin equi-convex lens is made of glass of refractive index 1.5 and its focal length is 0.2 m. If it acts as a concave lens of focal length 0.5 m when dipped in a liquid, the refractive index of the liquid is

(a)   17/8

(b)   15/8

(c)   13/8

(d)   9/8

Answer: (b)

18. A moving coil galvanometer has a resistance of 60 Ω and it indicates full deflection on passing a current of 4.5 mA. A voltmeter is made using this galvanometer and a 4.5 kΩ The maximum voltage, that can be measured using this voltmeter, will be close to

(a)   21 V

(b)   20.5 V

(c)   20 V

(d)   19.5 V

Answer: (b)

19. The combination of the gates shown in following figure yields

(a)   NAND gate

(b)   OR gate

(c)   NOT gate

(d)   XOR gate

Answer: (b)

20. A vessel contains one mole of O2 gas (molar mass 32) at a temperature T. The pressure of the gas is p. An identical vessel containing one mole of He gas (molar mass 4) at a temperature 2T has a pressure of

(a)   p/8

(b)   p

(c)   2p

(d)   8p

Answer: (c)

21. The acceleration of a particle in m/s2 is given by a = (3t2 – 2t + 1), where t is in second. If the particle starts with a velocity v = 1 m/s at t = 1s, then velocity of the particle at the end of 4s is

(a)   40 m/s

(b)   52 m/s

(c)   48 m/s

(d)   84 m/s

Answer: (b)

22. The figure shows graphs of pressure (p) versus density (d) for an ideal gas at two temperatures T1 and T2, then

(a)   T1 > T2

(b)   T1 < T2

(c)   T1 = T2

(d)   None of these

Answer: (b)

23. Two spheres of the same material and same radii r are touching each other. The gravitational force between the spheres is proportional to

(a)   1/r2

(b)   r2

(c)   1/r4

(d)   r4

Answer: (d)

24. A spherical lens of power −4 D is placed at a distance of 15 cm from another spherical lens of power 5 D. A beam of parallel light falls on the first spherical lens. The final image formed is

(a)   real and at a distance of 40 cm from the lens of power 5 D

(b)   real and at a distance of 10 cm from the lens of power −4 D

(c)   virtual and at a distance of 40 cm from the lens of power −5 D

(d)   None of the above

Answer: (a)

25. The wheel of a car, accelerated uniformly from rest, rotates through 5 rad during the first second. The angle (in rad) rotated during the next second is

(a)   15

(b)   7.5

(c)   12.5

(d)   20

Answer: (a)

26. Lights of two different frequencies whose photons have energies 1.5 eV and 2.5 eV respectively illuminate a metallic surface whose work function is 0.5 eV successively. Ratio of maximum speeds of emitted electrons will be

(a)   3 : 2

(b)   2 : 3

(c)   √3 : √2

(d)   √2 : √3

Answer: (d)

27. When a body is dropped from a height h, then it hits the ground with a momentum p. If the same body is dropped from a height which is three times more than previous height, the percentage change in momentum when it hits the ground is

(a)   25%

(b)   50%

(c)   75%

(d)   100%

Answer: (d)

28. The reading of the ammeter for a germanium diode in the given circuit is

(a)   0.94 A

(b)   0

(c)   2.8 A

(d)   5 A

Answer: (a)

29. The decay constants of two radioactive substances X and Y are 4λ and λ At t = 0, a sample has the same number of two nuclei. The time taken for the ratio of number of nuclei to become 1/e3 will be

(a)   1/3λ

(b)   1/2λ

(c)   2/3λ

(d)   3/2λ

Answer: (b)

30. The magnitude of the force vector acting on a unit length of a thin wire carrying a current I = 10 A at a point O, if the wire is bent in the form of a semi-circle (shown below) with radius R = 20 π cm, is

(a)   30 μN/m

(b)   40 μN/m

(c)   50 μN/m

(d)   60 μN/m

Answer: (c)

31. A long cylindrical iron core of cross-sectional area 5 cm2 is inserted into a long solenoid having 4000 turns/metre and carrying a current 5 A. The magnetic field inside the core is π Find the pole strength developed.

(a)   1000 A-m

(b)   1240 A-m

(c)   882 A-m

(d)   760 A-m

Answer: (b)

32. A plane requires for take off a speed of 72 kmh1, the run on the ground being 50 m. The mass of the plane is 10000 kg and the coefficient of friction between the plane and the ground is 0.2. Assume that the plane accelerates uniformly during take off. The minimum force required by the engine of the plane for take off is

(a)   4.43 × 104 N

(b)   5.96 × 104 N

(c)   2.25 × 104 N

(d)   3.45 × 104 N

Answer: (b)

33. In a fluorescent lamp choke, 120 V of reverse voltage is produced when the choke current changes uniformly 0.50 A to 0.20 A in a duration of 0.030 ms. The self inductance of the choke (in mH) is estimated to be

(a)   12 H

(b)   12 × 103 mH

(c)   12 × 103 H

(d)   0

Answer: (c)

34. A man grows into a giant such that his linear dimensions increase by a factor of 8. Assuming that his density remains same, the stress in his leg will change by a factor of

(a)   1/8

(b)   8

(c)   36

(d)   64

Answer: (b)

35. When 2 moles of a monoatomic gas are mixed with 3 moles of a diatomic gas, the value of adiabatic exponent for the mixture is

(a)   15/16

(b)   7/5

(c)   31/21

(d)   38/59

Answer: (c)

36. A simple pendulum is placed inside a lift, the lift is moving with a uniform acceleration. If the time periods of the pendulum, while the lift is moving upwards and downwards are in the ratio of 1 : 3, then the acceleration of the lift is

[Take, acceleration due to gravity, g = 10 m/s2]

(a)   4 m/s2

(b)   6 m/s2

(c)   8 m/s2

(d)   10 m/s2

Answer: (c)

37. A satellite is moving around the earth with speed υ in a circular orbit of radius r. If the orbit radius is decreased by 2%, its speed will increase by

(a)   1%

(b)   2%

(c)   1.5%

(d)   1.414%

Answer: (a)

38. In the circuit shown in figure, the AC source has angular frequency ω = 2000 rad s1. The amplitude of the current will be nearest to

(a)   2.85 A

(b)   3 A

(c)   0.5 A

(d)   3.625 A

Answer: (a)

39. The work done by a force acting on a body is as shown in the following graph. The total work done in covering an initial distance of 40 m is

(a)   500 J

(b)   600 J

(c)   400 J

(d)   800 J

Answer: (d)

40. A potential wire, 10 m long, has a resistance of 40 Ω. It is connected in series with a resistance box and a 4 V storage cell. If the potential gradient along the wire is 0.4 mVcm1, the resistance unplugged in the box is

(a)   220 Ω

(b)   360 Ω

(c)   760 Ω

(d)   848.3 Ω

Answer: (b)

PART II

Chemistry

41. In a set of reactions, m-bromobenzoic acid gives a product D. Identify the product D.

Answer: (d)

42. A volume of 50.00 mL of a weak acid of unknown concentration is titrated with 10 M solution of NaOH. The equivalence point is reached after 39.30 mL of NaOH solution h as been added. At the half equivalence point (19.65 mL) the pH is 4.85. Thus, initial concentration of the acid and its pKa values are

[HA]                pKa

(a)   0.1 M          4.85

(b)   0.079 M      4.85

(c)   0.1 M          3.70

(d)   0.097 M      2.93

Answer: (b)

43. 1-phenyl-1, 3-dibromopropane on treatment with alc. KOH gives diastereomeric mixture in which compound (A) is major product. (A) gives the following reaction

Answer: (d)

44. Which of the following statement is incorrect?

(a)   α-D-fructose and β-D-fructose are enantiomers of each other.

(b)   D-glyceraldehyde and L-glyceraldehyde are enantiomers of each other.

(c)   The reserve carbohydrate of animals is glycogen.

(d)   Aldohexoses which react with phenyl hydrazine to give identical osazones are C-2 epimers.

Answer: (a)

45. Which of the following is not a mixed pair of oxides?

(a)   Mn3O4 and Co3O4

(b)   Co3O4 and Pb3O4

(c)   Pb3O4 and Mn3O4

(d)   Fe3O4 and Fe2O3

Answer: (d)

46. A solution of copper sulphate is electrolyzed between copper electrodes by a current of 10.0A passing for one hour. Which of the following statements is correct regarding the changes that occur at the electrodes and in the solution?

(a)   11.84 g of copper will deposit on the cathode

(b)   11.84 g of copper will deposit on the anode

(c)   11.84 g copper will deposit on the anode as well as on the cathode

(d)   copper will not deposit on any of the electrode

Answer: (a)

47. 

Identify structure of compound (D).

Answer: (b)

48. Which of the following statement is correct?

(a)   ∆S for  is positive.

(b)   ∆E < 0 for combustion of CH4 (g) in a sealed container with rigid adiabatic system.

(c)   ∆G is always zero for a reversible process in a closed system.

(d)   ∆G° for an ideal gas reaction is a function of pressure.

Answer: (a)

49. Which of the following statements on critical constants of gases are correct?

(I) Larger the Tc/pc value of a gas, larger would be the included volume.

(II) Critical temperature (Tc) of g as is greater than its Boyle  temperature (TB).

(III) At the critical point in the van der Waal’s gas isotherm, 

Select the correct answer using the codes given below.

(a)   Both I and II

(b)   Both I and III

(c)   Both II and III

(d)   I, II and III

Answer: (b)

50. In the given reaction,

Product (D) is

(a)   a positional isomer of X

(b)   identical to X

(c)   chain isomer of X

(d)   an oxidation product of X

Answer: (a)

51. 

(B) is identified by the characteristic colour of the flame. (A) and (B) are respectively.

(a)   H3BO3 and Na2B4O7

(b)   B(OC2H5)3 and H3BO3

(c)   NaBO2 and H3BO3

(d)   H3BO3 and B(OC2H5)3

Answer: (d)

52. How many mL of perhydrol is required to produce sufficient oxygen which can be used to completely convert 2 L of SO2 gas?

(a)   10 mL

(b)   5 mL

(c)   20 mL

(d)   30 mL

Answer: (a)

53. At the top of a mountain the thermometer reads 0°C and the barometer reads 710 mm Hg. At the bottom of the mountain the temperature is 30°C and the pressure is 760 mm Hg. The ratio of the density of air at the top to that of the bottom is

(a)   1 : 1.04

(b)   0.4 : 1

(c)   1.04 : 1

(d)   1 : 04

Answer: (c)

54. Match the following columns.

Codes

(a)   A-5; B-4; C-3; D-2; E-1

(b)   A-5; B-3; C-4; D-2; E-1

(c)   A-5; B-3; C-4; D-1; E-2

(d)   A-4; B-3; C-5; D-2; E-1

Answer: (b)

55. Most acidic hydrogen is present in

Answer: (c)

56. The process of ‘eutrophication’ is due to the

(a)   increase in concentration of insecticide in H2O

(b)   increase in concentration of fluoride ion in H2O

(c)   reduction in concentration of the dissolved oxygen in water due to phosphate pollution in water.

(d)   attack of younger leaves of a plant by peroxyacetyl nitrate

Answer: (c)

57. Ionization energy of H-atom is 13.6 eV. The wavelengths of the spectral line emitted when an electron in Be3+ comes from 5th energy level to 2nd energy level is

(a)   43.5 nm

(b)   4350 nm

(c)   4.35 nm

(d)   435 nm

Answer: (d)

58. The enthalpies of combustion of carbon and carbon monoxide in excess of oxygen at 298 K and constant pressure are −5 kJ/mol and −280.0 kJ/mol respectively. The heat of formation of carbon monoxide at constant volume is

(a)   +111.7 kJ/mol

(b)   −1111.7 kJ/mol

(c)   −111.7 kJ/mol

(d)   −11.7 kJ/mol

Answer: (c)

59. If the quantum number l could have the value n also then, Sc(21) would have electronic configuration as (other rules strictly followed)

(a)   1s2 1p6 2s2 2p6 2d3 3s2

(b)   1s2 1p6 2s2 2p6 3s2 2d3

(c)   1s2 2s2 2p6 3s2 3p6 3d1 4s2

(d)   1s2 2s2 2p6 3s2 3p6 3d3

Answer: (b)

60. 1.1 mole of A mixed with 2.2 moles of B and the mixture is kept in a 1 L flask and the equilibrium, A + 2B ⇌ 2C + D is reached. If at equilibrium 0.2 mole of C is formed then the value of KC will be

(a)   0.1

(b)   0.01

(c)   0.001

(d)   1

Answer: (c)

61. Choose the correct chemical reaction among the following:

(a)   CaCN2 + H2O → Ca(OH)2 + C2H2 + NH3

(b)   2NH3 + CaSO4 + CO2 + H2O → CaCN2+(NH4)2SO4

(c)   CaCl2 + Na2SO4 → CaSO4 + 2NaCl

(d)   CaC2 + H2O → Ca(OH)2 + 2NaCl

Answer: (c)

62. Which of the following reactions represents disproportionation?

(a)   CrO5 → Cr3+ + O2

(b)   IO3 + I + H+ → I2

(c)   CrO2Cl2 + NaOH → Na2CrO4 + NaCl + H2O

(d)   Na2S2O3 + H2SO4 → Na2SO4 + SO2 + S8 + H2O

Answer: (d)

63. 

Compound C is

Answer: (d)

64. On the basis of Ellingham diagram, which of the following is not correct?

(a)   Entropy change for all metal oxides is roughly same.

(b)   Below the boiling point, T∆S factor is same irrespective of metal.

(c)   Above ∆G = 0 line, oxide decomposes into metal and oxygen.

(d)   If randomness increases the slope increases.

Answer: (a)

65. Which of the following is not explained by adsorption?

(a)   When acetic acid solution is shaken with charcoal the concentration of acid decreases.

(b)   The white ppt. of Mg(OH)2 attains blue colour when precipitated in the presence of magneson reagent.

(c)   An aqueous solution of NaOH attains pink colour with a drop of phenolphthalein.

(d)   When animal charcoal is shaken with coloured methylene blue solution, the solution turns colourless.

Answer: (c)

66. Identify ‘A’ and ‘B’ in the following reaction

Answer: (c)

67. An element occurs in two crystalline forms α and β. The α-from has an fcc with a = 3.68 Å and β form has a bcc with a = 2.92 Å. Calculate the ratio of their densities.

(a)   1 : 1

(b)   1 : 2

(c)   2 : 1

(d)   2 : 3

Answer: (a)

68. The molar heat of vaporization of water at 100°C is 40.585 kJ/mol. The temperature at which a solution containing 5.60 g of Al2(SO4)3 per 1000 g of water boil is …. . (Assuming the degree of ionization of salt to be 1).

(a)   1000.042°C

(b)   10.0042°C

(c)   100.042°C

(d)   105°C

Answer: (c)

69. An organic base C8H11(X) reacts with nitrous acid at 0°C to give a clear solution. Heating the solution with KCN and cuprous cyanide followed by continued heating with conc. HCl gives a crystalline solid. Heating this solid with alkaline potassium permanganate gives a compound which dehydrates on heating to an anhydride (C8H4O3). Compound X is

Answer: (d)

70. The final product of the following reaction is/are

Answer: (b)

71. The correct order of pseudohalide, polyhalide and interhalogen is

(a)   BrI2, OCN, IF5

(b)   IF5, BrI2, OCN

(c)   OCN, IF2, BrI2

(d)   OCN, BrI2, IF5

Answer: (d)

72. An inorganic halide (A) reacts with water to form two acids (B) and (C). (A) also reacts with NaOH to form two salts (D) and (E) which remain in solution. The solution gives white precipitate with both AgNO3 and BaCl2 solutions respectively. (A) is a useful organic reagent. The compound (A) is

(a)   SOCl2

(b)   SO2Cl2

(c)   S2Cl2

(d)   SF4

Answer: (b)

73. Following statements regarding the periodic trends of chemical reactivity to the alkali metals and the halogens are given. Which of these statements gives the correct picture?

(a)   The reactivity decreases in the alkali metals but increases in the halogens with increase in atomic number down the group.

(b)   In both the alkali metals and the halogens the chemical reactivity decreases with increase in atomic number down the group.

(c)   Chemical reactivity increases with increase in atomic number down the group in both the alkali metals and halogens.

(d)   In alkali metals the reactivity increases but in the halogens it decreases with increase in atomic number down the group.

Answer: (d)

74. According to IUPAC nomenclature sodium nitroprusside is named as

(a)   sodiumpentacyanonitrosylferrate (II)

(b)   sodiumpentacyanonitrosylferrate (III)

(c)   Sodiumnitroferricyanide

(d)   Sodiumnitroferrocyanide

Answer: (b)

75. A(g) → P(g) + Q(g) + R(g),

Follow first order kinetics with a half-life of 69.3 s at 500°C. Starting from the gas ‘A’ an container at 500°C and at a pressure of 0.4 atm, the total pressure of the system after 230 s will be

(a)   1.15 atm

(b)   1.32 atm

(c)   1.22 atm

(d)   1.12 atm

Answer: (d)

76. Which of the following gives paracetamol on acetylation?

Answer: (d)

77. In qualitative analysis when H2S passed through an aqueous solution of salt acidified with dilute HCl, a black ppt. is obtained. On boiling the precipitate with dil. HNO3, it forms a solution of blue colour. Addition of excess of aqueous solution of NH3 to this solution gives

(a)   deep blue ppt. of Cu(OH)2

(b)   deep blue solution of [Cu(NH3)4]2+

(c)   deep blue solution of Cu(NO3)2

(d)   deep blue solution of Cu(OH)2 ∙ Cu(NO3)2

Answer: (b)

78. Titration of 0.1467 g of primary standard Na2C2O4 required 28.85 mL of KMnO4 Calculate the molar concentration of KMnO4 solution.

(a)   0.01518 M

(b)   0.001518 M

(c)   0.15180 M

(d)   1.5180 M

Answer: (a)

79. Which of the following facts about the complex [Cr(NH3)6]Cl3 is wrong?

(a)   The complex involves d2sp3 hybridization and its octahedral in shape.

(b)   The complex is paramagnetic.

(c)   The complex is an outer orbital complex.

(d)   The complex gives white precipitate with silver nitrate solution.

Answer: (c)

80. Sulphuric acid is a diabasic acid. It ionizes in two stages and hence, has two dissociation constants Ka1 and Ka2. Which of the following is the correct observation regarding Ka1and Ka2?

(a)   Ka1 > Ka2

(b)   Ka1 < Ka2

(c)   Ka1 = Ka2

(d)   Ka1 = 1.2 × 102, Ka2 > 10

Answer: (a)

PART III

(a) English Proficiency

Directions (Q. Nos. 81-83) In the following questions, find out which part of a sentence has an error. If there is no error, mark part (d) as your answer.

81. The road a./to famous monument b./ passes through a forest. c./ No error d

Answer: (b)

82. The master did not known a./who of the servants b./broke the glass c./No error d

Answer: (b)

83. Had I come a./to know about his difficulties b./I would have certainly helped c./No error d

Answer: (c)

Directions (Q. Nos. 84 and 85) Fill in the blanks with suitable preposition from the alternatives given under each sentence.

84. He trekking was net ……………. Obstacles.

(a)   with

(b)   from

(c)   by

(d)   of

Answer: (a)

85. She has not got ….. the shock of losing her father.

(a)   over

(b)   at

(c)   from

(d)   with

Answer: (b)

Directions (Q. Nos. 86-88) Select the word or phrase which is closest to the opposite in meaning of the italicized word or phrase.

86. Yuvraj Singh is suffering from a BENIGN cancer.

(a)   Unfriendly

(b)   Friendly

(c)   Fatal

(d)   Malignant

Answer: (c)

87. He is a NOTED figure of film industry.

(a)   Known

(b)   Unknown

(c)   Famous

(d)   Infamous

Answer: (b)

88. SAGACIOUS decisions taken at right time in one’s career has long effects.

(a)   Foolish

(b)   Intelligent

(c)   Thoughtful

(d)   Intuitive

Answer: (a)

Directions (Q. Nos. 89 and 90) Choose the word nearest in meaning to the italicized word.

89. The actor got PEEVISH on asking personal questions.

(a)   Irritated

(b)   Happy

(c)   Shy

(d)   Satisfied

Answer: (a)

90. The engineer ROUGHED OUT his ideas on a piece of paper while he talked.

(a)   Shaped soughly

(b)   Rejected

(c)   Drew a quick plan

(d)   Describe inaccurately

Answer: (c)

Directions (Q. Nos. 91-95) Read the passage given below and answer the questions that follow.

The Centre and the States must become partners in the planning process to determine national priorities together. The process of planning would undergo a change in view of the changes in domestic economic situation and momentous trends emerging in the world. The development of human resource and the building up of an institutional framework would have to receive priority attention. The role of the government would also have to be examined so as to fully involve the people in the process of nation-building. The main task would be to ensure that the real initiative is transferred to the people. The private sector which would register expansion hereafter should keep this objective firmly in view. The need for an effective population policy is an urgent necessity in the country’s planning strategy. The family welfare programme should not be treated as the Centre’s responsibility alone. The States should evolve a suitable mechanism for closer involvement of the Government agencies, Zilla Parishads and Panchayats for making the family welfare programme a success.

91. Which one of the following statements is correct?

(a)   Effective family welfare programme is Centre’s responsibility alone.

(b)   Population policy and planning process are interlinked.

(c)   Family welfare programme should be left to the State Governments alone.

(d)   The State Government should use punitive measures to control population.

Answer: (b)

92. What should be given priority attention?

(a)   Role of the Government

(b)   Decentralization of power

(c)   Involvement of people in labour welfare

(d)   Human resource and institutional framework

Answer: (d)

93. Which one of the following statements is not correct?

(a)   Role of the government in nation-building should be examined.

(b)   Real initiatives should be transferred to the people.

(c)   There should be no role for the government as far as planning is concerned.

(d)   The Centre and the States must become equal partners in the planning process.

Answer: (c)

94. What would force the planning process to undergo a change?

(a)   Free market forces

(b)   Domestic economic situation and world trends

(c)   Domestic compulsions

(d)   International pressures

Answer: (b)

95. Which of the following is implied by the expression ‘momentous trends’?

(a)   GDP growth of the country

(b)   Memorable historical events

(c)   Important changes in the international scene

(d)   Improvement of foreign exchange reserves

Answer: (c)

(b) Logical Reasoning

96. Find out the wrong number.

2, 6, 12, 72, 865, 62208

(a)   72

(b)   12

(c)   62208

(d)   865

Answer: (d)

97. Each of P, Q, T, A and B has different heights. T is taller than P and B but shorter than A and Q. P is not the shortest, who is the tallest?

(a)   A

(b)   Q

(c)   A or Q

(d)   P or B

Answer: (c)

98. Identify the missing part of the question figure and select it from given answer figures.

Answer: (c)

99. Select the related word from the given alternatives.

Mechanic : Spanner : : Carpenter : ?

(a)   tree

(b)   wood

(c)   furniture

(d)   saw

Answer: (d)

100. How many rectangles are there in the following figure?

(a)   8

(b)   18

(c)   17

(d)   20

Answer: (b)

101. In the following question find the odd letters/group from the given alternatives.

(a)   BADC

(b)   JILK

(c)   NMPO

(d)   VUWX

Answer: (d)

102. Find out which of the answer figures (a), (b), (c) and (d) completes the figure matrix?

Answer: (d)

103. Among the four answer figures, which one can be formed from the cut out pieces given below in the question figures?

Answer: (c)

104. A piece of paper is folded and cut as shown below in the question figures. From the given answer figures, indicates how it will appear when opened.

Answer: (b)

105. In the following question three dots are placed in the figure marked as (A). The figure is followed by four alternatives marked as (a), (b), (c) and (d). One out of these four options contains region(s) common to the circle, square, triangle, similar to that marked by the dot in figure (A).

Answer: (b)

PART IV

Mathematics

106. If A = {x : x2 = 1} and B = {x : x4 = 1}, then A ∆ B is equal to

(a)   {−i, i}

(b)   {−1, i}

(c)   {−1, 1, −i, i}

(d)   None of these

Answer: (a)

107. If 2f(xy) = (f(x))y + (f(y))x for all x, y ∈ R and f(1) = a (≠1). Then  is equal to

(a)   (an – 1)/(a – 1)

(b)   a(an – 1 – 1)/(a – 1)

(c)   a(an – 1)/(a – 1)

(d)   (an – 1)/a + 1

Answer: (c)

108. Let f(x) = x – 3 and g(x) = 4 – x. Then the set of values of x for which

|f(x) + g(x)| < |f(x)| + |g(x)| is true, is given by:

(a)   R

(b)   R – (3, 4)

(c)   R – [3, 4]

(d)   None of these

Answer: (c)

109. If a1, a2, a3, …., a20 are AM’s between 13 and 67 then the maximum value of a1 ∙ a2 ∙ a3 …, a20 is

(a)   (20)20

(b)   (40)20

(c)   (60)20

(d)   (80)20

Answer: (b)

110. If p, q, r are in AP and the positive, the roots of the quadratic equation px2 + qx + r = 0 are all real for

(a) 

(b) 

(c)   All p and r

(d)   No p and r

Answer: (a)

111. The value of  is equal to

(a)   47C6

(b)   52C5

(c)   52C4

(d)   None of these

Answer: (c)

112. The number of numbers divisible by 3 that can be formed by four different even digits is

(a)   36

(b)   18

(c)   0

(d)   None of these

Answer: (a)

113. If n(A) = 1000, n(B) = 500, n (A ∩ B) ≥ 1 and n (A ∪ B) = P, then

(a)   500 ≤ P ≤ 1000

(b)   1001 ≤ P ≤ 1498

(c)   1000 ≤ P ≤ 1498

(d)   1000 ≤ P ≤ 1499

Answer: (d)

114. is equal to

(a)   R – {0}

(b)   R – {0, 1, 3}

(c)   R – 3{0, −1, −3}

(d)   R – {0, −1, −3, 1/2}

Answer: (c)

115. Let f(x) be a polynomial function of second degree. If f(1) = f(−1) and a, b, c are in AP, then f ꞌ(a), f ʹ(b) and f ʹ(c) are in

(a)   AP

(b)   GP

(c)   Arithmetic-Geometric progression

(d)   None of the above

Answer: (a)

116. The value of  is

(a)   5 – 2log 2

(b)   4 – 2log 2

(c)   3 – 2log 2

(d)   2 – 2log 2

Answer: (c)

117. The coefficient of x8 in the polynomial (x – 1) (x – 2) ….(x – 10)

(a)   2640

(b)   1320

(c)   1370

(d)   2740

Answer: (b)

118. If  then z14 is

(a)   27

(b)   27i

(c)   (−2)7

(d)   (−2)7i

Answer: (b)

119. The solution of the equation  is

(a)   2y = sin y(1 – 2cx2)

(b)   2x = cot y(1 + 2cx2)

(c)   2x = sin y(1 – 2cx2)

(d)   2x sin y  = 1 – 2cx2

Answer: (c)

120. The value of the definite integral 

(a)   1 – π/4

(b)   1 + π/4

(c)   π + 1/4

(d)   None of these

Answer: (a)

121. If a and b are two vectors such that |a| = 1, |b| = 4 and a ∙ b = 2, if c = (2a × b) – 3b, then angle between b and c

(a)   π/6

(b)   π/3

(c)   2π/3

(d)   5π/6

Answer: (d)

122. Let x1 and x2 be the real roots of the equation x2 – (k – 2)x + (k2 + 3k + 5) = 0, then maximum value of x12 + x22 is

(a)   19

(b)   22

(c)   18

(d)   17

Answer: (a)

123. Circle centered at origin and having radius π units is divided by the curve y = sin x in two parts. Then area of upper parts equals to

(a)   π2/2

(b)   π3/4

(c)   π3/2

(d)   π3/8

Answer: (c)

124. The root of the equation 2(1 + i)x2 – 4(2 – i)x – 5 – 3i = 0, where i = √−1, which has greater modulus, is

Answer: (a)

125. The equation (cos β – 1)x2 + (cos β)x + sin β = 0 in the variable x has real roots, then β lies in the interval

(a)   (0, 2π)

(b)   (−π, 0)

(c)   (−π/2, π/2)

(d)   (0, π)

Answer: (d)

126. A ordered pair (α, β) for which the system of linear (1 + α)x + βy + z = 2, αx + (1 + β)y + z = 3 and αx + βy + 2z = 2 has a unique solution.

(a)   (1, −3)

(b)   (−3, 1)

(c)   (2, 4)

(d)   (−4, 2)

Answer: (c)

127. A bird is sitting on the top of a vertical pole 20 m high and its elevation from a point O on the ground is 45°. If flies off horizontally straight way from the point O. After one second, the elevation of the bird from O is reduced to 30°, then the speed (in m/s) of the bird is

(a)   40(√2 – 1)

(b)   40(√3 – √2)

(c)   20√2

(d)   20(√3 – 1)

Answer: (d)

128. If one GM, g and two AM’s p and q are inserted between two numbers a and b, then (2p – q) (p – 2q) is equal to

(a)   g2

(b)   −g2

(c)   2g

(d)   3g2

Answer: (b)

129. When x100 is divided by x2 – 3x + 2 the remainder is (2k+1 – 1)x – 2(2k – 1), then k is

(a)   97

(b)   99

(c)   100

(d)   101

Answer: (b)

130. The mean of five observation is 5 and their variance is 9.20. If three of the given five observation are 1, 3 and 8, then a ratio of other two observations is

(a)   4 : 9

(b)   6 : 7

(c)   5 : 8

(d)   10 : 3

Answer: (a)

131. How many three digit number satisfy the property that the middle digit is arithmetic mean of the first and the last digit.

(a)   41

(b)   45

(c)   43

(d)   44

Answer: (b)

132. If z = reiθ, then arg(eiz) is

(a)   −r sin θ

(b)   r cos θ

(c)   er sin θ

(d)   −r cos θ

Answer: (b)

133. If 4 dice are rolled, then the number of ways of getting the sum 10 is

(a)   56

(b)   64

(c)   72

(d)   80

Answer: (d)

134. Distance of point A(1, 2) measured parallel to the line 3x – y = 10 from the line x + y + 5 = 0, is

(a)   2√5

(b)   2√10

(c)   4√5

(d)   4√10

Answer: (b)

135. Let f(x) = a0 + a1x2 + a2x4 + a3x6 + … + anx2n be a polynomial in a real variable x with 0 < a1 < a2 < a3 < … < an, the function f(x) has

(a)   neither a maxima nor a minima

(b)   only one maxima

(c)   both maxima and minima

(d)   only one minima

Answer: (d)

136. If f(x) = 2x3 + x4 + log x and g is the inverse of f, then g’(3) is equal to

(a)   1/9

(b)   1/7

(c)   1/11

(d)   1/8

Answer: (c)

137. A line passing through P(3, 7, 1) and R(2, 5, 7) meet the plane 3x + 2y + 11z – 9 = 0 at Q. Then PQ is equal to

(a)   5√41/59

(b)   √41/59

(c)   50√41/59

(d)   25√41/59

Answer: (d)

138. equals to

Answer: (a)

139. If  then find z component of a vector r, which is coplanar with a and b, r ∙ b = 0 and r ∙ a = 7.

(a)   0

(b)   3

(c)   6

(d)   5/2

Answer: (b)

140. If x, y, z are three consecutive positive integers, x – z + 2 = 0, then  is equal to

(a)   loge x

(b)   loge y

(c)   loge z

(d)   None of these

Answer: (b)

141. The solution of differential equation (xy5 + 2y)dx – xdy = 0, is

(a)   9x8 + 4x9y4 = 9y4C

(b)   9x8 – 4x9y4 – 9y4C = 0

(c)   x8(9 + 4y4) = 10y4C

(d)   None of these

Answer: (a)

142. The solution set of  is

(a)   [0, 1] ∪ (3, 4)

(b)   [0, 1] ∪ [3, 4]

(c)   [−1, 1] ∪ (3, 4]

(d)   None of these

Answer: (b)

143. Let  the  is

Answer: (b)

144. If  then  is equal to

(a)   50

(b)   47

(c)   44

(d)   53

Answer: (b)

145. The number of distinct solutions of the equation  in the interval [0, 2π] is

(a)   8

(b)   10

(c)   6

(d)   15

Answer: (a)

146. If the tangent at a point  to the ellipse 16x2 + 11y2 = 256 is also a tangent to x2 + y2 – 2x = 15, then ϕ equals

(a)   π/3

(b)   π/6

(c)   − π/6

(d)   π/4

Answer: (a)

147. The distance of point of intersection of the tangents to the parabola x = 4y – y2 drawn at the points where it is meet by Y-axis, from its focus is

(a)   11/4

(b)   17/4

(c)   13/4

(d)   3

Answer: (b)

148. The value of the sum  is

(a)   5

(b)   4

(c)   3

(d)   2

Answer: (d)

149. A curve passes through (2, 0) and the slope of the tangent at P(x, y) is equal to  then the equation of the curve is

(a)   y = x2 – 2x

(b)   y = x3 – 8

(c)   y2 = x2 + 2x

(d)   y2 = 5x2 – 6

Answer: (a)

150. Consider matrix  if A1 = αI + βA, when α, β ∉ R, then (α + β) is equal to (where A1 denotes the inverse of matrix A)

(a)   1

(b)   4/3

(c)   5/3

(d)   1/3

Answer: (a)

BITSAT Examination Previous Year Question Paper 2022 With Answer Key

BITSAT SOLVED PAPER-2022

PART-I

PHYSICS

1. The stopping potential (V0) versus frequency (v) of a graph of photoelectric effect in a metal. From the graph, the planck’s constant (h) is.

(a)   6.60 × 1034 J-s

(b)   6.69 × 1034 J-s

(c)   6.62 × 1034 J-s

(d)   6.63 × 1034 J-s

Answer: (a)

2. In a resonance column first and second resonance are obtained at depths 24 cm and 78 cm the third resonance will be obtained at depth.

(a)   160 cm

(b)   132 cm

(c)   131 cm

(d)   152 cm

Answer: (b)

3. A submarine A travelling at 17 m/s is being chased along the line of its velocity by another submarine B travelling at 34 m/s. B sends a sonar signal of 600 Hz to detect A and receives a reflected sound of frequency v. The of v is

[Speed of sound in water = 1500 ms1]

(a)   613.7 Hz

(b)   6137 Hz

(c)   62 Hz

(d)   539 Hz

Answer: (a)

4. Transverse waves of the same frequency are generated in two steel wires A and B. The diameter of A is twice that of B and the tension in A is half that in B. The ratio of the velocities of the waves in A and B is

(a)   1 : 2

(b)   1 : √2

(c)   1 : 2√2

(d)   3 : 2√2

Answer: (c)

5. In the diagram shown below, both the strings AB and CD are made of same material and have same cross-section. The pulleys are light and frictionless. If the speed of wave in string AB is v1 and in CD is v2, then v1/v2 is

(a)   1

(b)   √2

(c)   2

(d)   1/√2

Answer: (d)

6. What will be the acceleration due to gravity at a depth d, where g is acceleration due to gravity on the surface of earth?

Answer: (d)

7. A direct current of 6 A is superimposed on an alternating current I = 10 sin ωt flowing through a wire. The effective value of t he resulting current will be

(a)   5√2

(b)   5√3

(c)   9.27

(d)   8.37

Answer: (c)

8. Which one of the following graphs represents the variation of electric potential with distance r from the centre of a non-conducting charged sphere of radius R?

Answer: (d)

9. For an insulator, the forbidden energy gap is

(a)   Zero

(b)   1eV

(c)   2eV

(d)   5eV

Answer: (d)

10. A machine gun fires 300 bullets per min if the mass of each bullet is 10 g and the velocity of the bullets is 600 ms1, the power (in kW) of the gun is

(a)   43200

(b)   9

(c)   72

(d)   7.2

Answer: (b)

11. Four holes of radius 5 cm are cut from a thin square plate of 20 cm and mass 1 kg. The moment of inertia of the remaining portion about Z-axis is

(a)   15 kg-m2

(b)   0.37 kg-m2

(c)   0.0017 kg-m2

(d)   0.08 kg-m2

Answer: (c)

12. A particle of mass m is projected with velocity υ at an angle θ with the horizontal. At its highest point, it explodes into two pieces of equal mass, one of the piece continue to move on the original trajectory, then the velocity of second piece is.

(a)   2 v cos θ

(b)   v cos θ

(c)   3 v cos θ

(d) 

Answer: (b)

13. In the circuit shown assume the diode to be ideal. When Vi increases from −2V to 6V, the change in current is (in mA)

(a)   Zero

(b)   20

(c)   25/8

(d)   32

Answer: (b)

14. The de-Broglie wavelength of an electron moving with a velocity  is equal to the wavelength of photon. The ratio of the kinetic energies of electron and photon is

(a)   1 : 4

(b)   1 : 3

(c)   1 : 2

(d)   2 : 1

Answer: (b)

15. In the circuit shown in the figure, the AC source gives a voltage V = 20 cos (2000 t) neglecting source resistance, the voltmeter and ammeter reading will be

(a)   0V, 0.47 A

(b)   2.82 V, 1.41 A

(c)   1.41 V, 0.47 A

(d)   1.5 V, 8.37 A

Answer: (b)

16. An electromagnetic wave is propagating along X-axis. At x = 1 cm and t = 18s, its electric vector |E| = 8 V/m, then the magnitude of its magnetic vector is

(a)   2.66 × 108

(b)   3 × 107

(c)   3.14 × 108

(d)   3.16 × 107

Answer: (a)

17. In the following circuit the equivalent resistance between X and Y is …….. Ω

(a)   5

(b)   12

(c)   16

(d)   20

Answer: (c)

18. A monoatomic gas of molar mass m is kept in a insulated container. Container is moving with velocity v. If the container is suddenly stopped, then the change in the temperature of the gas is

(a)   mv2/4R

(b)   mv2/2R

(c)   mv2/R

(d)   mv2/3R

Answer: (d)

19. A projectile is projected with the velocity of  The horizontal range of the projectile will be

(a)   1.2 m

(b)   2.4 m

(c)   3.6 m

(d)   4.5 m

Answer: (b)

20. A transistor is connected in common-emitter (CE) configuration. The collector supply is 8V and the voltage drop across a resistor is 500 Ω in the collector circuit is 0.6 V. If the current gain factor α is 0.96, find the base current

(a)   25 μA

(b)   50 μA

(c)   20 μA

(d)   35 μA

Answer: (b)

21. A solid sphere of 80 kg radius 15 m moving in a space becomes a circular disc of radius 20 m in 1 h. The rate of change of moment of Inertia in this process is ………

Answer: (d)

22. If the B – H curves of two samples of X and Y of iron are as shown below, then which one of the following statement is correct?

(a)   Both X and Y are suitable for making electromagnets.

(b)   Both X and Y are suitable for making permanent magnet.

(c)   X is suitable for making permanent magnet and Y for making electromagnet.

(d)   X is suitable for making electromagnet and Y is suitable for permanent magnet.

Answer: (c)

23. In a radioactive material the activity at time t1, is A1 and at a later time t2, it is A2. If the decay constant of the material is λ, then

Answer: (a)

24. A mosquito O is sitting infront of a glass rod having spherical end of radius of curvature 40 cm. The image would be formed at

(a)   40 cm left

(b)   infinity

(c)   20 cm to the right

(d)   15 cm to the left

Answer: (a)

25. One mole of an ideal diatomic gas undergoes a process as shown in the figure. The molar specific heat of the gas in the process is

(a)   3R/2

(b)   R/2

(c)   5R/2

(d)   7R/2

Answer: (a)

26. A capillary tube is attached horizontally to a constant heat arrangement. If the radius of the capillary tube is increased by 25%, then the rate of flow of liquid will change nearly by

(a)   100%

(b)   112%

(c)   124%

(d)   144%

Answer: (d)

27. In the arrangement shown in figure, when the switch S2 is open, the galvanometer, shows no deflection of l = 50 cm when the switch S2 is closed, the galvanometer shows no deflection for l = 0.416 m. The internal resistance (r) of 6 V cell is

(a)   2 Ω

(b)   3 Ω

(c)   5 Ω

(d)   9 Ω

Answer: (a)

28. In a young’s double slit arrangement frings are produced using light of wavelength 4000 Å. One slit is covered by a thin plate of glass of refractive index 1.4 and the other with another glass plate of same thickness but of refractive index 1.7. By doing so the central bright shifts to original sixth fringe from centre. Thickness of glass plate is ………. .

(a)   2 μm

(b)   8 μm

(c)   11 μm

(d)   16 μm

Answer: (b)

29. An electric current I enters and leaves a uniform circular wire of radius r through diametrically opposite points. A charged particle q moves along the axis of circular wire passes through its centre at speed v. The magnetic force on the particle when it passes through the centre has a magnitude.

Answer: (d)

30. An achromatic convergent doublet of two lenses in contact has a power of +5D. The power of converging lens is +6D. The ratio of the dispersive power of the convergent and divergent lenses is

(a)   3 : 7

(b)   2 : 3

(c)   1 : 5

(d)   5 : 3

Answer: (c)

PART II

Chemistry

31. Which one of the following is correct or3der of given isotopes?

(I) T2 > D2 > P2 (order of boiling point)

(II) T2 > D2 > P2 (order of bond energy)

(III) T2 = D2 = P2 (order of bond length)

(IV) T2 < D2 < P2 (order of reactivity with Cl2)

(a)   I and II

(b)   III and IV

(c)   II, III and IV

(d)   All of these

Answer: (d)

32. Ninhydrin gives yellow colour in paper chromatography with which amino acid?

(a)   Tryptophan

(b)   Proline

(c)   Alanine

(d)   Tyrosine

Answer: (b)

33. How will raise in temperature affects the viscosity of liquids and gases?

(a)   Both increases

(b)   Both decreases

(c)   In case of liquids, decreases and in case of gases increases.

(d)   In case of liquid, increases and in case of gases, decreases.

Answer: (b)

34. Which of the following compounds is thermodynamically is the most stable?

(a)   BaCO3

(b)   MgCO3

(c)   SrCO3

(d)   CaCO3

Answer: (a)

35. Glucose reacts with X number of molecules of phenyl hydrazine to yield osazone. The value of X is,

(a)   three

(b)   two

(c)   one

(d)   four

Answer: (a)

36. Nylon-6, 6 is obtained from

(a)   adipic acid and hexamethylene diamine

(b)   tetrafluoroethylene

(c)   vinyl cyanide

(d)   vinyl benzene

Answer: (a)

37. What is the hybridization of [CrF6]3?

(a)   sp3d

(b)   sp3d2

(c)   d2sp3

(d)   d2sp

Answer: (c)

38. OF an F2 can be compared in terms of

(a)   OF is paramagnetic while F2 is diamagnetic

(b)   OF is more stable towards dissociation into atoms

(c)   Both (a) and (b) are correct

(d)   None of the above is correct

Answer: (c)

39. ortho and para form of hydrogen have

(a)   different physical and chemical properties

(b)   identical physical properties but different chemical properties

(c)   identical chemical properties but different physical properties

(d)   identical chemical and physical properties

Answer: (c)

40. The structure of H2O2 is

(a)   planar, linear

(b)   non-planar, linear

(c)   planar, non-linear

(d)   non-planar, non-linear

Answer: (d)

41. Match the species in Column I with their types in Column II.

Codes

(a)   A→4, B→3, C→2, D→1

(b)   A→1, B→2, C→3, D→1

(c)   A→2, B→3, C→1, D→4

(d)   A→3, B→1, C→2, D→4

Answer: (a)

42. In which pair or pairs is the stronger bond found in the first species?

(I) O22, O2;       (II) N2, N2+;     (III) NO+, NO

(a)   I only

(b)   II only

(c)   I and II only

(d)   II and III only

Answer: (d)

43. Select the correct statement about the complex [Co(NH3)5SO4]Br.

(a)   Its ionization isomer is [Co(NH3)5Br]SO4.

(b)   It gives yellow precipitate with AgNO3.

(c)   Its ionization isomer give while precipitate with BaCl2­.

(d)   All the above are correct statements.

Answer: (d)

44. A certain metal sulphide, MS2, is used extensively as a high temperature lubricant. If MS2 is 40.06% by mass sulphur, metal M has atomic mass.

(a)   160 u

(b)   64 u

(c)   40 u

(d)   96 u

Answer: (d)

45. 

X and Y are

(a)   benzene, benzaldehyde

(b)   toluene, benzaldehyde

(c)   toluene, benzoic acid

(d)   benzene, benzoic acid

Answer: (c)

46. Ge(II) compounds are powerful reducing agents whereas Pb(IV) compounds are strong oxidants. It can be because

(a)   Pb is more electropositive than Ge.

(b)   ionization potential of lead is less than that of Ge.

(c)   ionic radii of Pb2+ and Pb4+ are larger than that of Ge2+ and Ge4+

(d)   more pronounced inert pair effect in lead has.

Answer: (d)

47. Which compound has antifluorite structure?

(a)   MnO4

(b)   Na2O

(c)   Na2O2

(d)   Li2O2

Answer: (b)

48. 100 mL of 2 M of formic acid (pK­a = 3.74) is neutralize by NaOH, at the equivalence point pH is

(a)   7

(b)   6

(c)   9.5

(d)   8.87

Answer: (d)

49. The reaction of C6H5CH = CHCH3 with HBr produces

Answer: (a)

50. The number of 3C−2e bonds present in diborane is

(a)   1

(b)   2

(c)   3

(d)   4

Answer: (b)

51. Standard entropy of X2, Y2 and XY2 are 60, 40 and 50 JK1 mol1, respectively. For the reaction,  to be at equilibrium, the temperature will be

(a)   1250 K

(b)   500 K

(c)   750 K

(d)   1000 K

Answer: (c)

52. The total number of P−OH bonds for pyrophosphoric acid

(a)   4

(b)   5

(c)   6

(d)   8

Answer: (a)

53. Using the standard electrode potential, find out the pair between which redox reaction is not feasible.

E values Fe3+/Fe2+ = +0.77; I2/I = + 0.54 Cu2+/Cu = +0.34; Ag+/Ag = 0.80 V

(a)   Fe3+ and I

(b)   Ag+ and Cu

(c)   Fe3+ and Cu

(d)   Ag and Fe3+

Answer: (d)

54. What is [NH4+] in a solution that is 0.02 M NH301 M KOH ? [Kb(NH3) = 1.8 × 105]

(a)   3.6 × 105 M

(b)   1.8 × 105 M

(c)   0.9 × 105 M

(d)   7.2 × 105 M

Answer: (a)

55. For an isomerization reaction A ⇋ B, the temperature dependence of equilibrium constant is given by

The value of ∆S° at Hook is, therefore

(a)   4R

(b)   5R

(c)   400R

(d)   2000R

Answer: (a)

56. In an adiabatic process, no transfer of heat takes place between system and surrounding. Choose the correct option for free expansion of an ideal gas under adiabatic condition from the following.

(a)   q = 0, ∆T ≠ 0, W = 0

(b)   q ≠ 0, ∆T = 0, W = 0

(c)   q = 0, ∆T = 0, W = 0

(d)   q = 0, ∆T < 0, W ≠ 0

Answer: (c)

57. The given graph represents the variation of compressibility factor (Z) = pV/nRT, for three real gases A, B and C. Identify the only incorrect statement.

(a)   For the gas A, a = 0 and the dependence on p is linear at all pressure.

(b)   For the gas B, b = 0 and its dependence on p is linear at all pressure.

(c)   For the gas C, which is typical real gas for which neither, a nor b = 0. By knowing the minima and point of the intersection, with Z = 1, a and b can be calculated.

(d)   At high pressure the slope is positive for all real gases.

Answer: (b)

58. Which one of the following statements in relation to the hydrogen atom is correct?

(a)   3s, 3p and 3d-orbitals all have the same energy.

(b)   3s and 3p-orbitals are of lower energy than 3d-orbital.

(c)   3p-orbital is lower in energy than 3d-orbital.

(d)   3s-orbital is lower in energy than 3p-orbital.

Answer: (a)

59. In the molecules CH4, NF3, NH4+ and H2O

(a)   number of lone pairs are same

(b)   all have same hybridization of centre of atom

(c)   the bond angles are same

(d)   number of bond pairs are same

Answer: (b)

60. 0.20 g of an organic compound gave 0.12 g of AgBr By using Carius method, the percentage of bromine in the compound will be

(a)   34.06%

(b)   44.04%

(c)   54%

(d)   25%

Answer: (d)

PART III

(a) English Proficiency

Directions (Q. Nos. 61-64) Choose the word which best expresses the meaning of the  underlined word in the sentence.

61. Forthrightness in speech may not always be a desirable quality.

(a)   Outspokenness

(b)   Obliqueness

(c)   Mendacity

(d)   Equivocation

Answer: (a)

62. The inexorable demands of the workers brought the company to a closure.

(a)   Unreasonable

(b)   Relentless

(c)   Monetary

(d)   Violent

Answer: (b)

63. Select the one which best expresses the same sentence in Passive/Active voice.

Then her face was bowed.

(a)   Then she was being bowed her face.

(b)   Her face was bowed by them.

(c)   Then she bowed her face.

(d)   Then her face has been bowed.

Answer: (c)

64. The complex form of the sentence given below would be

Spare the rod and spoil the child.

(a)   The child is spoiled if the rod is spared.

(b)   The child becomes spoiled when the rod is spared.

(c)   The child is spoiled whenever the rod is spared.

(d)   The child is spoiled when the rod is spared.

Answer: (d)

Directions (Q. Nos. 65-66) Choose the word which is closest to the opposite in meaning of the given italicized word.

65. The attack on the freedom of the press is a retrograde

(a)   progressive

(b)   stubborn

(c)   punitive

(d)   aggressive

Answer: (a)

66. The leader might have had some covert reason for the change of his political affiliations.

(a)   Unjustifiable

(b)   Obvious

(c)   Inexplicable

(d)   Flimsy

Answer: (b)

Directions (Q. Nos. 67-68) In the following questions, out of the four alternatives, choose the one which can be substituted for the given word/sentence.

67. Regard for others as a principle of action or selflessly.

(a)   Gynicism

(b)   Nepotism

(c)   Philanthropy

(d)   Altruism

Answer: (d)

68. Code of diplomatic etiquette and precedence is

(a)   Formalism

(b)   Statesmanship

(c)   Protocol

(d)   Hierarchy

Answer: (c)

Directions (Q. Nos. 69-70) Choose the order of the sentences marked A, B, C and D to form a logical paragraph.

69. (A) Now under liberated economy they are learning to compete domestically and globally.

(B) In India corporations until recently achieved success by avoiding competition, using protected and regulated domestic markets.

(C) The trend is irreversible.

(D) Business leaders are preparing themselves to meet competitive challenges, and to avoid being swept away.

(a)   BADC

(b)   BDCA

(c)   BDAC

(d)   CDBA

Answer: (a)

70. (A) Recovery was given inadequate attention and consequently some bank branches regularly incurred heavy losses and their parent bodies had to bale them out.

(B) As a result, banks indulged in extensive lending to borrowers who had little or no potential to make repayments.

(C) To fulfill the social objectives laid down by the masters of nationalization, banks were asked to lend to identified priority sectors.

(D) 1992-93 results showed that the loss making branches of public sector banks increased from 10,000 to 13,000 and the quantum of losses showed at Rs. 3,369 crores.

(a)   BACD

(b)   DABC

(c)   CBAD

(d)   BCAD

Answer: (c)

(b) Logical Reasoning

71. Select the figure that can replace the question mark (?) in the following series.

Answer: (a)

72. ‘A + B’ means ‘A is the mother of B’.

‘A – B’ means ‘A is the brother of B’.

‘A × B’ means ‘A is the father of B’.

‘A ÷ B’ means ‘A is the daughter of B’.

If, P – K × Y – J ÷ S + R, then which of the following statement is not correct?

(a)   K is husband of S

(b)   Y is son of S

(c)   J is daughter of P

(d)   P is paternal uncle of R

Answer: (c)

73. Three different positions of the same dice are shown, the six faces of which are numbered from 1 to 6. Select the number that will be on the face opposite to the one showing ‘6’.

(a)   2

(b)   4

(c)   5

(d)   3

Answer: (d)

74. Select the option in which the given figure X is embedded (rotation is now allowed).

Answer: (b)

75. Select the letter-cluster that can replace the question mark (?) in the following series.

TULG, WRPC, ZOTY, CLXU, ?

(a)   FIBQ

(b)   FICR

(c)   FJCQ

(d)   GIAQ

Answer: (a)

76. How many triangles are there in the given figure?

(a)   33

(b)   18

(c)   31

(d)   29

Answer: (c)

77. The average marks of 50 students in a class was found to be 64. If the marks of two students were incorrectly entered as 38 and 42 instead of 83 and 24, respectively, then what is the correct average?

(a)   64.54

(b)   62.32

(c)   61.24

(d)   61.86

Answer: (a)

78. Select the correct mirror image of the given figure when the mirror is placed on the right of the figure.

Answer: (a)

79. Six friends A, B, C, D, E and F are sitting around a round table facing the centre. A sits second to the right of B, E sits second to the left of C. B doesn’t sit adjacent to E. D does not sit opposite to E or C. Who sits to the immediate left of E?

(a)   A

(b)   D

(c)   B

(d)   C

Answer: (a)

80. Five friends A, B, C, D and E bought cars which were priced differently. B’s car was costlier than C’s car but was less costly than E’s car. A’s car was costlier than D’s car but less costly than C’s car. Whose car was the 2nd costliest?

(a)   E

(b)   A

(c)   B

(d)   C

Answer: (c)

81. In the following questions, complete the missing segment by selecting the appropriate figure from the given alternatives, (a), (b), (c) and (d).

Answer: (b)

82. In each of the following question, find out which of the answer figures (a), (b), (c) and (d) completes the figure matrix?

Answer: (d)

Directions (Q. No. 83-84) In the following questions two statements are given followed by two conclusions I and II. You have to consider the two statements to be true even if they seem to be at variance from commonly known facts. You have to decide which of the given conclusions, if any follow from the given statements.

Give Answer

(a) Only conclusion I follows

(b) Only conclusion II follows

(c) Both conclusions I and II follow

(d) Either conclusion I or II follows

83. Statements 60% of government employees went on strike.

Mr. Gopal is government employee.

Conclusions

(I) Mr. Gopal went on strike.

(II) Mr. Gopal did not participate in the strike.

Answer: (d)

84. Statements

Lawyers marry only fair girls.

Shobha is very fair.

Conclusions

(I) Shobha is marked to a lawyer.

(II) Shobha is not married to a lawyer.

Answer: (d)

85. In the question given below, find out which of the figures can be formed from the pieces given in the problem figure.

Answer: (b)

86. Select the option in which the words share the same relationship as that shared by the given pair of words.

Barometer : Pressure

(a)   Ammeter : Current

(b)   Thermometer : Volume

(c)   Voltmeter : Heat

(d)   Scale : Seconds

Answer: (a)

87. Select the option in which the words share the same relationship as that shared by the given set of words.

Cat : Lion : Jaguar

(a)   Shark : Dolphin : Bat

(b)   Sport; Athlete : Javelin

(c)   Monkey : Chimpenzee : Gorilla

(d)   Reptile : Snake : Toad

Answer: (c)

88. ‘Needle’ is related to ‘Sew’ in the same way as ‘Microscope’ is related to ‘……….’.

(a)   Laboratory

(b)   Lens

(c)   Science

(d)   Magnify

Answer: (d)

89. Select the option that is related to the fifth number in the same way as the second number is related to the first number and the fourth number is related to the third number.

14 : 289 : : 17 : 400 : : 21 : ?

(a)   576

(b)   504

(c)   570

(d)   441

Answer: (a)

90. Select the letter-cluster that can replace the question mark (?) in the following series.

TXB, QWE, NVH, KUK, ?

(a)   ITM

(b)   JTM

(c)   HTN

(d)   HSN

Answer: (c)

PART IV

Mathematics

91. If α be a root of the equation 4x2 + 2x – 1 = 0, then the other root of the equation is

(a)   4α3 + 2α

(b)   4α2 – 2α

(c)   4α3 – 3α

(d)   4α3 + 3α

Answer: (c)

92. If A = {x : x is a multiple of 4}. And,

B + { x : x is a multiple of 6}, then A ∩ B consist of multiple of

(a)   16

(b)   12

(c)   8

(d)   4

Answer: (b)

93. If |w| = 2, then the set of points  is contained in or equal to the set of points z satisfying

(a)   Im(z) = 0

(b)   |Im(z)| ≤ 1

(c)   |Re(z)| ≤ 2

(d)   |z| ≤ 3

Answer: (d)

94. The value of  is

(a)   1/6

(b)   1/8

(c)   1/10

(d)   1/12

Answer: (b)

95. Let a1, a2, ….. a40 be in AP and h1­, h2, …. H10 be in HP. If a1 = h1 = 2 and a10 = h10 = 3, then a4h7 is

(a)   2

(b)   3

(c)   5

(d)   6

Answer: (d)

96. The number of terms in the expansion of (1 + 5√2x)9 + (1 – 5√2x)9, is

(a)   5

(b)   7

(c)   9

(d)   10

Answer: (a)

97. The number of different seven-digit numbers that can be written using only the three digit 1, 2 and 3 with the condition that the digit 2 occurs twice in each number is

(a)   7C225

(b)   7p225

(c)   7C252

(d)   None of these

Answer: (a)

98. Given 2x – y + 2z = 2, x – 2y + z = −4, x + y + λz = 4, then the value of λ such that the given system of equation has no solution is

(a)   −3

(b)   1

(c)   0

(d)   3

Answer: (d)

99. Let  and 10  If B is the inverse of A, then the value of α is

(a)   4

(b)   −4

(c)   3

(d)   5

Answer: (d)

100. If x ∈ (0, π/2), then the value of  is equal to

(a)   x − cos1 (7 cos x)

(b)   x + sin1 (7 cos x)

(c)   x + cos1 (6 cos x)

(d)   x + cos1 (7 cos x)

Answer: (a)

101. A running track of 440 ft is to be laid out enclosing a football field, the shape of which is a rectangle with a semi-circle at each end. If the area of the rectangular portion is to be maximum, then the lengths of its side are

(a)   70 ft and 110 ft

(b)   80 ft and 120 ft

(c)   35 ft and 110 ft

(d)   35 ft and 120 ft

Answer: (c)

102.  find general solution

(a)   y = tan x(log|cosec x – cot x| + cos x + c)

(b)   y = sec2 x + tan x + c

(c)   y = log|sec x + tan x| + cosec x + c

(d)   y = tan2 x + sin x + c

Answer: (a)

103. If the straight line y = mx + c touches the parabola y2 – 4ax + 4a3 = 0, then c is

Answer: (d)

104. A normal is drawn at the point P to the parabola y2 = 8x, which is inclined at 60° with the straight line y = 8. Then the point P lies on the straight line

(a)   2x + y – 12 – 4√3 = 0

(b)   2x – y – 12 + 4√3 = 0

(c)   2x – y – 12 – 4√3 = 0

(d)   None of these

Answer: (c)

105. The value of  is

Answer: (c)

106. The area of the region bounded by the parabola (y – 2)2 = (x – 1), the tangent to the parabola at the point (2, 3) and the X-axis is

(a)   3

(b)   6

(c)   9

(d)   12

Answer: (c)

107. are two non-collinear unit vectors such that  Then the value of  is equal to

Answer: (d)

108. A six faced die is a biased one. It is thrice more likely to show an odd numbers than show an even number. It is thrown twice. The probability that the sum of the numbers in two throws is even, is

(a)   5/9

(b)   5/8

(c)   1/2

(d)   None of these

Answer: (b)

109. The sum of all the solution of the equation  θ ∈ [0, 6π]

(a)   15π

(b)   30π

(c)   100π/3

(d)   None of these

Answer: (b)

110. Let α be the solution of  in (0, π/4). If the shadow of a vertical pole is 1/√3 of its height, then the altitude of the sun is

(a)   α

(b)   α/2

(c)   2α

(d)   α/3

Answer: (c)

111. For each parabola y = x2 + px + q, meeting coordinate axes at 3-distinct points, if circles are drawn through these points, then the family of circles must pass through

(a)   (1, 0)

(b)   (0, 1)

(c)   (1, 1)

(d)   (p, q)

Answer: (b)

112. The number of ways of arranging letters of the word HAVANA so that V and N do not appear together is

(a)   40

(b)   60

(c)   80

(d)   100

Answer: (c)

113. Let a1, a2, a3 …. Be a harmonic progression with a1 = 5 and a20 = 25. The least positive integer n for which an < 0, is

(a)   22

(b)   23

(c)   24

(d)   25

Answer: (d)

114. If the plane 3x + y + 2z + 6 = 0 is parallel to the line  then the value of 3a + 3b is

(a)   1/2

(b)   3/2

(c)   3

(d)   4

Answer: (b)

115. Let a, b be the solutions of x2 + px + 1 = 0 and c, d be the solution of x2 + qx + 1 = 0. If (a – c) (b – c) and (a + d) (b + d) are the solution of x2 + ax + β = 0, then β is equal to

(a)   p + q

(b)   p – q

(c)   p2 + q2

(d)   q2 – p2

Answer: (d)

116. If  then

(a)   a = 1, b = 1

(b)   a = sin 2θ, b = cos 2θ

(c)   a = cos 2θ, b = sin 2θ

(d)   None of these

Answer: (c)

117. The value of  is

(a) 

(b) 

(c)   e/24

(d)   None of these

Answer: (a)

118. The locus of the mid-point of the chord if contact of tangents drawn from points lying on the straight line 4x – 5y = 20 to the circle x2 + y2 = 9 is

(a)   20(x2 + y2) – 36x + 45y = 0

(b)   20(x2 + y2) + 36x – 45y = 0

(c)   36(x2 + y2) – 20x + 45y = 0

(d)   36(x2 + y2) + 20x – 45y = 0

Answer: (a)

119. Let  and f(0) = 0, then the value of f(1) be

(a)   log(1 + √2)

(b) 

(c) 

(d)   None of these

Answer: (b)

120. The mean of five observations is 4 and their variance is 5.2. If three of these observations are 1, 2 and 6, then the other two are

(a)   2 and 9

(b)   3 and 8

(c)   4 and 7

(d)   5 and 6

Answer: (c)

121. In a sequence of 21 terms, the first 11 terms are in AP with common difference 2 and the last 11 terms are in GP with common ratio 2. If the middle term of AP be equal to the middle term of the GP, then the middle term of the entire sequence is

(a)   −10/31

(b)   10/31

(c)   32/31

(d)   −31/32

Answer: (a)

122. If p ≠ a, q ≠ b, r ≠ c and the system of equations

px + ay + az = 0

bx + qy + bz = 0

cx + cy + rz = 0

has a non-trivial solution, then the value of  is

(a)   1

(b)   2

(c)   1/2

(d)   0

Answer: (b)

123. If g(x) = x2 + x – 2 and  then f(x) is equal to

(a)   2x – 3

(b)   2x + 3

(c)   2x2 + 3x + 1

(d)   2x2 – 3x + 1

Answer: (a)

124. The smallest positive integral value of n such that  is purely imaginary, is equal to

(a)   4

(b)   3

(c)   2

(d)   8

Answer: (a)

125. A house subtends a right angle at the window of a opposite house and the angle of elevation of the window from the bottom of the first house is 60. If the distance between two houses be 6 m, then the height of the first house is

(a)   8√3 m

(b)   6√3 m

(c)   4√3 m

(d)   None of these

Answer: (a)

126. A spherical balloon is filled with 4500π cubic meters of helium gas. If a leak in the balloon causes the gas to escape at the rate of 72π cubic meters per minute then the rate (in meters per minute) at which the radius of the balloon decreases 49 min after the leakage began is

(a)   9/7

(b)   7/9

(c)   2/9

(d)   9

Answer: (c)

127. If in a ∆ABC, 2b2 = a2 + c2, then  is equal to

Answer: (d)

128. If the sum of the coefficients in the expansion of (x + y)n is 1024, then the value of greatest coefficient in the expansion is

(a)   356

(b)   252

(c)   210

(d)   120

Answer: (b)

129. The area enclosed by the curves y = sin x + cos x and y = |cos x – sin x| over the interval [0, π/2] is

(a)   4(√2 – 1)

(b)   2√2(√2 – 1)

(c)   2(√2 + 1)

(d)   2√2(√2 + 1)

Answer: (b)

130. If α, β, γ ∈ [0, π] and if α, β, γ are in AP, then  is equal to

(a)   sin β

(b)   cos β

(c)   cot β

(d)   2 cos β

Answer: (c)

National Institute of Fashion & Technology (NIFT) Post Graduate 2020 Question Paper With Answer Key

NIFT (Post Graduate)

National Institute of Fashion and Technology

Solver Paper 2020

1. Three Science classes A, B and C take a Life Science test. The average score of class A is 83. The average score of class B is 76. The average score of class C is 85. The average score of classes A and B is 79 and average score of classes B and C is 81. Then, the average score of classes A, B and C is

(a)  80

(b)  80.5

(c)  81.5

(d)  81

Answer: (c)

2. In a library the ratio of story books and other books is 7 : 2 and there are 1512 story books. Due to collection of some more story books the said ratio becomes 15 : 4. The number of story books collected is

(a)  100

(b)  97

(c)  108

(d)  205

Answer: (c)

3. A and B have their monthly incomes in the ratio 8 : 5, while their monthly expenditures are in the ratio 5 : 3. If they have saved Rs. 12000 and Rs. 10000 monthly respectively, then the difference in their monthly incomes is

(a)  Rs. 52000

(b)  Rs. 46000

(c)  Rs. 44000

(d)  Rs. 42000

Answer: (d)

4. A farmer travelled a distance of 61 km in 9 h. He travelled partly on foot at the rate 4 km/h and partly on bicycle at the rate 9 km/h. The distance travelled on foot is

(a)  14 km

(b)  15 km

(c)  16 km

(d)  17 km

Answer: (c)

5. A sum of Rs. 7930 is divided into 3 parts and given on loan at 5% simple interest to A, B and C for 2 yr, 3 yr and 4 yr, respectively. If the amounts of all three are equal after their respective periods of loan, then the A received a loan of

(a)  Rs. 2760

(b)  Rs. 3050

(c)  Rs. 2800

(d)  Rs. 2750

Answer: (a)

6. A and B can do a piece of work in 30 and 36 days, respectively. They began the work together but A leaves after some days and B finished the remaining work in 25 days. After how many days did A leave?

(a)  6 days

(b)  11 days

(c)  10 days

(d)  5 days

Answer: (d)

7. From 1980-90, the population of a country was increased by 20%. From 1990-2000, the population of the country was increased by 20%. From 2000-10, the population of the country was increased by 20%. Then the overall increased population (in percentage) of the country from 1980-2010 was

(a)  60%

(b)  62.8%

(c)  72.2%

(d)  72.8%

Answer: (d)

8. If the product of two positive numbers be 1575 and their ratio is 7 : 9, then the greater number is

(a)  35

(b)  63

(c)  45

(d)  135

Answer: (c)

9. If A : B = 2 : 3 and B : C = 3 : 7, then

A + B : B + C : C + A is equal to

(a)  4 : 10 : 9

(b)  5 : 8 : 9

(c)  5 : 10 : 9

(d)  4 : 8 : 9

Answer: (c)

10. The HCF and LCM of two numbers are 21 and 84, respectively. If the ratio of the two numbers is 1 : 4, then the larger of the two numbers is

(a)  84

(b)  108

(c)  48

(d)  12

Answer: (a)

11. A sell a cycle to B at a profit of 20% and B sells it to C at a loss of 25%. If C bought the cycle for Rs. P, then the cost price of it for A was

(a)  Rs. 9/10 P

(b)  Rs. 10/9 P

(c)  Rs. 1/20P

(d)  Rs. 9/20 P

Answer: (b)

12. A number when divided by 361 gives a remainder 47. If the same number is divided by 19, then the remainder obtained is

(a)  8

(b)  1

(c)  3

(d)  9

Answer: (d)

13. If 64 buckets of water are removed from a cubical shaped water tank completely filled with water, 1/3 of the tank remains filled with water. The length of each side of the tank is 1.2 m. Assuming that all buckets are of the same measure, then the volume (in litre) of water contained by each bucket is

(a)  12

(b)  16

(c)  18

(d)  15

Answer: (c)

14. In a farm there are cows and hens. If heads are counted there are 180, if legs are counted there are 420. The number of cows in the farm is

(a)  150

(b)  30

(c)  130

(d)  50

Answer: (b)

15. If the arithmetic means of 3a and 4b is greater than 50, and a is twice of b, then the smallest possible integer value of a is

(a)  18

(b)  19

(c)  20

(d)  21

Answer: (d)

16. If the average of eight consecutive even number be 93, then the greatest number among them is

(a)  86

(b)  98

(c)  100

(d)  102

Answer: (c)

17. x can copy 80 pages in 20 h, x and y together can copy 135 pages in 27 h. Then y can copy 20 pages in

(a)  3 h

(b)  12 h

(c)  20 h

(d)  24 h

Answer: (c)

18. The average of the largest and smallest 3 digit number formed by 0, 2 and 4 would be

(a)  213

(b)  303

(c)  312

(d)  222

Answer: (c)

19. In what ratio two types of tea costing Rs. 180/kg and Rs. 280/kg should be mixed in the ratio so that the mixture obtained sold at Rs. 320/kg to earn a profit of 20% is

(a)  1 : 13

(b)  2 : 13

(c)  3 : 13      

(d)  4 : 13

Answer: (b)

20. If a person marked a product 25% above the cost price but allows 10% discount, then the percentage of profit is

(a)  15%

(b)  12.5%

(c)  35%

(d)  17.5%

Answer: (b)

21. x and Mr. y each bought the same motorcycle using a 10% off coupon. Mr. x’s cashier took 10% off the price and then added 8.5% sales tax, whereas Mr. y’s cashier first added the sales tax and then took 10% off the total price. The amount Mr. x paid is

(a)  same as the amount Mr. y paid

(b)  greater than Rs. 850 as the amount Mr. y paid

(c)  less than Rs. 550 as the amount Mr. y paid

(d)  greater than Rs. 85 as the amount Mr. y paid

Answer: (a)

22. Two workers A and B are engaged to do a piece of work A working alone would take 8 h more to complete the work than when work together. If B worked alone, would take  more than when work together. The time required to finish the work together is

(a)  4 h

(b)  6 h

(c)  5 h

(d)  8 h

Answer: (b)

23. A and B invest in the ratio 3 : 5. After 6 months, C joins the business investing an amount equal to amount of B. At the end of the year what will be the ratio of their profits?

(a)  3 : 5 : 2

(b)  3 : 5 : 5

(c)  6 : 10 : 5

(d)  8 : 10 : 5

Answer: (c)

24. If a man walks at the rate of 5 km/h, he misses a train by 7 min. However, if he walks at the rate of 6 km/h, he reaches the station 5 min before the arrival of the train. The distance covered by him to reach the station is

(a)  6 km

(b)  4 km

(c)  7 km

(d)  6.25 km

Answer: (a)

25. A dealer fixed the price of an article 40% above the cost of production. While selling it he allows a discount of 20% and makes a profit of Rs. 48. The cost of production (in Rs.) of the article is

(a)  400

(b)  360

(c)  320

(d)  420

Answer: (a)

26. Articles are marked at a price which gives a profit of 25%. After allowing a certain discount the profit reduces to  The discount per cent is

(a)

(b)  12%

(c)  10%

(d)  11.1%

Answer: (c)

27. The average age of 30 students of a class is 14 yr 4 months. After admission of 5 new students in the class the average becomes 13 yr 9 months. The youngest one of the five new students is 9 yr 11 months old. The average age of the remaining 4 new students is

(a)  13 yr 6 months

(b)  12 yr 4 months

(c)  10 yr 4 months

(d)  11 yr 4 months

Answer: (c)

28. A sum of money is paid back in two annual installments of Rs. 17640 each, allowing 5% compound interest compounded annually. The sum borrowed was

(a)  Rs. 32400

(b)  Rs. 32000

(c)  Rs. 32200

(d)  Rs. 32800

Answer: (d)

29. There is a number consisting of two digits, the digit in the unit’s place is twice that in the ten’s place and if 2 is subtracted from the sum of the digits, the difference is equal to 1/6th of the number. The number is

(a)  26

(b)  23

(c)  25

(d)  24

Answer: (d)

30. Average of n numbers is a. The first number is increased by 2, second one is increased by 4, the third one is increased by 8 and so on. The average of the new numbers is

Answer: (b)

Directions (Q. Nos. 31-34) In each of the following questions, out of the given alternatives, choose the one which best expresses the meaning of the given word.

31. ENDORSEMENT

(a)  Reprimand

(b)  Censure

(c)  Commendation

(d)  Reproach

Answer: (c)

32. COMMEMORATE

(a)  Boast

(b)  Remember

(c)  Manipulate

(d)  Harmonise

Answer: (b)

33. WHIM

(a)  Fancy

(b)  Clumsy

(c)  Desire     

(d)  Strange behaviour

Answer: (c)

34. CANDID

(a)  Rude

(b)  Impertinent

(c)  Blunt

(d)  Frank

Answer: (d)

Directions (Q. Nos. 35-38) In each of the following questions, a word has been spelt in four different ways out of which only one is correct. Find the correctly spelt word.

35.

(a)  Mountaineous

(b)  Mountaneous

(c)  Mountainous

(d)  Mountanous

Answer: (c)

36.

(a)  Hindrence

(b)  Hindrane

(c)  Hinderence

(d)  Hinderance

Answer: (b)

37.

(a)  Sacrosanct

(b)  Sacrosenct

(c)  Secrosanct

(d)  Sacrosantt

Answer: (a)

38.

(a)  Palid

(b)  Pallid

(c)  Palled

(d)  Pellid

Answer: (b)

Directions (Q. Nos. 39-42) In each of the following below four sentences are given which are denoted by (A), (B), (C), (D). By using all the four sentences you have to frame a meaningful paragraph. The correct order of the sentences is your answer. Choose from the four alternatives the one having the correct order of sentences and mark it as your answer.

39. (A) Now under liberated economy they are learning to compete domestically and globally.

(B) In India corporations until recently achieved success by avoiding competition, using protected and regulated domestic markets.

(C) The trend is irreversible.

(D) Business leaders are preparing themselves to meet competitive challenges, and to avoid being swept away.

(a)  BADC

(b)  BDCA

(c)  BDAC

(d)  CDBA

Answer: (a)

40. (A) Recovery was given inadequate attention and consequently some bank branches regularly incurred heavy losses and their parent bodies had to bale them out.

(B) As a result, banks indulged in extensive lending to borrowers who had little or no potential to make repayments.

(C) To fulfill the social objectives laid down by the master of nationalization, banks were asked to lend to identified priority sectors.

(D) 1992-93 results showed that the loss making branches of public sector banks increased from 10,000 to 13,000 and the quantum of losses showed at Rs. 3,369 crores.

(a)  BACD

(b)  DABC

(c)  CBAD

(d)  BCAD

Answer: (c)

41. (A) Finally the bureaucratic organization took over from the pioneering enterprise.

(B) The nineteenth century was the age of entrepreneur, the self-made man.

(C) Thoughtful business administration took over from action-centred business

(D) In the twentieth century the rational executive took command.

(a)  DBAC

(b)  CABD

(c)  BDCA

(d)  BCDA

Answer: (c)

42. (A) He was so busy with them that he did not get time to eat.

(B) Thousands of people came to him and asked different types of questions.

(C) No one cared to see that he had his food or rest that night.

(D) Swami Vivekanand once stayed in a small village.

(a)  BCDA

(b)  CBAD

(c)  DBAC

(d)  DBCA

Answer: (c)

Directions (Q. Nos. 43-47) Each item in this section consists of sentences with an underlined word followed by four words or group of words. Select the option that is opposite in meaning to the underlined word.

43. He speaks eloquently and can pull crowds.

(a)  confusingly

(b)  expressively

(c)  powerfully

(d)  fluently

Answer: (a)

44. Everyone has to fight the inertia in the system.

(a)  sluggishness

(b)  indolence

(c)  activity

(d)  torpor

Answer: (c)

45. There is a need to promote philanthropy in education.

(a)  charity

(b)  benevolence

(c)  nastiness

(d)  likeliness

Answer: (c)

46. What we lack in the current times is

(a)  empathy

(b)  carefulness

(c)  indifference

(d)  hardship

Answer: (c)

47. Tempestuous behavior would not yield much in any place.

(a)  relaxed

(b)  passionate

(c)  intense

(d)  windy

Answer: (a)

Directions (Q. Nos. 48-52) Each of the following sentences in this section has a blank space and four words or group of words given after the sentence. Select the word or group you consider most appropriate for the blank space.

48. The Election Commission on Saturday …….. that the Assembly elections in five states will be held from 12th November to 7th

(a)  pronounced

(b)  announced

(c)  promulgated

(d)  issued

Answer: (b)

49. The victims were fruit vendors and they were going in an auto when they ……….. an accident on the way.

(a)  met with

(b)  ran into

(c)  experienced

(d)  heard

Answer: (a)

50. Scores of villagers are …….. a sit-in protest against the construction of a new underpass.

(a)  performing

(b)  sitting

(c)  staging

(d)  standing

Answer: (c)

51. It is common for patients to stop ………… medicine as soon as they start feeling better.

(a)  eating

(b)  gulping

(c)  swallowing

(d)  taking

Answer: (d)

52. A four-year-old girl got a new lease of life after doctors at a hospital ………… a cancerous tumour from one of her kidneys.

(a)  rejuvenated

(b)  removed

(c)  displaced

(d)  replaced

Answer: (b)

Directions (Q. Nos. 53-55) Choose the correct plural form of the given words.

53. Loaf

(a)  Loaves

(b)  Loafs

(c)  Loavs

(d)  Lofes

Answer: (a)

54. Spy

(a)  Spis

(b)  Spys

(c)  Spies

(d)  Spiess

Answer: (c)

55. Vertex

(a)  Vertics

(b)  Vertexs

(c)  Verticess

(d)  Vertices

Answer: (d)

Directions (Q. Nos. 56-75) Read the following passages and answer the questions that follow.

PASSAGE 1

Indian nationalism emanated from ‘traditional patriotism’, a socially active sentiment of attachment to land, language and cult, that developed in the subcontinent long before the process of westernization had begun. In India of the 18th and early 19th centuries, such sentiments were emerging on a regional basis as homeland was being defined by various terms like desh, vatan or nadu. But although regionally centred at Bengal, Maharashtra, Awadh or Mysore, their isolation broke down through various means of communication, the political legitimacy of  the Mughal empire was recognized throughout Hindustan, which was thought to be the abode of both the Hindus and the Muslims. Cultural barriers melted down through commercialization and regular pilgrimages. It burst forth through numerous acts of resistance; participated by both princes and the commoners, culminating in the Revolt of 1857. After the revolt, a modern sector of politics gradually evolved in India, through rapid spread of education, development of communication systems, such as the railways and telegraph and the emergence of a new public space created by the colonial institutions.

56. Which of the following was the result of the East India Company’s rule in India?

(a)  Good government

(b)  Consolidation of patriotic sentiments

(c)  New ethical tradition

(d)  Evangelicalism

Answer: (b)

57. Which of the following contributed most to the dissolution of cultural barriers?

(a)  Pilgrimage

(b)  Creation of Mughal Empire

(c)  Commercialization

(d)  Communication networks

Answer: (c)

58. In pre-colonial India, what did the nationalism mean?

(a)  Cultural construction of homeland

(b)  Linguistic affinity

(c)  Patriotism

(d)  Social activism

Answer: (d)

PASSAGE 2

Anxiety and conscience are a pair of powerful dynamos. Of course, I can only speak for myself between them, they have ensured that one shall work at anything worthwhile. They are blind forces which drive but do not direct. Fortunately, I have also been moved by a third motive-the wish to see and understand. Curiosity is another motive for action. It is also one of the distinctive characteristics of human nature (keep) contrasted with the natures of non-human animals.

All human beings have curiosity in some degree and we also have it about things which are of no practical use. Curiosity may be focused on anything in the universe, but the spiritual reality of the phenomenon should be the ultimate objective of all curiosity for it to be fruitful. Thanks to my mother, my approach to this ultimate objective is through the story of human affairs.

59. What is the main objective of this passage?

(a)  Distinguish between human beings and animals

(b)  Project curiosity as a potent motivating factor

(c)  Project anxiety and conscience as inadequate motivators

(d)  Motivation and spiritual reality

Answer: (b)

60. A characteristic, peculiar to human beings that is referred to, is

(a)  superior intelligence

(b)  spirit of enquiry

(c)  capacity to rationalize and analyze

(d)  ability to shift the practical from the impractical

Answer: (b)

61. Which of the following statements is true according to the passage?

(a)  Animals are as curious as human beings

(b)  Curiosity is the only motive for action

(c)  People motivated by curiosity do not need other motives to guide them

(d)  People motivated by anxiety and conscience alone can be misdirected

Answer: (d)

62. According to the author

(a)  those who have little curiosity are curious about unimportant things

(b)  apart from humans no other living being have the gift of curiosity

(c)  the highest form of the curiosity can be satisfied by study of human affairs alone

(d)  spiritual reality is the ultimate goal of humans through action

Answer: (d)

63. The author subscribes to the view that

(a)  one’s curiosity should focus beyond the façade on the latent meaning of things

(b)  curiosity is an inherited family characteristic

(c)  a study of human affairs is the most effective method of satisfying one’s curiosity

(d)  in order to motivate, curiosity must be coupled with anxiety and conscience

Answer: (a)

PASSAGE 3

One simple physical concept lies behind the formation of the stars : gravitational instability. The concept is not new. Newton first perceived it late in the 17th century.

Imagine a uniform, static cloud of gas in space. Imagine then that the gas is somehow disturbed so that one small spherical region becomes a little denser than the gas around it so that the small region’s gravitational field becomes slightly stronger. It now attracts more matter to it and its gravity increases further, causing it to begin to contract. As it contracts, its density increases, which increase its gravity even more, so that it picks up even more matter and contracts even further. The process continues until the small regions of gas finally forms a gravitationally bound object.

64. The primary purpose of the passage is to

(a)  describe a static condition

(b)  support a theory considered outmoded

(c)  depict the successive stages of a phenomenon

(d)  demonstrate the evolution of the meaning of a term

Answer: (c)

65. It can be inferred from this passage that the author views the information contained within it as

(a)  lacking in elaboration

(b)  original but obscure

(c)  speculative and unprofitable

(d)  uncomplicated and traditional

Answer: (d)

66. With which of the following words can you replace the word ‘uniform’ as given in this passage?

(a)  Uniting

(b)  Varying

(c)  Gaseous

(d)  Unvarying

Answer: (d)

67. What does the word ‘It in bold type stand for in the passage?

(a)  Gravitational instability

(b)  Cloud of gas

(c)  Small spherical denser region

(d)  Matter

Answer: (c)

68. The author provides information that answers which of the following questions? Choose the correct option.

(i) What causes the disturbances that changes the cloud from its original static condition?

(ii) How does this small region’s increasing density affect its gravitational field?

(iii) What is the end result of the gradually increasing concentration of the small region of gas?

(a)  Only (i)

(b)  Only (ii)

(c)  (ii) and (iii)

(d)  All of these

Answer: (c)

PASSAGE 4

As heart disease continues to be the number one killer in the United States, researchers have become increasingly interested in identifying the potential risk factors that trigger heart attacks. High fat-diets and life in the fast lane have long been known to contribute to the high incidence of heart failure. But according to new studies, the list of risk factors may be significantly longer and quite surprising. Heart failure, e.g., appears to have seasonal and temporal patterns. A higher percentage of heart attacks occurs in cold weather and more people experience heart failure on Monday than on any other day of the week.

In addition. People are more susceptible to heart attacks in the first few hours after waking. Cardiologists first observed this morning phenomenon in the mid-1980 and have since discovered a number of possible causes. An early morning rise in blood pressure, heart rate and concentration of heart stimulating hormones, plus a reduction of blood flow to the heart, may all contribute to the higher incidence of heart attacks between the hours of 8:00 am and 10:00 am. In other studies, both birthday and bachelorhood have been implicated as risk factors.

Statistics reveal that heart attack rate increases significantly for both females and males in the few days immediately preceding and following their birthdays. And unmarried men are more at risk for heart attacks than their married counterparts. Though, stress is thought to be linked in some way to all of the aforementioned risk factors, intense research continues in the hope of future comprehending why and how heart failure is triggered.

69. What does the passage mainly discuss?

(a)  Cardiology

(b)  Diet and stress as factors in heart attacks

(c)  Seasonal and temporal patterns of heart attacks

(d)  Risk factors in heart attacks

Answer: (c)

70. The word ‘potential’ could best be replaced by which of the following?

(a)  Harmful

(b)  Possible

(c)  Unknown

(d)  Primary

Answer: (b)

71. According to the passage, which of the following is not a possible cause of morning heart attacks?

(a)  Decreased blood flow to the heart

(b)  Increased blood pressure

(c)  Lower heart rate

(d)  Increase in hormones

Answer: (c)

72. Which of the following is cited as possible risk factor?

(a)  Getting married

(b)  Rainy weather

(c)  Eating fatty foods

(d)  Driving fast

Answer: (c)

73. Which of the following does the passage infer?

(a)  We now fully understand how the risk factors trigger heart attacks.

(b)  We do not fully understand how the risk factors trigger heart attacks.

(c)  We have not identified risk factors associated with heart attacks.

(d)  We have recently begun to study how risk factors work.

Answer: (b)

74. What could be the suitable title of the passage?

(a)  Heart disease in the USA

(b)  Causes of Heart Attack

(c)  Stress

(d)  Risk of Heart attack in Humans

Answer: (b)

75. Which word can be the most opposite word of ‘unlikely’?

(a)  Susceptible

(b)  Stimulating

(c)  Implicated

(d)  Aforementioned

Answer: (a)

Directions (Q. Nos. 76-80) Study the following information carefully and answer the given questions.

Eight friends, P, Q, R, S, T, V, W and Y are sitting around a square table in such a way that four of them sit at four corners of the square while four sit in the middle of each of the four sides. The ones who sit at the four corners face the centre while those who sit in the middle of the sides face outside.

P, who faces are centre, sits third to the right of V. T, who faces the centre, is not an immediate neighbor of V. Only one person sits between V and W. S sits second to right of Q. Q faces the centre. R is not an immediate neighbor of P.

76. Who sits second to the left of Q?

(a)  V

(b)  P

(c)  T

(d)  Y

Answer: (b)

77. What is the position of T with respect to V?

(a)  Fourth to the left

(b)  Second to the left

(c)  Third to the left

(d)  Third to the right

Answer: (c)

78. Three of the following four are alike in a certain way and so form a group. Which is the one that does not belong to that group?

(a)  R

(b)  W

(c)  V

(d)  S

Answer: (d)

79. Which of the following will come in place of the question mark(?) based upon the given seating arrangement?

WP TR QW RS ?

(a)  YT

(b)  VY

(c)  VQ

(d)  PY

Answer: (a)

80. Which of the following is true regarding R?

(a)  R is an immediate neighbor of V

(b)  R faces the centre

(c)  R sits exactly between T and S

(d)  Q sits third to left of R

Answer: (c)

81. In the given figure, which letter represents those Actors who are also Dancers, Singers as well as Violinists?

(a)  S

(b)  Q

(c)  P

(d)  U

Answer: (b)

Directions (Q. Nos. 82 and 83) In the following questions, which one of the following diagram represents the correct relationship among :

82. Lion, Fox and Carnivorous

Answer: (c)

83. Manager, Labour Union and Worker

Answer: (a)

84. In a certain code language ‘INSTITUTION’ is written as ‘NOITUTITSNI’. How will ‘PERFECTION’ is written in that language?

(a)  NOICTEFREP

(b)  NOITCEFERP

(c)  NOITCEFRPE

(d)  NOITCEFREP

Answer: (d)

85. In a certain code language ‘RELATED’ is written as ‘EFUBKDQ’, then how will ‘RETAINS’ be written in that language?

(a)  SDQBTOJ

(b)  JOTBQDS

(c)  JOTBSDQ

(d)  TOJBSDQ

Answer: (d)

Directions (Q. Nos. 86-89) In these questions, select the related letters from the given alternatives.

86. KNQT : LORU : : ADGJ : ?

(a)  BEHK

(b)  FHLO

(c)  DGEF

(d)  MPVW

Answer: (a)

87. DRIVEN : NEVIRD : : BEGUM : ?

(a)  MEUBG

(b)  BGMUE

(c)  EBGMU

(d)  MUGEB

Answer: (d)

88. ADIP : DGLS : : BEJQ : ?

(a)  EHMT

(b)  EJQU

(c)  CGLS

(d)  FINU

Answer: (a)

89. ACE : BDF : : CEG : ?

(a)  DEF

(b)  DFH

(c)  DEH

(d)  DFE

Answer: (b)

Direction (Q. No. 90) A series is given with one term missing. Choose the correct alternative from the given ones that will complete the series.

90. 

Answer: (c)

91. Fatima while introducing Mustafa to her husband said, “His brother’s father is the only son of my grandfather”. How is Fatima related to Mustafa?

(a)  Aunt

(b)  Sister

(c)  Niece

(d)  Mother

Answer: (b)

92. A, Q, Y and Z are different persons. Z is the father of Q. A is the daughter of Y and Y is the son of Z. If P is the son of Y and B is the brother of P, then

(a)  B and Y are brothers

(b)  A is the sister of B

(c)  Z is the uncle of B

(d)  Q and Y are brothers

Answer: (b)

93. Radhika went 50 m South from her house, then turned left and went 20 m, then turning to North she went 30 m. In which direction is her home from this point?

(a)  North

(b)  South-West

(c)  East

(d)  North-West

Answer: (d)

94. If South-East becomes North and North-East becomes West and all the rest directions are changed in the same manner, what will be the direction for the West?

(a)  North-East

(b)  South

(c)  South-East

(d)  South-West

Answer: (c)

Directions (Q. Nos. 95-97) In the following questions a statement is given, you have to assume everything to be true, then decide which of the suggested courses of action logically follow(s) for pursuing.

Give Answer

(a) If only course of action I follows

(b) If only course of action II follows

(c) If either course of action I or II follows

(d) If both courses of action I and II follow

95. Statement Many pilgrims died in a stampede while boarding a private ferry to the holy place on the first day of the ten-day-long festival.

Course of action

(I) The Government should immediately cancel the licences of all the private ferry operators with immediate effect.

(II) The Government should deploy an. adequate number of its personnel to guide pilgrims on their journey to the holy place.

Answer: (b)

96. Statement Some students of the local college were caught travelling in the train without purchasing valid tickets.

Courses of action

(I) The parents of these students should be informed about the incident and requested to counsel their wards.

(II) These students should be put behind bars for travelling without bona fide credentials.

Answer: (a)

97. Statement A large part of the locality was flooded as the main pipe supplying drinking water burst while the workers of a utility company were laying cables in the area.

Courses of action

(I) The civic authority should immediately arrange to repair the damage and stop loss of water.

(II) The civic authority should seek an explanation and compensation from the utility company for the damage caused by them.

Answer: (d)

Directions (Q. Nos. 98 and 99) In each of the following questions, two statements are given followed by two Conclusions I and II. You have to consider the two statements to be true even if they seem to be at variance from commonly known facts. You have to decide which of the given conclusions, if any follow from the given statements.

98. Statements

(A) Education is a process of lighting.

(B) Mind requires light to enlighten the core of cognitive aspect.

Conclusions

(I) Education is a light which removes the darkness of mind.

(II) Education is a static process for mind.

(a)  Only Conclusion I follows

(b)  Only Conclusion II follows

(c)  Both Conclusions I and II follow

(d)  Neither Conclusion I nor II follows

Answer: (a)

99. Statements

(A) Best performance in Olympics fetches a gold medal.

(B) Player ‘X’ got god medal but later was found to be using a prohibited drug.

Conclusions

(I) ‘X’ should be allowed to keep the gold medal.

(II) Gold medal should be withdrawn and given to the next person.

(a)  Only Conclusion II follows

(b)  Neither Conclusion I nor II follows

(c)  Both Conclusions I and II follow

(d)  Only Conclusion I follows

Answer: (a)

100. In this question, two statements are given followed by some conclusions. You have to decide which of the given conclusion(s) follow(s) from the given statements.

Statements

(A) There are monks among those who are felicitated for remarkable social service.

(B) Jitananda and Vidyananda are among those felicitated.

Conclusions

(I) Jitananda and Vidyananda did remarkable social service

(II) All monks do social service

(III) Jitananda and Vidyananda are not monks

(IV) All monks are felicitated

(a)  Only conclusion I follows

(b)  Only conclusion II follows

(c)  Only conclusion III follows

(d)  Only conclusion IV follows

Answer: (a)

101. The tagline ‘The Complete Man’ is associated with which of the following fashion brand?

(a)  Siyaram

(b)  Raymond

(c)  Zara

(d)  Allen Solly

Answer: (b)

102. Who is the current CEO of Indian fashion e-commerce company Myntra?

(a)  Amar Nagaram

(b)  Ananth Narayanan

(c)  Vineet Saxena

(d)  Mukesh Bansal

Answer: (a)

103. Which of the following retail stores belong to Aditya Birla Group?

(a)  Big Bazar

(b)  Croma

(c)  Hypercity

(d)  Pantaloons

Answer: (d)

104. RADO, the famous international wrist watch range belongs to

(a)  Switzerland

(b)  Spain

(c)  Japan

(d)  France

Answer: (a)

105. Which Indian film actress is the brand ambassador of online portal ‘Make My Trip’?

(a)  Priyanaka Chopra

(b)  Shraddha Kapoor

(c)  Alia Bhatt

(d)  Deepika Padukone

Answer: (c)

106. Which among the following dance forms belongs to Andhra Pradesh?

(a)  Sattriya

(b)  Bharatnatyam

(c)  Kathak

(d)  Kuchipudi

Answer: (d)

107. Which among the following is an Indian Brand?

(a)  Chanel

(b)  Zara

(c)  Gucci

(d)  Allen Solly

Answer: (d)

108. Who is the first female Chief Economist of International Monetary Fund (IMF)?

(a)  Kristalina Georgieva

(b)  Anshula Kant

(c)  Gita Gopinath

(d)  Rohini Pandey

Answer: (c)

109. Which state is known for its renowned Maheshwari and Chanderi Sarees?

(a)  Karnataka

(b)  Madhya Pradesh

(c)  Tamil Nadu

(d)  Rajasthan

Answer: (b)

110. Which of these cities is a World Heritage city by declared by UNESCO?

(a)  Ahmedabad

(b)  Delhi

(c)  Chennai

(d)  Hyderabad

Answer: (a)

111. In which state is the Manas National Park located?

(a)  Assam

(b)  Rajasthan

(c)  Meghalaya

(d)  Manipur

Answer: (a)

112. Baggit is a well known brand which sells

(a)  bags

(b)  belts

(c)  wallets

(d)  All of these

Answer: (d)

113. Who became the first ever Indian to win a gold at Badminton World Championship?

(a)  PV Sindhu

(b)  Saina Nehwal

(c)  P Kashyap

(d)  Jwala Gutta

Answer: (a)

114. Where is Louis Vuitton headquarters located?

(a)  London

(b)  Milan

(c)  Paris

(d)  New York

Answer: (c)

115. Which is the world’s biggest retailer brand?

(a)  Target

(b)  Walmart  

(c)  JC Penny

(d)  Amazon

Answer: (b)

116. Which of the following desert is located in China?

(a)  Thar desert

(b)  Atacama desert

(c)  Kalahari desert

(d)  Gobi desert

Answer: (d)

117. In which year was the Lakme Fashion Week first started?

(a)  2006

(b)  2008

(c)  2000

(d)  1999

Answer: (d)

118. Which Constitutional Amendment Act gave Constitutional Status to Panchayati Raj Institutions?

(a)  73rd

(b)  74th

(c)  93rd

(d)  94th

Answer: (a)

119. Which of the following is not a member of the Shanghai Cooperation Organization?

(a)  India

(b)  China

(c)  Japan

(d)  Russia

Answer: (c)

120. Retail venture ‘Big Bazar’ is owned by

(a)  Future Group

(b)  Bharti Group

(c)  Sriram Group

(d)  Rahja Group

Answer: (a)

121. Which of these river is a tributary of Ganga?

(a)  Son

(b)  Narmada

(c)  Godavari

(d)  Mahi

Answer: (a)

122. Which Indian American economist was awarded the Nobel Prize of Economics in 2019?

(a)  Kaushik Basu

(b)  Raghuram Rajan

(c)  Abhijeet Banerjee

(d)  Urjit Patel

Answer: (c)

123. Who among the following is not an Indian fashion designer?

(a)  Manish Malhotra

(b)  Rohit Bal

(c)  Ritu Kumar

(d)  Bibi Russell

Answer: (d)

124. Which city is known as the ‘Manchester’ of South India?

(a)  Cochin

(b)  Coimbatore

(c)  Calicut

(d)  Chennai

Answer: (b)

125. Little black dress was introduced by

(a)  Zara

(b)  Gucci

(c)  Prada

(d)  Coco Chanel

Answer: (d)

Directions (Q. Nos. 126-139) Read each case very carefully and answer the questions that follow.

CASE 1

The Chinese krait and the Chinese cobra might be the original source of the newly discovered coronavirus that has triggered an outbreak of a deadly infectious respiratory illness in China in winter 2019. The many-banded krait (Bungarus multicinctus), also known as the Taiwanese krait or the Chinese krait, is a highly venomous species of elapid snake found in much of Central and Southern China and South-East Asia.

The illness was first reported in late December 2019 in Wuhan, a major city in central China and has been rapidly spreading. Since then, sick travellers from Wuhan have infected people in China and other countries, including the United States. India, Japan, Australia, Singapore, Hong Kong.

Using samples of the virus isolated from patients, scientists in China have determined the genetic code of the virus and used microscopes to photograph it. The pathogen responsible for this pandemic is a new coronavirus. It’s in the same family of viruses as the well-known Severe Acute Respiratory Syndrome Coronavirus (SARS-CoV) and Middle East Respiratory Syndrome Coronavirus (MERS-CoV), which have killed hundreds of people in the past 17 years. The World Health Organization (WHO) has named the new coronavirus 2019-nCoV.

The name of coronavirus comes from its shape, which resembles a crown or solar corona when imaged using an electron microscope. The electron microscopic image reveals the crown shape structural details for which the coronavirus was named. This image is of the Middle East Respiratory Syndrome Coronavirus (MERS-CoV).

Coronavirus is transmitted through the air and primarily infects the upper respiratory and gastrointestinal tract of mammals and birds. Though most of the members of the coronavirus family only cause mild flu-like symptoms during infection, SARS-CoV and MERS-CoV can infect both upper and lower airways and cause severe respiratory illness and other complications in humans. This new 2019-nCoV causes similar symptoms to SARS-CoV and MERS-CoV. People infected with these coronaviruses suffer a severe inflammatory response. Unfortunately, there is no approved vaccine or antiviral treatment available for coronavirus infection. A better understanding of the life cycle of 2019-nCoV, including the source of the virus, how it is transmitted and how it replicates are needed to both prevent and treat the disease.

126. Taiwanese Krait or the Chinese Krait, the original source of the newly discovered coronavirus, is a highly venomous species of elapid snake found in much of

(a)  Central China

(b)  Southern China

(c)  South-East Asia

(d)  All of the above

Answer: (d)

127. Coronavirus affected sick travellers from Wuhan have infected people in China and other countries, including

(a)  Tanzania

(b)  Australia

(c)  Uganda

(d)  Congo

Answer: (b)

128. The Pathogen responsible for the pandemic of China is a new coronavirus. The WHO has named the new coronavirus 2019 as

(a)  SARS-CoV

(b)  MERS-CoV

(c)  nCoV

(d)  W-CoV

Answer: (c)

129. The electron microscopic image of the virus depicts which shape and structure of coronavirus?

(a)  Helical shape

(b)  Icosahedral shape

(c)  Prolate shape

(d)  Crown shape

Answer: (d)

130. Severe Acute Respiratory Syndrome Coronavirus and Middle East Respiratory Syndrome Coronavirus can infect

(a)  upper airways

(b)  lower airways

(c)  Both (a) and (b)

(d)  None of these

Answer: (c)

CASE 2

The Sveriges Riksbank Prize in Economic Sciences in Memory of Alfred Nobel 2019 (commonly known as the Nobel Prize for economics) has been awarded to Abhijeet Banerjee, Esther Duflo and Michael Kremer “for their experimental approach to alleviating global poverty.”

Through the award, the Nobel committee recognized both the significance of development economics in the world today and the innovative approaches developed by these three economists.

Esther Duflo is only the second woman to be awarded the prize (after Elinor Ostrom in 2009). Banerjee, who is also her husband, is the third ever non-white recipient (after Arthur Lewis in 1979 and Amartya Sen in 1998).

In a recent issue of the journal ‘Nature’, Goran Hansson, head of the Royal Swedish Academy of Sciences that awards the Nobel, highlighted measures to address the imbalance in gender and ethnicity among winners.

He said “we are making sure to elect women to the academy” from which the prize-awarding committees for the chemistry, physics and economics Nobels are drawn. The first woman to win the John Bates Clark Medal for top economists under 40, an important indicator of who will be awarded the economics Nobel in the future, Susan Athey, only did so in 2007.

Esther Duflo was the second winner in 2010. Since then, women winners of the Clark medal have been more frequent.

Of course, award decisions are made strictly on significance of contributions. But, based on this evidence, perhaps Athey, Amy Finkelstein (who won the medal in 2012) and Emi Nakamura (who won it in 2019) will not be far behind.

131. The Sveriges Riksbank Prize in Economic Sciences has been awarded to

(a)  Abhijeet Banerjee

(b)  Esther Duflo

(c)  Michael Kremer

(d)  All of these

Answer: (d)

132. Esther Duflo is also only the second woman to be awarded the Nobel Prize in Economics after

(a)  Goran Hansson

(b)  Elinor Ostrom

(c)  Susan Athey

(d)  Emi Nakamura

Answer: (b)

133. Abhijeet Banerjee is the third ever non-white recipient of Nobel Prize for Economics after

(a)  Arthur Lewis and Amartya Sen

(b)  Goran Hansson and Amartya Sen

(c)  Elinor Ostrom and Amartya Sen

(d)  Susan Athey and Emi Nakamura

Answer: (a)

134. The first woman to win the John Bates Clark Medal for top economists under 40 is

(a)  Susan Athey

(b)  Goarn Hasson

(c)  Elinor Ostrom

(d)  None of these

Answer: (a)

CASE 3

In the 16th century, an age of great marine and terrestrial exploration, Ferdinand Magellan led the first expedition to sail around the world. As a young Portuguese noble, he served the king of Portugal, but he became involved in the quagmire of political intrigue at court and lost the king’s favor.

After he was dismissed from service by the king of Portugal, he offered to serve the future Emperor Charles V of Spain.

A papal decree of 1493 had assigned all land in the New World, West of 50 degrees W longitude to Spain and all the land East of that line to Portugal. Magellan offered to prove that the East Indies fell under Spanish authority. On 20th September, 1519, Magllan set sail from Spain with five ships.

More than a year later, one of these ships was exploring the topography of South America in search of a water route across the continent. This ship sank, but the remaining four ships searched along the Southern peninsula of South America.

Finally they found the passage they sought near 50 degrees S latitude. Magellan named this passage the ‘Strait of All Saints’, but today it is known as the “Strait of Magellan”.

One ship deserted while in this passage and returned to Spain, so fewer sailors were privileged to gaze at that first panorama of the Pacific Ocean. Those who remained, crossed the meridian, now known as the International Date Line in the early spring of 1521 after 98 days in the Pacific Ocean. During those long days at sea, many of Magellan’s men died of starvation and disease.

Later, Magellan became involved in an insular conflict in the Philippines and was killed in a tribal battle. Only one ship and 17 sailors under the command of the Basque navigator Elcano survived to complete the westward journey to Spain and thus prove once and for all that the world is round, with no precipice at the edge.

135. Which of the following century is known as ‘an age of great marine and terrestrial exploration?

(a)  17th century

(b)  18th century

(c)  16th century

(d)  15th century

Answer: (c)

136. After being dismissed from service of Portuguese king, Magellan served under which king?

(a)  Charles V of Britain

(b)  Charles V of Spain

(c)  George IV of Britain

(d)  Czar of Russia

Answer: (b)

137. The passage which was discovered by Magellan is known by which name today?

(a)  Strait of Hormuz

(b)  Strait of Gibraltar

(c)  Strait of Magellan

(d)  Strait of Charles

Answer: (c)

138. In which year, Spanish Ships crossed International Date Line

(a)  1525

(b)  1500

(c)  1498

(d)  1521

Answer: (d)

139. Under whose command, the only surviving ship returned to Spain and proved that world is round?

(a)  Ferdinand Magellan

(b)  Elcano

(c)  Christopher Columbus

(d)  John Cabot

Answer: (b)

Directions (Q. Nos. 140-150) In each of these questions a passage is followed by several inferences. You have to examine each inferences separately in the context of the passage and decide upon its degree of truth or falsity.

(a) If the inference is ‘definitely true’ i.e. it directly follows from the facts given in the passage.

(b) If the inference is ‘probably true’ i.e. through not definitely true in the light of facts given.

(c) If you think the inference is ‘probably false’ though not definitely false in the light of facts given.

(d) If you think the inference is ‘definitely’ false’ i.e. it contradicts the given facts.

CASELET 1

Sanitation is one of the biggest problems in India. There are about 700 million people who have no access to toilets at home. Slum areas do not have toilets. People are thus forced to defecate in the open, which causes numerous diseases like diarrhoea, cholera, dehydration etc. Many rural schools also have no toilets, because of which parents do not send their kids, especially girls, to school.

Due attention was drawn towards this problem by Gandhiji but nothing much was done. A growing population is the biggest challenge causing these problems.

For example, the sewage system in Delhi was designed to meet the needs of a population of three million people. But Delhi now has more than 14 million of population. This is not just the case of Delhi; every state and region in India is the same. Though 12 million toilets claim to have been built under Swachh Bharat Abhiyan in the last five years, as per a UN report, 44% of the population continues to defecate in the open. Sanitation, solid waste management and drainage continue to pose challenges.

A ranking exercise was taken up by the Government of India called ‘Swachh Survekshan” to assess rural and urban areas for their levels of cleanliness and active implementation of Swachhata mission initiatives in a timely and innovative manner.

The objective of the survey is to encourage large scale citizen participation and create awareness amongst all sections of society about the importance of working together towards making towns and cities a better place to live in. Additionally, the survey also intends to foster a spirit of healthy competition among towns and cities to  improve their service delivery to citizens, towards creating cleaner cities and towns.

140. A growing population is the biggest challenge causing problems related to sanitation.

Answer: (a)

141. There is a lack of good hygiene habits and inadequacy of social support in society that made sanitation a major problem in the country.

Answer: (b)

142. Objective of ‘Swachh Survekshan Mission’ is to encourage people to go for cleanliness activities individually and neglect group activities

Answer: (d)

143. ‘Swachh Survekshan’ exercise to assess rural and urban areas for their levels of cleanliness has increased the healthy competition among states.

Answer: (a)

144. Most of the cities are doing good in terms of sanitation except few such as Delhi and Mumbai.

Answer: (c)

CASELET 2

Availability of clean cooking fuel is a major challenge for rural households in India. This deprivation is predominantly suffered by women and children as they are the most exposed to harmful effects of smoke from burning unclean fuels in the households.

In order to safeguard the health of women and children by providing them with a clean cooking fuel-LPG, the Government of India (GoI) launched (1st May, 2016) Pradhan Mantri Ujjwala Yojana (PMUY). The scheme aims to provide five crore deposit-free LPG connections to women belonging to Below Poverty Line (BPL) households appearing in Socio-Economic and Caste Census (SECC2011) and suffering with at least one deprivation.

In SECC-2011 Census survey, Ministry of Rural Development (MoRD) enumerated 24.49 crore (17.97 crore rural and 6.52 crore urban) households in the country. Out of these, 10.31 crore households {8.72 crore rural (48.53 per cent) and 1.59 crore urban (24.39 per cent)} suffered with at least one deprivation, which were identified by the GoI for release of five crore LPG connections under PMUY.

PMUY provides that an amount of Rs. 1600 per LPG connection towards security deposit for LPG Cylinder, Pressure Regulator and Installation Charges etc. would be borne by the Government as one-time financial assistance to adult women of BPL households included in the SECC-2011 and who are not already having any LPG connection in their household. It also prescribed that the Oil Marketing Companies (OMCs) viz. Indian Oil Corporation Limited (IOCL), Bharat Petroleum Corporation Limited (BPCL) and Hindustan Petroleum Corporation Limited (HPCL) would provide an option to PMUY beneficiaries to opt for loan, if they so desire, to cover the cost of a cooking stove and first refill. EMI of loan amount would be recovered by the OMCs from the subsidy amount due to the beneficiaries on refills.

145. It was women and children who are mostly exposed due to burning of unclean fuel in household.

Answer: (a)

146. Giving gas cylinder in the name of women will strengthen their economic status in the family.

Answer: (b)

147. Though cylinder will be provided free of cost, the charges of pressure regulator and installation charges will be borne by beneficiary.

Answer: (d)

148. Putting cost of cooking stove and refill on beneficiary will become hindrace for BPLs to go for refilling of cylinders.

Answer: (b)

149. The major challenge for rural households in India is availability of cooking fuel.

Answer: (a)

150. PMUY also prescribed OMCs, IOCL, BPCL and HPCL for beneficiaries to opt for loan if they desire to cover the cost of a cooking stove and first refill.

Answer: (a)

National Institute of Fashion & Technology (NIFT) UNDER GRADUATE 2020 Question Paper With Answer Key

NIFT (Under Graduate)

National Institute of Fashion and Technology

Solver Paper 2020

1. A company employed 200 workers to complete a certain work in 150 days. If only 1/4th of the work has been done in 50 days, then in order to complete the whole work in time, the number of additional workers to be employed was

(a)  300

(b)  200

(c)  100

(d)  600

Answer: (c)

2. A boat moves downstream at the rate of 1 km in  and upstream at the rate of 5 km in an hour. What is the speed of the boat in the still water?

(a) 

(b)  8 km/h

(c) 

(d)  4 km/h

Answer: (c)

3. The average weight of first 11 persons among 12 persons is 95 kg. The weight of 12th person is 33 kg more than the average weight of all the 12 persons. The weight of the 12th person is

(a)  128 kg

(b)  97.45 kg

(c)  128.75 kg

(d)  131 kg

Answer: (d)

4. If a man earns Rs. 2000 for his first 50 h of work in a week and is then paid one and one half times his regular hourly rate for any additional hours, then the hours must he work to make Rs. 2300 in a week is

(a)  4 h

(b)  5 h

(c)  6 h

(d)  7 h

Answer: (b)

5. Pipe A can fill an empty tank in 6 h and pipe B in 8 h. If both the pipes are opened and after 2 h pipe A is closed, then how much time pipe B will take to fill the remaining tank?

Answer: (b)

6. A builder purchases 25 windows at 25% off the total price of Rs. 120000. If the builder receives an additional discount of Rs. 7500 for the purchase, then the cost of each window is

(a)  Rs. 3100

(b)  Rs. 3200

(c)  Rs. 3300

(d)  Rs. 3400

Answer: (c)

7. x does 1/4 of a job in 6 days, y completes rest of the job in 12 days. Then, x and y could complete the job together in

Answer: (a)

8. If 90 men can do a certain job in 16 days, working 12 h per day, then that part of that work which can be complete by 70 men in 24 days, working 8 h per day is

(a)  2/3

(b)  5/8

(c)  7/9

(d)  1/3

Answer: (c)

9. A profit of 12% is made when a mobile phone is sold at Rs. P and there is 4% loss when the phone is sold at Rs. Q. Then Q : P is

(a)  4 : 5

(b)  3 : 1

(c)  1 : 1

(d)  6 : 7

Answer: (d)

10. A man purchases some oranges at the rate of 3 for Rs. 40 and the same quantity at 5 for Rs. 60. If he sells all the oranges at the rate of 3 for Rs. 50, then find his gain or loss per cent (to the nearest integer).

(a)  31% loss

(b)  32% profit

(c)  34% loss

(d)  31% profit

Answer: (b)

11. Pooja wants to sell a watch at a profit of 20%. She bought it at 10% less and sold it at Rs. 30 less, but still she gained 20%. The cost price of watch is

(a)  Rs. 220

(b)  Rs. 225

(c)  Rs. 240

(d)  Rs. 250

Answer: (d)

12. The portion of a ditch 48 m long, 16.5 m wide and 4 m deep that can be filled with stones and earth available during excavation of a tunnel, cylindrical in shape, of diameter 4 m and length 56 m is (take π = 22/7)

(a)  1/9 part

(b)  1/4 part

(c)  2/9 part

(d)  1/2 part

Answer: (c)

13. A school group charters three identical buses and occupies 4/5 of the seats. After 1/4 of the passengers leave, the remaining passengers use only two of the buses. The fraction of the seats on the two buses that are now occupied is

(a)  7/10

(b)  9/10

(c)  8/9

(d)  7/9

Answer: (b)

14. Let x be the smallest number, which when added to 2000 makes the resulting number divisible by 12, 16, 18 and 21. The sum of the digits of x is

(a)  7

(b)  6

(c)  5

(d)  4

Answer: (a)

15. While solving a problem, by mistake, Anita squared a number and then subtracted 25 from it rather than first subtracting 25 from the number and then squaring it. But the got the right answer. What was the given number?

(a)  13

(b)  38

(c)  48

(d)  Cannot be determined

Answer: (a)

16. P and Q together can do a job in 6 days Q and R can finish the same job in 60/7 days. P started the work and worked for 3 days. Q and R continued for 6 days. Then, the difference of days in which R and P can complete the job, is

(a)  15

(b)  10

(c)  8

(d)  12

Answer: (b)

17. Two places P and Q are 162 km apart. A train leaves P for Q and simultaneously another train leaves Q for P. They meet at the end of 6 h. If the former train travels 8 km/h faster than the other, then speed of train from Q is

Answer: (b)

18. In an examination average marks obtained by the girls of a class is 85 and the average marks obtained by the of the same class is 87. If the girls and boys are in the ratio 4 : 5, then average marks of the whole class (approx.) is closest to

(a)  86.5

(b)  86.1

(c)  85.9

(d)  86.4

Answer: (b)

19. The price of an antique is reduced by 20% and then this price again reduced by 10%. The total reduction of the price is

(a)  30%

(b)  28%

(c)  25%

(d)  23%

Answer: (b)

20. The marked price of a tape recorder is Rs. 12600. A festival discount of 5% is allowed on it. Futher for cash payment, a second discount of 2% is given. The cash payment, is to be made for buying it, is

(a)  Rs. 11730.60

(b)  Rs. 11703.60

(c)  Rs. 11073.60

(d)  Rs. 11370.60

Answer: (a)

21. A shopkeeper sold his goods at half the list price and thus lost 20%. If he had sold on the listed price, his gain percentage would be

(a)  20%

(b)  35%

(c)  60%

(d)  72%

Answer: (c)

22. In a school there were 1554 students and the ratio of the number of the boys and girls was 4 : 3. After few days, 30 girls joined the school but few boys left, as a result the ratio of the boys and girls became 7 : 6. The number of boys who left the school is

(a)  84

(b)  76

(c)  74

(d)  86

Answer: (b)

23. The number, which can be written in the form of n(n + 1) (n + 2), where n is a natural number, is

(a)  3

(b)  6

(c)  7

(d)  5

Answer: (b)

24. If x can finish a job in 4 h and y can finish the same job in 8 h independently, then they together will finish the job in

(a)  160 min

(b)  150 min

(c)  140 min

(d)  120 min

Answer: (a)

25. In a partnership business, B’s capital was half of A’s. If after 8 months, B withdrew half of his capital and after 2 months more A withdrew one-fourth of his capital, then the profit ratio of A and B will be

(a)  10 : 23

(b)  23 : 10

(c)  5 : 2

(d)  2 : 5

Answer: (b)

26. A car covers four successive 7 km distances at speeds of 10 km/h, 20 km/h, 30 km/h, 60 km/h, respectively. Its average speed over this distance is

(a)  40 km/h

(b)  20 km/h

(c)  30 km/h

(d)  60 km/h

Answer: (b)

27. If a hemisphere is melted and four spheres of equal volume are made, then the radius of each sphere will be equal to

(a)  1/4th of the radius of the hemisphere

(b)  radius of the hemisphere

(c)  1/2 of the radius of the hemisphere

(d)  1/6th of the radius of the hemisphere

Answer: (c)

28. The average of six numbers is 3.95. The average of two of them is 3.4, while the average of the other two is 3.85. The average of the remaining two numbers is

(a)  4.5

(b)  4.7

(c)  4.6

(d)  4.8

Answer: (c)

29. If (x3 – y3) : (x2 + xy + y2) = 5 : 1 and (x2 – y2) : (x – y) = 7 : 1, then the ratio 2x : 3y equals

(a)  3 : 2

(b)  4 : 1

(c)  2 : 3

(d)  4 : 3

Answer: (b)

30. A manufacturer fixes his selling price at 33% over the cost of production. If cost of production goes up by 12% and manufacturer raises his selling price by 10%, his percentage profit is

Answer: (a)

Directions (Q. Nos. 31-33) In each of the following questions, an idiomatic expression/proverb has been given (underlined) followed by some alternatives. Choose the one that best expresses the meaning of the given idiom/proverb.

31. There is no cut and dried formula for success in life.

(a)  guaranteed

(b)  readymade form

(c)  compulsory

(d)  undefined

Answer: (b)

32. Meena is not highly qualified but she has the gift of the gab.

(a)  loves to gossip

(b)  speaks irrationally

(c)  art of speaking

(d)  constantly talk out of context

Answer: (c)

33. Rahul proved to be a fair-weather friend to Abhishek as he refused to support him in hard times.

(a)  a selfish friend

(b)  a faithful friend

(c)  a man of good heart

(d)  a man of wisdom

Answer: (a)

Directions (Q. Nos. 34 and 35) In each of the questions below, only one among the given alternatives is correctly spelt. Find out the word with correct spelling.

34.

(a)  Enterprenuer

(b)  Enterepraneur

(c)  Entrapreneur

(d)  Entrepreneur

Answer: (d)

35.

(a)  Itinarary

(b)  Itenerary

(c)  Itinerary

(d)  Itinarery

Answer: (c)

Directions (Q. Nos. 36 and 37) In each of the following questions, choose the wrongly spelt word.

36.

(a)  Conceit

(b)  Deceit

(c)  Receive

(d)  Percieve

Answer: (d)

37.

(a)  Coupious

(b)  Cautious

(c)  Captious 

(d)  Capricious

Answer: (a)

Directions (Q. Nos. 38 and 39) From amongst the options given below each word, choose the appropriate singular form of the given word.

38. Phenomena

(a)  Phenomen

(b)  Phenomenal

(c)  Phenomenon

(d)  Phenomenas

Answer: (c)

39. Strata

(a)  Stratum

(b)  Stratas

(c)  Stratal

(d)  Strati

Answer: (a)

Directions (Q. Nos. 40 and 41) From among the options given below each word, choose the appropriate plural form of the given word.

40. Medium

(a)  Mediums

(b)  Media

(c)  Medias

(d)  Mediumies

Answer: (b)

41. Crisis

(a)  Crisisess

(b)  Crisises

(c)  Crisies

(d)  Crises

Answer: (d)

Directions (Q. Nos. 42-45) in each of these question, choose the alternative which can replace the underlined word without changing the meaning of the sentence.

42. His boss’ criticism left him feeling rather

(a)  annoyed

(b)  arrogant

(c)  embarrassed

(d)  awakened

Answer: (c)

43. He wrote a scathing review of the prize winning novel.

(a)  biased

(b)  scornful

(c)  unbalanced

(d)  subjective

Answer: (b)

44. Even the most careful researcher cannot predict the possible future ramifications of his findings.

(a)  consequences

(b)  developments

(c)  uses

(d)  conclusions

Answer: (a)

45. After he came back from his evening walk, he felt famished.

(a)  exhausted

(b)  peevish

(c)  relaxed

(d)  hungry

Answer: (d)

Directions (Q. Nos. 46-49) Fill in the blanks.

46. I swam …….. the lake.

(a)  in

(b)  into

(c)  at

(d)  on

Answer: (a)

47. He lives ……. 48, MG Road.

(a)  in

(b)  at

(c)  on

(d)  for

Answer: (b)

48. Nobody …….. that he is innocent.

(a)  is believing

(b)  believes

(c)  believe

(d)  believed

Answer: (b)

49. Mother ……… busy cooking for the guests who are arriving in the evening.

(a)  was

(b)  has

(c)  had been

(d)  has been

Answer: (d)

Directions (Q. Nos. 50-53) In each of these questions, choose the alternative which is closest to the opposite in meaning of the underlined word.

50. He has a passion for indigenous

(a)  native

(b)  foreign

(c)  silly

(d)  cheap

Answer: (b)

51. The leader was pragmatic in his approach to the problems faced by the country.

(a)  indefinite

(b)  vague

(c)  idealistic

(d)  optimistic

Answer: (c)

52. That was dauntless

(a)  cowardly

(b)  secret

(c)  subtle

(d)  devious

Answer: (a)

53. It is obligatory for a common citizen to follow the rules.

(a)  advisable

(b)  unnecessary

(c)  superfluous

(d)  optional

Answer: (d)

Directions (Q. Nos. 54 and 55) In each of the following questions, choose the alternative that can be substituted for the given phrase/sentence.

54. Giving undue favours to one’s own kith and kin.

(a)  Favouritism

(b)  Nepotism

(c)  Corruption

(d)  Worldliness

Answer: (b)

55. That cannot be corrected

(a)  Invulnerable

(b)  Hardened

(c)  Incurable

(d)  Incorrigible

Answer: (d)

Directions (Q. Nos. 56-75) Read the following passages and answer the questions that follow.

PASSAGE 1

“The history of science is the real history of mankind.” In this striking epigram, a nineteenth century writer links science with its background. Like most epigrams, its power lies in emphasizing by contrast an aspect of truth which may be easily overlooked. In this case, it is easy to overlook the relations between science and mankind and to treat the former as some abstract third party, which can sometimes be praised for its beneficial influences, but frequently and conveniently blamed for the horrors of war.

Science and mankind cannot be divorced from time to time at man’s convenience. Yet, we have seen that, in spite of countless opportunities of improvement, the opening  years of the present period of civilization have been dominated by International conflict. Is this the inevitable result of the progress of science or does the fault lie somewhere else?

56. The sentence “The history of science is the real history of mankind” means

(a)  science has given man countless opportunities for improvement.

(b)  science and mankind cannot always be divorced.

(c)  mankind has progressed as science has developed.

(d)  the good and bad uses of science reflect the character of man.

Answer: (d)

57. The epigram given in the passage highlights

(a)  the evolution of science

(b)  the real history of man

(c)  the contrast between science and civilization

(d)  an elusive truth about human nature

Answer: (d)

58. The aspect of truth likely to be overlooked is that science

(a)  has made war horrible

(b)  is beneficial to man

(c)  is what man has made it

(d)  has created international conflicts

Answer: (c)

59. The writer implies that international conflict is the result of

(a)  faulty relations between nations

(b)  human weaknesses

(c)  invention of deadly weapons

(d)  progress of science

Answer: (b)

60. The last sentence of the passage suggests that

(a)  civilization could prosper well without scientific inventions.

(b)  the trouble lies with human beings themselves.

(c)  people have missed opportunities to improve themselves.

(d)  the horrors of modern life are the inevitable result of the progress of science.    

Answer: (b)

PASSAGE 2

There is more than a modicum of truth in the assertion that “a working knowledge of ancient history is necessary to the intelligent interpretation of current events”. But the sage who uttered these words of wisdom might well have added something on the benefits of studying, particularly, the famous battles of history for the lessons they contain for those of us who lead or aspire for leadership.

Such a study will reveal certain qualities and attributes which enabled the winners to win, and certain deficiencies which caused the losers to lose. And the student will see that the same pattern recur consistently, again and again, throughout the centuries.

61. The expression ‘more than a modicum of truth’ means

(a)  nothing but truth

(b)  some truth

(c)  much truth

(d)  more than a small amount of truth

Answer: (d)

62. In this context, ‘intelligent interpretation of current events’ means

(a)  skillful interpretation of events

(b)  intellectual outlook on events

(c)  appropriate understanding of events

(d)  rational explanation of events

Answer: (d)

63. According to the writer, a study of the famous battles of history would

(a)  provide food to modern leaders for reflection.

(b)  be beneficial to wise man.

(c)  help us understand the art of human warfare.

(d)  be more useful than a general knowledge of ancient history.

Answer: (a)

64. A person who aspires to lead could learn from the history of battles

(a)  the qualities and deficiencies of commanders of these battles.

(b)  what led the previous leaders win battle.

(c)  what made them lose a battle.

(d)  the strategies they evolved in course of these battles.

Answer: (a)

65. A knowledge of history is necessary to interpret current problems because

(a)  they may be repetitions of past events.

(b)  only then they can be put in a proper context.

(c)  they have roots in the past.

(d)  they can be contrasted with the past events.

Answer: (a)

PASSAGE 3

No one knows when or by whom rockets were invented. In all probability the rocket was not suddenly invented but evolved gradually over a long period of time. Perhaps in different parts of the world at the same time. Some historians of rocketry; notably Willy Ley, trace the development of rocket to the 13th century China, a land noted in ancient times for its firework display.

In the year AD 1232, when the Mongols laid siege to the city of Kai-Feng Fu, the capital of Homan province, the Chinese defenders used weapons that were described as ‘arrows of flying fire’.

There is no explicit statement that these arrows were rockets, but some students have concluded that they were because the record does not mention bows or other means of shooting the arrows.

In the same battle, we read the defenders dropped from the walls of the city a kind of bomb described as ‘heaven-shaking thunder’. From these meager references, some students have concluded that the Chinese, by the year 1232, had discovered gun powder and had learned to use it to make explosive bombs as well as propulsive charge for rockets.

66. The passage gives, primarily, a history of

(a)  the bravery of the Chinese

(b)  the invention of rockets

(c)  the attack on China by the Mongols

(d)  the battle against the Chinese wall

Answer: (b)

67. According to this passage, rockets were invented by

(a)  Willy Ley

(b)  unknown people

(c)  the Mongols

(d)  the ruler of Honan province

Answer: (b)

68. According to this passage, rockets were

(a)  a gift of God to the Chinese

(b)  invented in the twentieth century

(c)  invented in AD 1232

(d)  developed over many centuries

Answer: (d)

69. The phrase ‘arrows of flying fire’

(a)  means some ancient phenomenon in the skies

(b)  refers to lightning and thunder

(c)  is another name for rockets

(d)  is assumed to refer to rockets

Answer: (d)

70. The bombs have been referred to as ‘heaven shaking thunder’ because they

(a)  contain gunpowder

(b)  make thunderous noise

(c)  are propelled by rockets

(d)  seem to fall from heaven

Answer: (b)

PASSAGE 4

When we pick up a newspaper, a book or an article we come to our task with certain preconceptions and predispositions. We expect to find a specific piece of information or be presented with an argument or an analysis of something, say, the likelihood of recession in the next six months or the reasons why children can’t read. We probably know a little about the book or article we are reading even before we start. There was, after all, some reason why we chose to read one piece of writing rather than another.

Our expectations and predispositions may, however, blind us to what the article and its author is actually saying. If, for example, we are used to disagreeing with the author, we may see only what we expect to see and not what is actually there. Day after day in our routine pattern of life we expose ourselves to same newspaper, the same magazine, even books by authors with the same perspectives. In order to reflect on our reading habits and improve our skills we need to break out of this routine, step back and look at what we are doing when we read.

71. According to the author, which one of the following statements is not true?

(a)  Reader’s preconceptions influence their reading.

(b)  Readers have expectations when they read an article or a book.

(c)  Readers look for specific information in any of their readings.

(d)  Readers assume that everything they read will have new information.

Answer: (d)

72. Our expectations and predispositions may, however, blind us because

(a)  we may not get the actual ideas of the author.

(b)  we will get the actual ideas of the author.

(c)  we may disagree with the author.

(d)  we will agree with all the ideas of the author.

Answer: (a)

73. One of the ways to improve our reading habits is to

(a)  break the routine by changing the time of reading.

(b)  change the types of topics we read.

(c)  break the routine of reading the same newspaper.

(d)  stop reading for some time and then restart reading.

Answer: (d)

74. Which quality does the author here advocate, to be a good reader?

(a)  Being objective to the ideas of the author.

(b)  Having preconceptions and predispositions.

(c)  Having continuous routines.

(d)  Disagreeing with the author.

Answer: (a)

75. Which word the passage means ‘viewpoints’?

(a)  Preconceptions

(b)  Predispositions

(c)  Pattern

(d)  Perspectives

Answer: (d)

Directions (Q. Nos. 76-80) For the Assertion (A) and Reason (R) below, choose the correct alternative from the following.

Give Answer

(a) If (A) is true but (R) is false

(b) If both (A) and (R) are true but (R) is not the correct explanation of (A)

(c) If both (A) and (R) are true and (R) is the correct explanation of (A)

(d) If (A) is false but (R) true

76. Assertion (A) India celebrates its Independence day on 15th

Reason (R) India became independent on 15th August, 1947.

Answer: (c)

77. Assertion (A) Silver is not used to make wires.

Reason (R) Silver is a bad conductor.

Answer: (a)

78. Assertion (A) In India, people elect their own representatives.

Reason (R) India is a democratic country .

Answer: (c)

79. Assertion (A) A body weighs less when immersed in water.

Reason (R) Newton’s law explains the above phenomenon.

Answer: (a)

80. Assertion (A) The steam engine was invented by James Watt.

Reason (R) There was a problem of taking out water from flooded mines.

Answer: (c)

81. In a certain code language ‘KAMLESH’ is written as ‘GUJLMCO’, then how will ‘NATURAL’ be written in the language?

(a)  TCNUPCV

(b)  TCOUPVC

(c)  TCUOPVC

(d)  TCOUVCP

Answer: (a)

82. The time on the watch is 9 : 15 and the hour hand points towards West. The direction of the minute hand is

(a)  North

(b)  South

(c)  East

(d)  West

Answer: (c)

Directions (Q. Nos. 83-85) In the following questions, select the related letters from the given alternatives.

83. ? : ALKLO :: WOULD : TLRIA

(a)  BLOCK

(b)  DONOR

(c)  CONES

(d)  BARGE

Answer: (b)

84. BEAK : ORNX :: FILM : ?

(a)  RUXY

(b)  MLIF

(c)  SVYZ

(d)  URON

Answer: (c)

85. AB : ZY :: CD : ?

(a)  WX

(b)  UV

(c)  XW

(d)  VU

Answer: (c)

86. Complete the following series by choosing the correct term.

ABC, PQR, DEF, STU, ?

(a)  VWX

(b)  GHI

(c)  IJK

(d)  GKL

Answer: (b)

87. If WOOD is coded as 23|225|4, then MEET is coded as

(a)  13|5|5|20

(b)  13|10|20

(c)  13|25|20

(d)  None of these

Answer: (c)

88. What comes in place of question mark (?) in the alphabetical series.

W, T, P, M, I, F, B, ?, ?

(a)  Z, V

(b)  X, U

(c)  Y, U

(d)  Y, V

Answer: (c)

89. City C is to the South of city B and city A is to the North of city C. In which direction city A is located in respect of city B?

(a)  North

(b)  South

(c)  East

(d)  Cannot be determined

Answer: (d)

90. Surbhi ranks 18th in a class of 49 students. What is her rank from the last?

(a)  31

(b)  28

(c)  35

(d)  32

Answer: (d)

91. There are 35 students in a class. Suman ranks third among the girls in the class. Amit ranks 5th among the boys in the class. Suman is one rank below Amit in the class. No, two students hold the same rank in the class. What is Amit’s rank in the class?

(a)  7th

(b)  5th

(c)  8th

(d)  Cannot be determined

Answer: (a)

92. What comes in place of question mark (?) in the following number series?

4, 7, 12, 21, 38, ?

(a)  75

(b)  71

(c)  78

(d)  77

Answer: (b)

93. What comes in place of question mark (?) in the following number series?

4, 6, 16, 62, 308, ?

(a)  990

(b)  1721

(c)  698

(d)  1846

Answer: (d)

94. If A is B’s sister, C is B’s mother, D is C’s father and E is D’s mother, then how is A related to D?

(a)  Granddaughter

(b)  Daughter

(c)  Aunt

(d)  Father

Answer: (a)

95. Rajesh is the brother of Ankit. Shano is the sister of Shubham. Ankit is the son of Shano. How is Rajesh related to Shano?

(a)  Father

(b)  Brother

(c)  Son

(d)  Nephew

Answer: (c)

Directions (Q. Nos. 96-98) In each question below is given a statement followed by two assumptions numbered I and II. An assumption is something supposed or taken for granted. You have to consider the statements and the following assumptions and decide which of the assumptions is implicit in the statement.

Give Answer

(a) if only Assumption I is implicit

(b) if only Assumption II is implicit

(c) if either Assumption I or II is implicit

(d) if both Assumption I and II are implicit

96. Statement The driver of the huge truck pulled the emergency brakes to avoid hitting the auto rickshaw which suddenly came in front of the truck.

Assumptions

(I) The auto rickshaw driver may be able to steer his vehicle away from the oncoming truck.

(II) The truck driver may be able to stop the truck before it hits the auto rickshaw.

Answer: (b)

97. Statement The doctor warned the patient against any further consumption of alcohol if he desired to get cured from the ailment and live a longer life.

Assumptions

(I) The patient may follow the doctor’s advice and stop consuming alcohol.

(II) The doctor may be able to cure the patient from the ailment if the patient stops consuming alcohol.

Answer: (d)

98. Statement The Chairman of the company urged all the employees to refrain from making long personal calls during working hours in order to boost productivity.

Assumptions

(I) Majority of the employees may respond positively to the Chairman’s appeal.

(II) Most of the employees may continue to make long personal calls during working hours.

Answer: (a)

Directions (Q. Nos. 99 and 100) In these questions, identify the diagram that represents the best relationship among classes given below.

99. Women, Sisters and Wives

Answer: (a)

100. Pulses, Redgram, Moongdal

Answer: (a)

101. Myntra, the popular fashion e-commerce platform is an entity of which company?

(a)  Amazon

(b)  Flipkart

(c)  Snapdeal

(d)  Google

Answer: (b)

102. Which among the following brands is not an Indian brand?

(a)  Allen Solly

(b)  Da Milano

(c)  Monte Carlo

(d)  Steve Madden

Answer: (d)

103. ‘Carat Lane’, the biggest online jewellery company in India is associated with which brand?

(a)  Kalyan

(b)  Malabar Gold

(c)  Tanishq

(d)  PC Jewellers

Answer: (c)

104. The brand ‘ZARA’ is associated with which country?

(a)  India

(b)  France

(c)  England

(d)  Spain

Answer: (d)

105. National Gallery of Modern Art is located at

(a)  Mumbai

(b)  Kolkata

(c)  New Delhi

(d)  Lucknow

Answer: (c)

106. Who among the following is the brand ambassador of Men’s clothing brand “Wrogn’?

(a)  Hritik Roshan

(b)  Virat Kohli

(c)  Ranveer Singh

(d)  Rohit Sharma

Answer: (b)

107. ‘Your Style, Your Store’ is the tag line of which of the following retail chains?

(a)  Lifestyle

(b)  Pantaloons

(c)  Landmark

(d)  Shoppers shop

Answer: (a)

108. Peter England, one of India’s premium Men’s wear brands, belongs to which group?

(a)  Reliance Industries

(b)  Aditya Birla Group

(c)  Tata Group

(d)  Adani Group

Answer: (b)

109. Nike, a popular fashion and sportswear brand belongs to which country?

(a)  USA

(b)  UK

(c)  Japan

(d)  Spain

Answer: (a)

110. Mirabai Chanu is a famous Indian sportsperson of

(a)  Hockey

(b)  Boxing

(c)  Weightlifting

(d)  Badminton

Answer: (c)

111. What type of fabric is Calico?

(a)  Silk

(b)  Cotton

(c)  Linen

(d)  Wool

Answer: (b)

112. Who was won the Australian open Tennis Title 2020 in the men’s category?

(a)  Rafael Nadal

(b)  Novak Djokovic

(c)  Roger Federer

(d)  Dominic Thiem

Answer: (b)

113. Lakme Fashion week winter festive 2019 took place at

(a)  Bangaluru

(b)  Mumbai

(c)  Delhi

(d)  Pune

Answer: (b)

114. Which of the following is a West flowing river?

(a)  Godavari

(b)  Kaveri

(c)  Son

(d)  Narmada

Answer: (d)

115. Sariska Tiger Reserve is located in which of the following states?

(a)  Maharashtra

(b)  Uttar Pradesh

(c)  Rajasthan

(d)  Tamil Nadu

Answer: (c)

116. Where is the headquarter of World Trade Organization located?

(a)  Vienna

(b)  Geneva

(c)  New York

(d)  Philippines

Answer: (b)

117. Who was sworn in as the Prime Minister of Great Britain in July 2019?

(a)  Theresa May

(b)  Tony Blair

(c)  Boris Johnson

(d)  Bonar Law

Answer: (c)

118. Which part of Indian Constitution deals with the Fundamental Rights?

(a)  III

(b)  IV

(c)  IV A

(d)  II

Answer: (a)

119. Pravasi Bharatiya Divas is celebrated to commemorate the return of whom to India?

(a)  Subhash Chandra Bose

(b)  Mahatma Gandhi

(c)  Rakesh Sharma

(d)  Bal Gangadhar Tilak

Answer: (b)

120. Which of the following Union Territories are recently merged with Daman and Diu?

(a)  Dadar and Nagar Haveli

(b)  Andaman and Nicobar island

(c)  Lakshadweep

(d)  Chandigarh

Answer: (a)

121. Which is the first Design School in India?

(a)  NID

(b)  NIFT

(c)  IDC

(d)  SID

Answer: (a)

122. Which country is known as the ‘Land of the Rising Sun’?

(a)  Norway

(b)  Australia

(c)  Japan

(d)  Mexico

Answer: (c)

123. The 2024, Olympic Games are scheduled to be held in

(a)  Tokyo

(b)  Paris

(c)  Los Angles

(d)  Shanghai

Answer: (b)

124. Van Heusen is a leading international brand of

(a)  Cars

(b)  Bikes

(c)  Clothing

(d)  Sports accessories

Answer: (c)

125. Who was awarded the 2019 Ramon Magsaysay award for journalism?

(a)  Ravish Kumar

(b)  Bhavesh Kumar

(c)  Abhisar Sharma

(d)  Vinod Dua

Answer: (a)

CASE 1

Nike used factory scraps and recycled “space waste  yarn” to create the Space Hippie sneakers in an experimental project to reduce the carbon impact of its products. Each of the four different designs in the collection – named Space Hippie 01, 02, 03 and 04 – are made from scrap material taken from Nike’s factory floors. Which the brand has named “space junk”, and other recycled materials. The result is the creation of Nike footwear with Nike’s lowest carbon footprint scores ever. Nike said the shoe is a first step towards employing the process of the circular economy, which aims to eliminate waste and pollution from manufacturing. The upper part of Space Hippie is knitted from what Nike calls “space waster yarn”. This is made from 100 per cent recycled material including plastic water bottles, T-shirts and textile scraps.

When combined with other design elements, Nike claims the shoe upper consists of a total of 90 per cent recycled content. It redesigned this lightweight foam in a manner that uses around half the carbon dioxide equivalent as compared to normal Nike foams. The outsole part of each shoe in the Space Hippie collection is built from “crater foam”, made from a mixture of standard Nike foams and 15 per cent recycled waste rubber that has been ground down into granules, which Nike has branded as Nike Grind.

126. Nike has used recycled materials to create the space hippie sneakers for which purpose?

(a)  To change its product strategy

(b)  To reduce carbon impact

(c)  To support waste collectors

(d)  To create new designs

Answer: (b)

127. According to Nike, the space hippie sneakers are the first step towards

(a)  A clean environment

(b)  A circular economy

(c)  A sustainable ecosystem

(d)  A new brand to be launched by Nike

Answer: (b)

128. The outsole part of the Nike sneakers is built from which foam?

(a)  Space waste foam

(b)  Crater foam

(c)  Rubber foam

(d)  Junk foam

Answer: (b)

129. The space waste yarn used by Nike is

(a)  waste collected from outer space

(b)  waste collected from waste collectors

(c)  waste from Nike’s factories

(d)  waste from other brands

Answer: (c)

130. The Recycled waste rubber used by Nike in their sneakers, has been named as

(a)  Nike waste

(b)  Nike granules

(c)  Nike rubber

(d)  Nike grind

Answer: (d)

CASE 2

Swedish fast-fashion major H and M has announced its first global collaboration with celebrated Indian couture and jewellery designer Sabyasachi Mukherjee. The new collection will be sold under the label Wanderlust. The collection will be out on 16th April, 2020. The high-end Indian designer wants to make the designs accessible to a larger set of people, both in India and worldwide.

The Sabyasachi plus H and M collection will be offering clothes for both ladies and men, including accessories and will give a modern twist to its inspiration-Indian textile, craft and history. Taking cues from India’s rich textile, craft and history, the collection mixes modern and traditional silhouettes with a tilt towards athleisure and glamping. A key highlight of this collection will be Indian textile and print traditions made by the Sabyasachi Art Foundation, meticulously crafted, embroidery and multicultural silhouettes. The collection will be available at all H and M stores in India as well as selected H and M flagship stores around the world, and online on HM.com and Myntra.

Sabyasachi has previously collaborated with various international luxury designers and brands such as KarI Lagerfeld, Christian Louboutin, Pottery Barn, Balmain and Versace.

131. Sabyasachi has collaborated with H and M, which is based in which of these countries

(a)  Norway

(b)  Australia

(c)  Sweden

(d)  India

Answer: (c)

132. Under the collaboration of Sabyasachi and H and M, their new product range will be known as

(a)  Global star

(b)  Cosmos  

(c)  Korona

(d)  Wanderlust

Answer: (d)

133. The Sabyasachi and H and M collection will offer which of these products

(a)  Ladies clothes

(b)  Men clothes

(c)  Accessories

(d)  All of these

Answer: (d)

134. Apart from HM.com, which other online portal would sell the products of their collaboration?

(a)  Flipkart

(b)  e-bay

(c)  Myntra

(d)  Amazon

Answer: (c)

135. The aim of Sabyasachi and H and M collaboration is

(a)  Mixing Indian and Western culture

(b)  Showing traditional arts of India

(c)  Giving an old glance to its products

(d)  Making designs accessible to larger set of people

Answer: (d)

CASE 3

Prada has become the first luxury brand to sign a loan tied to sustainability targets. Prada has signed a £42.9 million loan with banking group Credit Agricole, with repayment terms conditional to meeting key targets around the sustainability of its products and operations. Prada’s interest payments on the five-year loan will be determined annually based on whether the company has hit three specific objectives.

The first focuses on its physical shops, setting out that a certain number of them need to be certified gold or platinum according to the green-building rating system, Leadership in Energy and Environmental Design (LEED). This evaluates everything from the design and construction of a building to its management and the extent to which it uses resources and produces waste. The other targets are related to the amount of training hours given to employees, and reinforce a pledge made by Prada earlier this year which involves phasing out the use of virgin nylon by 2021.

According to Prada’s chief financial officer Alessandra Cozzani, linking more sustainable business practices to concrete financial rewards is an attempt to engrain these values into the very functioning of the company.

This transaction demonstrates that sustainability is a key element for the development of the Prada Group. Beyond substituting nylon, Prada has also gone fur-free, switching to faux fur, although these are often made of plastic.

136. Prada has become the first luxury brand to sign a loan that is tied to its

(a)  economic performance

(b)  sustainability targets

(c)  stocks in markets

(d)  None of the above

Answer: (b)

137. Prada has to design its shops to meet the sustainability targets according to which rating system?

(a)  BIS

(b)  LEED

(c)  ISO

(d)  None of the above

Answer: (b)

138. Prada has promised to phase out virgin nylon by which year?

(a)  2020

(b)  2021

(c)  2025

(d)  2050

Answer: (b)

139. According to Prada, what is the purpose of linking the sustainable practices to financial rewards?

(a)  To save the environment

(b)  To achieve the sustainable development

(c)  The engrain sustainable values in functioning of the company

(d)  To develop Prada Group

Answer: (c)

140. Apart from substitution of the Nylon, Prada has also eliminated which of the following elements from its processes?

(a)  Wool

(b)  Cotton

(c)  Plastic

(d)  Fur

Answer: (d)

Directions (Q. Nos. 141-150) In each of these questions, a passage is followed by several inferences. You  have to examine each inference separately in the context of the passage and secede upon its degree of truth or falsity.

(a) If the inference is definitely true i.e. it directly follows from the facts given the passage.

(b) If the inference is probably true i.e. though not definitely true in the light of facts given

(c) If the inference is probably false i.e. though not definitely false in the light of facts given

(d) If the inference is definitely false i.e. it contradicts the facts given.

CASELET 1

In India, the proportion of women in paid work is among the lowest in the world, at just over 23% – a figure which contrasts sharply with the corresponding rate of over 78% for  men. Opportunities for women to enter employment in the country are limited by a range of factors. These include a dominant tradition of female domestic responsibility, and a prevailing social patriarchy.

Deeply entrenched cultural expectations mean that women are more likely to stay at home. And when they do work, it is mainly on an informal basis, without the luxury of secured wages and contracts.

Against this backdrop, the idea of female entrepreneurship in India faces major challenges. Setting up a business can require significant efforts outside of normal work times, and can lead to women being perceived as irresponsible if they dedicate time to entrepreneurial activities.

But the scenario is still changing. Improved access to social media, education, and social enterprises are all contributing to change.

These are giving momentum to the aspirations of women entrepreneurs in India. Their stories will hopefully inspire women entrepreneurs from around the world, while encouraging policy makers to create avenues that support their aspirations.

141. In India, the proportion of women in paid work is just 23% as compared to 78% for men.

Answer: (a)

142. The women have low participation rate in paid work as they do not have necessary skill set.

Answer: (d)

143. A large number of women are employed in the informal sector of the economy that do not provide regular wages.

Answer: (b)

CASELET 2

In the wake of the recent brutal murder of a  young woman in Hyderabad, much of the public discourse on the subject has been confined to outrage, policing, punishment and tougher laws. All this sound and fury suggest that few really want to address the basic flaws in our culture that breed male violence against women.

Holding up a mirror to ourselves can be a painful exercise as it is safer to vent against the government of the day, lack of law enforcement personnel and unutilized budget allocations for women’s safety programmes.

Deep-rooted social prejudice against women reflects in India’s adverse sex ratio, particularly in the age group 0-6. Families prefer boys: female foetus are selectively aborted, infant girls allowed to die. An official campaign of the government has been addressing this most violent form of discrimination against the female sex.

However, other egregious forms of discrimination that push women to a subordinate role in society and train men to see themselves as entitled to favourable treatment fail to get addressed. Only rigorous democratization of society can bring about sustainable improvement in the conditions of women. The sooner we realize this, the better.

144. There has been a critical discussion on the issues that affect women in the country.

Answer: (d)

145. Families prefer a boy child over a girl child and discriminate against the women.

Answer: (a)

146. The problems of women can be addressed when they get truly represented in the society.

Answer: (b)

147. There are basic flaws in the culture and society of India that are only responsible for perpetuating male violence.

Answer: (b)

CASELET 3

In the decade between 2005 and 2016, India lifted 27 crore people out of extreme poverty-more than the combined population of Congo, Uganda, Iraq, Venezuela, Australia and Italy. India more than doubled its per capita GDP moving up from being a poor country to a middle-income one. An average Indian now earns slightly more than Rs. 10,500 a month, up from around Rs. 4,000 in 2005. The numbers are flattering but deceiving. More than 36 crore Indians still cannot afford three square meals a day, which is why the World Bank continues to bracket India with Nicaragua, Honduras, Kenya and Kiribati. On an average, even people in war-torn Libya and sanctions-plagued Cuba continue to earn more than Indians.

There is another worry line for India. The Oxford Poverty and Human Development Initiative, which annually publishes the most authoritative index of poverty across the world, has found that the poorest among India’s poor did not increase their income as much as the other sections did. It means that the gap between the poor and the rich is widening fast. The last time the government appointed an expert committee to estimate poverty in India was seven years ago. The committee, headed by former Reserve Bank of India Governor C. Rangarajan submitted its report but the report has neither been rejected nor been released, which means that economists in India still baser their research largely on data that date back to 2012.

148. India has been able to lift almost 27 crore people out of extreme poverty.

Answer: (a)

149. India’s average per capita income has increased in the decade 2005-2016 because of reduction in its population.

Answer: (c)

150. The gap between poor and rich is widening fast because of reducing income of the poor.

Answer: (b)

National Institute of Fashion & Technology (NIFT) Post Graduate 2021 Question Paper With Answer Key

NIFT (Post Graduate)

National Institute of Fashion and Technology

Solver Paper 2021

1. The length, breadth and height of a room are in the ratio of 3 : 2 : 1. If its volume be 1296 m3, find its breadth.

(a)  24 m

(b)  15 m

(c)  16 m

(d)  12 m

Answer: (d)

2. Ram and Shyam run a race of 2000 m. First, Ram gives Shyam a start of 200 m and beats him by 30 s. Next, Ram gives Shyam a start of 3 min and is beaten by 1000 m. Find the time in and is beaten by 1000 m. Find the time in minute in which Ram and Shyam can run the race separately?

(a)  8 min, 10 min,

(b)  4 min, 5 min

(c)  5 min, 9 min

(d)  6 min, 9 min

Answer: (b)

3. There is a number lock with four rings. How many attempts at the maximum would have to be made before getting the right number?

(a)  104

(b)  255

(c)  104 – 1

(d)  256

Answer: (c)

4. If the curved surface area of a cone is thrice that of another cone and slant height of the second cone is thrice that of the first, find the ratio of the area of their base.

(a)  9 : 1

(b)  81 : 1

(c)  3 : 1

(d)  27 : 1

Answer: (b)

5. Seven equal cubes each of side 5 cm are joined end to end. Find the surface area of the resulting cuboid.

(a)  750 cm2

(b)  1500 cm2

(c)  2250 cm2

(d)  700 cm2

Answer: (a)

6. 400 students took the mock test 60% of the boys and 80% of the girls cleared the cut off in the test. If the total percentage of students qualifying is 65%, how many girls appeared in the test?

(a)  100

(b)  120

(c)  150

(d)  300

Answer: (a)

7. Two fair dices are thrown. Given that, the sum of the dice is less than or equal to 4, find the probability that only one dice shows two.

(a)  1/4

(b)  1/2

(c)  2/3

(d)  1/3

Answer: (d)

8. There are 10 persons P1, P2, ….,P9, P10. Out of these 10 persons, 5 persons are to be arranged in a line such that in each arrangement P1 must occur whereas P4 and P5 do not occur. The number of such possible arrangements are

(a)  7C4 × 5!

(b)  9C5 × 5!

(c)  8C5 × 5!  

(d)  9C5 × 4!

Answer: (a)

9. In a business, A and C invested amounts in the ratio 2 : 1, whereas the ratio between amounts invested by A and B was 3 : 2. If Rs. 157300 was their profit, how much amount did B receive?

(a)  Rs. 24200

(b)  Rs. 36300

(c)  Rs. 48400

(d)  Rs. 72600

Answer: (c)

10. What will be the ratio of petrol and kerosene in the final solution formed by mixing petrol and kerosene that are present in three vessels in the ratio 4 : 1, 5 : 2 and 6 : 1, respectively?

(a)  166 : 22

(b)  83 : 22

(c)  83 : 44

(d)  None of these

Answer: (b)

11. A cistern contains 50 L of water. 5 L of water is taken out of it and replaced with wine. The process is repeated again. Find the proportion of wine and water in the resulting mixture.

(a)  1 : 4

(b)  41 : 50

(c)  19 : 81

(d)  81 : 19

Answer: (c)

12. A person saves 6% of his income, 2 yr later, his income shoots up by 15% but his savings remain the same. Find the hike in his expenditure (in approx.. per cent).

(a)  13.65%

(b)  12.45%

(c)  14.85%

(d)  15.95%

Answer: (d)

13. A reduction in the price of petrol by 10% enables a motorist to buy 5 gallons more for Rs. 180. Find the original price of petrol (in Rs. Per gallon).

(a)  20

(b)  30

(c)  40

(d)  50

Answer: (c)

14. A shopkeeper allows a discount of 10% on the marked price of an item but charges a sales tax of 8% on the discounted price. If the customer pays Rs. 680.40 as the price including the sales tax, then what is the marked price of the item?

(a)  Rs. 630

(b)  Rs. 700

(c)  Rs. 780

(d)  None of these   

Answer: (b)

15. Ashok borrows Rs. 1500 from two moneylenders. He pays interest at the rate of 12% per annum for one loan and at the rate of 14% per annum for the other. The total interest he pays for the entire year is Rs. 186. How much does he borrow at the rate of 12%?

(a)  Rs. 1200

(b)  Rs. 1300

(c)  Rs. 1400

(d)  Rs. 1000

Answer: (a)

16. A driver of an auto-rickshaw makes a profit of 20% on every trip when he carries three passengers and the price of petrol is Rs. 30/L. Find the percentage profit for the same journey if he goes for four passengers per trip and the price of petrol reduces Rs. 24/L? (Assume that revenue per passenger is the same in both the cases).

(a)  33.33%

(b)  65.66%

(c)  100%

(d)  Data inadequate

Answer: (c)

17. After receiving two successive rises, Ajitha’s salary became equal to 15/8 times of her initial salary. By how much per cent was the salary rised the first time if the second rise was twice as high (in per cent) as the first?

(a)  15

(b)  20

(c)  25

(d)  30

Answer: (d)

18. Find the principal if compound interest is charged on the principal at the rate of  per annum for two years and the sum becomes Rs. 196.

(a)  Rs. 140

(b)  Rs. 154

(c)  Rs. 150

(d)  None of these

Answer: (d)

19. A tank of 4800 m3 capacity is full of water . The discharging capacity of the pump is 10 m3/min higher than its filling capacity. As a result the pump needs 16 min less to discharge the fuel, then to fill up the tank. Find the filling capacity of the pump.

(a)  50 m3/min

(b)  25 m3/min

(c)  55 m3/min

(d)  None of these

Answer: (a)

20. Divide Rs. 6000 into two parts, so that simple interest on the first part for 2 yr at 6% per annum may be equal to the simple interest on the second part for 3 yr at 8% per annum?

(a)  Rs. 4000, Rs. 2000

(b)  Rs. 5000, Rs. 1000

(c)  Rs. 3000, Rs. 3000

(d)  None of these

Answer: (a)

21. A part of Rs. 38800 is lent out at 6% per six months. The rest of the amount is lent out at 5% per annum after 1 yr. The ratio of interest after 3 yr from the time when first amount was lent out is 5 : 4. Find the second part that was lent out at 5%.

(a)  Rs. 26600

(b)  Rs. 28800

(c)  Rs. 7500

(d)  Rs. 28000

Answer: (b)

22. A person draws a card from a pack of 52, replaces it and shuffles it. He continues doing it until he draws a heart. What is the probability that he has to make 3 trials?

(a)  9/64

(b)  3/64

(c)  5/64

(d)  1/64

Answer: (a)

23. The cost of manufacture of an article is made up of four components A, B, C and D which have a ratio of 3 : 4 : 5 : 6, respectively. If there are respective changes in the cost of +10%, −20%, −30% and +40%, then what would be the percentage change in the cost?

(a)  2.22%

(b)  1.80%

(c)  3.28%

(d)  0.95%

Answer: (a)

24. A railway passenger counts the telegraph poles on the rail road as he passes them. The telegraph poles are at a distance of 50 m. What will be his count in 4 h, if the speed of the train is 45 km/h?

(a)  600

(b)  2500

(c)  3600

(d)  5000

Answer: (c)

25. In a zoo, there are healthy deers and ducks. If the heads are counted, there are 180, while the legs are 448. What will be the number of deers in the zoo?

(a)  156

(b)  68

(c)  22

(d)  44

Answer: (d)

26. Aditi bought an article and spent Rs. 110 on its repairs. She then sold it to Samir at a profit of 20%. Samir sold it to Vikas at a lost of 10%. Vikas finally sold it for Rs. 1188 at a profit of 10%. How much did Aditi pay for the article?

(a)  Rs. 890

(b)  Rs. 1000

(c)  Rs. 780

(d)  Rs. 840

Answer: (a)

27. Mukesh borrows a certain sum of money from the ABC Bank at 10% per annum at compound interest. The entire debt is discharged in full by Mukesh on payment of two equal amounts of Rs. 1000 each, one at the end of the first year and the other at the end of the second year. What is the approximate value of the amount borrowed by him?

(a)  Rs. 1852

(b)  Rs. 1694

(c)  Rs. 1736

(d)  Rs. 1792

Answer: (c)

28. After selling a watch, Sultan found that he had made a loss of 10%. He also found that had he sold it for Rs. 27 more, he would have made a profit of 5%. The actual initial loss was what percentage of the profit earned, had he sold the watch for a 5% profit?

(a)  23%

(b)  150%

(c)  180%

(d)  200%

Answer: (d)

29. A cistern is normally filled in 6 h but takes 4 h longer to fill because of a leak in its bottom. If the cistern is full, the leak will empty it in how much time?

(a)  20 h

(b)  15 h

(c)  23 h

(d)  17 h

Answer: (b)

30. A and B completed a work together in 5 days. Had A worked at twice the speed and B at half the speed, it would have taken them 4 days to complete the job. How much time would A alone take to do the work?

(a)  10 days

(b)  20 days

(c)  15 days

(d)  25 days

Answer: (a)

Directions (Q. Nos. 31-33) In each of the following questions, an idiomatic expression/proverb has been given followed by some alternatives. Choose the one that best expresses the meaning of the given bold idiom/proverb.

31. Don’t mix with the bad hats.

(a)  people of bad character

(b)  people selling bad hats

(c)  people of poor status

(d)  people with bad hats

Answer: (a)

32. The scientist worked for donkey’s years to arrive at the formula.

(a)  a short time

(b)  for donkeys

(c)  for few years

(d)  a long time

Answer: (d)

33. The President gave away the prizes to the winners.

(a)  released

(b)  gave back

(c)  distributed

(d)  let go

Answer: (c)

Directions (Q. Nos. 34 and 35) In each of the questions below, only one among the given alternatives is correctly spelt. Find out the word with the correct spelling.

34.

(a)  Adulation

(b)  Adlation

(c)  Aduletion

(d)  Addulation

Answer: (a)

35.

(a)  Indipensable

(b)  Indipenseble

(c)  Indispansible

(d)  Indispensable

Answer: (d)

Directions (Q. Nos. 36 and 37) In each of the questions below, choose the wrongly spelt word.

36.

(a)  Potassium

(b)  Possibility

(c)  Parasology

(d)  Preamble

Answer: (c)

37.

(a)  Sanguinery

(b)  Itinerary

(c)  Temporary

(d)  Necessary

Answer: (a)

Directions (Q. Nos. 38 and 39) From amongst the options given below each word, choose the appropriate singular form.

38. Lice

(a)  Lices

(b)  Louse

(c)  Licen

(d)  Leece

Answer: (b)

39. Loaves

(a)  Loaf

(b)  Loav

(c)  Loave

(d)  Loafs

Answer: (a)

Directions (Q. Nos. 40 and 41) From amongst the options given below each word, choose the appropriate plural form.

40. Chateau

(a)  Chateaus

(b)  Chateaux

(c)  Chatea

(d)  Chateaues

Answer: (b)

41. Nucleus

(a)  Nucleuses

(b)  Nucleus

(c)  Nuclei

(d)  Nucleius

Answer: (c)

Directions (Q. Nos. 42-45) In each of these questions, choose the alternative which can replace the underlined word without changing the meaning of the sentence.

42. Tax evasion amounts to larcenous accumulation of public money by some unscrupulous individuals.

(a)  miserly

(b)  extravagant

(c)  theft

(d)  shrewd

Answer: (c)

43. I cannot believe in the veracity of his statement.

(a)  truth

(b)  usefulness

(c)  sincerity

(d)  falsity

Answer: (a)

44. People thronged to pay homage to the departed leader.

(a)  humility

(b)  tribute

(c)  obedience

(d)  allegiance

Answer: (b)

45. He is a sycophant who tries to win over politicians.

(a)  a psychologist

(b)  an opportunist

(c)  an unscrupulous man

(d)  a flatterer

Answer: (d)

Directions (Q. Nos. 46-49) Fill in the blanks.

46. Did the boys turn ……… for football practice?

(a)  up

(b)  on

(c)  back

(d)  in

Answer: (a)

47. Corruption is a standing hindrance ………. The nation’s development.

(a)  of

(b)  over

(c)  to

(d)  upon

Answer: (c)

48. You ……… mad if you think I’m going to show my answer-sheet.

(a)  are supposed to be

(b)  must be

(c)  will be

(d)  ought to be

Answer: (b)

49. At one time Mr. Nigel ……… this supermarket.

(a)  was owing

(b)  used to own

(c)  had owned

(d)  owned

Answer: (d)

Directions (Q. Nos. 50-53) In each of these questions, choose the alternative which is closest to the opposite in meaning to the underlined word.

50. There was marked deterioration in his condition.

(a)  reformation

(b)  amendment

(c)  improvement

(d)  revision

Answer: (c)

51. I find his views repugnant.

(a)  amiable

(b)  repulsive

(c)  amoral

(d)  apolitical

Answer: (a)

52. There was not a single bibulous adventurer in our expedition.

(a)  fearful

(b)  cowardly

(c)  sober

(d)  unenergetic

Answer: (c)

53. This author has perspicuity in his style.

(a)  frankness

(b)  bluntness

(c)  obtuseness

(d)  vivacity

Answer: (c)

Directions (Q. Nos. 54 and 55) In each of the following questions, choose the alternative that can be substitutes for the given word/sentence.

54. A person who eats too much

(a)  Glutton

(b)  Nibbler   

(c)  Cannibal

(d)  Omnivore

Answer: (a)

55. Fear of spiders

(a)  Hydrophobia

(b)  Social Phobia

(c)  Agoraphobia

(d)  Arachnophobia

Answer: (d)

Directions (Q. Nos. 56-75) Read the following passages and answer the questions that follow.

PASSAGE 1

Organisations are institutions in which members compete for status and power. They compete for the resources of the organization, for example, finance to expand their own departments, for career advancement and for power to control the activities of others. In pursuit of these aims, groups are formed and sectional interests emerge. As a result, policy decisions may serve the ends of the political and career systems rather than those of the concern. In this way, the goals of the organization may be displaced in favour of sectional interests and individual ambition. These preoccupations sometimes prevent the emergence of organic systems. Many of the electronics firms in their study had recently created research and development departments employing highly qualified and well-paid scientists and technicians. Their high pay and expert knowledge were sometimes seen as a threat to the established order of rank, power and privilege. Many senior managers had little knowledge of the technicalities and possibilities of new developments and electronics. Some felt that close cooperation with the experts in an organic system would reveal their ignorance and show that their experience was now redundant.

56. The author makes out a case for

(a)  organic system

(b)  research and development in organizations

(c)  an understanding between senior and middle level executives

(d)  a refresher course for senior

Answer: (a)

57. The theme of the passage is

(a)  groupism in organisations

(b)  individual ambitions in organizations.

(c)  frustration of senior managers.

(d)  emergence of sectional interests in organizations.

Answer: (d)

58. Policy decision in organization should involve

(a)  cooperation at all levels in the organization.

(b)  modernization of the organization.

(c)  attracting highly qualified personnel.

(d)  keeping in view the larger objectives of the organization.

Answer: (b)

59. “Organic system” as related to the organization implies is

(a)  growth with the help of expert knowledge.

(b)  growth with inputs from science and technology

(c)  steady all-round development.

(d)  natural and unimpeded growth.

Answer: (a)

60. The author tends to see the senior managers as

(a)  ignorant and incompetent.

(b)  a little out of step with their work environment.

(c)  jealous of their younger colleagues.

(d)  robbed of their rank, power and privilege

Answer: (a)

PASSAGE 2

Teaching more than most other professions, has been transformed during the last hundred years from a small, highly skilled profession concerned with a minority of the population, to a large and important branch of public service. The profession has a great and honourable tradition, extending from the dawn of history until recent times, but any teacher in the modern world who allows himself to be inspired by the ideals of his predecessors is likely to be made sharply aware that it is not his function to teach what he thinks, but to instill such beliefs and prejudices as are thought useful by his employers.

61. In ancient times, the teaching profession was

(a)  reserved for the upper class.

(b)  reserved for a privileged few.

(c)  open to everyone irrespective of any differences

(d)  limited to a highly skilled minority.

Answer: (d)

62. The phrase ‘Extending from the drawn of history’ indicates

(a)  tradition

(b)  culture

(c)  antiquity

(d)  All of these

Answer: (d)

63. In modern times, a successful teacher is primarily supposed to

(a)  impart knowledge.

(b)  impart new and latest skills.

(c)  toe the lines preferred by those in authority.

(d)  instill values he cherishes the most.

Answer: (c)

64. The modern teacher is not able to follow the ideals of his predecessors because

(a)  of more interest in politics than in academic activity.

(b)  of tremendous advancements in professional skills.

(c)  of social and financial constraints.

(d)  the students are not serious about studies.

Answer: (c)

65. The author seems to

(a)  be against the current trend in the teaching profession.

(b)  approve the recent developments in the mode of teaching.

(c)  be a traditionalist in his views.

(d)  consider education as a part of public service.

Answer: (a)

PASSAGE 3

Speech is a great blessing but it can also be a great curse, for while it helps us to make our intentions and desires known to our fellows, it can also, if we use it carelessly, make our attitude completely misunderstood. A slip of the tongue, the use of an unusual word or of an ambiguous word, and so on, may create an enemy where we had hoped to win a friend. Again, different classes of people use different vocabularies, and the ordinary speech of an educated man may strike an uneducated listener as pompous. Unwittingly, we may use a word which bears a different meaning to our listener from what it does to men of our own class. Thus speech is not a gift to use lightly without thought, but one which demands careful handling. Only a fool will express himself alike to all kinds and conditions of men.

66. Speech can be a curse, because it can

(a)  hurt others.

(b)  lead to carelessness.

(c)  create misunderstanding.

(d)  reveal our intentions

Answer: (c)

67. A “slip of the tongue” means something said

(a)  unintentionally.

(b)  without giving proper thought.

(c)  to hurt another person.

(d)  Both (a) and (b)

Answer: (d)

68. While talking to an uneducated person, we should use

(a)  ordinary speech.

(b)  his vocabulary.

(c)  simple words.

(d)  polite language.

Answer: (b)

69. If one used the same style of language with everyone, one would sound

(a)  flat

(b)  boring

(c)  foolish

(d)  democratic

Answer: (c)

70. The best way to win a friend is to avoid

(a)  irony in speech.

(b)  pomposity in speech.

(c)  verbosity in speech.

(d)  ambiguity in speech.

Answer: (d)

PASSAGE 4

If you are running out of ideas for your New Year’s resolution, consider running. Researchers have found that runners show greater functional connectivity in brain regions important for tasks such as planning and decision-making. “These activities, such as running, that people consider repetitive actually involve many complex cognitive functions – like planning and decision-making – that may have effects on the brain,” said one of the researchers, David Raichlen, associate professor at University of Arizona, at Tucson in the United States. For the study, the researchers compared brain scans of young adults engaged in cross-country running to young adults who do not engage in regular physical activity.

Participants were roughly of the same age – 18 to 25 – with comparable body mass index and educational levels. The runners, overall showed greater functional connectivity or connections between distinct brain regions within several areas of the brain, including the frontal cortex, which is important for cognitive functions such as planning decision-making and the ability to switch attention between tasks.

71. Research has shown that running

(a)  may weaken your heart.

(b)  could deteriorate your eye-sight.

(c)  improves your brain-functioning.

(d)  has no effect on your health.

Answer: (c)

72. Cognitive functioning of your brain impacts your

(a)  ability to take decisions.

(b)  capability to plan things.

(c)  ability to shift  your attention from one task to another.

(d)  All of the above

Answer: (d)

73. Generally, running is considered

(a)  not a good physical activity.

(b)  an activity which repeats itself regularly.

(c)  a useless activity for health.

(d)  None of the above

Answer: (b)

74. While selecting participants to conduct research, it was ensured that participants were of similar

(a)  academic background.

(b)  age group.

(c)  body mass index.

(d)  All of these

Answer: (d)

75. Which of the following statements is not true?

(a)  Running hampers various connections between different brain regions.

(b)  Running is recommended as a good hobby for people.

(c)  Participants for research included those who took part in cross-country running.

(d)  All of the above

Answer: (d)

Directions (Q. Nos. 76-79) Each of these questions has an assertion (A) and reason (R). Mark answer as

(a) if both (A) and (R) are true and (R) is the correct explanation of (A).

(b) if both (A) and (R) are true but (R) is not the correct explanation of (A).

(c) if (A) is true but (R) is false.

(d) if (A) is false but (R) is true.

76. Assertion (A) Devaluation of a currency may promote export.

Reason (R) Price of the country’s products in the international market may fall due to devaluation.

Answer: (a)

77. Assertion (A) Fiscal deficit is greater than budgetary deficit.

Reason (R) Fiscal deficit is the borrowing from the Reserve Bank of India plus other liabilities’ of the government to meet its expenditure.

Answer: (d)

78. Assertion (A) According to statistics, more female children are born each year than male children in India.

Reason (R) In India, the death rate of a male child is higher than that of the female child.

Answer: (c)

79. Assertion (A) Insect resistant transgenic cotton has been produced by inserting Bt gene.

Reason (R) The Bt gene is derived from a bacterium.

Answer: (a)

80. T is the son of P, S is the son of Q. T is married to R. R is Q’s daughter. How is S related T?

(a)  Brother

(b)  Uncle

(c)  Father-in-law

(d)  Brother-in-law

Answer: (d)

Directions (Q. Nos. 81-84) In each of these questions a statement is followed by two arguments numbered I and II.

Mark answer as

(a) if only Argument I is strong

(b) if only Argument II is strong

(c) if both the Argument are strong

(d) if neither Argument I nor II is strong

81. Statement Has the Medical Science really lengthened the span of life?

Argument I Yes, the new drugs have been able to combat the diseases and increases the span of life.

Argument II No, the eternal truth that every person’s days are numbered cannot be denied.

Answer: (a)

82. Statement Should mass media be fully controlled by the government?

Argument I Yes, the contradictory news only confuses the people.

Argument II No, its credibility will be doubtful if it confuses people.

Answer: (d)

83. Statement Should sales tax be abolished?

Argument I Yes, it will eliminate an important sources of corruption

Argument II Yes, it will bring the prices of commodities down and hence consumers will be benefited.

Answer: (b)

84. Statement Computer based technology is very fruitful for industrial development in India.

Argument I Yes, accuracy, fast production and fineness are possible through computer technology.

Argument II No, it will increase unemployment in the country.

Answer: (c)

85. A and B start walking from point O. A covers a distance of 300 m in North direction and B covers a distance of 400 m towards West. How far are they now from each other?

(a)  700 m

(b)  500 m

(c)  50 m

(d)  None of these

Answer: (b)

86. Aisha, who is Mohan’s daughter, says to Reena, “Your mother Riya is younger sister of my father, who is third child of Sanjay”. How is Sanjay related to Reena?

(a)  Uncle

(b)  Father

(c)  Grandfather

(d)  Father-in-law

Answer: (c)

87. How many 7’s not immediately preceded by 4 but immediately followed by 2 are there in the following series?

3 4 7 2 8 7 2 9 4 7 1 3 5 7 2 9 9 7 7 2 5 1 4 7 2 3

(a)  1

(b)  2

(c)  3

(d)  4

Answer: (c)

88. In a row of boys, Anand is eleventh from the left and Deepak is fifteenth from the right. When Anand and Deepak interchange their positions Anand will be fifth from the left. Which of the following will be Deepak’s position from the right after rearrangement?

(a)  Seventh

(b)  Seventeenth

(c)  Eleventh

(d)  None of these

Answer: (d)

89. Shiva walks 10 km towards North and then turns right. After walking 3 km, he again turns right and walks 7 km. Now he turns left and walks 1 km. How far is he from the starting point?

(a)  10 km

(b)  7 km

(c)  20 km

(d)  5 km

Answer: (d)

90. If in a certain language SURVIVE is coded as 9182723 and MONSOON is coded as 6549554, how is RUMOUR coded in that language?

(a)  518618

(b)  581618

(c)  865118

(d)  816518

Answer: (d)

91. Neeraj starts walking towards South. After walking 15 m, he turns towards North. After walking 20 m, he turns towards East and walks 10 m. He then turns towards South and walks 5 m. How far is he from his original position and in which direction?

(a)  10 m, East

(b)  10 m, South-East

(c)  10 m, West

(d)  10 m, North-East

Answer: (a)

92. If Neena says, “Anita’s father Raman is the only son of my father-in-law Mahipal”, then how is Bindu, who is the sister of Anita, related to Mahipal?

(a)  Niece

(b)  Daughter

(c)  Daughter-in-law

(d)  None of these

Answer: (d)

93. Thirteen students are standing in horizontal row from left to right. If all the odd numbered students in the row are shifted to successive odd numbered positions, what will be the position of Sahil who was 7th in the row initially?

(a)  9th from right

(b)  6th from left

(c)  5th from right

(d)  8th from left

Answer: (c)

94. In the following number series how many such 7’s are there which are immediately preceded by a pair of numbers whose product is more than the product of pair of numbers immediately following 7?

2 2 7 1 3 9 4 8 7 6 5 4 2 8 3 5 7 4 6 5 9 7 8 6 4 3 9 7 4 6 5 2

(a)  1

(b)  4

(c)  2

(d)  3

Answer: (d)

Directions (Q. Nos. 95-100) Complete the series replacing?

95. HSJ, IRL, JQN, ?

(a)  KPR

(b)  KPP

(c)  PKP

(d)  KOP

Answer: (b)

96. 2, 4, 7, 14, 17, 34, ?

(a)  37

(b)  39

(c)  68

(d)  70

Answer: (a)

97. 2, 4, 12, 48, 240,

(a)  960

(b)  1440

(c)  1080

(d)  1920

Answer: (b)

98. 71, 76, 69, 74, 67, 72, ?

(a)  77

(b)  80

(c)  65

(d)  76

Answer: (c)

99. In a code language, ‘mok dan sil’ means ‘nice big house’, ‘fit kon dan’ means ‘house is good’ and ‘warm tir fit’ means ‘cost is high’. Which word stands for ‘good’ in that language?

(a)  mok

(b)  dan

(c)  fit

(d)  kon

Answer: (d)

100. If in a certain language code ‘nee tim see’ means ‘how are you’, ‘ble nee see’ means ‘where are you’, then what is the code for ‘where’?

(a)  see

(b)  tim

(c)  nee

(d)  ble

Answer: (d)

101. Which company has roped in Saikhom Mirabai Chanu as its brand ambassador?

(a)  Patanjali

(b)  Amway India

(c)  Bigbasket

(d)  Modicare

Answer: (b)

102. The tagline Bleed Blue is associated with which of the following fashion brand?

(a)  Zara

(b)  Allen Solly

(c)  Nike

(d)  Adidas

Answer: (c)

103. Which of the person is the brand ambassador of Siyaram fashion brand?

(a)  Mahendra Singh Dhoni

(b)  Rohit Shetty

(c)  Virat Kohli

(d)  Ranbir Kapoor

Answer: (a)

104. Who has been roped in as the chairman of the committee formed by government to double and quadruple the exports of handlooms in 3 years?

(a)  Dinesh Bhargav

(b)  Sunil Sethi

(c)  Anamika Khanna

(d)  Ashish Soni

Answer: (b)

105. Amazon has appointed which Indian celebrity to lend voice for Alexa?

(a)  Shatrughan Sinha

(b)  Rajnikanth

(c)  Shahrukh Khan

(d)  Amitabh Bachchan

Answer: (d)

106. The ‘Jogajog’ is a platform similar to Facebook. The new platform is being developed by which country ?

(a)  Pakistan

(b)  India

(c)  Nepal

(d)  Bangladesh

Answer: (d)

107. Which of the following retail stores belong to Ambani’s Reliance Industries?

(a)  Pantaloons

(b)  Big Bazar

(c)  Shoppers Stop

(d)  Westside

Answer: (b)

108. Which of the following is an emirati retail fashion brand which comes under Dubai-based retail company?

(a)  Westside

(b)  Lifestyle

(c)  Pantaloons

(d)  Max

Answer: (b)

109. Which states is known for its renowned Tant saree and Batik saree?

(a)  Assam

(b)  Odisha

(c)  West Bengal

(d)  Gujarat

Answer: (c)

110. Which of the following brand is the famous for ladies suits in India?

(a)  Baluchari

(b)  Gucci

(c)  Flormar

(d)  Biba

Answer: (d)

111. How many Indian companies features in the 2021 Fortune’s Global 500 list?

(a)  9

(b)  5

(c)  7

(d)  11

Answer: (c)

112. Which is the first Indian city to become 100 percent COVID-19 vaccinated?

(a)  Hyderabad

(b)  Bhubaneswar

(c)  Pune

(d)  Indore

Answer: (b)

113. Neeraj Chopra has claimed the gold medal for India at Tokyo Olympics in which event?

(a)  Boxing

(b)  Shooting 

(c)  Wrestling

(d)  Javelin Throw

Answer: (d)

114. Government of India launched brand name and logo for marketing of the urban Self-Help Group (SHG) products. What brand name has been given to these products?

(a)  SonChiraiya

(b)  Neelkanth

(c)  Saras

(d)  Gauraiya

Answer: (a)

115. My e-Haat portal has been launched by which company to empower artisans?

(a)  Reliance Foundation

(b)  Infosys Foundation

(c)  Adani Foundation

(d)  HCL Foundation

Answer: (d)

116. Dholavira is the newest Indian site to be inscribed on the UNESCO’s World Heritage list. Where is Dholavira located?

(a)  Ladakh

(b)  Mizoram

(c)  Telangana

(d)  Gujarat

Answer: (d)

117. Which country has assumed the Presidency of the United Nations Security Council (UNSC)?

(a)  India

(b)  Russia

(c)  France

(d)  Germany

Answer: (a)

118. Which of the following brand belongs to Titan as sub-brand?

(a)  Chumbak

(b)  Dressberry

(c)  Fastrack

(d)  Roadster

Answer: (c)

119. Good Morning is the sub-brand of the following brands

(a)  Gucci

(b)  Park Avenue

(c)  Axe

(d)  Old Spice

Answer: (b)

120. Zodiac is the famous brand in which categories of following?

(a)  T-shirt

(b)  Jeans

(c)  Tie

(d)  Kurti

Answer: (c)

121. Who among these is the noted environmentalist and the leader of the Chipko Movement?

(a)  Mohan Dharia

(b)  Chandi Prasad Bhatt

(c)  Mike Pandey

(d)  Sunderlal Bahuguna

Answer: (d)

122. Raimona Reserve Forest has been declared as a National Park. Where is it located?

(a)  Sikkim

(b)  Assam

(c)  Tamil Nadu

(d)  Punjab

Answer: (b)

123. Who among the following has won the Miss Universe title in May 2021?

(a)  Julia Gama

(b)  Andrea Meza

(c)  Adline Castelino

(d)  Kimberly Jimenez

Answer: (b)

124. miniTV is a newly launched free video streaming service. The service has been launched by which entity?

(a)  Amazon India

(b)  Star India

(c)  Reliance

(d)  Zee Entertainment

Answer: (a)

125. The book title “Elephant In The Womb” is a debut book by which Indian celebrity?

(a)  Shilpa Shetty

(b)  Kalki Koechlin

(c)  Twinkle Khanna

(d)  Anushka Sharma

Answer: (b)

CASE 1

American scientists David Julius and Ardem Patapoutian won the 2021 Nobel Prize for Medicine for the discovery of receptors in the skin that sense temperature and touch and could pave the way for new pain-killers. Patrik Ernfors, a member of the Nobel Committee, in Oslo, Sweden, recognized the work by David Julius and Ardem Patapoutian as it has unlocked one of the secrets of nature.

Humans’ abilities to sense heat, cold, pressure and position are vital for perceiving and reacting to our surroundings. Understanding how they work is critical for treating chronic pain and other conditions. Their work, carried out independently, for discovering the molecular bases of how nerves convert stimuli-the burn of a chili pepper or the soft pressure of a hug-into signals that can be sensed by the brain.

Julius and his colleagues worked to find the receptor for capsaicin, component in chilies that causes a painful burning sensation. They identified the gene that encodes a new protein, called TRPV1-an ion channel in the membranes of cells that opens in response to heat. Julius got the idea to do his capsaicin experiments while shopping in a grocery store: “Walking through the supermarket aisle one day, seeing all these hot chili pepper sauces, et cetera, I was thinking, “We really have to get his project done,” . “And my wife said, ‘Well, then  you should get on it!”

Julius and Patapoutian independently identified another protein: TRPM8, which is sensitive to cold and menthol. Additionally, Patapoutian and his colleagues identified the genes for proteins that sense touch, known as Piezo 1 and Piezo 2. He showed that these two proteins were force activated ion channels. Piezo2 was also found to be important for sensing the positions of limbs in space, an ability known as proprioception.

126. Most of the Nobel Prizes in Medicines won by Americans and Europeans since its inception. The award is given by the Nobel Committee for Medicine based at

(a)  London

(b)  New York

(c)  Lisbon

(d)  Stockholm

Answer: (d)

127. American scientists won the Nobel Prize for the discovery of receptors in the skin that sense temperature, this will help in

(a)  New kinds of Pain Killer

(b)  Chemo Therapy

(c)  Treating Gene Structures

(d)  All of the above

Answer: (a)

128. The new identified gene that encodes a new protein, called TRPV1 found at

(a)  Berberine

(b)  Piperine

(c)  Curcumin

(d)  Capsaici

Answer: (d)

129. Julius and Patapoutian independently identified another protein named TRPM8, which is sensitive to

(a)  Cold and Menthol

(b)  Pressure and Position

(c)  Burning Sensation

(d)  All of these

Answer: (a)

130. Patapoutian and his colleagues identified the genes known as Piezol and Piezoz. Which of these two proteins were ion channels of the above two?

(a)  Shape-activated ion channels

(b)  Force-activated ion channels

(c)  Channel-activated ion channels

(d)  None of the above

Answer: (b)

CASE 2

In the year-2020, the fashion industry was trying to hold back its position, as the concept of work from home (WFH) became inevitable. Most of the people started working from home. Retail shops were shut down due to the pandemic and online shopping increased to encourage social distancing. Due to cost-cutting and new responsibilities, expensive purchases were curtailed, which subsequently affected the fashion industry.

Now that the world is slowly returning to normalcy, we have all started to regain and reshape ourselves in the new economic sphere. The designers have to come forward in terms of competitive clothing. Designer now uses Indian handloom fabric to make it more comfortable as it absorbs sweat, so it can be worn at home all day long. However, for now, designing competitive clothing is a challenge. The pandemic has taught us a huge lesson. It has given us time to rethink and reinvent ourselves.

Also, it is undeniable that clothes shape our personality, bringing out the best version of ourselves.

Also, it is undeniable that clothes shape our personality, bringing out the best version of ourselves. Our clothes and jewellery have always been inspired by our cultures and traditions. As humans started to get civilized, presenting oneself was always a prime psychological and emotional point within a human being. Moreover, the psychology of presenting oneself lies at the core of human existence.

During the 80s and 90s, Indian people were clueless about fashion. In today’s generation, our lifestyle is a reference to previous cultural influences. We need to get inspired by classical music, 500-year-old Amir Khusrow’s poetry, and Mughal influences in our craft. So, I feel that in today’s generation, our lifestyle is a reference to previous cultural influences. Even we can get inspirations from our kings and royal families, the way they were wearing their clothes, the jewellery, turbans, attire, shoes, and/or makeup.

However, we need to reinvent and make garments that are wearable and relevant in today’s time. However fashion industry in India has explored many avenues, but the real challenge is to make clothing more relevant to the modem lifestyle. Many designers has worked in each region with each design, including lehengas, cholis, anarkali, Hyderabadi and Pakistani shararas, and farshi. So, a lot of areas have already been explored.

The major question in fashion industry is how do we take out the traditional attire and make it relevant in contemporary dressing sense. Since the younger generation is fast-paced, they don’t want so much fabric and weight on themselves. One needs to select the best textile and make it into a stylish garment.

131. The Fashion Industry has evolved from normalcy to WFH culture with the concept of

(a)  Online Purchasing

(b)  Staggered Fashion

(c)  Competitive Clothing

(d)  None of these

Answer: (c)

132. Which of following have highly influenced our cultures and traditions?

(a)  Clothes and Jewellery

(b)  International Exposure

(c)  Wealth and Prosperity

(d)  Religion and Society

Answer: (a)

133. Which element/s can help us for crafting right fashion?

(a)  Classical Music and Poetry

(b)  Mughal Art and Craft

(c)  Kings and Royal Families

(d)  None of the above

Answer: (d)

134. The real challenge lies in today’s fashion industry in making relevant to

(a)  with cost cutting

(b)  modern lifestyle

(c)  contemporary and fusion

(d)  None of the above

Answer: (b)

135. What sort of fashion, younger generation wants these days?

(a)  Lighter and Stylish

(b)  Smart and Durable

(c)  Costly and Rich Work

(d)  Both (a) and (b)

Answer: (d)

CASE 3

A series of latest scams have exposed the vulnerabilities of the Aadhaar-enabled Payment System (AePS). The AePS enables a person to withdraw money from her bank account anywhere in the country using a local “business correspondent” (BC). A BC is an informal bank agent equipped with a biometric Point-of-Sale (PoS) machine-a kind of micro-ATM.

If you want to withdraw, say, Rs. 500 from your bank account using a BC, you just have to give him the name of your bank and submit yourself to Aadhaar-based biometric authentication (ABBA). The BC will give you Rs. 500 in cash, and his own account will be credited with the same amount. For this to be possible, your bank account must be linked with Aadhaar. But what if the BC enters “one thousand rupees” in the PoS machine even as he gives you five hundred? In that case, one thousand will be debited from your account, and credited to the BC’s account, but you will only get five hundred.

A corrupt BC can even get away with asking a gullible customer to put her finger in the PoS machine under some pretext, without giving her any money. Many similar cases of AePSenabled fraud have emerged in Indial. Most of them are unresolved. Even if the BC can be traced, it is easy for him to claim that he did disburse cash as per records-it is his word against the victim’s. In short, corrupt BCs operate with virtual impunity.

The AePS is a a valuable facility for those who are able to use it safely. Like other micro-A TM systems, it has helped to decongest banks. It can be particularly useful to migrant workers who have no ATM facility. But AePS can be improved through number of measures, for those who are not clear about how it works.

For instance, BCs could be required to make manual if not digital entries into printed customer passbooks. That would act as a permanent, verifiable receipt that cannot be denied to the customer so easily (a blank entry would be incriminating).

Ensuring that BCs are clearly identified in transaction records would also help. So would SMS alerts, when the customer has a mobile number. Roaming BCs should perhaps be banned, at least in states with low literacy levels. And most importantly, better grievance redressal facilities must be made available to the victims of AePS fraud.

136. A Business Correspondent (BC) is equipped with PoS machine, which help in

(a)  Withdraw money without Bank

(b)  Aadhaar-linked Payment

(c)  B2B Transfer

(d)  None of the above

Answer: (d)

137. How Aadhaar-based biometric authentication (ABBA) functions?

(a)  Through Smart Cards with chip

(b)  Through QR Code

(c)  Through biometric based Authentication

(d)  None of the above

Answer: (c)

138. The AcPS are important tool to provide assistance to poor people as it maintains

(a)  Confidentiality

(b)  Authentication

(c)  Hassle-free

(d)  None of the above

Answer: (d)

139. Various cases of AePS-enabled fraud have emerged in India because of

(a)  a corrupt BC

(b)  AePS-related irregularities

(c)  untraceable in nature

(d)  None of the above

Answer: (d)

140. The BCs can be made accountable with

(a)  Manual Entries of Details

(b)  Clear Transaction Details

(c)  Switching to Traditional Methods

(d)  Both (a) and (b)

Answer: (d)

Directions (Q. Nos. 141-150) In each of these questions a passage is followed by several inferences. You have to examine each inferences separately in the context of the passage and decide upon it degree of truth or Falsity.

(a) If the inference is ‘definitely true’ i.e. it directly follows from the facts given in the passage.

(b) If the inference is ‘probably true’ i.e. through not definitely true in the light of facts given.

(c) If you think the inference is ‘probably false’ though not definitely false in the light of facts given.

(d) If you think the inference is ‘definitely false’ i.e. it contradicts the given facts.

CASELET 1

With a combination of rapid turnover of the latest trends, affordable prices, and matchless convenience, online shopping apps are set to take over the fashion industry in India. In the last decade, e-commerce has transformed the business landscape worldwide, driven by the almost complete shift to online shopping during the Covid-19 pandemic. Sites like Ajio, owned by billionaire Mukesh Ambani’s Reliance Industries, and Mytra.com, a subsidiary of Walmart owned Flipkart, the largest player in Indian e-commerce space.

The main reasons behind the growing popularity of such platforms are competitive prices, steep discounting and true to its moniker of “fast fashion,” its ability to create and release the latest styles on demand. Though, online shopping does have its drawbacks, with numerous customers complaining about fraud pictures that show completely different colours or flawed sizing issues. These fast fashion apps may offer good deals to consumers; the speedy turnover and the short life of these garments come at big cost to the environment and ecology.

However, environmentalists are concerned about the impact of such large-scale e-commerce. The micro-plastics released from clothes are not as visible as large plastic packaging. Ingestion of these has catastrophic effects on marine life and even humans. Globally, the fashion industry is responsible for 20 pc of wastewater. To combat this, Indian designers have introduced sustainable fashion e-commerce that offers a wide range of vegan apparel created from 10 pc organic cotton in a fair-trade factory.

Fast or slow, the online fashion industry will undoubtedly continue to expand exponentially. According to IBEF, the Indian e-commerce market expected to grow from USD 46.2 billion in 2020 to USD 111.40 billion by 2025, largely led by giants such as Flipkart and Amazon. Ultimately, it may be to the consumers whether the current e-commerce giants will continue to outpace their local, smaller counterparts, or whether they will be forced to alter some of their controversially unethical manufacturing processes.

141. The growing popularity of e-Commerce platforms are due to steep discounts.

Answer: (b)

142. E-Commerce has transformed the business landscape worldwide, through completely shift to online shopping during the Covid-19 pandemic.

Answer: (a)

143. Online shopping doesn’t have any drawbacks as it offers good deals to consumers.

Answer: (d)

144. Environmentalists are concerned about e-commerce, because of different forms of environment pollution.

Answer: (a)

145. The future of E-Commerce is not bright in India as it will lead by giants like Flipkart and Amazon.

Answer: (c)

CASELET 2

Every year, millions of people are forced to abandon their homes in search of safer places to rebuild their lives. According to the UN, over 82.4 million people were force to leave their homes in 2020 and more than 20 million of them are refugees. Over 200,000 of these refuges are currently in India.

Through its history, India has hosted people fleeing war, conflict and persecution many times-Zoroastrians from Iran, Sri Lankans in the 1980s or Afghans during varied waves of displacement, including the current one in 2021. The country also has the experience of rehabilitating Partition refugees.

Welcoming refuges lies at the core of India’s secular, spiritual and cultural values. India has taken part in 49 peacekeeping missions, in which more than 195,000 troops and a significant number of police personnel assisted the UN and international NGOs in conflict-ridden lands. The paradox, therefore, of such a welcoming country not having its own homegrown national refugee framework requires a rethink.

A sustainable refugee policy is a necessary step intelligently manage population movements and ensure transparency and predictability in our administrative actions. Treatment of refugees must receive the same attention that other human rights protection issues receive-this is consistent with the constitutional emphasis on the rule of law.

The legislation will also clarify the roles of different agencies-governmental, judicial, UN – involved in refugee protection and lay down the procedures of coordination amongst them. It would also help avoid friction between the host country and the country of origin. Other states would recognize the move to grant asylum as a peaceful, humanitarian and legal act, and not an arbitrary political gesture. It will also provide a platform for dialogue on sharing responsibility and aid the search of durable solutions to the root causes of a refugee problem.

Welcoming refugees generally implies an initial investment of public funds. Once refugees start working, this investment may harvest dividends.

Refugees can fill gaps in the labour market or start trades that create wealth and help improve international trade and investment. Thanks to their diverse experiences, refugees bring new concepts, technologies and innovative ideas.

146. Accepting refuges is against the core of India’s cultural values.

Answer: (d)

147. India has given shelter to Sri Lankans in the 1980s and Afghans during the current one in 2021.

Answer: (a)

148. An urgent and temporary refugee policy is required in India to manage refugee movements.

Answer: (c)

149. In India, there is need for a legislative law to deal with the refugee crisis will and also clarify the roles of different agencies-governmental, judicial, UN involved in refugee protection.

Answer: (a)

150. If refugees shall work and invest in India, this will harvest dividends and prosperity.

Answer: (b)

National Institute of Fashion & Technology (NIFT) UNDER GRADUATE 2021 Question Paper With Answer Key

NIFT (Under Graduate)

National Institute of Fashion and Technology

Solver Paper 2021

Directions (Q. Nos. 1-25) Study the passages given below to answer the questions that follow each passage.

PASSAGE I

Opera refers to a dramatic art form, originating in Europe, in which the emotional content is conveyed to the audience as much through the lyrics. By contrast, in musical theatre an actor’s dramatic performance is primary, and the music plays a lesser role. The drama in opera is presented using the primary elements of theatre such as scenery, costumes and acting. However, the words of the opera or libretto are sung rather than spoken. The singers are accompanied by a musical ensemble ranging from a small instrumental ensemble to a full symphonic orchestra.

1. It is pointed out in the passage that Opera

(a)  has developed under the influence of musical theatre.

(b)  is a drama sung with the accompaniment of an orchestra.

(c)  is often performed in Europe.

(d)  is the most complex of all the performing arts.

Answer: (b)

2. We can understand from the passage that

(a)  drama in opera is more important than the music.

(b)  people are captivated more by opera than musical theatre.

(c)  orchestras in operas can vary considerably in size.

(d)  musical theatre relies above all on music.

Answer: (c)

3. It is stated in the passage that

(a)  an opera requires an orchestra.

(b)  many people find musical theatre more captivating than opera.

(c)  music in musical theatre is not as important as it is in opera.

(d)  opera doesn’t have any properties in common with musical theatre.

Answer: (a)

4. A good opera performance requires

(a)  dramatic skills

(b)  singing skills

(c)  musical instrumental skills

(d)  All of the above

Answer: (d)

PASSAGE II

Dolphins are regarded as the friendliest creatures in the sea and stories of them helping drowning sailors have been common since Roman times. The more we learn about dolphins, the more we realize that their society is more complex than people previously imagined. They look after other dolphins when they are ill, care for pregnant mothers and protect the weakest in the community, as we do. Some scientists have suggested that dolphins have a language but it is much more probable that they communicate with each other without needing words. Could any of these mammals be more intelligent than man?

Certainly the most common argument in favour of man’s superiority over them, that we can kill them more easily than they can kill us, is the least satisfactory. On the contrary, the more we discover about these remarkable creatures, the less we appear superior when we destroy them.

5. It is clear from the passage that dolphins

(a)  are proven to be less intelligent than once thought.

(b)  have a reputation for being friendly to humans.

(c)  are the most powerful creatures that live in the oceans.

(d)  are capable of learning a language and communicating with humans.

Answer: (b)

6. The fact that the writer of the passage thinks that we can kill dolphins more easily than they can kill us

(a)  means that they are less adapted to their environment than humans are to theirs.

(b)  proves that dolphins are not the most intelligent species at sea.

(c)  does not mean that we are superior to them.

(d)  proves that the linguistic skills of dolphins are not comprehensible to humans.

Answer: (c)

7. One can infer from the passage that

(a)  dolphins are quite abundant in some areas of the world.

(b)  communication is the most fascinating aspect of the dolphins.

(c)  the ability to think is unique to dolphins among all mammals.

(d)  dolphins have some social traits are similar to those of humans.

Answer: (d)

8. If a dolphin gets sick, the other dolphins

(a)  take care of the sick one

(b)  move on to keep away

(c)  simply keep looking at the sick member

(d)  None of the above

Answer: (a)

PASSAGE III

Naval architects never claim that a ship is unsinkable, but the sinking of the passenger-and-car ferry Estonia in the Baltic surely should never have happened. It was well designed and carefully maintained. It carried the proper number of lifeboats. It had been thoroughly inspected on the day of I ts fatal voyage. Yet hours later, the Estonia rolled over and sank on a cold, stormy night. It went down so quickly that most of those on board, caught in their dark, flooding cabins, had no chance to save themselves. Of those who managed to scramble overboard, only 139 survived The rest died of hypothermia before the rescuers could pluck them from the cold sea. The final death toll was 912 souls. However, there were an unpleasant number of questions about why the Estonia sank and why so many survivors were men in the prime of their lives, while most of the dead were women, children and the elderly.

9. One can understand from the passage that

(a)  most victims were trapped inside the ship as they were in their cabins.

(b)  the lifesaving equipment did not work well and lifeboats could not be lowered.

(c)  design faults and incompetent crew contributed to the sinking of the Estonia ferry.

(d)  naval architects claimed that the Estonia was unsinkable.

Answer: (a)

10. It is clear from the passage that the survivors of the accident

(a)  were immune to hypothermia.

(b)  were mostly young men.

(c)  told the investigators nothing about the accident

(d)  All of the above

Answer: (b)

11. According to the passage, when the Estonia sank,

(a)  there were only 139 passengers on board.

(b)  few of the passengers were asleep.

(c)  there were enough lifeboats for the number of people on board.

(d)  faster reaction by the crew could have increased the Estonia’s chances of survival.

Answer: (c)

12. Estonia appears to be a

(a)  naval ship

(b)  naval submarine

(c)  cargo ship

(d)  passenger ferry

Answer: (d)

PASSAGE IV

Erosion of America’s farmland by wind and water has been a problem since settlers first put the prairies and grasslands under the plough in the nineteenth century. By the 1930s, more than 282 million acres of farmlands were damaged by erosion. After 40 years of conservation efforts, now soil erosion has again accelerated due to new demands placed on the land by heavy crop production. In the years ahead, soil erosion and the pollution problems it causes are likely to replace petroleum scarcity as the nation’s most critical natural resource problem.

13. As we understand from the passage, today, soil erosion in America

(a)  causes humans to place new demands on the land

(b)  happened so slowly that it took forty years to get noticed.

(c)  is the most critical problem that the nation faces.

(d)  is worse than it was in the nineteenth century.

Answer: (d)

14. The author points out in the passage the soil erosion in America

(a)  has damaged 282 million acres ever since settlers first put the prairies and grasslands under the plough.

(b)  could become a more serious problem in the future.

(c)  has been so severe that it has forced people to abandon their settlements.

(d)  was on the decline before 1930s.

Answer: (b)

15. It is pointed out in the passage that in America

(a)  petroleum is causing heavy soil erosion and pollution problems.

(b)  soil erosion has been hastened due to the overuse of farming lands.

(c)  heavy crop production is necessary to meet the demands and to prevent a disaster.

(d)  water is undoubtedly the largest cause of erosion.

Answer: (b)

16. From the passage, it appears that the profession of early settlers in America was

(a)  mining.

(b)  conservation work.

(c)  agriculture.

(d)  All of the above

Answer: (c)

PASSAGE V

The Gandharva Music and Art Fair-better known to its participants and to history simply as “Gandharva”-should have been a colossal failure. Just a month prior to its August 15th, 1969 opening, the fair’s organisers were informed by the Council of Gandharva Music and Art, Allahabad that permission to hold the festival was withdrawn. Amazingly, not only was a new site found, but word spread to the public of the fair’s new location. At the new site, fences that were supposed to facilitate ticket collection never materialized and all attempts at gathering tickets were abandoned. Crowd estimates of 30,000 kept rising; by the end of the three days, some estimated the crowd at 500,000. Then, on the opening night, it began to rain. Off and on, throughout all three days, huge summer storms rolled over the gathering. In spite of these problems, most people think of Gandharva not only as a fond memory but also as the defining moment for an entire generation.

17. Which of the following would be closest in meaning to the word ‘colossal’ as used in this passage?

(a)  Definite

(b)  Surprising

(c)  Massive

(d)  Disastrous

Answer: (c)

18. Which of the following is implied in the passage, about The Gandharva Music and Art Fair held in August 1969?

(a)  It was a colossal failure.

(b)  It was the defining moment for an entire generation.

(c)  Ticket sales never materialized as per expectations.

(d)  All of the above

Answer: (b)

19. In what context is the word ‘amazingly’ used in the passage?

(a)  The fair drew such an unexpectedly enormous crowd

(b)  The event was moved and word sent out in a very short time.

(c)  Despite pressure by Allahabad officials, the fair was held successfully.

(d)  The stormy weather was so unfavourable.

Answer: (b)

20. As per the passage, most people think of Gandharva as a fond memory because the

(a)  event was held for the first time in August, 1969.       

(b)  crowds that thronged the venue were larger than anticipated.

(c)  performers at the fair were popular amongst the youth.

(d)  None of the above

Answer: (d)

PASSAGE VI

Everybody earns the living in their own way to support themselves as well as the family. If you are employed and bored with what you do, take a look at the weirdest jobs below to know that you are luckier than many people. Those who are not well educated or have strange hobbies take up to following jobs t make ends meet:

(a) Fire-Observer Sounds interesting! This job requires the candidate to sit and watch for any upcoming fires in forests. You sit on a tower and warn the authorities about any impending dangers. It’s quite a lonely job. So, if there are any loners out there who seek complete loneliness they can try their hands at this weird job.

(b) Ice-Cream Taster A sweet job! But it takes more than just tasting the ice-creams to get your pay in this job. There can be no compromise about the quality of the ice-cream, so one has to keep all the taste-buds on his tongue alive and active. This requires you to follow a strict diet, not eating anything too spicy or hot. You cannot smoke, drink or even use strong perfumes!

.(c) Sports Mascot Getting dressed in an often silly-looking costume, entertaining thousands of screaming fans and shadow-punching umpires are all in a day’s work for the Sports Mascot.

The term, mascot, or in this case, the sports mascot is used to describe a person, animal or object that is thought to bring luck. Sporting entities are usually the home for these “luck” bringers, but mascots can also be found in other areas of society, such as military units and charity organisations.

The next time you sit at your desk considering a job change, sneak a peek at this list of weird jobs that give a complete twist to your career.

21. A word close in meaning to ‘weird’ is …

(a)  bored

(b)  luckier

(c)  strange

(d)  well educated

Answer: (c)

22. A fire observer

(a)  works alone

(b)  works with lots of people

(c)  has to hate loneliness

(d)  is a lonely person

Answer: (a)

23. A mascot is a/an ….. symbol.

(a)  bad luck

(b)  good luck

(c)  melancholic

(d)  elegant

Answer: (b)

24. An ice-cream taster

(a)  eats whatever he wants

(b)  drinks alcohol.

(c)  has a strict diet.

(d)  is a heavy smoker.

Answer: (c)

25. Getting dressed in an often silly-looking costume probably means to

(a)  dress elegantly

(b)  dress casually

(c)  be naked

(d)  dress ridiculously

Answer: (d)

Directions (Q. Nos. 26 and 27) Choose the option that is the plural form of the given word.

26. Bunch

(a)  Bunches

(b)  Bunch

(c)  Bunchs

(d)  Bunchies

Answer: (a)

27. Shelf

(a)  Shelf

(b)  Shelves

(c)  Shelfes

(d)  Shelfies

Answer: (b)

Directions (Q. Nos. 28-30) Fill in the blanks.

28. He ……… from fever

(a)  has been suffering

(b)  has suffered

(c)  is suffering

(d)  was suffering

Answer: (a)

29. I shall get a job before the week ……….

(a)  will end

(b)  ends

(c)  had ended

(d)  is ending

Answer: (b)

30. The servant had finished his work when I …….. him.

(a)  had called

(b)  has called

(c)  have called

(d)  called

Answer: (d)

Directions (Q. Nos. 31-34) Choose the word which is similar in meaning to the given word.

31. Contrary

(a)  Adverse

(b)  Kind

(c)  Harmonious

(d)  Consistent

Answer: (a)

32. Desolate

(a)  Bright

(b)  Lonely

(c)  Inhabited

(d)  Intact

Answer: (b)

33. Despise

(a)  Adore

(b)  Cherish

(c)  Hate

(d)  Praise

Answer: (c)

34. Eccentric

(a)  Regular

(b)  Ordinary

(c)  Standard

(d)  Erratic

Answer: (d)

Directions (Q. Nos. 35-38) Choose the word which is opposite in meaning to the given word.

35. Abate

(a)  Advance

(b)  Recede

(c)  Diminish

(d)  Slacken

Answer: (a)

36. Bizarre

(a)  Peculiar

(b)  Common

(c)  Weird

(d)  Curious

Answer: (b)

37. Zest

(a)  Passion

(b)  Relish

(c)  Gusto

(d)  Dullness

Answer: (d)

38. Dainty

(a)  Exquisite

(b)  Lacy

(c)  Crude

(d)  Petite

Answer: (c)

Directions (Q. Nos. 39-41) In each of these questions, out of the given group of words, choose the correctly spelt word.

39.

(a)  Acknowledgement

(b)  Acknowledgemant

(c)  Acknawledgement

(d)  Acknowladgemnt

Answer: (a)

40.

(a)  Publesher

(b)  Publisher

(c)  Publishar

(d)  Pablishar

Answer: (b)

41.

(a)  Anouncement

(b)  Announcement

(c)  Annoncement

(d)  Announcment

Answer: (b)

Directions (Q. Nos. 42-44) Choose the option which best expresses the meaning of the given idiom/phrase.

42. Pay through one’s nose

(a)  To pay heavily

(b)  To criticize someone

(c)  To get ahead secretly

(d)  To be under suspicion

Answer: (a)

43. To turn over a new leaf

(a)  To want something that is not possible

(b)  To change for the better

(c)  To do all possible efforts

(d)  To put an end to something before it develops

Answer: (b)

44. To see the light of the day

(a)  To be in control of the situation

(b)  To gain an advantage on

(c)  To be made available

(d)  To change for a better

Answer: (c)

Directions (Q. Nos. 45-47) A sentence has been divided into four parts and one part has a grammatical error. Choose the part that has an error.

45.

(a)  Post COVID-19

(b)  the Government of India

(c)  made the test and treatment

(d)  free under the Ayushman Bharat Scheme.

Answer: (c)

46.

(a)  I am so glad that

(b)  my husband spoke in on my behalf

(c)  because I felt awful

(d)  that I couldn’t make it to the event.

Answer: (b)

47.

(a)  We thought our company

(b)  was going to expand this year

(c)  but the company actually has

(d)  less people now than it did two years ago.

Answer: (d)

Directions (Q. Nos. 48-50) In each of these questions, choose the option which can be substituted for the given words.

48. One who is able to use the right and left hands equally well.

(a)  Disingenuous

(b)  Multi-tasker

(c)  Ambidextrous  

(d)  Equipoised       

Answer: (c)

49. The study of the origin of humankind.

(a)  Anthropogeny

(b)  Archaeology

(c)  Anthropology

(d)  Psephology

Answer: (a)

50. A person who is lacking in or hostile or indifferent to cultural values.

(a)  Imposter

(b)  Cannibal

(c)  Iconoclast

(d)  Philistine

Answer: (d)

51. What is the unit digit in the expansion of (89628)402?

(a)  8

(b)  6

(c)  4

(d)  2

Answer: (c)

52. Find the sum of all possible values of x if 25849 x 4 is divisible by 4.

(a)  8

(b)  20

(c)  12

(d)  24

Answer: (b)

53. What is the remainder if 293 is divided by 9?

(a)  2

(b)  6

(c)  7

(d)  8

Answer: (d)

54. If  then find the value of a.

(a)  4/5

(b)  2/3

(c)  8/5

(d)  3/5

Answer: (a)

55. The number of unskilled workers in a factory is six times the number of skilled workers. Which of the following cannot be the total number of workers in the factory?

(a)  35

(b)  40

(c)  70

(d)  84

Answer: (b)

56. Average of eleven observations is 30. Average of first five observations is 25 and last five is 28. Find the 6th

(a)  65

(b)  60

(c)  75

(d)  78

Answer: (a)

57. Virat obtained 18% marks in an examination and he failed by 60 marks. If passing marks are 33%, then total marks are ……….. .

(a)  350

(b)  400

(c)  450

(d)  500

Answer: (b)

58. The simple interest on Rs 200 for 7 months at 5 paisa per rupee per month is ……….. .

(a)  Rs. 70

(b)  Rs. 7

(c)  Rs. 35

(d)  Rs. 30.50

Answer: (a)

59. The difference between compound interest and simple interest on a sum of Rs 50000 for 2 yr at 6.5% p.a. is ……… .

(a)  Rs. 201.125

(b)  Rs. 210.25

(c)  Rs. 211.25

(d)  Rs. 225.125

Answer: (c)

60. Rahul travels first 50 km at 10 km/h, next 40 km at 20 km/h and then next 60 km at 15 km/h. Find his average speed for the whole journey.

Answer: (d)

61. The ratio of average temperature between Jaipur and Delhi is 9 : 7 and that between Jaipur and Mumbai is 11 : 8. Find the ratio of average temperature between Delhi and Mumbai.

(a)  7 : 9

(b)  77 : 72

(c)  9 : 8

(d)  11 : 8

Answer: (b)

62. The ratio of three numbers is 3 : 4 : 7 and their product is 18144. The numbers are ……… .

(a)  9, 12 and 21

(b)  15, 20 and 25

(c)  18, 24 and 42

(d)  None of these

Answer: (c)

63. What is the single discount that is equivalent to two successive discounts of 12% and 20%?

Answer: (d)

64. A shopkeeper buys a mobile phone for Rs 12500 and sell sit for Rs. 13350. Find his profit percent.

(a)  6%

(b)  6.3%

(c)  6.5%

(d)  6.8%

Answer: (d)

65. How many minutes will a motor car travelling at 45 km/h take to cover a distance of 15 km?

(a)  15 min

(b)  20 min

(c)  10 min

(d)  30 min

Answer: (b)

66. If the speed of a 100 m long train is 144 km/h, then in how much time will this train cross a standing pole?

(a)  2 sec

(b)  1.5 sec

(c)  2.5 sec

(d)  3.5 sec

Answer: (c)

67. How many zeroes are there in the LCM of 150, 70 and 2500?

(a)  1

(b)  2

(c)  3

(d)  4

Answer: (b)

68. The HCF of 80, 120 and 200 = 10 + ‘?’.

(a)  10

(b)  40

(c)  30

(d)  20

Answer: (c)

69. Find the greatest number of four digits, which is exactly divisible by 18, 24 and 45.

(a)  9580       

(b)  9640

(c)  9720

(d)  9860

Answer: (c)

70. Satish can do a job in 15 days and Kapil works twice as fast as Satish. Then, in how many days can Satish and Kapil together complete the job?

(a)  4.5 days

(b)  5 days

(c)  5.5 days

(d)  6 days

Answer: (b)

Directions (Q. Nos. 71 and 72) Find what will come at the place of question mark ‘?’ in the following questions which will complete the series.

71. 2, 7, 17, 32, 52, ?

(a)  77

(b)  64

(c)  72

(d)  81

Answer: (a)

72. AZ, GT, MN, ‘?’, YB

(a)  KF

(b)  RX

(c)  SH

(d)  TS

Answer: (c)

Directions (Q. Nos. 73 and 74) In the given questions, which number/number-alphabet combination is different from the others.

73.

(a)  Seven

(b)  Two

(c)  Three

(d)  Five

Answer: (b)

74.

(a)  a

(b)  d

(c)  p

(d)  u

Answer: (d)

75. In a certain code language, BEGK is written as ADFJ. How will PSVY will be written in that code language?

(a)  LOQT

(b)  ROUX

(c)  OTUZ

(d)  ORUX

Answer: (d)

76. Pointing towards a man in the photograph Ami said, “He is the only son of the father of the brother of my sister”. How is that person related to Ami?

(a)  Mother

(b)  Father

(c)  Uncle

(d)  None of these

Answer: (d)

77. What is the minimum number of colours required to colour the faces of a cube so that no adjacent faces have the same colour?

(a)  2

(b)  3

(c)  5

(d)  6

Answer: (b)

78. A, B, C, D and E are five cities. There is a way which join two cities. How many total number of ways are there?

(a)  12

(b)  10

(c)  5

(d)  2

Answer: (b)

79. Vishal walks 8 km towards East. He turns left and walks 5 km. Then he rotates 270° Now in which direction is he facing?

(a)  West

(b)  East

(c)  North

(d)  South

Answer: (b)

Directions (Q. Nos. 80-82) Study the information carefully to answer the following questions.

Out of four persons Arun, Bharat, Chandu and Dharma, two are interested in hockey, two are painters, one is a singer and one is a dancer. Each man has one outdoor interest (either cricket or hockey) and one hobby concerned with fine arts (either painting or dancing or signing).

(i) Arun is neither a painter nor he plays hockey.

(ii) Bharat does not dance.

(iii) The person who is a dancer plays hockey.

(iv) Bharat and Dharma do not play cricket.

80. Who among the following plays cricket and is interested in painting?

(a)  Arun

(b)  Bharat

(c)  Chandu

(d)  Arun and Chandu

Answer: (c)

81. Who among the following is a singer?

(a)  Dharma

(b)  Arun

(c)  Chandu

(d)  Bharat

Answer: (b)

82. Which pair shows the correct relationship of the game and hobby of Dharma?

(a)  Cricket and Singing

(b)  Cricket and Painting

(c)  Hockey and Singing

(d)  Hockey and Dancing

Answer: (d)

Directions (Q. Nos. 83-85) For the Assertion (A) and Reason (R) below, choose the correct alternative from the following mark answer as

(a) If both (A) and (R) are true and (R) is the correct explanation of (A).

(b) If both (A) and (R) are true and (R) is not the correct explanation of (A).

(c) If (A) is true but (R) is false.

(d) If (A) is false but (R) is true.

83. Assertion (A) Indian Constitution is the largest written Constitution in the world.

Reason (R) India follows bi-cameral system of Legislative House.

Answer: (b)

84. Assertion (A) Carrying fountain pen while flying in an aeroplane may be a risky proposition.

Reason (R) Due to low pressure, the ink of a fountain pen may leak or overflow.

Answer: (a)

85. Assertion (A) The states in Eastern part of India produce more rice than those in Western part.

Reason (R) The states in eastern part receive more rainfall than those in western part of India.

Answer: (a)

86. Which state will host the 4th edition of Khelo India Youth Games (KIYG) in the year 2021?

(a)  Uttarakhand

(b)  Haryana

(c)  Assam

(d)  None of these

Answer: (b)

87. Who has/have won the 2020 Nobel Prize in Physiology or Medicine for discovering the Hepatitis C Virus?

(a)  Michael Houghton

(b)  Harvey J Alter

(c)  Charles M Rice

(d)  All of these

Answer: (d)

88. Joe Biden of Democratic Party won the 2020 United States (US) Presidential election to become ……. President of the United States.

(a)  44th

(b)  45th

(c)  46th

(d)  47th

Answer: (c)

89. Which Bollywood actress is recently seen in advertisements of the Kalyan Jewellers ‘Muhurat’ Wedding Jewellery along with Amitabh Bachchan?

(a)  Katrina Kaif

(b)  Karishma Kapoor

(c)  Sonam Kapoor

(d)  Disha Patani

Answer: (a)

90. ‘Urban Ladder’ is a well-known online company dealing with ………. .

(a)  Fashion garments and accessories

(b)  Furniture-Home Decor

(c)  Fashion jewellery and accessories

(d)  None of the above

Answer: (b)

91. Which former Indian cricketer has paired with Arvind Fashion to bring out the label ‘True Blue’, a range of summer suits, t-shirts, chinos etc.?

(a)  MS Dhoni

(b)  Virender Sehwag

(c)  Rahul Dravid

(d)  Sachin Tendulkar

Answer: (d)

92. Who among the following was popularly known as ‘Tiger of Mysore’?

(a)  Haider Ali

(b)  Tipu Sultan

(c)  Krishnadevaraya

(d)  Achyuta Deva Raya

Answer: (b)

93. Who was the first Indian to win the Nobel Prize?

(a)  Rabindranath Tagore

(b)  C V Raman

(c)  Amartya Sen

(d)  Mahatma Gandhi

Answer: (a)

94. India celebrates birthday of ………. On 12th January every year as the “National Youth Day”.

(a)  Swami Vivekananda

(b)  Chandra Shekhar Azad

(c)  Subhash Chandra Bose

(d)  Bhagat Singh

Answer: (a)

95. The famous monument ‘Leaning Tower of Pisa’ is located at …………. .

(a)  France

(b)  USA

(c)  Japan

(d)  Italy

Answer: (d)

96. Which of the following is not a function of the RBI?

(a)  Issue of currency

(b)  Conducting monetary policy

(c)  Accepting deposits from the public

(d)  Lender of last resort for the banks

Answer: (c)

97. Durand Cup is associated with which national sports/games?

(a)  Football

(b)  Badminton

(c)  Cricket

(d)  Golf

Answer: (a)

98. Which of the following is not a common property of plastics?

(a)  Non-reactive

(b)  Light in weight

(c)  Durable

(d)  Good conductor of electricity

Answer: (d)

99. Based on their origin, resources can be abiotic and biotic, and biotic resources are ………… .

(a)  derived from non-living things

(b)  made by human beings

(c)  derived from living things

(d)  None of the above

Answer: (c)

100. Parliament of India consists of ………. .

(a)  Rajya Sabha, Lok Sabha and State Assemblies

(b)  President, Prime Minister and Speaker

(c)  President, Rajya Sabha and Lok Sabha

(d)  Prime Minister, Cabinet Ministers and Opposition

Answer: (c)

National Institute of Fashion & Technology (NIFT) Post Graduate 2022 Question Paper With Answer Key

NIFT (Post Graduate)

National Institute of Fashion and Technology

Solver Paper 2022

1. If a + b – c : b + c – a : a + c – b = 5 : 6 : 7, then find a : b : c.

(a)  12 : 13 : 11

(b)  12 : 11 : 13

(c)  13 : 12 : 11

(d)  13 : 11 : 12

Answer: (b)

2. The cost price of 80 articles is Rs. 12.50 per article. 20 opf them were sold for Rs. 18 each. At what price should each of the remaining articles be sold so as to get an overall profit of Rs 4.50 per article?

(a)  Rs. 15

(b) 

(c) 

(d)  Rs. 18

Answer: (b)

3. Divide Rs 6600 into two parts so that the simple interest on the first part for 5 yr at 6% per annum is equal to the simple interest on the second part for 3 yr at 12% per annum.

(a)  Rs. 4000, Rs. 2600

(b)  Rs. 3500, Rs 3100

(c)  Rs. 3800, Rs. 2800

(d)  Rs. 3600, Rs. 3000

Answer: (d)

4. A vessel is full of a mixture of milk and water with 9% milk. 9 L are withdrawn and then replaced with pure water. If the milk is now 6%, then how much does the vessel hold?

(a)  27 L

(b)  18 L

(c)  36 L

(d)  40 L

Answer: (a)

5. There are two containers having mixtures of hydrochloric acid and water. In container 1, the ratio of hydrochloric acid and water is 1 : 2 and in container 2, the ratio of hydrochloric acid and water is 4 : 1. Find the amount of mixture that should be taken from container 1 in order to make 28 L of a mixture containing equal amount of water and hydrochloric acid.

(a)  15 L

(b)  14 L

(c)  20 L

(d)  18 L

Answer: (d)

6. P, Q and R can together earn Rs 3100 in 10 days. Q and R together can earn Rs 1320 in 6 days. P and R together can earn Rs 1050 in 5 days. Find R’s daily earning.

(a)  Rs 100

(b)  Rs 110

(c)  Rs 120

(d)  Rs 90

Answer: (c)

7. A train overtakes two persons, cycling at 9 km/h and 18 km/h in 40s and 48 s respectively. Find the length of the train.

(a)  550 m

(b)  580 m

(c)  625 m

(d)  600 m

Answer: (d)

8. A, B, C and D play four different games among Baseball, Cricket, Kabaddi and Volleyball. A does not play Baseball or Cricket. B does not play Kabaddi or Volleyball. C plays Volleyball and D plays either Baseball or Volleyball. Who plays Cricket?

(a)  A

(b)  B

(c)  C

(d)  D

Answer: (b)

9. What is the angle between the minute hand and the hour hand of a clock at 3 h 4 min ?

(a)  20°

(b)  70°

(c)  90°

(d)  130°

Answer: (d)

10. If 1st January, 1992 is a Tuesday, then on which day of the week will 1st January, 1993 fall?

(a)  Tuesday

(b)  Thursday

(c)  Friday

(d)  Saturday

Answer: (b)

11. Five villages P, Q, R, S and T situated close to each other. P is to the West of Q, R is to the South of P, T is to the North of Q and S is to the East of T. In which direction R is with respect to S?

(a)  North-West

(b)  South-East

(c)  South-West

(d)  Data inadequate

Answer: (c)

Directions (Q. Nos. 12-16) Study the following information carefully and answer the given questions.

A, B, C, D, E, F and G are seven members of a club. Each of them likes one day of the week, viz. Monday, Tuesday, Wednesday, Thursday, Friday, Saturday and Sunday, not necessarily in the same order. Each of them owns a different car, viz. Swift, Alto, Figo, Beat, SX4, Estilo and Optra, not necessarily in the same order.

C likes Wednesday and his favolurite car is neither SX4 nor Optra. E does not like Monday and his favourite car is Beat. The favourite car of one who likes Friday is Figo. The one whose favourite car of one who likes Friday is Figo. The one whose favourite car is Estilo likes Tuesday. D likes Saturday and D’s favourite car is not SX4. G’s favourite car is Alto. F likes Thursday. B does not like Estilo.

12. Who among the following likes Tuesday?

(a)  A

(b)  B

(c)  D

(d)  Data inadequate

Answer: (a)

13. Whose favourite car is Figo?

(a)  A

(b)  B

(c)  F

(d)  C

Answer: (b)

14. Who among the following likes Sunday?

(a)  A

(b)  C

(c)  F

(d)  E

Answer: (d)

15. Whose favourite car is SX4?

(a)  B

(b)  A

(c)  F

(d)  D

Answer: (c)

16. Which of the following combinations is correct?

(a)  F-Thursday-Estilo

(b)  C-Wednesday-Alto

(c)  D-Saturday-Beat

(d)  All are incorrect

Answer: (d)

Directions (Q. Nos. 17-21) Study the following information carefully and answer the given questions.

A, B, C, D, E, F and G are standing in a straight line facing North with equal distances between them, but not necessarily in the same order.

Each one is pursuing a different profession-Actor, Reporter, Doctor, Engineer, Lawyer, Teacher and Painter but not necessarily in the same order.

G is fifth to the left of C. The Reporter is third to the right of G. F is fifth to the right of A. E is second to the left of B. The Engineer is second to the left of D. There are only three people between the Engineer and the Painter. The Doctor is to the immediate left of the Engineer. The Lawyer is to the immediate right of the Teacher.

17. What is A’s profession?

(a)  Painter

(b)  Doctor

(c)  Teacher

(d)  Actor

Answer: (b)

18. Which one is true according to the given arrangement?

(a)  F is the Teacher

(b)  F is third to the left of E

(c)  The Painter is to the immediate left of B

(d)  The Lawyer is standing in the exact middle of the arrangement

Answer: (d)

19. Who among the following is an Actor?

(a)  E

(b)  F   

(c)  C

(d)  B

Answer: (c)

20. What is D’s position with respect to the Painter?

(a)  Third to the left

(b)  Second to the right

(c)  Third to the right

(d)  Second to the left

Answer: (d)

21. Who is an Engineer?

(a)  G

(b)  E

(c)  B

(d)  F

Answer: (a)

22. The first n even natural numbers is equal to k times the sum of first n odd natural numbers.

Answer: (a)

23. If 12 pumps working 7 h a day can lift 2800 tonnes of water in 20 days, then in how many days can 20 pumps working 9 h a day lift 3000 tonnes?

(a)  12

(b)  14

(c)  10

(d)  8

Answer: (c)

24. The ratio of present age of two brothers is 1 : 2 and 5 yr back the ratio was 1 : 3. What will be the ratio their age after 5 yr?

(a)  2 : 5

(b)  4 : 5

(c)  3 : 5

(d)  1 : 5

Answer: (c)

25. The price of petrol increased by 2% in a certain week and increased by 4% in the next week. Find the net percentage increase in the price of petrol over these two weeks.

(a)  6.12%

(b)  6.08%

(c)  6.16%

(d)  6.20%

Answer: (b)

26. At what per cent will simple interest on Rs 1950 amount to Rs 253.50 in 

(a)  3%

(b)  5%

(c)  4%

(d)  4.5%

Answer: (c)

27. What annual installment will discharge a debt of Rs 3094 due in 4 yr at 7% simple interest?

(a)  Rs 650

(b)  Rs 1200

(c)  Rs 900

(d)  Rs 700

Answer: (d)

28. In how many years interest of a sum will be (3/5)th of the total amount at 10% simple interest?

(a)  10  yr

(b)  12 yr

(c)  15 yr

(d)  13 yr

Answer: (c)

29. P and Q working together can complete a job in 16 days. P alone can complete it in 18 days. Both work together for 4 days and then Q leaves. Find the time taken by P to complete the remaining work.

(a)  12.5 days

(b)  13.5 days

(c)  14 days

(d)  14.5 days

Answer: (b)

30. A tank has a leak at its bottom which empties it at 6 L/min. It also has a filling tap which can fill the tank in 6 h. The tank takes 18 h to become full. Find the capacity of the tank.

(a)  3403 L

(b)  4023 L

(c)  3402 L

(d)  3240 L

[bg_colla pse view=”button-orange” color=”#4a4949″ icon=”arrow” expand_text=”Show Answer” collapse_text=”Hide Answer” ]

Answer: (d)

[/bg_collapse]

31. A person covered a certain distance at a certain speed. If his speed was 20% more, he would take 10 min less to cover the same distance. Find the time he takes to cover the distance.

(a)  60 min

(b)  50 min

(c)  45 min

(d)  42 min

Answer: (a)

32. What number will replace the question mark?

(a)  5

(b)  6

(c)  7

(d)  12

Answer: (c)

33. Which of the following countries is known as the Clove Bowl of the World?

(a)  Baharin

(b)  Zanzibar

(c)  Gibralter

(d)  Brazil

Answer: (b)

34. The land of Thousand Islands is

(a)  Malaysia

(b)  Indonesia

(c)  Ireland

(d)  Finland

Answer: (b)

35. Which of the following countries is known as Cockpit of Europe?

(a)  Sweden

(b)  Netherlands

(c)  Belgium

(d)  Germany

Answer: (c)

36. National Start Up day is

(a)  January 13

(b)  January 14

(c)  January 15

(d)  January 16

Answer: (d)

37. Where Atacama Desert is located?

(a)  Peru

(b)  Iran

(c)  Chile

(d)  Namibia

Answer: (c)

38. Who is the Managing Director of International Monetary Fund?

(a)  Kristalina Georgieva

(b)  Gita Gopinath

(c)  Geoffery W.S. Okamoto

(d)  Rodrigo Rato

Answer: (a)

39. The singer of Samaveda is known as

(a)  Ardhavayu

(b)  Udgatr

(c)  Hotra

(d)  None of these

Answer: (b)

40. The river Parushini is known as

(a)  Swati

(b)  Ravi

(c)  Ghaggar

(d)  Kabul

Answer: (b)

41. Who wrote the book “Hasanat-ul-Arifin”?

(a)  Abul Fazl

(b)  Dara Shikoh

(c)  Nizamuddin Ahmed

(d)  Abbas Khan Sherwani

Answer: (b)

42. What is the capital of Macedonia?

(a)  Vilnius

(b)  Riga

(c)  Valletta

(d)  Skopje

Answer: (d)

43. The imaginary line in southern hemisphere that can be drawn at  of equator is known as

(a)  Tropic of Cancer

(b)  Tropic of Capricorn

(c)  Antarctic Circle

(d)  Arctic Circle

Answer: (c)

44. Damon Galgut won the Booker Prize, 2021 for the book

(a)  In a Strange Room

(b)  The Good Doctor

(c)  Arctic Summer

(d)  The Promise

Answer: (d)

45. The Study of spiders and related animals is known as

(a)  Ophiology

(b)  Arachnology

(c)  Ethology

(d)  Myology

Answer: (b)

46. The capital of the country Mozambique is

(a)  Maputo

(b)  Rabat

(c)  Lilongwe

(d)  Accra

Answer: (a)

47. The Asiatic Society in Kolkata was established on

(a)  15 January 1854

(b)  25 January 1784

(c)  15 January 1874

(d)  15 January 1784

Answer: (d)

48. A car covered a certain distance at 90 km/h and returned back at 60 km/h. Find his average speed (in km/h) for the entire journey.

(a)  80

(b)  70

(c)  72

(d)  75

Answer: (c)

49. Find the time taken by a 200 m long train running at 36 km/h to cross a boy standing on a platform.

(a)  15 s

(b)  20 s

(c)  23 s

(d)  25 s

Answer: (b)

50. In a 600 m race, P gives Q a start of 200 m. Ratio of speeds of P and Q is 5 : 4. By what distance Q wins the race?

(a)  100 m

(b)  125 m

(c)  150 m

(d)  175 m

Answer: (a)

Directions (Q. Nos. 51-55) Study the following information carefully and answer the given questions.

P, Q, R, S, T, U, V and W are eight employees of a concern. Each is allotted a different locker out of eight lockers numbered 1 to 8 in a cupboard. The lockers are arranged in four rows with two lockers in each now.

Lockers 1 and 2 are in the top row from left to right, respectively while lockers 7 and 8 are in the bottom row arranged from left to right, respectively. Lockers 3 and 4 are in the second row from the top – arranged from right to left, respectively. So are lockers 5 and 6 – arranged from right to left, respectively – in the second row from the bottom. P has been allotted locker 1 while V has been allotted locker 8. T’s locker is just above that of Q which is just above that of R, whereas W’s locker is in the bottom row.

51. Which of the following cannot be the correct locker number-occupant pair?

(a)  3-Q

(b)  7-W

(c)  4-U

(d)  6-R

Answer: (d)

52. If U’s locker is not beside Q’s locker, whose locker is just above the of W’s locker?

(a)  U

(b)  S

(c)  R

(d)  Q

Answer: (a)

53. Which of these pairs cannot have lockers that are diagonally placed?

(a)  P-Q

(b)  S-R

(c)  U-R

(d)  Either option (b) or (c)

Answer: (d)

54. Which of the following groups consists only occupants of odd numbered lockers?

(a)  Q, R, W

(b)  R, V, W

(c)  T, R, Q

(d)  P, T, Q

Answer: (a)

55. If U’s locker is in the same row as that of R and S exchanges his locker with V, then who is the new neighbor of V in the same row? (assume that nothing else is distributed from the original arrangement)

(a)  P

(b)  Q

(c)  R

(d)  U

Answer: (b)

Directions (Q. Nos. 56-60) Study the following information carefully and answer the given questions.

P, Q, R, S and T sit around a table. P sits two seats to the left of R and Q sits two seats to the right of R.

56. If S sits in between Q and R, who sits to the immediate right of P?

(a)  T

(b)  S

(c)  Q

(d)  R

Answer: (a)

57. Which of the following cannot be the correct seating arrangement of the five persons in either the clockwise direction or the anti-clockwise direction?

(a)  P, Q, R, S, T

(b)  P, S, R, T, Q

(c)  P, Q, S, R, T

(d)  P, T, R, S, Q

Answer: (a)

58. If S is not sitting next to Q, who is sitting between Q and S ?

(a)  R

(b)  P

(c)  T

(d)  Both R and P

Answer: (b)

59. If a new person U joins the group such that the initial conditions for the seating arrangement should be observed and also a new condition that U does not sit next to R be satisfied, then which of the following statements is true?

(a)  U sits to the immediate right of S

(b)  U sits to the immediate left of T

(c)  U sits to the immediate left of P

(d)  Either a or b

Answer: (c)

60. If a new person U joins the group such that the initial conditions for the seating arrangement should be observed and also a new condition that U does not sit next to P, S or T be satisfied, then who will be the neighbours of P (one on either side) ?

(a)  S and T

(b)  S and Q

(c)  T and R

(d)  R and Q

Answer: (a)

Directions (Q. Nos. 61-65) Choose the one which can be substituted for the given words/sentence.

61. Commencement of adjacent words with the same letter.

(a)  Consonance

(b)  Concubine

(c)  Alliteration

(d)  Euphemism

Answer: (c)

62. A wall built to prevent the sea or a river from flooding an area-

(a)  Dyke

(b)  Surveillance

(c)  Nuance

(d)  Reconnaissance

Answer: (a)

63. An entertainer who performs difficult physical actions

(a)  Clown

(b)  Gymnast

(c)  Nuance

(d)  Acrobat

Answer: (d)

64. A white skinned woman with dark brown hair-

(a)  Amazon

(b)  Brunette

(c)  Swashbuckler

(d)  Anodyne

Answer: (b)

65. Bitter and violent attack in words-

(a)  Pandemonium

(b)  Prototype

(c)  Diatribe

(d)  Profanity

Answer: (c)

Directions (Q. Nos. 66-70) Groups of four words are given. In each group one word is wrongly spelt, Find out the wrongly spelt word.

66.

(a)  Connoisseur

(b)  Insolvent

(c)  Jubillee

(d)  Spendthrift

Answer: (c)

67.

(a)  Renunciation

(b)  Quotiant

(c)  Survivor

(d)  Efficient

Answer: (b)

68.

(a)  Commemoration

(b)  Accommodation

(c)  Asassination

(d)  Association

Answer: (c)

69.

(a)  Abarrant

(b)  Abecedarian

(c)  Barouche

(d)  Barbarian

Answer: (a)

70.

(a)  Replaceable

(b)  Presumptuous

(c)  Prerogative

(d)  Predater

Answer: (d)

Directions (Q. Nos. 71-75) Choose the one which best expresses the meaning of the given word.

71. INADVERTENT

(a)  Thoughtless

(b)  Unintentional

(c)  Insane

(d)  Unintelligent

Answer: (b)

72. INFURIATE

(a)  Burn

(b)  Disgrace

(c)  Threaten

(d)  Enrage

Answer: (d)

73. PROSPECTIVE

(a)  Potential

(b)  Prosperous

(c)  Perplexed

(d)  Possible

Answer: (a, d)

74. LOUSY

(a)  Unbearable

(b)  Unpleasant

(c)  Awful

(d)  Stinking

Answer: (c)

75. ACCENTUATED

(a)  Exhibited

(b)  Devalued

(c)  Mitigated

(d)  Sharpened        

Answer: (a)

Directions (Q. Nos. 76-80) Choose the word opposite in meaning to the given word.

76. LACERATION

(a)  Convalescence

(b)  Recuperation

(c)  Healing

(d)  Palpitation

Answer: (c)

77. IMPLICATE

(a)  Disentangle

(b)  Impersonate

(c)  Complicate

(d)  Impose

Answer: (a)

78. INGEST

(a)  Disrupt

(b)  Disgorge

(c)  Dismiss

(d)  Display

Answer: (b)

79. HERETICAL

(a)  Contradictory

(b)  Doubtful

(c)  Impious

(d)  Orthodox

Answer: (d)

80. PHILISTINE

(a)  Cultured

(b)  Libertine

(c)  Sober

(d)  Educated

Answer: (a)

Directions (Q. Nos. 81-85) In the following question a part of the sentence is bold. Below are given alternatives to the bold part as (a), (b), (c), and (d) which may improve the sentence. Choose the correct alternative.

81. The young boy was considered uncorrectable by his parents.

(a)  impolite

(b)  incorrigible

(c)  unruly

(d)  no improvement

Answer: (b)

82. Write down the address lest you may forget.

(a)  you may not forget

(b)  you cannot forget

(c)  you will forget

(d)  you should forget

Answer: (d)

83. He is addicted to smoke.

(a)  addicted to smoking

(b)  used to smoke

(c)  addicted of smoking

(d)  addicted with smoking

Answer: (a)

84. I will be with you in one quarter of an hour

(a)  a quarter of one hour

(b)  a quarter of an hour

(c)  a quarter of hour

(d)  no improvement

Answer: (b)

85. I am thinking to do an M.A. in English

(a)  wondering to do

(b)  thinking doing

(c)  thinking of doing

(d)  no improvement

Answer: (c)

Directions (Q. Nos. 86-90) Choose the alternative which best expresses the meaning of the idiom/phrase.

86. There is no love lost between any two neighbouring countries in the world.

(a)  stop loving

(b)  not on good terms

(c)  forming a group

(d)  have good understanding

Answer: (b)

87. When it was the peak season of his business, he played ducks and drakes with money.

(a)  distributed among the poor

(b)  spent money carelessly

(c)  played childish game

(d)  closed his season

Answer: (b)

88. The story of his survival in Tsunami beggars description.

(a)  is beyond description

(b)  is interesting

(c)  is described by a beggar

(d)  not described by anyone

Answer: (a)

89. I don’t know why she has become stand-offish recently.

(a)  angry

(b)  hilarious

(c)  indifferent

(d)  unmanageable

Answer: (c)

90. The president of this club should be someone who is open and above board.

(a)  far better

(b)  highly qualified

(c)  honest

(d)  rich and hard working

Answer: (c)

Directions (Q. Nos. 91-95) Read the following passage carefully and Answer the Questions given below:

Many people are aware of the famous “witch trials” that occurred at the end of the seventeenth century in Salem, Massachusetts. But fewer people know that men, women, and children had been persecuted in New England for practicing witchcraft decades before the Salem trials ever took place. The events in Salem happened between February 1692 and May 1693 and resulted in twenty deaths, but colonies in Connecticut, New Hampshire, and Vermont accused women of practicing witchcraft as early a the 1640s. Colonies in Connecticut executed eleven women on these charges between 1647 and 1663.

Colonists brought with them to North America the widespread European belief in dark magic and the supernatural. Suspicious behavior, sudden illnesses or deaths in families, and rivalries among neighbours often led to accusations of witchcraft. Witchcraft was a capital crime in colonial North America until 1750, and though the judicial procedure varied from colony to colony, often the only evidence needed to start a trial was the testimony of a single witness. The accused woman was ordered to appear before a judge to confess her wrongdoing. If she did not confess, she was brought before a jury, where she endured questioning, torture, and tests that would supposedly reveal her affiliation with the devil. If the woman was found guilty, she was sentenced to death.

The trials ceased in the mid-1690s after thirty-five people had been killed. The governor of Massachusetts put an end to the process after receiving pleas from legal scholars-though it should be noted that the governor’s own wife had been accused of witchcraft right before he called off the trials. Colonists began demanding public apologies from the judges and jury members in 1695, and petitions were filed seeking to reverse the alleged witches convictions as early as 1700. The Massachusetts, New Hampshire, and Vermont governments have formally acknowledged the harm done to the women, but the Connecticut government has yet to clear their names. To this day, descendants of the eleven women put to death in Connecticut hundreds of years ago are fighting to clear their ancestor’s names.

91. According to the passage, in which year did the Salem witch trials end?

(a)  1647

(b)  1663

(c)  1692

(d)  1693

Answer: (d)

92. Which of the following would be the most appropriate subtitle for this passage?

(a)  Seeking Justice for Connecticut’s Convicted Witches

(b)  The Lost Souls of Salem

(c)  The Witch Trials of Colonial New England

(d)  Religion and Witchcraft in Early New England

Answer: (c)

93. Which of the following words best describes the public apology given to victims of the colonial witch trials?

(a)  partial

(b)  delayed

(c)  inclusive

(d)  appropriate

Answer: (a)

94. Based on the information in the final paragraph, it can be inferred that the author believes

(a)  the trials should have continued well into the 1700s

(b)  the Massachusetts governor should have formed an improved legal system       

(c)  the descendants of the Connecticut witches are fighting a useless battle

(d)  the governor’s decision to end the trials may have been unduly influenced

Answer: (d)

95. In paragraph 2, we learn that “Colonists brought with them to North America the widespread European belief in dark magic and the supernatural.” Based on its use in paragraph 2, which of the following examples accurately describes a scenario involving the supernatural?

(a)  Ben can read books faster than anyone else in class.

(b)  Jamie told me she talks to her guardian angel every night.

(c)  The mansion on the hill has been abandoned for over a century.

(d)  Calvin expected the house to be full of people, but it was dark and silent

Answer: (b)

Directions (Q. Nos. 96-100) Read the following passage carefully and answer the questions given below.

In the height of the Enlightenment, men influenced by the new political theories of the era launched two of the largest revolutions in history. These two conflicts, on two separate continents, were both initially successful in forming new forms of government. And yet, the two conflicts, though merely a decade apart, had-radically different conclusions. How do two wars inspired by more or less the same ideals end up so completely different? Why was the American Revolution largely a success and the French Revolution largely a failure?

Historians have pointed to myriad reasons-far too various to be listed here. However, the most frequently cited are worth mentioning. For one, the American Revolution was far removed from the Old World; that is, since it was on a different continent, other European nations did not attempt to interfere with it. However, in the French Revolution, there were immediate cries for war from neighbouring nations. Early on, for instance, the ousted king attempted to flee to neighbouring Austria and the army waiting there. The newly formed French Republic also warred with Belgium, and a conflict with Britain loomed. Thus, the French had the burden not only of winning a revolution but also defending it from outside. The Americans simply had to win a revolution.

Secondly, the American Revolution seemed to have a better chance for success from the get-go, due to the fact that Americans already saw themselves as something other than British subjects. Thus, there was already a uniquely American character, so, there was not as loud a cry to preserve the British way of life. In France, several thousands of people still supported the king, largely because the king was seen as an essential part of French life. And when the king was first ousted and then killed, some believed that character itself was corrupted. Remember, the Americans did not oust a king or kill him-they merely separated from him.

Finally, there is a general agreement that the French were not as unified as the Americans, who, for the most part, put aside their political differences until after they had already formed a new nation. The French, despite their Tennis Court, Oath, could not do so. Infighting led to inner turmoil, civil war, and eventually the Reign of Terror, in which political dissidents were executed in large numbers. Additionally, the French people themselves were not unified. The nation had so much stratification that it was impossible to unite all of them-the workers, the peasants, the middleclass, the nobles, the clergy-into one cause. And the attempts to do so under a new religion, the Divine Cult of Reason, certainly did not help. The Americans, remember, never attempted to change the society at large; rather, they merely attempted to change the government.

96. It can be inferred from paragraph 1 that the author most likely believes that

(a)  historians have not come to a conclusion about what separates the French and American Revolutions

(b)  the French Revolution could have been just as successful as the American Revolution was

(c)  two events based on the same philosophy are generally not that common

(d)  two events that are similar in one way ought to be similar in other ways

Answer: (d)

97. What function do the questions that end paragraph I play in the passage?

(a)  They introduce the topic the author of the passage will explore, even though he or she will not answer the questions.

(b)  They introduce the author’s opinions on the main topic and summarise his or her main argument.

(c)  They present rhetorical questions that are used only for the purposes of engaging the reader, even though the questions will not be answered.

(d)  They refocus the passage into its original topic after a detour into historical context.

Answer: (b)

98. As used in paragraph 2, the word myriad most nearly means

(a)  an unknowable secret

(b)  an uncountable number

(c)  an unrealistic ideal

(d)  an unusually rare occurrence

Answer: (b)

99. According to the passage, the Reign of Terror happened because of

(a)  social stratification in America

(b)  infighting in the American Revolution

(c)  the decision to kill the king in the French Revolution

(d)  infighting in the French Revolution

Answer: (d)

100. According to the author, which of the following best summarises the most frequently cited explanations historians give for why the French and American Revolution did not turn out the same way?

(a)  Locations relative to Europe, social equality in America, and the lack of a unique French identity

(b)  Social unity among the Americans, social stratification among the French, and the character of the French and Americans in general

(c)  Locations relative to Europe, American and French characters, and political cohesion or lack thereof

(d)  Locations relative to Europe, the decision to oust or not oust a king, and a lack of unity amongst all the revolutionaries

Answer: (c)

Directions (Q. Nos. 101-105) Read the following passage and answer the questions given below:

By the mid-nineteenth century, mass production of paper patterns, the emergence of the home sewing machine, and the convenience of mail order catalogs brought fashionable clothing into the American home. By the early twentieth century, home economists working in extension and outreach programs taught women how to use paper patterns to improve the fit and efficiency of new garments as well as how to update existing ones.

Teachers of home economics traditionally made home sewing a critical part of their curriculum, emphasizing self-sufficiency and resourcefulness for your women. However, with the increasing availability of mass-produced clothing in catalogs and department stores, more and more women preferred buying garments to making them. As a result, home economist shifted their attention to consumer education. Through field study, analysis, and research, they became experts on the purchase and preservation of ready-to-wear clothing for the family, offering budgeting instruction targeted at adolescent girls. Modern home sewing made it possible for American women to transcend their economic differences and geographic locations with clothing that was increasingly standardized. The democratisation of fashion continued through the twentieth century as the ready-to-wear market expanded and home sewing became more of a pastime than a necessity.

101. According to the passage, the advent of mail order catalogs altered the role of home economists because

(a)  mass-produced clothing rendered their jobs obsolete

(b)  women ceased sewing so home economists had to teach other subjects

(c)  their focus shifted to instruction on budgeting and buying and preserving clothing

(d)  home economists had to compete with the ready-to-wear marketplace.

Answer: (b)

102. The passage focuses on the

(a)  historical shifts in home sewing from the mid-nineteenth century through the twentieth century

(b)  changing role of home economists as a result of changes in the world

(c)  modernization of home sewing

(d)  effects of home economists on home sewing

Answer: (b)

103. As used in paragraph 2 which is the best definition for democratization?

(a)  Transitioning to a more democratic political regime

(b)  Altering or modifying in a beneficial way

(c)  Becoming more affordable to the lower class

(d)  Becoming widely available to a populace

Answer: (d)

104. Based on information in the passage, it can be inferred that home sewing allowed American women to do all of the following except

(a)  continue to wear clothing that had gone out of style in stores

(b)  copy fashions they had seen elsewhere

(c)  create less expensive versions of current styles

(d)  become experts on budgeting and consumerism

Answer: (d)

105. According to the passage, which of the following led to a decline in home sewing?

(I) mail order catalogs

(II) the availability of apparel in retail outlets

(III) curriculum changes in home economics

(a)  Only I

(b)  Only II

(c)  Both I and II

(d)  Both II and III

Answer: (c)

Directions (Q. Nos. 106-110) Read the following passage and answer the questions given below:

Ancient Greek doctors relied heavily on the theory of humorism, a belief that the body was essentially balanced by four substances known as humours and that any ailme was simply the result of an imbalance of these humours. The four substances-black bile, yellow bile, phlegm, and blood-were all found in the body, and each was associated with a different element-air, fire, earth, and water, respectively; too much of one would cause disease. An excess of black bile in your body, for instance, meant you were melancholic; too much blood made you amorous and sanguine. While it is easy today to sneer at such beliefs, theories like this one; though ludicrous, matter historically. The beauty of the ancient wisdom is not that it was actually wise but that the logic behind it opened the door for future scientific inquiry. Thus, while my cough might not be caused by a lack of blood, I can thank Hippoerates and his cohorts for encouraging later doctors to find the actual source of my cough.

106. According to the passage, which of the following correctly matches a humour to its element?

(a)  Phlegm : fire

(b)  Black bile : water

(c)  Yellow bile : fire

(d)  Blood : earth

Answer: (c)

107. As used near the end of the passage, the phrase ‘sneer at’ most nearly means

(a)  deplore

(b)  deride

(c)  debunk

(d)  decline

Answer: (b)

108. It can be inferred from the passage that Hippocrates was

(a)  an ancient philosopher         

(b)  a disease researcher

(c)  a modern humanist

(d)  a Greek physician

Answer: (d)

109. Which of the following would be the most appropriate subtitle for this passage?

(a)  Ancient Humor, Modern Applications    

(b)  Greek Philosophy and the Origin of Science

(c)  The Humorists and the Scientists

(d)  The Humors and Why They Matter

Answer: (d)

110. It can be inferred from the penultimate (second-to-last) sentence of the passage that the author believes the hunours

(a)  deserve to be studied today for historical reasons

(b)  are laughably inaccurate as explanations for ailments

(c)  can offer alternative treatment for diseases

(d)  should continue to be taught in medical schools

Answer: (a)

Directions (Q. Nos. 111-115) Read the following passage and answer the questions given below:

Imagine a mythical beast rising before you on two legs that end in cloven hooves. Sharp, curled horns and pointed ears sit atop its head, while a long tongue lolls out of a mouth that seems perpetually frozen in a sinister smile. It carries rusty chains and a pack upon its back, and it waits until nightfall to enter homes and terrorize youngsters, For centuries, horrifying tales of this creature induced nightmares in children during a certain holiday. But while this monster seems like it would be right at home in a Halloween horror story, children across Europe know the creature as Krampus, a demonic creature that punishes misbehaving youth during Christmas.

The legend of Krampus has its roots in Germanic folklore and was popularized in central European countries such as Austria, Hungary, and Slovenia during the seventeenth century. The beast was said to be the uncharitable counterpart to the magnanimous St. Nicholas. While St. Nicholas filled children’s wooden shoes and stockings with treats, Krampus sought out the children who didn’t deserve gifts and delivered spankings and whippings. Children who had been especially bad during the year were in danger of being kidnapped and taken back to the creature’s lair for further punishment until they showed true remorse for their misbehaviour. By the mid-twentieth century, however, people were actively trying to bring an end to perpetuating the story of Krampus. Prominent newspapers across Europe published editorials discouraging parents to share the legend with their children, and the Austrian government distributed “Krampus Is an Evil Man,” a cautionary pamphlet written by psychologists. Thankfully, fewer children grow up fearing Krampus is modern times. But fans of history, folk tales, and offbeat holiday traditions still take time every December to reflect on one of the most frightening characters ever associated with a holiday.

111. Using the passage as a guide, it can be understood that

(I) efforts taken in the mid-twentieth century to discourage sharing the tale of Krampus were effective

(II) the legend of Krampus was popularized when the Austrian government distributed pamphlets

(III) children in Central European countries were afraid of getting eaten by Krampus every Christmas

(a)  I only

(b)  II only

(c)  Both I and II

(d)  Both II and III

Answer: (a)

112. Based on its use in paragraph 2, it can be inferred that the word ‘magnanimous’ belongs to which of the following word groups?

(a)  dastardly, vile, detestable

(b)  generous, compassionate, benevolent

(c)  refined, polish, genteel

(d)  astute, perceptive, intelligent

Answer: (b)

113. Which of the following statements would the author most likely agree with?

(a)  It is for the best that fewer children grow up knowing the legend of Krampus.

(b)  The image of Krampus was not very frightening and shouldn’t have bothered children.

(c)  Children should not expect gifts during the holidays unless they behave all year.

(d)  The legend of Krampus is too juvenile to be discussed in prominent newspapers.

Answer: (a)

114. In the final sentence of the passage, we learn that, “fans of history, folk tales, and offbeat holiday traditions still take time every December to reflect on one of the most frightening characters ever associated with a holiday.” Based on its use in this sentence, which of the following accurately describes something that is offbeat?

(a)  The new restaurant downtown offer simple, traditional American cuisine.

(b)  The toddler had a tantrum after his mother refused to buy him a new toy.

(c)  I ate turkey, stuffing, and pumpkin pie on Thanksgiving last year.

(d)  My cousin’s handmade clothing combines contrasting colors and patterns.

Answer: (d)

115. Which of the following would be the most appropriate subtitle for this passage?

(a)  Holiday Monsters from Halloween to Christmas         

(b)  The Yuletide Legends of Central Europe

(c)  Austria’s Holiday Traditions and Rituals

(d)  On the Dark Side of Christmas

Answer: (d)

Directions (Q. Nos. 116-120) Read the following passage and answer the questions given below:

By far, the most popular exhibit at the National Zoo in Washington, DC is the zoo’s pair of giant pandas, Mei Xiang and Tian Tian. These are the second set of pandas government made to the Nixons during then-President Nixon’s historic trip to China in 1972. Nixon was the first sitting President to visit China since the nation became Communist. The popularity of the exhibit today is easy to understand: The zoo’s giant pandas are two of only 300 captive pandas in the world, two of only 1,900 total pandas in either China or captivity! Yet, while literally thousands of people ‘bear’ witness to the giant pandas each day, few can ‘bear’ to face a simple reality: the socalled ‘panda bears’ may not be bears to all.

The debate began in 1869, when the first European witness of a giant panda likened the strange animal to a bear. However, Alphonse Milne-Edwards, a French scientist, challenged that classification after reviewing the remains of a giant panda and concluding that it was physiologically closer to the red panda, a member of the raccoon family. To this day, scientists are not certain about how to classify the giant panda.

But the giant panda is plainly a bear on sight, so how can it be classified with raccoons? Well, part of it is because taxonomy is hardly a perfect science, and classification occurs on many levels; appearance, genetics, behaviour, evolution, etc. In appearance, the red and giant pandas are as similar as they are dissimilar. Likewise, DNA research has revealed similar links between giant pandas and both bears and raccoons. Behaviourally, the only animal that is clearly similar to the giant panda is the red panda. So, even though the giant panda moves like a bear, it eats bamboo in the same manner as the red panda. Eating bamboo could, however, be a trait that developed independently in both species, meaning that tracing evolution many likewise be an inconclusive path. Perhaps, in the end it is best to merely classify both red and giant pandas as, simply, ‘pandas’. Certainly, the classification does not make the animals any less majestic.

116. Which sentence, if removed from the first paragraph, would strengthen the passage?

(a)  “By far, the most popular exhibit at the National Zoo in Washington, DC is the zoo’s pair of giant pandas. Mei Xiang and Tian Tian.”

(b)  “These are the second set of pandas the zoo has owned; the first two were a gift the Chinese government made to the Nixons during then-President Nixon’s a historic trip to China  in 1972.”

(c)  “Nixon was the first sitting president to visit China since the nation became Communist.”

(d)  “The popularity of the exhibit today is easy to understand: The zoo’s giant pandas are two of only 300 captive pandas in the world, and two of only 1,900 total pandas in either China or captivity!”

Answer: (c)

117. The author traces the popularity of the giant panda exhibit at the National Zoo to the

(a)  debate over classifying giant pandas

(b)  fact that the pandas were gifts to the United States

(c)  small number of giant pandas in the wild

(d)  scarcity of giant pandas in zoos

Answer: (d)

118. Each of the following can be inferred from the passage except that

(a)  giant pandas primarily live in China

(b)  the first giant panda was seen by a non-European

(c)  giant pandas are clearly members of the raccoon family

(d)  scientists are still not certain how to classify giant pandas

Answer: (c)

119. Which of the following statements made in the passage least summarizes the author’s main point?

(a)  “to this day, scientists are not certain about how to classify the giant panda”

(b)  “taxonomy is hardly a perfect science”

(c)  “the so-called’ panda bears’ may not be bears at all”

(d)  “it is best to merely classify both the red and giant pandas as, simply, pandas”

Answer: (b)

120. The author’s purpose in writing this passage is most likely to

(a)  suggest that a species has been misclassified

(b)  inform readers about an important scientific debate

(c)  argue that a species is difficult to classify

(d)  refute an argument about two species

Answer: (c)

© Copyright Entrance India - Engineering and Medical Entrance Exams in India | Website Maintained by Firewall Firm - IT Monteur